Resolved, That For His Refusal To Reveal The Name of The Person To Whom He Gave The P440,000 Jean L

You might also like

Download as docx, pdf, or txt
Download as docx, pdf, or txt
You are on page 1of 157

G.R. No.

L-3820             July 18, 1950 It was in the face of the antecedents sketched in the last three preceding paragraphs that the Philippine
Government, through the Secretary of Justice as Chairman of the Board of Directors of the Rural
JEAN L. ARNAULT vs. LEON NAZARENO, Progress Administration and as Chairman of the Board of Directors of the Philippine National Bank, from
which the money was borrowed, accomplished the purchase of the two estates in the latter part of
October, 1949, as stated at the outset.
This is an original petition for habeas corpus to relieve the petitioner from his confinement in the New
Bilibid Prison to which he has been committed by virtue of a resolution adopted by the Senate on May 15,
1950, which reads as follows: On February 27, 1950, the Senate adopted its Resolution No. 8, which reads as follows:

Whereas, Jean L. Arnault refused to reveal the name of the person to whom he gave the P440,000, as RESOLUTION CREATING A SPECIAL COMMITTEE TO INVESTIGATE THE BUENAVISTA AND THE
well as answer other pertinent questions related to the said amount; Now, therefore, be it. TAMBOBONG ESTATES DEAL.

Resolved, that for his refusal to reveal the name of the person to whom he gave the P440,000 Jean L. WHEREAS, it is reported that the Philippine government, through the Rural Progress Administration, has
Arnault be committed to the custody of the Sergeant-at-Arms and imprisoned in the New Bilibid Prison, bought the Buenavista and the Tambobong Estates for the aggregate sum of five million pesos;
Muntinlupa, Rizal, until discharged by further order of the Senate or by the special committee created by
Senate Resolution No. 8, such discharge to be ordered when he shall have purged the contempt by WHEREAS, it is reported that under the decision of the Supreme Court dated October 31, 1949, the
revealing to the Senate or to the said special committee the name of the person to whom he gave the Buenavista Estate could have been bought for three million pesos by virtue of a contract entered into
P440,000, as well as answer other pertinent questions in connection therewith. between the San Juan de Dios Hospital and Philippine Government in 1939;

The facts that gave rise to the adoption of said resolution, insofar as pertinent here, may be briefly stated WHEREAS, it is even alleged that the Philippine Government did not have to purchase the Buenavista
as follows: Estate because the occupation government had made tender of payment in the amount of three million
pesos, Japanese currency, which fact is believed sufficient to vest title of Ownership in the Republic of the
In the latter part of October, 1949, the Philippine Government, through the Rural Progress Administration, Philippines pursuant to decisions of the Supreme Court sustaining the validity of payments made in
bought two estates known as Buenavista and Tambobong for the sums of P4,500,000 and P500,000, Japanese military notes during the occupation;
respectively. Of the first sum, P1,000,000 was paid to Ernest H. Burt, a nonresident American, thru his
attorney-in-fact in the Philippines, the Associated Estates, Inc., represented by Jean L. Arnault, for alleged WHEREAS, it is reported that the Philippine Government did not have to pay a single centavo for the
interest of the said Burt in the Buenavista Estate. The second sum of P500,000 was all paid to the same Tambobong Estate as it was already practically owned by virtue of a deed of sale from the Philippine
Ernest H. Burt through his other attorney-in-fact, the North Manila Development Co., Inc., also Trust Company dated September 3, 194, for seven hundred and fifty thousand pesos, and by virtue of the
represented by Jean L. Arnault, for the alleged interest of the said Burt in the Tambobong Estate. recission of the contract through which Ernest H. Burt had an interest in the estate; Now, therefore, be it.

The original owner of the Buenavista Estate was the San Juan de Dios Hospital. The Philippine RESOLVED, That a Special Committee, be, as it hereby is, created, composed of five members to be
Government held a 25-year lease contract on said estate, with an option to purchase it for P3,000,000 appointed by the President of the Senate to investigate the Buenavista and Tambobong Estate deals. It
within the same period of 25 years counted from January 1, 1939. The occupation Republic of the shall be the duty of the said Committee to determine whether the said purchase was honest, valid, and
Philippines purported to exercise that option by tendering to the owner the sum of P3,000,000 and, upon proper and whether the price involved in the deal was fair and just, the parties responsible therefor, and
its rejection, by depositing it in court on June 21, 1944, together with the accrued rentals amounting to any other facts the Committee may deem proper in the premises. Said Committee shall have the power to
P3224,000. Since 1939 the Government has remained in possession of the estate. conduct public hearings; issue subpoena or subpoena duces tecum to compel the attendance of
witnesses or the production of documents before it; and may require any official or employee of any
On June 29, 1946, the San Juan de Dios Hospital sold the Buenavista Estate for P5,000,000 to Ernest H. bureau, office, branch, subdivision, agency, or instrumentality of the Government to assist or otherwise
Burt, who made a down payment of P10,000 only and agreed to pay P5000,000 within one year and the cooperate with the Special Committee in the performance of its functions and duties. Said Committee
remainder in annual installments of P500,000 each, with the stipulation that failure on his part to make any shall submit its report of findings and recommendations within two weeks from the adoption of this
of said payments would cause the forfeiture of his down payment of P10,000 and would entitle the Resolution.
Hospital to rescind to sale to him. Aside from the down payment of P10,000, Burt has made no other
payment on account of the purchase price of said estate. The special committee created by the above resolution called and examined various witnesses, among
the most important of whom was the herein petitioner, Jean L. Arnault. An intriguing question which the
The original owner of the Tambobong Estate was the Philippine Trust Company. On May 14, 1946, the committee sought to resolve was that involved in the apparent unnecessariness and irregularity of the
Philippine Trust Company sold estate for the sum of P1,200,000 to Ernest H. Burt, who paid P10,000 Government's paying to Burt the total sum of P1,500,000 for his alleged interest of only P20,000 in the
down and promise to pay P90,000 within nine months and the balance of P1,100,000 in ten successive two estates, which he seemed to have forfeited anyway long before October, 1949. The committee sought
installments of P110,000 each. The nine-month period within which to pay the first installment of P90,000 to determine who were responsible for and who benefited from the transaction at the expense of the
expired on February 14, 1947, without Burt's having paid the said or any other amount then or afterwards. Government.
On September 4, 1947, the Philippine Trust Company sold, conveyed, and delivered the Tambobong
Estate to the Rural Progress Administration by an absolute deed of sale in consideration of the sum of Arnault testified that two checks payable to Burt aggregating P1,500,000 were delivered to him on the
P750,000. On February 5, 1948, the Rural Progress Administration made, under article 1504 of the Civil afternoon of October 29, 1949; that on the same date he opened a new account in the name of Ernest H.
Code, a notarial demand upon Burt for the resolution and cancellation of his contract of purchase with the Burt with the Philippine National Bank in which he deposited the two checks aggregating P1,500,000; and
Philippine Trust Company due to his failure to pay the installment of P90,000 within the period of nine that on the same occasion he draw on said account two checks; one for P500,000, which he transferred
months. Subsequently the Court of First Instance of Rizal ordered the cancellation of Burt's certificate of to the account of the Associated Agencies, Inc., with the Philippine National Bank, and another for
title and the issuance of a new one in the name of the Rural Progress Administration, from which order he P440,000 payable to cash, which he himself cashed. It was the desire of the committee to determine the
appealed to the Supreme Court.1 ultimate recipient of this sum of P440,000 that gave rise to the present case.

At first the petitioner claimed before the Committee:

1
Mr. ARNAULT (reading from a note). Mr. Chairman, for questions involving the disposition of funds, I take The CHAIRMAN. When was that instruction given to you by Burt?
the position that the transactions were legal, that no laws were being violated, and that all requisites had Mr. ARNAULT. Long time ago.
been complied with. Here also I acted in a purely functional capacity of representative. I beg to be The CHAIRMAN. In what year did Burt give you that verbal instruction; when Burt was still here in the
excused from making answer which might later be used against me. I have been assured that it is my Philippines?
constitutional right to refuse to incriminate myself, and I am certain that the Honorable Members of this Mr. ARNAULT. Yes.
Committee, who, I understand, are lawyers, will see the justness of my position. The CHAIRMAN. But at that time Burt already knew that he would receive the money?
Mr. ARNAULT. No.
At as subsequent session of the committee (March 16) Senator De Vera, a member of the committee, The CHAIRMAN. In what year was that when Burt while he was here in the Philippines gave you the
interrogated him as follows: verbal instruction?
Mr. ARNAULT. In 1946.
The CHAIRMAN. And what has that certain person done for Burt to merit receiving these P440,000?
Senator DE VERA. Now these transactions, according to your own typewritten statement, were legal? Mr. ARNAULT. I absolutely do not know.
Mr. ARNAULT. I believe so. The CHAIRMAN. You do not know?
Senator DE VERA. And the disposition of that fund involved, according to your own statement, did not Mr. ARNAULT. I do not know.
violate any law? The CHAIRMAN. Burt did not tell you when he gave you the verbal instruction why that certain person
Mr. ARNAULT. I believe so. should receive these P440,000?
xxx     xxx     xxx Mr. ARNAULT. He did not tell me.
Senator DE VERA. So that if the funds were disposed of in such a manner that no laws were violated, The CHAIRMAN. And Burt also authorized you to give this big amount to that certain person without
how is it that when you were asked by the Committee to tell what steps you took to have this money receipt?
delivered to Burt, you refused to answer the questions, saying that it would incriminate you? Mr. ARNAULT. He told me that a certain person would represent him and where could I meet him.
Mr. ARNAULT. Because it violates the rights of a citizen to privacy in his dealings with other people. The CHAIRMAN. Did Burt know already that certain person as early as 1946?
xxx     xxx     xxx Mr. ARNAULT. I presume much before that.
Senator DE VERA. Are you afraid to state how the money was disposed of because you would be The CHAIRMAN. Did that certain person have any intervention in the prosecution of the two cases
incriminated, or you would be incriminating somebody? involving the Buenavista and Tambobong estates?
Mr. ARNAULT. I am not afraid; I simply stand on the privilege to dispose of the money that has been paid Mr. ARNAULT. Not that I know of.
to me as a result of a legal transaction without having to account for any use of it. The CHAIRMAN. Is that certain person related to any high government official?
But when in the same session the chairman of the committee, Senator Sumulong, interrogated the Mr. ARNAULT. No, I do not know.
petitioner, the latter testified as follows: The CHAIRMAN. Why can you not tell us the name of that certain person?
The CHAIRMAN. The other check of P440,000 which you also made on October 29, 1949, is payable to Mr. ARNAULT. Because I am not sure of his name; I cannot remember the name.
cash; and upon cashing this P440,000 on October 29, 1949, what did you do with that amount? The CHAIRMAN. When gave that certain person that P440,000 on October 29, 1949, you knew already
Mr. ARNAULT. I turned it over to a certain person. that person?
The CHAIRMAN. The whole amount of P440,000? Mr. ARNAULT. Yes, I have seen him several times.
Mr. ARNAULT. Yes. The CHAIRMAN. And the name of that certain person is a Filipino name?
The CHAIRMAN. Who was that certain person to whom you delivered these P440,000 which you cashed Mr. ARNAULT. I would say Spanish name.
on October 29, 1949? The CHAIRMAN. And how aneroibout his Christian name; is it also a Spanish name?
Mr. ARNAULT. I don't remember the name; he was a representative of Burt. Mr. ARNAULT. I am not sure; I think the initial is J.
The CHAIRMAN. That representative of Burt to whom you delivered the P440,000 was a Filipino? The CHAIRMAN. Did he have a middle name?
Mr. ARNAULT. I don't know. Mr. ARNAULT. I never knew it.
The CHAIRMAN. You do not remember the name of that representative of Burt to whom you delivered The CHAIRMAN. And how about his family name which according to your recollection is Spanish; can you
this big amount of P440,000? remember the first letter with which that family name begins?
Mr. ARNAULT. I am not sure; I do not remember the name. Mr. ARNAULT. S, D or F.
The CHAIRMAN. That certain person who represented Burt to whom you delivered the big amount on The CHAIRMAN. And what was the last letter of the family name?
October 29, 1949, gave you a receipt for the amount? Mr. ARNAULT. I do not know.
Mr. ARNAULT. No. The CHAIRMAN. Have you seen that person again after you have delivered this P440,000?
The CHAIRMAN. Neither did you ask a receipt? Mr. ARNAULT. Yes.
Mr. ARNAULT. I didn't ask. The CHAIRMAN. Several times?
The CHAIRMAN. And why did you give that certain person, representative of Burt, this big amount of Mr. ARNAULT. Two or three times.
P440,000 which forms part of the P1-½ million paid to Burt? The CHAIRMAN. Here in Manila?
Mr. ARNAULT. Because I have instructions to that effect. Mr. ARNAULT. Yes.
The CHAIRMAN. Who gave you the instruction? The CHAIRMAN. And in spite of the fact that you met that person two or three times, you never were able
Mr. ARNAULT. Burt. to find out what was his name?
The CHAIRMAN. Where is the instruction; was that in writing? Mr. ARNAULT. If I knew, I would [have] taken it down. Mr. Peralta knows my name; of course, we have
Mr. ARNAULT. No. not done business. Lots of people in Manila know me, but they don't know my name, and I don't know
The CHAIRMAN. By cable? them. They sa{ I am "chiflado" because I don't know their names.
Mr. ARNAULT. No. The CHAIRMAN. That certain person is a male or female?
The CHAIRMAN. In what form did you receive that instruction? Mr. ARNAULT. He is a male.
Mr. ARNAULT. Verbal instruction. The CHAIRMAN. You are sure that he is a male at least?
The CHAIRMAN. When did you receive this verbal instruction from Burt to deliver these P440,000 to a Mr. ARNAULT. Let us say 38 or 40 years, more or less.
certain person whose name you do not like to reveal? The CHAIRMAN. Can you give us, more or less, a description of that certain person? What is his
Mr. ARNAULT. I have instruction to comply with the request of the person. complexion: light, dark or light brown?
The CHAIRMAN. Now, you said that instruction given to you by Burt was verbal?
Mr. ARNAULT. Yes.

2
Mr. ARNAULT. He is like the gentleman there (pointing to Senator Cabili), but smaller. He walks very Mr. ARNAULT. I do not remember .
straight, with military bearing.
The CHAIRMAN. Do you know the residence of that certain person to whom you gave the P440,000? Sen. SUMULONG. Now, if you do not remember the name of that person, how can you say that your
Mr. ARNAULT. No. answer might be incriminating? If you do not remember his name, you cannot answer the question; so
The CHAIRMAN. During these frequent times that you met that certain person, you never came to know how could your answer be self-incriminating? What do you say to that?
his residence?
Mr. ARNAULT. No, because he was coming to the office.
The CHAIRMAN. How tall is that certain person? Mr. ARNAULT. This is too complicated for me to explain. Please, I do not see how to answer those
Mr. ARNAULT. Between 5-2 and 5-6. questions. That is why I asked for a lawyer, so he can help me. I have no means of knowing what the
On May 15, 1950, the petitioner was haled before the bar of the Senate, which approved and read to him situation is about. I have been in jail 13 days without communication with the outside. How could I answer
the following resolution: the question? I have no knowledge of legal procedure or rule, of which I am completely ignorant.
Be it resolved by the Senate of the Philippines in Session assembled:
That Jean L. Arnault, now at the bar of the Senate, be arraigned for contempt consisting of contumacious xxx     xxx     xxx
acts committed by him during the investigation conducted by the Special Committee created by Senate
Resolution No. 8 to probe the Tambobong and Buenavista estates deal of October 21, 1949, and that the Sen. SUMULONG. Mr. President, I ask that the question be answered.
President of the Senate propounded to him the following interrogatories:
1. What excuse have you for persistently refusing to reveal the name of the person to whom you gave the
The PRESIDENT. The witness is ordered to answer the question. It is very clear. It does not incriminate
P440,000 on October 29, 1949, a person whose name it is impossible for you not to remember not only
the witness.
because of the big amount of money you gave to him without receipt, but also by your own statements
you knew him as early as 1946 when General Ernest H. Burt was still in the Philippines, you made two
other deliveries of money to him without receipt, and the last time you saw him was in December 1949? xxx     xxx     xxx
Thereupon petitioner's attorney, Mr. Orendain, submitted for him a written answer alleging that the
questions were incriminatory in nature and begging leave to be allowed to stand on his constitutional right Mr. ARNAULT. I do not remember. I stand on my constitutional rights. I beg to be excused from making
not to be compelled to be a witness against himself. Not satisfied with that written answer Senator further answer, please.
Sumulong, over the objection of counsel for the petitioner, propounded to the latter the following question:
Sen. SUMULONG. During the investigation, when the Committee asked you for the name of that person Sen. SUMULONG. In that mimeographed letter that you sent addressed to the President of the Senate,
to whom you gave the P440,000, you said that you can [could] not remember his name. That was the dated May 2, 1950, you stated there that you cannot reveal the name of the person to whom you gave the
reason then for refusing to reveal the name of the person. Now, in the answer that you have just cited, P440,000 because if he is a public official you might render yourself liable for prosecution for bribery, and
you are refusing to reveal the name of that person to whom you gave the P440,000 on the ground that that if he is a private individual you might render yourself liable for prosecution for slander. Why did you
your answer will be self-incriminating. Now, do I understand from you that you are abandoning your make those statements when you cannot even tell us whether that person to whom you gave the
former claim that you cannot remember the name of that person, and that your reason now for your P440,000 is a public official or a private individual ? We are giving you this chance to convince the Senate
refusal to reveal the name of that person is that your answer might be self-incriminating? In other words, that all these allegations of yours that your answers might incriminate you are given by you honestly or
the question is this: What is your real reason for refusing to reveal the name of that person to whom you you are just trying to make a pretext for not revealing the information desired by the Senate.
gave the P440,000: that you do not remember his name or that your answer would be self-incriminating?
xxx     xxx     xxx
Mr. ORENDAIN. Mr. President, we are begging for the rules of procedure that the accused should not be The PRESIDENT. You are ordered to answer the question.
required to testify unless he so desires.
The PRESIDENT. It is the duty of the respondent to answer the question. The question is very clear. It Mr. ARNAULT. I do not even understand the question. (The question is restated and explained.)
does not incriminate him.
xxx     xxx     xxx Mr. ARNAULT. That letter of May 2, was prepared by a lawyer for me and signed it. That is all I can say
how I stand about this letter. I have no knowledge myself enough to write such a letter, so I had to secure
Mr. ARNAULT. I stand by every statement that I have made before the Senate Committee on the first, the help of a lawyer to help me in my period of distress.
second, and third hearings to which I was made in my letter to this Senate of May 2, 1950, in which I gave
all the reasons that were in my powers to give, as requested. I cannot change anything in those In that same session of the Senate before which the petitioner was called to show cause why he should
statements that I made because they represent the best that I can do , to the best of my ability. not be adjudged guilty of contempt of the Senate, Senator Sumulong propounded to the petitioner
questions tending to elicit information from him as to the identity of the person to whom he delivered the
The PRESIDENT. You are not answering the question. The answer has nothing to do with the question. P440,000; but the petitioner refused to reveal it by saying that he did not remember. The President of the
Senate then propounded to him various questions concerning his past activities dating as far back as
Sen. SUMULONG. I would like to remind you , Mr. Arnault, that the reason that you gave during the when witness was seven years of age and ending as recently as the post liberation period, all of which
investigation for not revealing the name of the person to whom you gave the P440,000 is not the same questions the witness answered satisfactorily. In view thereof, the President of the Senate also made an
reason that you are now alleging because during the investigation you told us: "I do not remember his attempt to illicit the desired information from the witness, as follows:
name." But, now, you are now saying: "My answer might incriminate me." What is your real position?
The PRESIDENT. Now I am convinced that you have a good memory. Answer: Did you deliver the
Mr. ARNAULT. I have just stated that I stand by my statements that I made at the first, second, and third P440,000 as a gift, or of any consideration?
hearings. I said that I wanted to be excused from answering the question. I beg to be excused from
making any answer that might be incriminating in nature. However, in this answer, if the detail of not Mr. ARNAULT. I have said that I had instructions to deliver it to that person, that is all.
remembering the name of the person has not been included, it is an oversight.
The PRESIDENT. Was it the first time you saw that person?
Sen. SUMULONG. Mr. Arnault, will you kindly answer a simple question: Do you remember or not the
name of the person to whom you gave the P440,000? Mr. ARNAULT. I saw him various times, I have already said.
3
The PRESIDENT. In spite of that, you do not have the least remembrance of the name of that person? information which is volunteered is not always accurate or complete; so some means of compulsion is
essential to obtain what is needed. (McGrain vs. Daugherty, 273 U.S., 135; 71 L. ed., 580; 50 A.L R., 1.)
Mr. ARNAULT. I cannot remember. The fact that the Constitution expressly gives to Congress the power to punish its Members for disorderly
behavior, does not by necessary implication exclude the power to punish for contempt any other person.
(Anderson vs. Dunn, 6, Wheaton, 204; 5 L. ed., 242.) But no person can be punished for contumacy as a
The PRESIDENT. How is it that you do not remember events that happened a short time ago and, on the witness before either House, unless his testimony is required in a matter into which that House has
other hand, you remember events that occurred during your childhood? jurisdiction to inquire. (Kilbourn vs. Thompson, 26 L. ed., 377.).

Mr. ARNAULT. I cannot explain. Since, as we have noted, the Congress of the Philippines has a wider range of legislative field than either
the Congress of the United States or a State Legislature, we think it is correct to say that the field of
The Senate then deliberated and adopted the resolution of May 15 hereinabove quoted whereby the inquiry into which it may enter is also wider. It would be difficult to define any limits by which the subject
petitioner was committed to the custody of the Sergeant-at-Arms and imprisoned until "he shall have matter of its inquiry can be bounded. It is not necessary to do so in this case. Suffice it to say that it must
purged the contempt by revealing to the Senate or to the aforesaid Special Committee the name of the be coextensive with the range of the legislative power.
person to whom he gave the P440,000, as well as answer other pertinent questions in connection
therewith." In the present case the jurisdiction of the Senate, thru the Special Committee created by it, to investigate
the Buenavista and Tambobong Estates deal is not challenged by the petitioner; and we entertain no
The Senate also adopted on the same date another resolution (No. 16) , to wit: doubt as to the Senate's authority to do so and as to the validity of Resolution No. 8 hereinabove quoted.
The transaction involved a questionable and allegedly unnecessary and irregular expenditure of no less
That the Special Committee created by Senate Resolution No. 8 be empowered and directed to continue than P5,000,000 of public funds, of which Congress is the constitutional guardian. It also involved
its investigation of the Tambobong and Buenavista Estates deal of October 21, 1949, more particularly to government agencies created by Congress to regulate or even abolish. As a result of the yet uncompleted
continue the examination of Jean L. Arnault regarding the name of the person to whom he gave the investigation, the investigating committee has recommended and the Senate approved three bills (1)
P440,000 and other matters related therewith. prohibiting the Secretary of Justice or any other department head from discharging functions and
exercising powers other than those attached to his own office, without ]previous congressional
authorization; (2) prohibiting brothers and near relatives of any President of the Philippines from
The first session of the Second Congress was adjourned at midnight on May 18, 1950.
intervening directly or indirectly and in whatever capacity in transactions in which the Government is a
party, more particularly where the decision lies in the hands of executive or administrative officers who are
The case was argued twice before us. We have given its earnest and prolonged consideration because it appointees of the President; and (3) providing that purchases of the Rural Progress Administration of big
is the first of its kind to arise since the Constitution of the Republic of the Philippines was adopted. For the landed estates at a price of P100,000 or more, shall not become effective without previous congressional
first time this Court is called upon to define the power of either House of Congress to punish a person not confirmation.2
a member for contempt; and we are fully conscious that our pronouncements here will set an important
precedent for the future guidance of all concerned.
We shall now consider and pass upon each of the questions raised by the petitioner in support of his
contention that his commitment is unlawful.
Before discussing the specific issues raised by the parties, we deem it necessary to lay down the general
principles of law which form the background of those issues.
First He contends that the Senate has no power to punish him for contempt for refusing to reveal the
name of the person to whom he gave the P440,000, because such information is immaterial to, and will
Patterned after the American system, our Constitution vests the powers of the Government in three not serve, any intended or purported legislation and his refusal to answer the question has not
independent but coordinate Departments — Legislative, Executive, and Judicial. The legislative power is embarrassed, obstructed, or impeded the legislative process. It is argued that since the investigating
vested in the Congress, which consists of the Senate and the House of Representatives. (Section 1, committee has already rendered its report and has made all its recommendations as to what legislative
Article VI.) Each house may determine the rules of its proceedings, punish its Members for disorderly measures should be taken pursuant to its findings, there is no necessity to force the petitioner to give the
behavior, and, with the concurrence of two-thirds of all its Members, expel a Member. (Section 10, Article information desired other than that mentioned in its report, to wit: "In justice to Judge Quirino and to
VI.) The judicial power is vested in the Supreme Court and in such inferior courts as may be established Secretary Nepomuceno, this atmosphere of suspicion that now pervades the public mind must be
by law. (Section 1, Article VIII.) Like the Constitution of the United States, ours does not contain an dissipated, and it can only be done if appropriate steps are taken by the Senate to compel Arnault to stop
express provision empowering either of the two Houses of Congress to punish nonmembers for contempt. pretending that he cannot remember the name of the person to whom he gave the P440,000 and answer
It may also be noted that whereas in the United States the legislative power is shared by and between the the questions which will definitely establish the identity of that person . . ." Senator Sumulong, Chairman
Congress of the United States, on the one hand, and the respective legislatures of the different States, on of the Committee, who appeared and argued the case for the respondents, denied that that was the only
the other — the powers not delegated to the United States by the Constitution nor prohibited by it to purpose of the Senate in seeking the information from the witness. He said that the investigation had not
States being reserved to the States, respectively, or to the people — in the Philippines, the legislative been completed, because, due to the contumacy of the witness, his committee had not yet determined the
power is vested in the Congress of the Philippines alone. It may therefore be said that the Congress of the parties responsible for the anomalous transaction as required by Resolution No. 8; that, by Resolution No.
Philippines has a wider range of legislative field than the Congress of the United States or any State 16, his committee was empowered and directed to continue its investigation, more particularly to continue
Legislature. Our form of Government being patterned after the American system — the framers of our its examination of the witness regarding the name of the person to whom he gave the P440,000 and other
Constitution having drawn largely from American institutions and practices — we can, in this case, matters related therewith; that the bills recommended by his committee had not been approved by the
properly draw also from American precedents in interpreting analogous provisions of our Constitution, as House and might not be approved pending the completion of the investigation; and that those bills were
we have done in other cases in the past. Although there is no provision in the Constitution expressly not necessarily all the measures that Congress might deem it necessary to pass after the investigation is
investing either House of Congress with power to make investigations and exact testimony to the end that finished.
it may exercise its legislative functions as to be implied. In other words, the power of inquiry — with
process to enforce it — is an essential and appropriate auxiliary to the legislative function. A legislative
Once an inquiry is admitted or established to be within the jurisdiction of a legislative body to make, we
body cannot legislate wisely or effectively in the absence of information respecting the conditions which
think the investigating committee has the power to require a witness to answer any question pertinent to
the legislation is intended to effect or change; and where the legislative body does not itself possess the
that inquiry, subject of course to his constitutional right against self-incrimination. The inquiry, to be within
requisite information — which is not infrequently true — recourse must be had to others who do possess
the jurisdiction of the legislative body to make, must be material or necessary to the exercise of a power in
it. Experience has shown that mere requests for such information are often unavailing, and also that
it vested by the Constitution, such as to legislate, or to expel a Member; and every question which the

4
investigator is empowered to coerce a witness to answer must be material or pertinent to the subject of He refused to answer the questions and was prosecuted under an Act of Congress for contempt of the
the inquiry or investigation. So a witness may not be coerced to answer a question that obviously has no Senate. Upon being convicted and sent to jail he petitioned the Supreme Court of the United States for a
relation to the subject of the inquiry. But from this it does not follow that every question that may be writ of habeas corpus. One of the questions decided by the Supreme Court of the United States in that
propounded to a witness must be material to any proposed or possible legislation. In other words, the case was whether the committee had the right to compel the witness to answer said questions, and the
materiality of the question must be determined by its direct relation to any proposed or possible Court held that the committee did have such right, saying:
legislation. The reason is, that the necessity or lack of necessity for legislative action and the form and
character of the action itself are determined by the sum total of the information to be gathered as a result The questions were undoubtedly pertinent to the subject-matter of the inquiry. The resolution directed the
of the investigation, and not by a fraction of such information elicited from a single question. committee to inquire whether any senator has been, or is, speculating in what are known as sugar stocks
during the consideration of the tariff bill now before the Senate." What the Senate might or might not do
In this connection, it is suggested by counsel for the respondents that the power of the Court is limited to upon the facts when ascertained, we cannot say, nor are we called upon to inquire whether such ventures
determining whether the legislative body has jurisdiction to institute the inquiry or investigation; that once might be defensible, as contended in argument, but is plain that negative answers would have cleared
that jurisdiction is conceded, this Court cannot control the exercise of that jurisdiction; and it is insinuated, that body of what the Senate regarded as offensive imputations, while affirmative answers might have led
that the ruling of the Senate on the materiality of the question propounded to the witness is not subject to to further action on the part of the Senate within its constitutional powers. (Emphasis supplied.)
review by this Court under the principle of the separation of powers. We have to qualify this proposition.
As was said by the Court of Appeals of New York: "We are bound to presume that the action of the It may be contended that the determination of the parties responsible for the deal is incumbent upon the
legislative body was with a legitimate object if it is capable of being so construed, and we have no right to judicial rather than upon the legislative branch. But we think there is no basis in fact or in law for such
assume that the contrary was intended." (People ex rel. McDonald vs. Keeler, 99 N.Y., 463; 52 Am. Rep., assumption. The petitioner has not challenged the validity of Senate Resolution No. 8, and that resolution
49; 2 N.E., 615, quoted with approval by the Supreme Court of the United States in the said case of expressly requires the committee to determine the parties responsible for the deal. We are bound to
McGrain vs. Daugherty, it is necessary deduction from the decision in Re Chapman, 41 L. ed., 1154, that presume that the Senate has acted in the due performance of its constitutional function in instituting the
where the questions are not pertinent to the matter under inquiry a witness rightfully may refuse to inquiry, if the act is capable of being so construed. On the other hand, there is no suggestion that the
answer. So we are of the opinion that where the alleged immateriality of the information sought by the judiciary has instituted an inquiry to determine the parties responsible for the deal. Under the
legislative body from a witness is relied upon to contest its jurisdiction, the court is in duty bound to pass circumstances of the case, it appearing that the questioned transaction was affected by the head of the
upon the contention. The fact that the legislative body has jurisdiction or the power to make the inquiry Department of Justice himself, it is not reasonable to expect that the Fiscal or the Court of First Instance
would not preclude judicial intervention to correct a clear abuse of discretion in the exercise of that power. of Manila will take the initiative to investigate and prosecute the parties responsible for the deal until and
unless the Senate shall determined those parties are and shall taken such measures as may be within its
Applying the criterion laid down in the last two preceding paragraphs to the resolution of the issue under competence to take the redress the wrong that may have been committed against the people as a result
consideration, we find that the question for the refusal to answer which the petitioner was held in contempt of the transaction. As we have said, the transaction involved no less than P5,000,000 of public funds. That
by the Senate is pertinent to the matter under inquiry. In fact, this is not and cannot be disputed. Senate certainly is a matter of a public concern which it is the duty of the constitutional guardian of the treasury to
Resolution No. 8, the validity of which is not challenged by the petitioner, requires the Special Committee, investigate.
among other things, to determine the parties responsible for the Buenavista and Tambobong estates deal,
and it is obvious that the name of the person to whom the witness gave the P440,000 involved in said If the subject of investigation before the committee is within the range of legitimate legislative inquiry and
deal is pertinent to that determination — it is in fact the very thing sought to be determined. The the proposed testimony of the witness called relates to that subject, obedience, to its process may be
contention is not that the question is impertinent to the subject of the inquiry but that it has no relation or enforced by the committee by imprisonment. (Sullivan vs. Hill, 73 W. Va., 49; 79 S.E., 670; 40 Ann. Cas.
materiality to any proposed legislation. We have already indicated that it is not necessary for the [1916 B.], 1115.)
legislative body to show that every question propounded to a witness is material to any proposed or
possible legislation; what is required is that is that it be pertinent to the matter under inquiry.
The decision in the case of Kilbourn vs. Thompson, 26 L. ed., 377, relied upon by the petitioner, is not
applicable here. In that case the inquiry instituted by the House of Representatives of the United States
It is said that the Senate has already approved the three bills recommended by the Committee as a result related to a private real-estate pool or partnership in the District of Columbia. Jay Cook and Company had
of the uncompleted investigation and that there is no need for it to know the name of the person to whom had an interest in the pool but become bankrupts, and their estate was in course of administration in a
the witness gave the P440,000. But aside from the fact that those bills have not yet been approved by the federal bankruptcy court in Pennsylvania. The United States was one of their creditors. The trustee in the
lower house and by the President and that they may be withdrawn or modified if after the inquiry is bankruptcy proceeding had effected a settlement of the bankrupts' interest in the pool, and of course his
completed they should be found unnecessary or inadequate, there is nothing to prevent the Congress action was subject to examination and approval or disapproval by the bankruptcy court. Some of the
from approving other measures it may deem necessary after completing the investigation. We are not creditors, including the United States, were dissatisfied with the settlement. The resolution of the House
called upon, nor is it within our province, to determine or imagine what those measures may be. And our directed the Committee "to inquire into the nature and history of said real-estate pool and the character of
inability to do so is no reason for overruling the question propounded by the Senate to the witness. said settlement, with the amount of property involve, in which Jay Cooke and Co. were interested, and the
amount paid or to be paid in said settlement, with power to send for persons and papers, and report to this
The case of Re Chapman , 166 U.S., 661; 41 L. ed., 1154, is in point here. The inquiry there in question House." The Supreme Court of the United States, speaking thru Mr. Justice Miller, pointed out that the
was conducted under a resolution of the Senate and related to charges, published in the press, that resolution contained no suggestion of contemplated legislation; that the matter was one in respect of
senators were yielding to corrupt influences in considering a tariff bill then before the Senate and were which no valid legislation could be had; that the bankrupts' estate and the trustee's settlement were still
speculating in stocks the value of which would be affected by pending amendments to the bill. Chapman, pending in the bankruptcy court; and that the United States and other creditors were free to press their
a member of a firm of stock brokers dealing in the stock of the American Sugar Refining Company, claims in that proceeding. And on these grounds the court held that in undertaking the investigation "the
appeared before the committee in response to a subpoena and asked, among others, the following House of Representatives not only exceeded the limit of its own authority, but assumed a power which
questions: could only be properly exercised by another branch of the government, because the power was in its
nature clearly judicial." The principles announced and applied in that case are: that neither House of
Had the firm, during the month of March, 1894, bought or sold any stock or securities, known as sugar Congress possesses a "general power of making inquiry into the private affairs of the citizen"; that the
stocks, for or in the interest, directly or indirectly, of any United Senate senator? power actually possessed is limited to inquires relating to matters of which the particular House has
jurisdiction, and in respect of which it rightfully may take other action; that if the inquiry relates to a matter
wherein relief or redress could be had only by judicial proceeding, it is not within the range of this power ,
Was the said firm at that time carrying any sugar stock for the benefit of, or in the interest, directly or but must be left to the court, conformably to the constitutional separation of government powers.
indirectly, of any United Senate senator?

5
That case differs from the present case in two important respects: (1) There the court found that the Second. It is next contended for the petitioner that the Senate lacks authority to commit him for contempt
subject of the inquiry, which related to a private real-estate pool or partnership, was not within the for a term beyond its period of legislative session, which ended on May 18, 1950. This contention is based
jurisdiction of either House of Congress; while here if it is not disputed that the subject of the inquiry, on the opinion of Mr. Justice Malcolm, concurred in by Justices Street and Villa-Real, in the case of Lopez
which relates to a transaction involving a questionable expenditure by the Government of P5,000,000 of vs. De los Reyes (1930), 55 Phil., 170. In that case it appears that on October 23, 1929, Candido Lopez
public funds, is within the jurisdiction of the Senate, (2) There the claim of the Government as a creditor of assaulted a member of the House of Representatives while the latter was going to the hall of the House of
Jay Cooke and Company, which had had an interest in the pool, was pending adjudication by the court; Representatives to attend the session which was then about to begin, as a result of which assault said
while here the interposition of the judicial power on the subject of the inquiry cannot be expected, as we representative was unable to attend the sessions on that day and those of the two days next following by
have pointed out above, until after the Senate shall have determined who the parties responsible are and reason of the threats which Candido Lopez made against him. By the resolution of the House adopted
shall have taken such measures as may be within its competence to take to redress the wrong that may November 6, 1929, Lopez was declared guilty of contempt of the House of Representatives and ordered
have been committed against the people as a result of the transaction. punished by confinement in Bilibid Prison for a period of twenty-four hours. That resolution was not
complied with because the session of the House of Representatives adjourned at midnight on November
It is interesting to note that the decision in the case of Killbourn vs. Thompson has evoked strong 8, 1929, and was reiterated at the next session on September 16, 1930. Lopez was subsequently
criticisms from legal scholars. (See Potts, Power of Legislative Bodies to Punish for Contempt [1926], 74 arrested, whereupon he applied for the writ of habeas corpus in the Court of First Instance of Manila,
U. Pa. L. Rev., 692-699; James L. Land is, Constitutional Limitations on the Congressional Power of which denied the application. Upon appeal to the Supreme Court, six justices voted to grant the writ:
Investigation [1926], 40 Harvard L. Rev., 153, 154, 214-220.) We quoted the following from Professor Justice Malcolm, Street, and Villa-real, on the ground that the term of imprisonment meted out to the
Land is' criticism: "Mr. Justice Miller saw the case purely as an attempt by the House to secure to the petitioner could not legally be extended beyond the session of the body in which the contempt occurred;
Government certain priority rights as creditor of the bankrupt concern. To him it assumed the character of and Justices Johns, Villamor, and Ostrand, on the ground that the Philippine Legislature had no power to
a lawsuit between the Government and Jay Cooke and Co., with the Government, acting through the punish for contempt because it was a creature merely of an Act of the Congress of the United States and
House, attempting to override the orderliness of established procedure and thereby prefer a creditors' bill not of a Constitution adopted by the people. Chief Justice Avanceña, Justice Johnson, and Justice
not before the courts but before Congress. That bankruptcy proceedings had already been instituted Romualdez wrote separate opinions, concurring with Justice Malcolm, Street, and Villa-Real, that the
against Jay Cooke and Co., in a federal court gave added impetus to such a conception. The House was Legislature had inherent power to punish for contempt but dissenting from the opinion that the order of
seeking to oust a court of prior acquired jurisdiction by an extraordinary and unwarranted assumption of commitment could only be executed during the particular session in which the act of contempt was
"judicial power"! The broader aspect of the investigation had not been disclosed to the Court. That Jay committed.
Cooke and Co.'s indebtedness and the particular funds in question were only part of the great
administrative problem connected with the use and disposition of public monies, that the particular failure Thus, on the question under consideration, the Court was equally divided and no decisive pronouncement
was of consequence mainly in relation to the security demanded for all government deposits, that the was made. The opinion of Mr. Justice Malcolm is based mainly on the following passage in the case of
facts connected with one such default revealed the possibility of other and greater maladministration, such Anderson vs. Dunn, supra:
considerations had not been put before the Court. Nor had it been acquainted with the every-day nature of
the particular investigation and the powers there exerted by the House, powers whose exercise was And although the legislative power continues perpetual, the legislative body ceases to exist on the
customary and familiar in legislative practice. Instead of assuming the character of an extraordinary moment of its adjournment or periodical dissolution. It follows that imprisonment must terminate with that
judicial proceeding, the inquiry, place in its proper background, should have been regarded as a normal adjournment.
and customary part of the legislative process. Detailed definiteness of legislative purpose was thus made
the demand of the court in Killbourn vs. Thompson. But investigators cannot foretell the results that may
be achieved. The power of Congress to exercise control over a real-estate pool is not a matter for abstract as well as on the following quotation from Marshall vs. Gordon, supra:
speculation but one to be determined only after an exhaustive examination of the problem. Relationship,
and not their possibilities, determine the extent of congressional power. Constitutionality depends upon And the essential nature of the power also makes clear the cogency and application of the two limitations
such disclosures. Their presence, whether determinative of legislative or judicial power, cannot be which were expressly pointed out in Anderson vs. Dunn, supra, that is, that the power even when applied
relegated to guesswork. Neither Congress nor the Court can predict, prior to the event, the result of the to subjects which justified its exercise is limited to imprisonment and such imprisonment may not be
investigation." extended beyond the session of the body in which the contempt occurred.

The other case relied upon by the petitioner is Marshall vs. Gordon, 243 U.S., 521; 61. ed., 881. The Interpreting the above quotations, Chief Justice Avanceña held:
question there was whether the House of Representatives exceeded its power in punishing, as for
contempt of its authority, the District Attorney of the Southern District of New York, who had written, From this doctrine it follows, in my judgement, that the imposition of the penalty is limited to the existence
published, and sent to the chairman of one of its committees an ill-tempered and irritating letter respecting of the legislative body, which ceases to function upon its final periodical dissolution. The doctrine refers to
the action and purposes of the committee in interfering with the investigation by the grand jury of alleged its existence and not to any particular session thereof. This must be so, inasmuch as the basis of the
illegal activities of a member of the House of Representatives. Power to make inquires and obtain power to impose such penalty is the right which the Legislature has to self-preservation, and which right is
evidence by compulsory process was not involved. The court recognized distinctly that the House of enforceable during the existence of the legislative body. Many causes might be conceived to constitute
Representatives had implied power to punish a person not a member for contempt, but held that its action contempt to the Legislature, which would continue to be a menace to its preservation during the existence
in this instance was without constitutional justification. The decision was put on the ground that the letter, of the legislative body against which contempt was committed.
while offensive and vexatious, was not calculated or likely to affect the House in any of its proceedings or
in the exercise of any of its functions. This brief statement of the facts and the issues decided in that case
If the basis of the power of the legislature to punish for contempt exists while the legislative body
is sufficient to show the inapplicability thereof to the present case. There the contempt involved consisted
exercising it is in session, then that power and the exercise thereof must perforce continue until the final
in the district attorney's writing to the chairman of the committee an offensive and vexatious letter, while
adjournment and the election of its successor.
here the contempt involved consists in the refusal of the witness to answer questions pertinent to the
subject of an inquiry which the Senate has the power and jurisdiction to make . But in that case, it was
recognized that the House of Representatives has implied power to punish a person not a member of Mr. Justice Johnson's more elaborate opinion, supported by quotations from Cooley's Constitutional
contempt. In that respect the case is applicable here in favor of the Senate's (and not of the Petitioner's ) Limitations and from Jefferson's Manual, is to the same effect. Mr. Justice Romualdez said: "In my
contention. opinion, where as in the case before us, the members composing the legislative body against which the
contempt was committed have not yet completed their three-year term, the House may take action against
the petitioner herein."

6
We note that the quotations from Anderson vs. Dunn and Marshall vs. Gordon relied upon by Justice Commerce Commission, 219 U. S., 498, 514-516; 55 L. ed., 310, 315, 316; 31 Sup. Ct. Rep., 279, where
Malcolm are obiter dicta. Anderson vs. Dunn was an action of trespass against the Sergeant-at-Arms of it was held that a suit to enjoin the enforcement of an order of the Interstate Commerce Commission did
the House of Representatives of the United States for assault and battery and false imprisonment. The not become moot through the expiration of the order where it was capable of repetition by the
plaintiff had been arrested for contempt of the House, brought before the bar of the House, and Commission and was a matter of public interest. Our judgment may yet be carried into effect and the
reprimanded by the Speaker, and then discharged from custody. The question as to the duration of the investigation proceeded with from the point at which it apparently was interrupted by reason of the habeas
penalty was not involved in that case. The question there was "whether the House of Representatives can corpus proceedings. In these circumstances we think a judgment should be rendered as was done in the
take cognizance of contempt committed against themselves, under any circumstances." The court there case cited.
held that the House of Representatives had the power to punish for contempt, and affirmed the judgment
of the lower court in favor of the defendant. In Marshall vs. Gordon, the question presented was whether What has been said requires that the final order in the District Court discharging the witness from custody
the House had the power under the Constitution to deal with the conduct of the district attorney in writing a be reversed.
vexatious letter as a contempt of its authority, and to inflict punishment upon the writer for such contempt
as a matter of legislative power. The court held that the House had no such power because the writing of
the letter did not obstruct the performance of legislative duty and did not endanger the preservation of the Like the Senate of the United States , the Senate of the Philippines is a continuing body whose members
power of the House to carry out its legislative authority. Upon that ground alone, and not because the are elected for a term of six years and so divided that the seats of only one-third become vacant every two
House had adjourned, the court ordered the discharge of the petitioner from custody. years, two-thirds always continuing into the next Congress save as vacancies may occur thru death or
resignation. Members of the House of Representatives are all elected for a term of four years; so that the
term of every Congress is four years. The Second Congress of the Philippines was constituted on
The case where the question was squarely decided is McGrain vs. Daugherty, supra. There it appears December 30, 1949, and will expire on December 30, 1953. The resolution of the Senate committing the
that the Senate had adopted a resolution authorizing and directing a select committee of five senators to Petitioner was adopted during the first session of the Second Congress, which began on the fourth
investigate various charges of misfeasance and nonfeasance in the Department of Justice after Attorney Monday of January and ended in May 18, 1950.
General Harry M. Daugherty became its supervising head. In the course of the investigation the
committee caused to be served on Mally S. Daugherty, brother of Harry M. Daugherty and president of
the Midland National Bank of Washington Court House, Ohio, a subpoena commanding him to appear Had said resolution of commitment been adopted by the House of Representatives, we think it could be
before it for the purpose of giving testimony relating to the subject under consideration. The witness failed enforced until the final adjournment of the last session of the Second Congress in 1953. We find no sound
to appear without offering any excuse for his failure. The committee reported the matter to the Senate and reason to limit the power of the legislative body to punish for contempt to the end of every session and not
the latter adopted a resolution, "That the President of the Senate pro tempore issue his warrant to the end of the last session terminating the existence of that body. The very reason for the exercise of
commanding the Sergeant-at-Arms or his deputy to take into custody the body of the said M.S. Daugherty the power to punish for contempt is to enable the legislative body to perform its constitutional function
wherever found, and to bring the said M.S. Daugherty before the bar of the Senate, then and there to without impediment or obstruction. Legislative functions may be and in practice are performed during
answer such questions pertinent to the matter under inquiry as the Senate may order the President of the recess by duly constituted committees charged with the duty of performing investigations or conducting
Senate pro tempore to propound; and to keep the said M.S. Daugherty in custody to await the further hearing relative to any proposed legislation. To deny to such committees the power of inquiry with process
order of the Senate." Upon being arrested, the witness petitioned the federal court in Cincinnati for a writ to enforce it would be to defeat the very purpose for which that the power is recognized in the legislative
of habeas corpus. The federal court granted the writ and discharged the witness on the ground that the body as an essential and appropriate auxiliary to is legislative function. It is but logical to say that the
Senate, in directing the investigation and in ordering the arrest, exceeded its power under the power of self-preservation is coexistent with the life to be preserved.
Constitution. Upon appeal to the Supreme Court of the United States, one of the contentions of the
witness was that the case ha become moot because the investigation was ordered and the committee But the resolution of commitment here in question was adopted by the Senate, which is a continuing body
was appointed during the Sixty-eighth Congress, which expired on March 4, 1926. In overruling the and which does not cease exist upon the periodical dissolution of the Congress or of the House of
contention, the court said: Representatives. There is no limit as to time to the Senate's power to punish for contempt in cases where
that power may constitutionally be exerted as in the present case.
. . . The resolution ordering the investigation in terms limited the committee's authority to the period of the
Sixty-eighth Congress; but this apparently was changed by a later and amendatory resolution authorizing Mere reflection upon the situation at hand convinces us of the soundness of this proposition. The Senate
the committee to sit at such times and places as it might deem advisable or necessary. It is said in has ordered an investigation of the Buenavista and Tambobong estates deal, which we have found it is
Jefferson's Manual: "Neither House can continue any portion of itself in any parliamentary function beyond within its competence to make. That investigation has not been completed because of the refusal of the
the end of the session without the consent of the other two branches. When done, it is by a bill petitioner as a witness to answer certain questions pertinent to the subject of the inquiry. The Senate has
constituting them commissioners for the particular purpose." But the context shows that the reference is to empowered the committee to continue the investigation during the recess. By refusing to answer the
the two houses of Parliament when adjourned by prorogation or dissolution by the King. The rule may be questions, the witness has obstructed the performance by the Senate of its legislative function, and the
the same with the House of Representatives whose members are all elected for the period of a single Senate has the power to remove the obstruction by compelling the witness to answer the questions thru
Congress: but it cannot well be the same with the Senate, which is a continuing body whose members are restraint of his liberty until he shall have answered them. That power subsists as long as the Senate,
elected for a term of six years and so divided into classes that the seats of one third only become vacant which is a continuing body, persists in performing the particular legislative function involved. To hold that it
at the end of each Congress, two thirds always continuing into the next Congress, save as vacancies may may punish the witness for contempt only during the session in which investigation was begun, would be
occur through death or resignation. to recognize the right of the Senate to perform its function but at the same time to deny to it an essential
and appropriate means for its performance. Aside from this, if we should hold that the power to punish for
Mr. Hinds in his collection of precedents, says: "The Senate, as a continuing body, may continue its contempt terminates upon the adjournment of the session, the Senate would have to resume the
committees through the recess following the expiration of a Congress;" and, after quoting the above investigation at the next and succeeding sessions and repeat the contempt proceedings against the
statement from Jefferson's Manual, he says: "The Senate, however being a continuing body, gives witness until the investigation is completed-an absurd, unnecessary, and vexatious procedure, which
authority to its committees during the recess after the expiration of a Congress." So far as we are advised should be avoided.
the select committee having this investigation in charge has neither made a final report nor been
discharged; nor has been continued by an affirmative order. Apparently its activities have been As against the foregoing conclusion it is argued for the petitioner that the power may be abusively and
suspended pending the decision of this case. But, be this as it may, it is certain that the committee may be oppressively exerted by the Senate which might keep the witness in prison for life. But we must assume
continued or revived now by motion to that effect, and if, continued or revived, will have all its original that the Senate will not be disposed to exert the power beyond its proper bounds. And if, contrary to this
powers. This being so, and the Senate being a continuing body, the case cannot be said to have become assumption, proper limitations are disregarded, the portals of this Court are always open to those whose
moot in the ordinary sense. The situation is measurably like that in Southern P. Terminal Co. vs. Interstate rights might thus be transgressed.

7
Third. Lastly, the petitioner invokes the privilege against self-incrimination. He contends that he would against an imaginary danger, or to secure immunity to a third person. ( 3 Wharton's Criminal Evidence,
incriminate himself if he should reveal the name of the person to whom he gave the P440,000 if that 11th ed., secs. 1135,1136.)
person be a public official be (witness) might be accused of bribery, and if that person be a private
individual the latter might accuse him of oral defamation. It is the province of the trial judge to determine from all the facts and circumstances of the case whether
the witness is justified in refusing to answer. (People vs. Gonzo, 23 N.E. [2d], 210 [Ill. App., 1939].) A
The ground upon which the witness' claim is based is too shaky, in firm, and slippery to afford him safety. witness is not relieved from answering merely on his own declaration that an answer might incriminate
At first he told the Committee that the transactions were legal, that no laws were violated, and that all him, but rather it is for the trial judge to decide that question. (Mason vs. U.S., 244 U. S., 362; 61 L. ed.,
requisites had been replied with; but at the time he begged to be excused from making answers "which 1193, 1200.)
might later be used against me." A little later he explained that although the transactions were legal he
refused to answer questions concerning them "because it violates the right of a citizen to privacy in his As against witness's inconsistent and unjustified claim to a constitutional right, is his clear duty as a citizen
dealings with other people . . . I simply stand on my privilege to dispose of the money that has been paid to give frank, sincere, and truthful testimony before a competent authority. The state has the right to exact
to me as a result of a legal transaction without having to account for the use of it." But after being fulfillment of a citizen's obligation, consistent of course with his right under the Constitution. The witness in
apparently convinced by the Committee that his position was untenable, the witness testified that, without this case has been vociferous and militant in claiming constitutional rights and privileges but patently
securing any receipt, he turned over the P440,000 to a certain person, a representative of Burt, in recreant to his duties and obligations to the Government which protects those rights under the law. When
compliance with Burt's verbal instruction made in 1946; that as far as he know, that certain person had a specific right and a specific obligation conflict with each other, and one is doubtful or uncertain while the
nothing to do with the negotiations for the settlement of the Buenavista and Tambobong cases; that he other is clear and imperative, the former must give way to the latter. The right to life is one of the most
had seen that person several times before he gave him the P440,000 on October 29, 1949, and that since sacred that the citizen may claim, and yet the state may deprive him of it if he violates his corresponding
then he had seen him again two or three times, the last time being in December, 1949, in Manila; that the obligation to respect the life of others. As Mr. Justice Johnson said in Anderson vs. Dunn: "The wretch
person was a male, 39 to 40 years of age, between 5 feet, 2 inches and 5 feet, 6 inches in height. Butt the beneath the gallows may repine at the fate which awaits him, and yet it is not certain that the laws under
witness would not reveal the name of that person on these pretexts: " I don't remember the name; he was which he suffers were made for the security." Paraphrasing and applying that pronouncement here, the
a representative of Burt." "I am not sure; I don't remember the name." petitioner may not relish the restraint of his liberty pending the fulfillment by him of his duty, but it is no
less certain that the laws under which his liberty is restrained were made for his welfare.
We are satisfied that those answers of the witness to the important question, what is the name of that
person to whom you gave the P440,000? were obviously false. His insistent claim before the bar of the From all the foregoing, it follows that the petition must be denied, and it is so ordered, with costs.
Senate that if he should reveal the name he would incriminate himself, necessarily implied that he knew
the name. Moreover, it is unbelievable that he gave the P440,000 to a person to him unknown.
G.R. No. 89914 November 20, 1991
"Testimony which is obviously false or evasive is equivalent to a refusal to testify and is punishable as
contempt, assuming that a refusal to testify would be so punishable." (12 Am. Jur., sec. 15, Contempt, pp. JOSE F.S. BENGZON vs. THE SENATE BLUE RIBBON COMMITTEE AND ITS MEMBERS,
399-400.) In the case of Mason vs. U.S., 61 L. ed., 1198, it appears that Mason was called to testify
before a grand jury engaged in investigating a charge of gambling against six other men. After stating that This is a petition for prohibition with prayer for the issuance of a temporary restraining order and/or
he was sitting at a table with said men when they were arrested, he refused to answer two questions, injuective relief, to enjoin the respondent Senate Blue Ribbon committee from requiring the petitioners to
claiming so to do might tend to incriminate him: (1) "Was there a game of cards being played on this testify and produce evidence at its inquiry into the alleged sale of the equity of Benjamin "Kokoy"
particular evening at the table at which you are sitting?" (2) "Was there a game of cards being played at Romualdez to the Lopa Group in thirty-six (36) or thirty-nine (39) corporations.
another table at this time?" The foreman of the grand jury reported the matter to the judge, who ruled "that
each and all of said questions are proper and that the answers thereto would not tend to incriminate the On 30 July 1987, the Republic of the Philippines, represented by the Presidential Commission on Good
witness." Mason was again called and refused to answer the first question propounded to him, but, half Government (PCGG), assisted by the Solicitor General, filed with the Sandiganbayan Civil Case No. 0035
yielding to frustration, he said in response to the second question: "I don't know." In affirming the (PCGG Case No. 35) entitled "Republic of the Philippines vs. Benjamin "Kokoy" Romualdez, et al.", for
conviction for contempt, the Supreme Court of the United States among other things said: reconveyance, reversion, accounting, restitution and damages.

In the present case, the witness certainly were not relieved from answering merely because they declared The complaint was amended several times by impleading new defendants and/or amplifying the
that so to do might incriminate them. The wisdom of the rule in this regard is well illustrated by the allegations therein. Under the Second Amended Complaint, 1 the herein petitioners were impleaded as
enforced answer, "I don't know ," given by Mason to the second question, after he had refused to reply party defendants.
under a claim of constitutional privilege.
The complaint insofar as pertinent to herein petitioners, as defendants, alleges among others that:
Since according to the witness himself the transaction was legal, and that he gave the P440,000 to a
representative of Burt in compliance with the latter's verbal instruction, we find no basis upon which to
14. Defendants Benjamin (Kokoy) Romualdez and Juliette Gomez Romualdez, acting by themselves
sustain his claim that to reveal the name of that person might incriminate him. There is no conflict of
and/or in unlawful concert with Defendants Ferdinand E. Marcos and Imelda R. Marcos, and taking undue
authorities on the applicable rule, to wit:
advantage of their relationship, influence and connection with the latter Defendant spouses, engaged in
devices, schemes and strategems to unjuestly enrigh themselves at the expense of Plaintiff and the
Generally, the question whether testimony is privileged is for the determination of the Court. At least, it is Filipino people, among others:
not enough for the witness to say that the answer will incriminate him. as he is not the sole judge of his
liability. The danger of self-incrimination must appear reasonable and real to the court, from all the
(a) Obatained, with the active collaboration of Defendants Sene J. Gabaldon, Mario D. Camacho,
circumstances, and from the whole case, as well as from his general conception of the relations of the
Mamerto Nepomuceno, Carlos J. Valdez, Cesar C. Zalamea and Francisco Tantuico, Atty. Jose Bengzon,
witness. Upon the facts thus developed, it is the province of the court to determine whether a direct
Jr. and his law partners, namely: Edilberto S. Narciso, Jr., Jose Vicente E. Jimenez, Amando V. Faustino,
answer to a question may criminate or not. . . . The fact that the testimony of a witness may tend to show
Jr., and Leonardo C. Cruz; Jose S. Sandejas and his fellow senior managers of FMMC/PNI Holdings
that he has violated the law is not sufficient to entitle him to claim the protection of the constitutional
groups of companies such as Leonardo Gamboa, Vicente T. Mills, Jr., Jose M. Mantecon, Abelardo S.
provision against self-incrimination, unless he is at the same time liable to prosecution and punishment for
Termulo, Rex C. Drilon II and Kurt Bachmann, Jr., control of some of the biggest business enterprises in
such violation. The witness cannot assert his privilege by reason of some fanciful excuse, for protection
the Philippines, such as the Manila Corporation (MERALCO), Benguet Consolidated and the Philippine

8
Commercial International Bank (PCI Bank) by employing devious financial schemes and techniques including, among others, the 6,229,177 shares in PCIB registered in the names of Trans Middle East
calculated to require the massive infusion and hemorrhage of government funds with minimum or Phils. Equities, Inc. and Edilberto S. Narciso, Jr. which they refused to surrender to PCGG despite their
negligible "cashout" from Defendant Benjamin Romualdez... disclosure as they tried and continue to exert efforts in getting hold of the same as well as the shares in
Benguet registered in the names of Palm Avenue Holdings and Palm Avenue Realty Development Corp.
x x x           x x x          x x x purportedly to be applied as payment for the claim of P70 million of a "merger company of the First Manila
Managerment Corp. group" supposedly owned by them although the truth is that all the said firms are still
beneficially owned by defendants Benjamin Romualdez.
(m) manipulated, with the support, assistance and collaboration of Philgurantee officials led by chairman
Cesar E.A. Virata and the Senior managers of FMMC/PNI Holdings, Inc. led by Jose S. Sandejas, Jr.,
Jose M. Mantecom and Kurt S. Bachmann, Jr., among others, the formation of Erectors Holdings, Inc. x x x           x x x          x x x
without infusing additional capital solely for the purpose of Erectors Incorporated with Philguarantee in the
amount of P527,387,440.71 with insufficient securities/collaterals just to enable Erectors Inc, to appear On 28 September 1988, petitioner (as defendants) filed their respective answers. 2 Meanwhile, from 2 to
viable and to borrow more capitals, so much so that its obligation with Philgurantee has reached a total of 6 August 1988, conflicting reports on the disposition by the PCGG of the "Romualdez corporations" were
more than P2 Billion as of June 30, 1987. carried in various metropolitan newspapers. Thus, one newspaper reported that the Romuladez firms had
not been sequestered because of the opposition of certain PCGG officials who "had worked prviously as
(n) at the onset of the present Administration and/or within the week following the February 1986 People's lawyers of the Marcos crony firms." Another daily reported otherwise, while others declared that on 3
Revolution, in conspiracy with, supoort, assistance and collaboration of the abovenamed lawyers of the March 1986, or shortly after the EDSA February 1986 revolution, the Romualdez companies" were sold
Bengzon Law Offices, or specifically Defendants Jose F.S. Bengzon, Jr., V.E. Jimenez, Amando V. for P5 million, without PCGG approval, to a holding company controlled by Romualdez, and that Ricardo
Faustino, Jr., and Edilberto S. Narciso, Jr., manipulated, shcemed, and/or executed a series of devices Lopa, the President's brother-in-law, had effectively taken over the firms, even pending negotiations for
intended to conceal and place, and/or for the purpose of concealing and placing, beyond the inquiry and the purchase of the corporations, for the same price of P5 million which was reportedly way below the fair
jurisdiction of the Presidential Commission on Good Government (PCGG) herein Defendant's individual value of their assets. 3
and collective funds, properties, and assets subject of and/or suited int he instant Complaint.
On 13 September 1988, the Senate Minority Floor Leader, Hon. Juan Ponce Enrile delivered a speech
(o) manuevered, with the technical know-how and legalitic talents of the FMMC senior manager and some "on a matter of personal privilege" before the Senate on the alleged "take-over personal privilege" before
of the Bengzon law partners, such as Attys. Jose F.S. Bengzon, Jr., Edilberto S. Narciso, Jr., Amando V. the Senate on the alleged "take-over of SOLOIL Incorporated, the flaship of the First Manila Management
Faustino, Jose Vicente E. Jimenez and Leonardo C. Cruz, the purported sale of defendant Benjamin of Companies (FMMC) by Ricardo Lopa" and called upon "the Senate to look into the possible violation of
Romualdez's interests in the (i) Professional Managers, (ii) A & E International Corporation (A & E), (iii) the law in the case, particularly with regard to Republic Act No. 3019, the Anti-Graft and Corrupt Practices
First Manila Managerment Corporation (FMMC), (iv) Philippine World Travel Inc. (PWTI) and its Act." 4
subsidiaries consisting of 36 corporations in all, to PNI Holdings, Inc. (wjose purported incorporations are
all members of Atty. Jose F.S. Bengzon's law firm) for only P5 million on March 3, 1986 or three days after On motion of Senator Orlando Mercado, the matter was referred by the Senate to the Committee on
the creation of the Presidential Commission on Good Government on February 28, 1986, for the sole Accountability of Public Officers (Blue Ribbon Committee). 5 Thereafter, the Senate Blue Ribbon
purpose of deceiving and preempting the Government, particularly the PCGG, and making it appear that Committee started its investigation on the matter. Petitioners and Ricardo Lopa were subpoenaed by the
defendant Benjamin Romualdez had already divested himself of his ownership of the same when in truth Committee to appear before it and testify on "what they know" regarding the "sale of thirty-six (36)
and in fact, his interests are well intact and being protected by Atty. Jose F.S. Bengzon, Jr. and some of corporations belonging to Benjamin "Kokoy" Romualdez."
his law partners, together with the FMMC senior managers who still control and run the affiars of said
corporations, and in order to entice the PCGG to approve the said fictitious sale, the above-named At the hearing held on 23 May 1989, Ricardo Lopa declined to testify on the ground that his testimony
defendants offered P20 million as "donation" to the Government; may "unduly prejudice" the defendants in Civil Case No. 0035 before the Sandiganbayan. Petitioner Jose
F.S. Bengzon, Jr. likewise refused to testify involing his constitutional right to due process, and averring
(p) misused, with the connivance, support and technical assitance of the Bengzon law firm represented by that the publicity generated by respondents Committee's inquiry could adversely affect his rights as well
Atty. Jose F.S. Bengzon, Jr. as legal counsel, together with defendants Cesar Zalamea, Antonio Ozaeta, as those of the other petitioners who are his co-defendants in Civil Case No. 0035 before the
Mario D. Camacho amd Senen J. Gabaldon as members of the Board of Directors of the Philippine Sandiganbayan.
Commercial International bank (PCIB), the Meralco Pension Fund (Fund, for short) in the amount of P25
million by cuasing it to be invested in the PCIB and through the Bank's TSG, assigned to PCI The Senate Blue Ribbon Committee, thereupon, suspended its inquiry and directed the petitioners to file
Development and PCI Equity at 50% each, the Fund's (a) 8,028.011 common shares in the Bank and (b) their memorandum on the constitutional issues raised, after which, it issued a resolution 6 dated 5 June
"Deposit in Subscription" in the amount of P4,929.972.50 but of the agreed consideration of P28 million 1989 rejecting the petitioner's plea to be excused from testifying, and the Committee voted to pursue and
for the said assignment, PCI Development and PCI Equity were able to pay only P5,500.00 downpayment continue its investigation of the matter. Senator Neptali Gonzales dissented. 7
and the first amortization of P3,937,500.00 thus prompting the Fund to rescind its assignment, and the
consequent reversion of the assigned brought the total shareholding of the Fund to 11,470,555 voting
shares or 36.8% of the voting stock of the PCIB, and this development (which the defendants themselves Claiming that the Senate Blue Ribbon Committee is poised to subpoena them and required their
orchestrated or allowed to happen) was used by them as an excuse for the unlawful dismantling or attendance and testimony in proceedings before the Committee, in excess of its jurisdiction and legislative
cancellation of the Fund's 10 million shares for allegedly exceeding the 30-percent ceiling prescribed by purpose, in clear and blatant disregard of their constitutional rights, and to their grave and irreparable
Section 12-B of the General Banking Act, although they know for a fact that what the law declares as damager, prejudice and injury, and that there is no appeal nor any other plain, speedy and adequate
unlawful and void ab initio are the subscriptions in excess of the 30% ceiling "to the extent of the excess remedy in the ordinary course of law, the petitioners filed the present petition for prohibition with a prayer
over any of the ceilings prescribed ..." and not the whole or entire stockholding which they allowed to stay for temporary restraning order and/or injunctive relief.
for six years (from June 30, 1980 to March 24, 1986);

(q) cleverly hid behind the veil of corporate entity, through the use of the names and managerial expertise
of the FMMC senior manager and lawyers identified as Jose B. Sandejas, Leonardo Gamboa, Vicente T. Meanwhile, one of the defendants in Civil Case No. 0035 before the Sandiganbayan, Jose S. Sandejas,
Mills, Abelardo S, Termulo, Edilberto S. Narciso, Jr., Jose M. Mantecon, Rex C. Drilon II, Kurt Bachmann, filed with the Court of motion for intervention, 8 which the Court granted in the resolution 9 of 21
Jr. together with the legal talents of corporate lawyers, such as Attys. Jose F.S. Bengzon, Jr., Jose V.E. December 1989, and required the respondent Senate Blue Ribbon Committee to comment on the petition
Jimenez, Amando V. Faustino, Jr., and Leonardo C. Cruz, the ill-gotten wealth of Benjamin T. Romualdez
9
in intervention. In compliance, therewith, respondent Senate Blue Ribbon Committee filed its The Court is thus of the considered view that it has jurisdiction over the present controversy for the
comment 10 thereon. purpose of determining the scope and extent of the power of the Senate Blue Ribbon Committee to
conduct inquiries into private affirs in purported aid of legislation.
Before discussing the issues raised by petitioner and intervenor, we will first tackle the jurisdictional
question raised by the respondent Committee. Coming to the specific issues raised in this case, petitioners contend that (1) the Senate Blue Ribbon
Committee's inquiry has no valid legislative purpose, i.e., it is not done in aid of legislation; (2) the sale or
In its comment, respondent Committee claims that this court cannot properly inquire into the motives of disposition of hte Romualdez corporations is a "purely private transaction" which is beyond the power of
the lawmakers in conducting legislative investigations, much less cna it enjoin the Congress or any its the Senate Blue Ribbon Committee to inquire into; and (3) the inquiry violates their right to due process.
regular and special commitees — like what petitioners seek — from making inquiries in aid of legislation,
under the doctrine of separation of powers, which obtaines in our present system of government. The 1987 Constition expressly recognizes the power of both houses of Congress to conduct
inquiries in aid of legislation. 14 Thus, Section 21, Article VI thereof provides:
The contention is untenable. In Angara vs. Electoral Commission, 11 the Court held:
The Senate or the House of Representatives or any of its respective committee may conduct inquiries in
The separation of powers is a fundamental principle in our system of government. It obtains not hrough aid of legislation in accordance with its duly published rules of procedure. The rights of persons appearing
express provision but by actual division in our Constitution. Each department of the government has in or affected by such inquiries shall be respected. 15
exclusive cognizance of matters wihtin its jurisdiction, and is supreme within its own sphere. But it does
not follow from the fact that the three powers are to be kept separate and distinct that the Constitution
intended them to be absolutely unrestrained and independent of each other. The Constitution has
The power of both houses of Congress to conduct inquiries in aid of legislation is not, therefore, absolute
provided for an elaborate system of checks and balances to secure coordination in the workings of the
or unlimited. Its exercise is circumscribed by the afore-quoted provision of the Constitution. Thus, as
various departments of the government...
provided therein, the investigation must be "in aid of legislation in accordance with its duly published rules
of procedure" and that "the rights of persons appearing in or affected by such inquiries shall be
x x x           x x x          x x x respected." It follows then that the rights of persons under the Bill of Rights must be respected, including
the right to due process and the right not to be compelled to testify against one's self.
But in the main, the Constitution has blocked out with deft strokes and in bold lines, allotment of power to The power to conduct formal inquiries or investigations in specifically provided for in Sec. 1 of the Senate
the executive, the legislative and the judicial departments of the government. The ovelapping and Rules of Procedure Governing Inquiries in Aid of Legislation. Such inquiries may refer to the
interlacing of funcstions and duties between the several deaprtments, however, sometimes makes it hard implementation or re-examination of any law or in connection with any proposed legislation or the
to say just where the political excitement, the great landmarks of the Constitution are apt to be forgotten or formulation of future legislation. They may also extend to any and all matters vested by the Constitution in
marred, if not entirely obliterated, in cases of conflict, the judicial departments is the only constitutional Congress and/or in the Seante alone.
organ which can be called upon to determine the proper allocation of powers between the several
departments and among the integral or constituent units thereof. As held in Jean L. Arnault vs. Leon Nazareno, et al., 16 the inquiry, to be within the jurisdiction of the
legislative body making it, must be material or necessary to the exervise of a power in it vested by the
x x x           x x x          x x x Constitution, such as to legislate or to expel a member.

The Constitution is a definition of the powers of government. Who is to determine the nature, scope and Under Sec. 4 of the aforementioned Rules, the Senate may refer to any committee or committees any
extent of such powers? The Constitution itself has provided for the instrumentality of the judiciary as the speech or resolution filed by any Senator which in tis judgment requires an appropriate inquiry in aid of
rational way. And when the judiciary mediates to allocate constitutional boundaries; it does not assert any legislation. In order therefore to ascertain the character or nature of an inquiry, resort must be had to the
superiority over the other departments; it does not inr eality nullify or invalidate an act of the legislature, speech or resolution under which such an inquiry is proposed to be made.
but only asserts the solemn and sacred obligation assigned to it by tyhe Constitution to determine
conflicting claims of authority under the Constitution and to established for the parties in an actual A perusal of the speech of Senator Enrile reveals that he (Senator Enrile) made a statement which was
controversy the rights which that instrument secures and guarantess to them. This is in thruth all that is published in various newspapers on 2 September 1988 accusing Mr. Ricardo "Baby" Lopa of "having
involved in what is termed "judicial supremacy" which properly is the power of judicial review under the taken over the FMMC Group of Companies." As a consequence thereof, Mr. Lopa wrote a letter to
Constitution. Even the, this power of judicial review is limited to actual cases and controversies to be Senator Enrile on 4 September 1988 categorically denying that he had "taken over " the FMMC Group of
exercised after full opportunity of argument by the parties, and limited further to the constitutional question Companies; that former PCGG Chairman Ramon Diaz himself categorically stated in a telecast interview
raised or the very lis mota presented. Any attempt at abstraction could only lead to dialectics and barren by Mr. Luis Beltran on Channel 7 on 31 August 1988 that there has been no takeover by him (Lopa); and
legal questions and to sterile conclusions unrelated to actualities. Narrowed as its function is in this that theses repeated allegations of a "takeover" on his (Lopa's) part of FMMC are baseless as they are
manner, the judiciary does not pass upon questions of wisdom, justice or expediency of legislation. More malicious.
thatn that, courts accord the presumption of constitutionality to legislative enactments, not only because
the legislature is presumed to abide by the Constitution but also becuase the judiciary in the determination
of actual cases and controversies must reflect the wisdom and justice of the people as expressed through The Lopa reply prompted Senator Enrile, during the session of the Senate on 13 September 1988, to avail
their representatives in the executive and legislative departments of the government. of the privilege hour, 17 so that he could repond to the said Lopa letter, and also to vindicate his
reputation as a Member of the Senate of the Philippines, considering the claim of Mr. Lopa that his
(Enrile's) charges that he (Lopa) had taken over the FMMC Group of Companies are "baseless" and
The "allocation of constituional boundaries" is a task that this Court must perfomr under the Constitution. "malicious." Thus, in his speech, 18 Senator Enrile said, among others, as follows:
Moreowever, as held in a recent case, 12 "(t)he political question doctrine neither interposes an obstacle
to judicial determination of the rival claims. The jurisdiction to delimit constitutional boundaries has been
given to this Court. It cannot abdicate that obligation mandated by the 1987 Constitution, although said Mr. President, I rise this afternnon on a matter of personal privilege; the privilege being that I received, Mr.
provision by no means does away with kthe applicability of the principle in appropriate cases." 13 President, a letter dated September 4, 1988, signed by Mr. ricardo A. Lopa, a.k.a. or Baby Lopa, wherein
he denied categorically that he has taken over the First Manila Management Group of Companies which
includes SOLOIL Incorporated.

10
xxx xxxx xxx Mr. President, it may be worthwhile for the Senate to look into the possible violation of the law in the case
particularly with regard to Republic Act No. 3019, the Anti-Graft and Corrupt Practices Act, Section 5 of
In answer to Mr. Lopa, I will quote pertinent portions from an Official Memorandum to the Presidential which reads as follows and I quote:
Commission of Good Government written and signed by former Governor, now Congressman Jose
Ramirez, in his capacity as head of the PCGG Task Force for Region VIII. In his memorandum dated July Sec. 5. Prohibition on certain relatives. — It shall be unlawful for the spouse or for nay relative, by
3, 1986, then Governor Ramirez stated that when he and the members of his task force sought to serve a consanguinity or affinity, within the third civil degree, of the President of the Philippines, the Vice-
sequestration order on the management of SOLOIL in Tanauan, Leyte, management officials assured him President of the Philippines, the President of the Senate, or the Speaker of the House of Representatives,
that relatives of the President of the Philippines were personally discussing and representing SOLOIL so to intervene directly or indirectly, in any business, transaction, contract or application with the
that the order of sequestration would be lifted and that the new owner was Mr. Ricardo A. Lopa. Government: Provided, that this section shall not apply to any person who prior to the assumption of office
of any of the above officials to whom he is related, has been already dealing with the Government along
I will quote the pertinent portions in the Ramire's memorandum. the same line of business, nor to any transaction, contract or application filed by him for approval of which
is not discretionary on the part of the officials concerned but depends upon compliance with requisites
provided by law, nor to any act lawfully performed in an official capacity or in the exercise of a profession.
The first paragraph of the memorandum reads as follows and I quote, Mr. President:
Mr. President, I have done duty to this Senate and to myself. I leave it to this august Body to make its own
"Our sequestration work of SOLOIL in Tanauan, Leyte was not heeded by management because they conclusion.
said another representation was being made to this Commission for the ventual lifting of our
sequestrationorder. They even assured us that Mr. Ricardo Lopa and Peping Cojunangco were personally
discussing and representing SOLOIL, so the order of sequestration will finally be lifted. While we Verily, the speech of Senator Enrile contained no suggestion of contemplated legislation; he merely called
attempted to carry on our order, management refused to cooperate and vehemently turned down our upon the Senate to look into a possible violation of Sec. 5 of RA No. 3019, otherwise known as "The Anti-
request to make available to us the records of the company. In fact it was obviously clear that they will Graft and Corrupt Practices Act." I other words, the purpose of the inquiry to be conducted by respondent
meet us with forcethe moment we insist on doing normally our assigned task. In view of the impending Blue Ribbon commitee was to find out whether or not the relatives of President Aquino, particularly Mr.
threat, and to avoid any untoward incident we decided to temporarily suspend our work until there is a ricardo Lopa, had violated the law in connection with the alleged sale of the 36 or 39 corporations
more categorical stand of this Commission in view of the seemingly influential represetation being made belonging to Benjamin "Kokoy" Romualdez to the Lopaa Group. There appears to be, therefore, no
by SOLOIL for us not to continue our work." intended legislation involved.

Another pertinent portion of the same memorandum is paragraph five, which reads as follows, and I quote The Court is also not impressed with the respondent Committee's argument that the questioned inquiry is
Mr. President: to be conducted pursuant to Senate Resolution No. 212. The said resolution was introduced by Senator
Jose D. Lina in view of the representaions made by leaders of school youth, community groups and youth
of non-governmental organizations to the Senate Committee on Youth and Sports Development, to look
"The President, Mr. Gamboa, this is, I understand, the President of SOLOIL, and the Plant into the charges against the PCGG filed by three (3) stockholders of Oriental petroleum, i.e., that it has
Superintendent, Mr. Jimenez including their chief counsel, Atty. Mandong Mendiola are now saying that adopted a "get-rich-quick scheme" for its nominee-directors in a sequestered oil exploration firm.The
there have been divestment, and that the new owner is now Mr. Ricardo Lopa who according to them, is pertinent portion of Senate Resolution No. 212 reads as follows:
the brother-in-law of the President. They even went further by telling us that even Peping Cojuangco who
we know is the brother of her excellency is also interested in the ownership and management of SOLOIL.
When he demanded for supporting papers which will indicate aforesaid divestment, Messrs. Gamboa, x x x           x x x          x x x
Jimenez and Mendiola refused vehemently to submit these papers to us, instead they said it will be
submitted directly to this Commission. To our mind their continuous dropping of names is not good for this WHEREAS, recent developments have shown that no less than the Solicitor-General has stated that the
Commission and even to the President if our dersire is to achieve respectability and stability of the PCGG Chairman and at least three Commissioners should resign and that the agency should rid itself of
government." "ineptness, incompetence and corruption" and that the Sandiganbayan has reportedly ordered the PCGG
to answer charges filed by three stockholders of Oriental Petroleum that it has adopted a "get-rich-quick
The contents of the memorandum of then Governor and now Congressman Jose Ramirez were scheme" for its nominee-directors in a sequestered oil exploration firm;
personally confirmed by him in a news interview last September 7, 1988.
WHEREAS, leaders of school youth, community groups and youth of non-governmental organization had
xxx xxxx xxx made representations to the Senate Committee on Youth and Sports Development to look into the
charges against the PCGG since said agency is a symbol of the changes expected by the people when
the EDSA revolution took place and that the ill-gotten wealth to be recovered will fund priority projects
Also relevant to this case, Mr. President, is a letter of Mr. Ricardo Lopa himself in August 11, 1988 issue which will benefit our people such as CARP, free education in the elementary and secondary levels
of the newspaper Malaya headlined "On Alleged Takeover of Romualdez Firms." reforestration, and employment generation for rural and urban workers;

Mr. Lopa states in the last paragraph of the published letter and I quote him: WHEREAS, the government and the present leadeship must demonstrate in their public and private lives
integrity, honor and efficient management of government services lest our youth become disillusioned and
12. As of this writing, the sales agreement is under review by the PCGG solely to determine the lose hope and return to an Idelogy and form of government which is repugnant to true freedom,
appropriate price. The sale of these companies and our prior rigtht to requires them have never been at democratic participation and human rights: Now, therefore, be it.
issue.
Resolved by the Senate, That the activities of the Presidential Commission on Good Government be
Perhaps I could not make it any clearer to Mr. Lopa that I was not really making baseless and malicious investigated by the appropriate Committee in connection with the implementation of Section 26, Article
statements. XVIII of the Constitution. 19

Senator Enrile concluded his privilege speech in the following tenor:

11
Thus, the inquiry under Senate Resolution No. 212 is to look into the charges against the PCGG In another case —
filed by the three (3) stockholders of Oriental Petroleum in connection with the implementation of
Section 26, Article XVIII of the Constitution.

It cannot, therefore, be said that the contemplated inquiry on the subject of the privilege speech of
Senator Juan Ponce Enrile, i.e., the alleged sale of the 36 (or 39) corporations belonging to Benjamin ... the mere semblance of legislative purpose would not justify an inquiry in the face of the Bill of Rights.
"Kokoy" Romualdez to the Lopa Group is to be conducted pursuant to Senate Resolution No. 212 The critical element is the exeistence of, and the weight to be ascribed to, the interest of the Congress in
because, firstly, Senator Enrile did not indict the PCGG, and, secondly, neither Mr. Ricardo Lopa nor the demanding disclosures from an unwilling witness. We cannot simply assume, however, that every
herein petitioners are connected with the government but are private citizens. congressional investigation is justified by a public need that over-balances any private rights affected. To
do so would be to abdicate the responsibility placed by the Constitution upon the judiciary to insure that
the Congress does not unjustifiably encroah upon an individual's right to privacy nor abridge his liberty of
It appeals, therefore, that the contemplated inquiry by respondent Committee is not really "in aid of
speech, press, religion or assembly. 23
legislation" becuase it is not related to a purpose within the jurisdiction of Congress, since the aim of
the investigation is to find out whether or not the ralatives of the President or Mr. Ricardo Lopa had
violated Section 5 RA No. 3019, the "Anti-Graft and Corrupt Practices Act", a matter that appears One of the basic rights guaranteed by the Constitution to an individual is the right against self-
more within the province of the courts rather than of the legislature. Besides, the Court may take incrimination. 24 Thir right constured as the right to remain completely silent may be availed of by the
judicial notice that Mr. Ricardo Lopa died during the pendency of this case. In John T. Watkins vs. accused in a criminal case; but kit may be invoked by other witnesses only as questions are asked of
United States, 20 it was held held: them.

... The power of congress to conduct investigations in inherent in the legislative process. That power is This distinction was enunciated by the Court in Romeo Chavez vs. The Honorable Court of Appeals, et
broad. it encompasses inquiries concerning the administration of existing laws as well as proposed, or al. 25 thus —
possibly needed statutes. It includes surveys of defects in our social,economic, or political system for the
purpose of enabling Congress to remedy them. It comprehends probes into departments of the Federal Petitioner, as accused, occupies a different tier of protection from an ordinary witness. Whereas an
Government to expose corruption, inefficiency or waste. But broad asis this power of inquiry, it is not ordinary witness may be compelled to take the witness stand and claim the privilege as each question
unlimited. There is no general authority to expose the private affairs ofindividuals without justification in requiring an incriminating answer is hot at him, an accused may altother refuse to take the witness stand
terms of the functions of congress. This was freely conceded by Solicitor General in his argument in this and refuse to answer any all questions.
case. Nor is the Congress a law enforcement or trial agency. These are functions of the executive and
judicial departments of government. No inquiry is an end in itself; it must be related to and in furtherance Moreover, this right of the accused is extended to respondents in administrative investigations but only if
of a legitimate task of Congress. Investigations conducted soly for the personal aggrandizement of the they partake of the nature of a criminal proceeding or analogous to a criminal proceeding. In Galman vs.
investigators or to "punish" those investigated are indefensible. (emphasis supplied) Pamaran, 26 the Court reiterated the doctrine in Cabal vs. Kapuanan (6 SCRA 1059) to illustrate the right
of witnesses to invoke the right against self-incrimination not only in criminal proceedings but also in all
It can not be overlooked that when respondent Committee decide to conduct its investigation of the other types of suit
petitioners, the complaint in Civil No. 0035 had already been filed with the Sandiganbayan. A perusal of
that complaint shows that one of its principal causes of action against herein petitioners, as defendants It was held that:
therein, is the alleged sale of the 36 (or 39) corporations belonging to Benjamin "Kokoy" Romualdez.
Since the issues in said complaint had long been joined by the filing of petitioner's respective answers
thereto, the issue sought to be investigated by the respondent Commitee is one over which jurisdiction We did not therein state that since he is not an accused and the case is not a criminal case, Cabal cannot
had been acquired by the Sandiganbayan. In short, the issue had been pre-empted by that court. To allow refuse to take the witness stand and testify, and that he can invoke his right against self-incrimination only
the respondent Committee to conduct its own investigation of an issue already before the Sandiganbayan when a question which tends to elicit an answer that will incriminate him is propounded to him. Clearly
would not only pose the possibility of conflicting judgments betweena legislative commitee and a judicial then, it is not the characeter of the suit involved but the nature of the proceedings that controls. The
tribunal, but if the Committee's judgment were to be reached before that of the Sandiganbayan, the privilege has consistenly been held to extend to all proceedings sanctioned by law and to all cases in
possibility of its influence being made to bear on the ultimate judgment of the Sandiganbayan can not be which punishment is sought to be visited upon a witness, whether a party or not.
discounted.
We do not here modify these doctrines. If we presently rule that petitioners may not be compelled by the
In fine, for the rspondent Committee to probe and inquire into the same justiciable controversy already respondent Committee to appear, testify and produce evidenc before it, it is only becuase we hold that the
before the Sandiganbayan, would be an encroachment into the exclusive domain of judicial jurisdiction questioned inquiry is not in aid of legislation and, if pursued, would be violative of the principle of
that had much earlier set in. In Baremblatt vs. United States, 21 it was held that: separation of powers between the legislative and the judicial departments of government, ordained by the
Constitution.
Broad as it is, the power is not, howevern, without limitations. Since congress may only investigate into
those areas in which it may potentially legislate or appropriate, it cannot inquire into matters which are WHEREFORE, the petition is GRANTED. The Court holds that, under the facts, including the circumtance
within the exclusive province of one of the other branches of the government. Lacking the judicial power that petitioners are presently impleaded as defendants in a case before the Sandiganbayan, which
given to the Judiciary, it cannot inquire into mattes that are exclusively the concern of the Judiciary. involves issues intimately related to the subject of contemplated inquiry before the respondet Committee,
Neither can it suplant the Executive in what exclusively belongs to the Executive. ... the respondent Senate Blue Ribbon Committee is hereby enjoined from compelling the petitioners and
intervenor to testify before it and produce evidence at the said inquiry.
Now to another matter. It has been held that "a congressional committee's right to inquire is 'subject to all
relevant limitations placed by the Constitution on governmental action,' including "'the relevant limitations SO ORDERED.
of the Bill of Rights'." 22
G.R. No. 180643             March 25, 2008

12
ROMULO L. NERI, petitioner, INFRASTRUCTURE PROJECTS, GOODS, AND CONSULTING SERVICES TO BE INCLUDED IN THE
vs. SCOPE AND APPLICATION OF PHILIPPINE PROCUREMENT LAWS, AMENDING FOR THE
SENATE COMMITTEE ON ACCOUNTABILITY OF PUBLIC OFFICERS AND INVESTIGATIONS, PURPOSE REPUBLIC ACT NO. 9184, OTHERWISE KNOWN AS THE GOVERNMENT
SENATE COMMITTEE ON TRADE AND COMMERCE, AND SENATE COMMITTEE ON NATIONAL PROCUREMENT REFORM ACT, AND FOR OTHER PURPOSES;
DEFENSE AND SECURITY, respondents.
2. Senate Bill No. 1794, introduced by Senator Mar Roxas, entitled AN ACT IMPOSING SAFEGUARDS
DECISION IN CONTRACTING LOANS CLASSIFIED AS OFFICIAL DEVELOPMENT ASSISTANCE, AMENDING
FOR THE PURPOSE REPUBLIC ACT NO. 8182, AS AMENDED BY REPUBLIC ACT NO. 8555,
LEONARDO-DE CASTRO, J.: OTHERWISE KNOWN AS THE OFFICIAL DEVELOPMENT ASSISTANCE ACT OF 1996, AND FOR
OTHER PURPOSES; and
At bar is a petition for certiorari under Rule 65 of the Rules of Court assailing the show
cause Letter1 dated November 22, 2007 and contempt Order2 dated January 30, 2008 concurrently 3. Senate Bill No. 1317, introduced by Senator Miriam Defensor Santiago, entitled AN ACT MANDATING
issued by respondent CONCURRENCE TO INTERNATIONAL AGREEMENTS AND EXECUTIVE AGREEMENTS.

Senate Committees on Accountability of Public Officers and Investigations,3 Trade and Commerce,4 and Respondent Committees initiated the investigation by sending invitations to certain personalities and
National Defense and Security5 against petitioner Romulo L. Neri, former Director General of the National cabinet officials involved in the NBN Project. Petitioner was among those invited. He was summoned to
Economic and Development Authority (NEDA). appear and testify on September 18, 20, and 26 and October 25, 2007. However, he attended only the
September 26 hearing, claiming he was "out of town" during the other dates.
The facts, as culled from the pleadings, are as follows:
In the September 18, 2007 hearing, businessman Jose de Venecia III testified that several high executive
officials and power brokers were using their influence to push the approval of the NBN Project by the
On April 21, 2007, the Department of Transportation and Communication (DOTC) entered into a contract NEDA. It appeared that the Project was initially approved as a Build-Operate-Transfer (BOT) project but,
with Zhong Xing Telecommunications Equipment (ZTE) for the supply of equipment and services for the on March 29, 2007, the NEDA acquiesced to convert it into a government-to-government project, to be
National Broadband Network (NBN) Project in the amount of U.S. $ 329,481,290 (approximately P16 financed through a loan from the Chinese Government.
Billion Pesos). The Project was to be financed by the People's Republic of China.
On September 26, 2007, petitioner testified before respondent Committees for eleven (11) hours. He
In connection with this NBN Project, various Resolutions were introduced in the Senate, as follows: disclosed that then Commission on Elections (COMELEC) Chairman Benjamin Abalos offered him P200
Million in exchange for his approval of the NBN Project. He further narrated that he informed President
(1) P.S. Res. No. 127, introduced by Senator Aquilino Q. Pimentel, Jr., entitled RESOLUTION Arroyo about the bribery attempt and that she instructed him not to accept the bribe. However, when
DIRECTING THE BLUE RIBBON COMMITTEE AND THE COMMITTEE ON TRADE AND INDUSTRY TO probed further on what they discussed about the NBN Project, petitioner refused to answer, invoking
INVESTIGATE, IN AID OF LEGISLATION, THE CIRCUMSTANCES LEADING TO THE APPROVAL OF "executive privilege". In particular, he refused to answer the questions on (a) whether or not President
THE BROADBAND CONTRACT WITH ZTE AND THE ROLE PLAYED BY THE OFFICIALS Arroyo followed up the NBN Project,6 (b) whether or not she directed him to prioritize it,7 and (c) whether
CONCERNED IN GETTING IT CONSUMMATED AND TO MAKE RECOMMENDATIONS TO HALE TO or not she directed him to approve.8
THE COURTS OF LAW THE PERSONS RESPONSIBLE FOR ANY ANOMALY IN CONNECTION
THEREWITH AND TO PLUG THE LOOPHOLES, IF ANY IN THE BOT LAW AND OTHER PERTINENT Unrelenting, respondent Committees issued a Subpoena Ad Testificandum to petitioner, requiring him to
LEGISLATIONS. appear and testify on November 20, 2007.

(2) P.S. Res. No. 144, introduced by Senator Mar Roxas, entitled Á RESOLUTION URGING PRESIDENT However, in the Letter dated November 15, 2007, Executive Secretary Eduardo R. Ermita requested
GLORIA MACAPAGAL ARROYO TO DIRECT THE CANCELLATION OF THE ZTE CONTRACT respondent Committees to dispense with petitioner's testimony on the ground of executive privilege. The
pertinent portion of the letter reads:
(3) P.S. Res. No. 129, introduced by Senator Panfilo M. Lacson, entitled RESOLUTION DIRECTING THE
COMMITTEE ON NATIONAL DEFENSE AND SECURITY TO CONDUCT AN INQUIRY IN AID OF With reference to the subpoena ad testificandum issued to Secretary Romulo Neri to appear and testify
LEGISLATION INTO THE NATIONAL SECURITY IMPLICATIONS OF AWARDING THE NATIONAL again on 20 November 2007 before the Joint Committees you chair, it will be recalled that Sec. Neri had
BROADBAND NETWORK CONTRACT TO THE CHINESE FIRM ZHONG XING already testified and exhaustively discussed the ZTE / NBN project, including his conversation with the
TELECOMMUNICATIONS EQUIPMENT COMPANY LIMITED (ZTE CORPORATION) WITH THE END IN President thereon last 26 September 2007.
VIEW OF PROVIDING REMEDIAL LEGISLATION THAT WILL PROTECT OUR NATIONAL
SOVEREIGNTY, SECURITY AND TERRITORIAL INTEGRITY.
Asked to elaborate further on his conversation with the President, Sec. Neri asked for time to consult with
his superiors in line with the ruling of the Supreme Court in Senate v. Ermita, 488 SCRA 1 (2006).
(4) P.S. Res. No. 136, introduced by Senator Miriam Defensor Santiago, entitled RESOLUTION
DIRECTING THE PROPER SENATE COMMITTEE TO CONDUCT AN INQUIRY, IN AID OF
LEGISLATION, ON THE LEGAL AND ECONOMIC JUSTIFICATION OF THE NATIONAL BROADBAND Specifically, Sec. Neri sought guidance on the possible invocation of executive privilege on the following
NETWORK (NBN) PROJECT OF THE NATIONAL GOVERNMENT. questions, to wit:

At the same time, the investigation was claimed to be relevant to the consideration of three (3) pending a) Whether the President followed up the (NBN) project?
bills in the Senate, to wit:
b) Were you dictated to prioritize the ZTE?
1. Senate Bill No. 1793, introduced by Senator Mar Roxas, entitled AN ACT SUBJECTING TREATIES,
INTERNATIONAL OR EXECUTIVE AGREEMENTS INVOLVING FUNDING IN THE PROCUREMENT OF

13
c) Whether the President said to go ahead and approve the project after being told about the matters relating to the impact of the bribery scandal involving high government officials and the possible
alleged bribe? loss of confidence of foreign investors and lenders in the Philippines. The letter ended with a reiteration of
petitioner's request that he "be furnished in advance" as to what else he needs to clarify so that he may
Following the ruling in Senate v. Ermita, the foregoing questions fall under conversations and adequately prepare for the hearing.
correspondence between the President and public officials which are considered executive privilege
(Almonte v. Vasquez, G.R. 95637, 23 May 1995; Chavez v. PEA, G.R. 133250, July 9, 2002). Maintaining In the interim, on December 7, 2007, petitioner filed with this Court the present petition
the confidentiality of conversations of the President is necessary in the exercise of her executive and for certiorari assailing the show cause Letter dated November 22, 2007.
policy decision making process. The expectation of a President to the confidentiality of her conversations
and correspondences, like the value which we accord deference for the privacy of all citizens, is the Respondent Committees found petitioner's explanations unsatisfactory. Without responding to his request
necessity for protection of the public interest in candid, objective, and even blunt or harsh opinions in for advance notice of the matters that he should still clarify, they issued the Order dated January 30,
Presidential decision-making. Disclosure of conversations of the President will have a chilling effect on the 2008, citing him in contempt of respondent Committees and ordering his arrest and detention at the Office
President, and will hamper her in the effective discharge of her duties and responsibilities, if she is not of the Senate Sergeant-At-Arms until such time that he would appear and give his testimony. The said
protected by the confidentiality of her conversations. Order states:

The context in which executive privilege is being invoked is that the information sought to be disclosed ORDER
might impair our diplomatic as well as economic relations with the People's Republic of China. Given the
confidential nature in which these information were conveyed to the President, he cannot provide the
Committee any further details of these conversations, without disclosing the very thing the privilege is For failure to appear and testify in the Committee's hearing on Tuesday, September 18, 2007; Thursday,
designed to protect. September 20, 2007; Thursday, October 25, 2007; and Tuesday, November 20, 2007, despite personal
notice and Subpoenas Ad Testificandum sent to and received by him, which thereby delays, impedes and
obstructs, as it has in fact delayed, impeded and obstructed the inquiry into the subject reported
In light of the above considerations, this Office is constrained to invoke the settled doctrine of executive irregularities, AND for failure to explain satisfactorily why he should not be cited for contempt (Neri letter of
privilege as refined in Senate v. Ermita, and has advised Secretary Neri accordingly. 29 November 2007), herein attached) ROMULO L. NERI is hereby cited in contempt of this (sic)
Committees and ordered arrested and detained in the Office of the Senate Sergeant-At-Arms until
Considering that Sec. Neri has been lengthily interrogated on the subject in an unprecedented 11-hour such time that he will appear and give his testimony.
hearing, wherein he has answered all questions propounded to him except the foregoing questions
involving executive privilege, we therefore request that his testimony on 20 November 2007 on the ZTE / The Sergeant-At-Arms is hereby directed to carry out and implement this Order and make a return hereof
NBN project be dispensed with. within twenty four (24) hours from its enforcement.

On November 20, 2007, petitioner did not appear before respondent Committees. Thus, on November 22, SO ORDERED.
2007, the latter issued the show cause Letter requiring him to explain why he should not be cited in
contempt. The Letter reads:
On the same date, petitioner moved for the reconsideration of the above Order.9 He insisted that he has
not shown "any contemptible conduct worthy of contempt and arrest." He emphasized his willingness to
Since you have failed to appear in the said hearing, the Committees on Accountability of Public Officers testify on new matters, however, respondent Committees did not respond to his request for advance
and Investigations (Blue Ribbon), Trade and Commerce and National Defense and Security require you to notice of questions. He also mentioned the petition for certiorari he filed on December 7, 2007. According
show cause why you should not be cited in contempt under Section 6, Article 6 of the Rules of the to him, this should restrain respondent Committees from enforcing the show cause Letter "through the
Committee on Accountability of Public Officers and Investigations (Blue Ribbon). issuance of declaration of contempt" and arrest.

The Senate expects your explanation on or before 2 December 2007. In view of respondent Committees' issuance of the contempt Order, petitioner filed on February 1, 2008
a Supplemental Petition for Certiorari (With Urgent Application for TRO/Preliminary Injunction), seeking to
On November 29, 2007, petitioner replied to respondent Committees, manifesting that it was not his restrain the implementation of the said contempt Order.
intention to ignore the Senate hearing and that he thought the only remaining questions were those he
claimed to be covered by executive privilege, thus: On February 5, 2008, the Court issued a Status Quo Ante Order (a) enjoining respondent Committees
from implementing their contempt Order, (b) requiring the parties to observe the status quo prevailing
It was not my intention to snub the last Senate hearing. In fact, I have cooperated with the task of the prior to the issuance of the assailed order, and (c) requiring respondent Committees to file their comment.
Senate in its inquiry in aid of legislation as shown by my almost 11 hours stay during the hearing on 26
September 2007. During said hearing, I answered all the questions that were asked of me, save for those Petitioner contends that respondent Committees' show cause Letter and contempt Order were issued
which I thought was covered by executive privilege, and which was confirmed by the Executive Secretary with grave abuse of discretion amounting to lack or excess of jurisdiction. He stresses that his
in his Letter 15 November 2007. In good faith, after that exhaustive testimony, I thought that what conversations with President Arroyo are "candid discussions meant to explore options in making
remained were only the three questions, where the Executive Secretary claimed executive privilege. policy decisions." According to him, these discussions "dwelt on the impact of the bribery scandal
Hence, his request that my presence be dispensed with. involving high government officials on the country's diplomatic relations and economic and
military affairs and the possible loss of confidence of foreign investors and lenders in the
Be that as it may, should there be new matters that were not yet taken up during the 26 September 2007 Philippines." He also emphasizes that his claim of executive privilege is upon the order of the President
hearing, may I be furnished in advance as to what else I need to clarify, so that as a resource person, I and within the parameters laid down in Senate v. Ermita10 and United States v. Reynolds.11 Lastly, he
may adequately prepare myself. argues that he is precluded from disclosing communications made

In addition, petitioner submitted a letter prepared by his counsel, Atty. Antonio R. Bautista, stating, among to him in official confidence under Section 712 of Republic Act No. 6713, otherwise known as Code of
others that: (1) his (petitioner) non-appearance was upon the order of the President; and (2) his Conduct and Ethical Standards for Public Officials and Employees, and Section 2413 (e) of Rule 130 of
conversation with President Arroyo dealt with delicate and sensitive national security and diplomatic the Rules of Court.

14
Respondent Committees assert the contrary. They argue that (1) petitioner's testimony is material and (1) The communications between petitioner and the President are covered by the principle of "executive
pertinent in the investigation conducted in aid of legislation; (2) there is no valid justification for petitioner privilege."
to claim executive privilege; (3) there is no abuse of their authority to order petitioner's arrest; and (4)
petitioner has not come to court with clean hands. (2) Petitioner was not summoned by respondent Senate Committees in accordance with the law-making
body's power to conduct inquiries in aid of legislation as laid down in Section 21, Article VI of the
In the oral argument held last March 4, 2008, the following issues were ventilated: Constitution and Senate v. Ermita.

1. What communications between the President and petitioner Neri are covered by the principle of (3) Respondent Senate Committees gravely abused its discretion for alleged non-compliance with
'executive privilege'? the Subpoena dated November 13, 2007.

1.a Did Executive Secretary Ermita correctly invoke the principle of executive privilege, by order of the The Court granted the OSG's motion the next day, March 18, 2008.
President, to cover (i) conversations of the President in the exercise of her executive and policy decision-
making and (ii) information, which might impair our diplomatic as well as economic relations with the As the foregoing facts unfold, related events transpired.
People's Republic of China?
On March 6, 2008, President Arroyo issued Memorandum Circular No. 151, revoking Executive Order No.
1.b. Did petitioner Neri correctly invoke executive privilege to avoid testifying on his conversations with the 464 and Memorandum Circular No. 108. She advised executive officials and employees to follow and
President on the NBN contract on his assertions that the said conversations "dealt with delicate and abide by the Constitution, existing laws and jurisprudence, including, among others, the case of Senate v.
sensitive national security and diplomatic matters relating to the impact of bribery scandal Ermita17 when they are invited to legislative inquiries in aid of legislation.
involving high government officials and the possible loss of confidence of foreign investors and
lenders in the Philippines" x x x within the principles laid down in Senate v. Ermita (488 SCRA 1
[2006])? At the core of this controversy are the two (2) crucial queries, to wit:

1.c Will the claim of executive privilege in this case violate the following provisions of the Constitution: First, are the communications elicited by the subject three (3) questions covered by executive privilege?

Sec. 28, Art. II (Full public disclosure of all transactions involving public interest) And second, did respondent Committees commit grave abuse of discretion in issuing the
contempt Order?
Sec. 7, Art. III (The right of the people to information on matters of public concern)
We grant the petition.
Sec. 1, Art. XI (Public office is a public trust)
At the outset, a glimpse at the landmark case of Senate v. Ermita18 becomes imperative. Senate draws in
bold strokes the distinction between the legislative and oversight powers of the Congress, as embodied
Sec. 17, Art. VII (The President shall ensure that the laws be faithfully executed) under Sections 21 and 22, respectively, of Article VI of the Constitution, to wit:

and the due process clause and the principle of separation of powers? SECTION 21. The Senate or the House of Representatives or any of its respective committees may
conduct inquiries in aid of legislation in accordance with its duly published rules of procedure. The rights
2. What is the proper procedure to be followed in invoking executive privilege? of persons appearing in or affected by such inquiries shall be respected.

3. Did the Senate Committees gravely abuse their discretion in ordering the arrest of petitioner for non- SECTION 22. The heads of department may upon their own initiative, with the consent of the President,
compliance with the subpoena? or upon the request of either House, or as the rules of each House shall provide, appear before and be
heard by such House on any matter pertaining to their departments. Written questions shall be submitted
After the oral argument, the parties were directed to manifest to the Court within twenty-four (24) hours if to the President of the Senate or the Speaker of the House of Representatives at least three days before
they are amenable to the Court's proposal of allowing petitioner to immediately resume his testimony their scheduled appearance. Interpellations shall not be limited to written questions, but may cover
before the Senate Committees to answer the other questions of the Senators without prejudice to the matters related thereto. When the security of the state or the public interest so requires and the President
decision on the merits of this pending petition. It was understood that petitioner may invoke executive so states in writing, the appearance shall be conducted in executive session.
privilege in the course of the Senate Committees proceedings, and if the respondent Committees
disagree thereto, the unanswered questions will be the subject of a supplemental pleading to be resolved Senate cautions that while the above provisions are closely related and complementary to each other,
along with the three (3) questions subject of the present petition.14 At the same time, respondent they should not be considered as pertaining to the same power of Congress. Section 21 relates to the
Committees were directed to submit several pertinent documents.15 power to conduct inquiries in aid of legislation. Its aim is to elicit information that may be used for
legislation. On the other hand, Section 22 pertains to the power to conduct a question hour, the objective
The Senate did not agree with the proposal for the reasons stated in the Manifestation dated March 5, of which is to obtain information in pursuit of Congress' oversight function.19 Simply stated, while both
2008. As to the required documents, the Senate and respondent Committees manifested that they would powers allow Congress or any of its committees to conduct inquiry, their objectives are different.
not be able to submit the latter's "Minutes of all meetings" and the "Minute Book" because it has never
been the "historical and traditional legislative practice to keep them."16 They instead submitted the This distinction gives birth to another distinction with regard to the use of compulsory process. Unlike in
Transcript of Stenographic Notes of respondent Committees' joint public hearings. Section 21, Congress cannot compel the appearance of executive officials under Section 22. The Court's
pronouncement in Senate v. Ermita20 is clear:
On March 17, 2008, the Office of the Solicitor General (OSG) filed a Motion for Leave to Intervene and to
Admit Attached Memorandum, founded on the following arguments: When Congress merely seeks to be informed on how department heads are implementing the statutes
which it has issued, its right to such information is not as imperative as that of the President to whom, as

15
Chief Executive, such department heads must give a report of their performance as a matter of duty. In The Nixon and post-Watergate cases established the broad contours of the presidential
such instances, Section 22, in keeping with the separation of powers, states that Congress may communications privilege.28 In United States v. Nixon,29 the U.S. Court recognized a great public
only request their appearance. Nonetheless, when the inquiry in which Congress requires their interest in preserving "the confidentiality of conversations that take place in the President's
appearance is 'in aid of legislation' under Section 21, the appearance is mandatory for the same reasons performance of his official duties." It thus considered presidential communications as "presumptively
stated in Arnault. privileged." Apparently, the presumption is founded on the "President's generalized interest in
confidentiality." The privilege is said to be necessary to guarantee the candor of presidential advisors
In fine, the oversight function of Congress may be facilitated by compulsory process only to the and to provide "the President and those who assist him… with freedom to explore alternatives in
extent that it is performed in pursuit of legislation. This is consistent with the intent discerned from the the process of shaping policies and making decisions and to do so in a way many would be
deliberations of the Constitutional Commission unwilling to express except privately."

Ultimately, the power of Congress to compel the appearance of executive officials under section 21 and In In Re: Sealed Case,30 the U.S. Court of Appeals delved deeper. It ruled that there are two (2) kinds of
the lack of it under Section 22 find their basis in the principle of separation of powers. While the executive executive privilege; one is the presidential communications privilege and, the other is the
branch is a co-equal branch of the legislature, it cannot frustrate the power of Congress to legislate by deliberative process privilege. The former pertains to "communications, documents or other
refusing to comply with its demands for information. (Emphasis supplied.) materials that reflect presidential decision-making and deliberations and that the President
believes should remain confidential." The latter includes 'advisory opinions, recommendations and
deliberations comprising part of a process by which governmental decisions and policies are
The availability of the power of judicial review to resolve the issues raised in this case has also been formulated."
settled in Senate v. Ermita, when it held:
Accordingly, they are characterized by marked distinctions. Presidential communications
As evidenced by the American experience during the so-called "McCarthy era," however, the right of privilege applies to decision-making of the President while, the deliberative process privilege,
Congress to conduct inquiries in aid of legislation is, in theory, no less susceptible to abuse than executive to decision-making of executive officials. The first is rooted in the constitutional principle of separation
or judicial power. It may thus be subjected to judicial review pursuant to the Court's certiorari powers of power and the President's unique constitutional role; the second on common law privilege. Unlike
under Section 1, Article VIII of the Constitution. the deliberative process privilege, the presidential communications privilege applies to documents
in their entirety, and covers final and post-decisional materials as well as pre-deliberative
Hence, this decision. ones31 As a consequence, congressional or judicial negation of the presidential communications
privilege is always subject to greater scrutiny than denial of the deliberative process privilege.
I-The Communications Elicited by the Three (3) Questions are Covered by Executive Privilege
Turning on who are the officials covered by the presidential communications privilege, In Re: Sealed
We start with the basic premises where the parties have conceded. Case confines the privilege only to White House Staff that has "operational proximity" to direct presidential
decision-making. Thus, the privilege is meant to encompass only those functions that form the core of
presidential authority, involving what the court characterized as "quintessential and non-delegable
The power of Congress to conduct inquiries in aid of legislation is broad. This is based on the proposition
Presidential power," such as commander-in-chief power, appointment and removal power, the power to
that a legislative body cannot legislate wisely or effectively in the absence of information respecting the
grant pardons and reprieves, the sole-authority to receive ambassadors and other public officers, the
conditions which the legislation is intended to affect or change.21 Inevitably, adjunct thereto is the
power to negotiate treaties, etc.32
compulsory process to enforce it. But, the power, broad as it is, has limitations. To be valid, it is imperative
that it is done in accordance with the Senate or House duly published rules of procedure and that the
rights of the persons appearing in or affected by such inquiries be respected. The situation in Judicial Watch, Inc. v. Department of Justice33 tested the In Re: Sealed Case principles.
There, while the presidential decision involved is the exercise of the President's pardon power, a non-
delegable, core-presidential function, the Deputy Attorney General and the Pardon Attorney were deemed
The power extends even to executive officials and the only way for them to be exempted is through a valid
to be too remote from the President and his senior White House advisors to be protected. The Court
claim of executive privilege.22 This directs us to the consideration of the question -- is there a
conceded that
recognized claim of executive privilege despite the revocation of E.O. 464?

functionally those officials were performing a task directly related to the President's pardon power, but
A- There is a Recognized Claim of Executive Privilege Despite the Revocation of E.O. 464
concluded that an organizational test was more appropriate for confining the potentially broad sweep that
would result from the In Re: Sealed Case's functional test. The majority concluded that, the lesser
At this juncture, it must be stressed that the revocation of E.O. 464 does not in any way diminish our protections of the deliberative process privilege would suffice. That privilege was, however, found
concept of executive privilege. This is because this concept has Constitutional underpinnings. Unlike the insufficient to justify the confidentiality of the 4,341 withheld documents.
United States which has further accorded the concept with statutory status by enacting the Freedom of
Information Act23 and the Federal Advisory Committee Act,24 the Philippines has retained its
But more specific classifications of communications covered by executive privilege are made in older
constitutional origination, occasionally interpreted only by this Court in various cases. The most recent of
cases. Courts ruled early that the Executive has a right to withhold documents that might reveal military
these is the case of Senate v. Ermita where this Court declared unconstitutional substantial portions of
or state secrets,34 identity of government informers in some circumstances,,35 and information
E.O. 464. In this regard, it is worthy to note that Executive Ermita's Letter dated November 15, 2007 limits
related to pending investigations.36 An area where the privilege is highly revered is in foreign
its bases for the claim of executive privilege to Senate v. Ermita, Almonte v. Vasquez,25 and Chavez v.
relations. In United States v. Curtiss-Wright Export Corp.37 the U.S. Court, citing President George
PEA.26 There was never a mention of E.O. 464.
Washington, pronounced:

While these cases, especially Senate v. Ermita,27 have comprehensively discussed the concept of


The nature of foreign negotiations requires caution, and their success must often depend on secrecy, and
executive privilege, we deem it imperative to explore it once more in view of the clamor for this Court to
even when brought to a conclusion, a full disclosure of all the measures, demands, or eventual
clearly define the communications covered by executive privilege.
concessions which may have been proposed or contemplated would be extremely impolitic, for this might
have a pernicious influence on future negotiations or produce immediate inconveniences, perhaps danger
and mischief, in relation to other powers. The necessity of such caution and secrecy was one cogent

16
reason for vesting the power of making treaties in the President, with the advice and consent of the [N]either the doctrine of separation of powers, nor the need for confidentiality of high-level
Senate, the principle on which the body was formed confining it to a small number of members. To admit, communications, without more, can sustain an absolute, unqualified Presidential privilege of immunity
then, a right in the House of Representatives to demand and to have as a matter of course all the papers from judicial process under all circumstances.
respecting a negotiation with a foreign power would be to establish a dangerous precedent.
The foregoing is consistent with the earlier case of Nixon v. Sirica,46 where it was held that presidential
Majority of the above jurisprudence have found their way in our jurisdiction. In Chavez v. PCGG38, this communications are presumptively privileged and that the presumption can be overcome only by mere
Court held that there is a "governmental privilege against public disclosure with respect to state secrets showing of public need by the branch seeking access to conversations. The courts are enjoined to resolve
regarding military, diplomatic and other security matters." In Chavez v. PEA,39 there is also a recognition the competing interests of the political branches of the government "in the manner that preserves the
of the confidentiality of Presidential conversations, correspondences, and discussions in closed-door essential functions of each Branch."47 Here, the record is bereft of any categorical explanation from
Cabinet meetings. In Senate v. Ermita, the concept of presidential communications privilege is fully respondent Committees to show a compelling or citical need for the answers to the three (3) questions in
discussed. the enactment of a law. Instead, the questions veer more towards the exercise of the legislative oversight
function under Section 22 of Article VI rather than Section 21 of the same Article. Senate v. Ermita ruled
As may be gleaned from the above discussion, the claim of executive privilege is highly recognized in that the "the oversight function of Congress may be facilitated by compulsory process only to the
cases where the subject of inquiry relates to a power textually committed by the Constitution to the extent that it is performed in pursuit of legislation." It is conceded that it is difficult to draw the line
President, such as the area of military and foreign relations. Under our Constitution, the President is the between an inquiry in aid of legislation and an inquiry in the exercise of oversight function of Congress. In
repository of the commander-in-chief,40 appointing,41 pardoning,42 and diplomatic43 powers. Consistent this regard, much will depend on the content of the questions and the manner the inquiry is conducted.
with the doctrine of separation of powers, the information relating to these powers may enjoy greater
confidentiality than others. Respondent Committees argue that a claim of executive privilege does not guard against a possible
disclosure of a crime or wrongdoing. We see no dispute on this. It is settled in United States v.
The above cases, especially, Nixon, In Re Sealed Case and Judicial Watch, somehow provide the Nixon48 that "demonstrated, specific need for evidence in pending criminal trial" outweighs the
elements of presidential communications privilege, to wit: President's "generalized interest in confidentiality." However, the present case's distinction with
the Nixon case is very evident. In Nixon, there is a pending criminal proceeding where the information is
requested and it is the demands of due process of law and the fair administration of criminal justice that
1) The protected communication must relate to a "quintessential and non-delegable presidential power." the information be disclosed. This is the reason why the U.S. Court was quick to "limit the scope of its
decision." It stressed that it is "not concerned here with the balance between the President's
2) The communication must be authored or "solicited and received" by a close advisor of the President or generalized interest in confidentiality x x x and congressional demands for information." Unlike in
the President himself. The judicial test is that an advisor must be in "operational proximity" with the Nixon, the information here is elicited, not in a criminal proceeding, but in a legislative inquiry. In this
President. regard, Senate v. Ermita stressed that the validity of the claim of executive privilege depends not only on
the ground invoked but, also, on the procedural setting or the context in which the claim is made.
3) The presidential communications privilege remains a qualified privilege that may be overcome by a Furthermore, in Nixon, the President did not interpose any claim of need to protect military, diplomatic or
showing of adequate need, such that the information sought "likely contains important evidence" and by sensitive national security secrets. In the present case, Executive Secretary Ermita categorically claims
the unavailability of the information elsewhere by an appropriate investigating authority.44 executive privilege on the grounds of presidential communications privilege in relation to her executive
and policy decision-making process and diplomatic secrets.
In the case at bar, Executive Secretary Ermita premised his claim of executive privilege on the ground that
the communications elicited by the three (3) questions "fall under conversation and correspondence The respondent Committees should cautiously tread into the investigation of matters which may present a
between the President and public officials" necessary in "her executive and policy decision-making conflict of interest that may provide a ground to inhibit the Senators participating in the inquiry if later on
process" and, that "the information sought to be disclosed might impair our diplomatic as well as an impeachment proceeding is initiated on the same subject matter of the present Senate inquiry.
economic relations with the People's Republic of China." Simply put, the bases are presidential Pertinently, in Senate Select Committee on Presidential Campaign Activities v. Nixon,49 it was held that
communications privilege and executive privilege on matters relating to diplomacy or foreign since an impeachment proceeding had been initiated by a House Committee, the Senate Select
relations. Committee's immediate oversight need for five presidential tapes should give way to the House Judiciary
Committee which has the constitutional authority to inquire into presidential impeachment. The Court
expounded on this issue in this wise:
Using the above elements, we are convinced that, indeed, the communications elicited by the three (3)
questions are covered by the presidential communications privilege. First, the communications relate
to a "quintessential and non-delegable power" of the President, i.e. the power to enter into an executive It is true, of course, that the Executive cannot, any more than the other branches of government, invoke a
agreement with other countries. This authority of the President to enter into executive agreements without general confidentiality privilege to shield its officials and employees from investigations by the proper
the concurrence of the Legislature has traditionally been recognized in Philippine governmental institutions into possible criminal wrongdoing. The Congress learned this as to its own
jurisprudence.45 Second, the communications are "received" by a close advisor of the President. Under privileges in Gravel v. United States, as did the judicial branch, in a sense, in Clark v. United States, and
the "operational proximity" test, petitioner can be considered a close advisor, being a member of the executive branch itself in Nixon v. Sirica. But under Nixon v. Sirica, the showing required to
President Arroyo's cabinet. And third, there is no adequate showing of a compelling need that would overcome the presumption favoring confidentiality turned, not on the nature of the presidential
justify the limitation of the privilege and of the unavailability of the information elsewhere by an conduct that the subpoenaed material might reveal, but, instead, on the nature and appropriateness of
appropriate investigating authority. the function in the performance of which the material was sought, and the degree to which the
material was necessary to its fulfillment. Here also our task requires and our decision implies no
judgment whatever concerning possible presidential involvement in culpable activity. On the
The third element deserves a lengthy discussion.
contrary, we think the sufficiency of the Committee's showing must depend solely on whether the
subpoenaed evidence is demonstrably critical to the responsible fulfillment of the Committee's
United States v. Nixon held that a claim of executive privilege is subject to balancing against other functions.
interest. In other words, confidentiality in executive privilege is not absolutely protected by the
Constitution. The U.S. Court held:
In its initial briefs here, the Committee argued that it has shown exactly this. It contended that resolution,
on the basis of the subpoenaed tapes, of the conflicts in the testimony before it 'would aid in a
determination whether legislative involvement in political campaigns is necessary' and 'could help

17
engender the public support needed for basic reforms in our electoral system.' Moreover, Congress has, There are, it bears noting, clear distinctions between the right of Congress to information which underlies
according to the Committee, power to oversee the operations of the executive branch, to investigate the power of inquiry and the right of people to information on matters of public concern. For one, the
instances of possible corruption and malfeasance in office, and to expose the results of its investigations demand of a citizen for the production of documents pursuant to his right to information does not have the
to public view. The Committee says that with respect to Watergate-related matters, this power has been same obligatory force as a subpoena duces tecum issued by Congress. Neither does the right to
delegated to it by the Senate, and that to exercise its power responsibly, it must have access to the information grant a citizen the power to exact testimony from government officials. These powers belong
subpoenaed tapes. only to Congress, not to an individual citizen.

We turn first to the latter contention. In the circumstances of this case, we need neither deny that the Thus, while Congress is composed of representatives elected by the people, it does not follow,
Congress may have, quite apart from its legislative responsibilities, a general oversight power, nor explore except in a highly qualified sense, that in every exercise of its power of inquiry, the people are
what the lawful reach of that power might be under the Committee's constituent resolution. Since passage exercising their right to information.
of that resolution, the House Committee on the Judiciary has begun an inquiry into presidential
impeachment. The investigative authority of the Judiciary Committee with respect to presidential conduct The members of respondent Committees should not invoke as justification in their exercise of power a
has an express constitutional source. x x x We have been shown no evidence indicating that right properly belonging to the people in general. This is because when they discharge their power, they
Congress itself attaches any particular value to this interest. In these circumstances, we think the do so as public officials and members of Congress. Be that as it may, the right to information must be
need for the tapes premised solely on an asserted power to investigate and inform cannot justify balanced with and should give way, in appropriate cases, to constitutional precepts particularly those
enforcement of the Committee's subpoena. pertaining to delicate interplay of executive-legislative powers and privileges which is the subject of careful
review by numerous decided cases.
The sufficiency of the Committee's showing of need has come to depend, therefore, entirely on whether
the subpoenaed materials are critical to the performance of its legislative functions. There is a clear B- The Claim of Executive Privilege is Properly Invoked
difference between Congress' legislative tasks and the responsibility of a grand jury, or any institution
engaged in like functions. While fact-finding by a legislative committee is undeniably a part of its
task, legislative judgments normally depend more on the predicted consequences of proposed We now proceed to the issue -- whether the claim is properly invoked by the President. Jurisprudence
legislative actions and their political acceptability, than on precise reconstruction of past events; teaches that for the claim to be properly invoked, there must be a formal claim of privilege, lodged by the
Congress frequently legislates on the basis of conflicting information provided in its hearings. In contrast, head of the department which has control over the matter."56 A formal and proper claim of executive
the responsibility of the grand jury turns entirely on its ability to determine whether there is probable cause privilege requires a "precise and certain reason" for preserving their confidentiality.57
to believe that certain named individuals did or did not commit specific crimes. If, for example, as in Nixon
v. Sirica, one of those crimes is perjury concerning the content of certain conversations, the grand jury's The Letter dated November 17, 2007 of Executive Secretary Ermita satisfies the requirement. It serves as
need for the most precise evidence, the exact text of oral statements recorded in their original form, is the formal claim of privilege. There, he expressly states that "this Office is constrained to invoke the
undeniable. We see no comparable need in the legislative process, at least not in the settled doctrine of executive privilege as refined in Senate v. Ermita, and has advised Secretary
circumstances of this case. Indeed, whatever force there might once have been in the Committee's Neri accordingly." Obviously, he is referring to the Office of the President. That is more than enough
argument that the subpoenaed materials are necessary to its legislative judgments has been substantially compliance. In Senate v. Ermita, a less categorical letter was even adjudged to be sufficient.
undermined by subsequent events. (Emphasis supplied)
With regard to the existence of "precise and certain reason," we find the grounds relied upon by Executive
Respondent Committees further contend that the grant of petitioner's claim of executive privilege violates Secretary Ermita specific enough so as not "to leave respondent Committees in the dark on how the
the constitutional provisions on the right of the people to information on matters of public concern.50 We requested information could be classified as privileged." The case of Senate v. Ermita only requires that
might have agreed with such contention if petitioner did not appear before them at all. But petitioner made an allegation be made "whether the information demanded involves military or diplomatic secrets, closed-
himself available to them during the September 26 hearing, where he was questioned for eleven (11) door Cabinet meetings, etc." The particular ground must only be specified. The enumeration is not even
hours. Not only that, he expressly manifested his willingness to answer more questions from the Senators, intended to be comprehensive."58 The following statement of grounds satisfies the requirement:
with the exception only of those covered by his claim of executive privilege.
The context in which executive privilege is being invoked is that the information sought to be disclosed
The right to public information, like any other right, is subject to limitation. Section 7 of Article III provides: might impair our diplomatic as well as economic relations with the People's Republic of China. Given the
confidential nature in which these information were conveyed to the President, he cannot provide the
The right of the people to information on matters of public concern shall be recognized. Access to official Committee any further details of these conversations, without disclosing the very thing the privilege is
records, and to documents, and papers pertaining to official acts, transactions, or decisions, as well as to designed to protect.At any rate, as held further in Senate v. Ermita, 59 the Congress must not require the
government research data used as basis for policy development, shall be afforded the citizen, subject to executive to state the reasons for the claim with such particularity as to compel disclosure of the
such limitations as may be provided by law. information which the privilege is meant to protect. This is a matter of respect to a coordinate and co-
equal department.
The provision itself expressly provides the limitation, i.e. as may be provided by law. Some of these laws
are Section 7 of Republic Act (R.A.) No. 6713,51 Article 22952 of the Revised Penal Code, Section 3 IIRespondent Committees Committed Grave Abuse of Discretion in Issuing the Contempt Order
(k)53 of R.A. No. 3019, and Section 24(e)54 of Rule 130 of the Rules of Court. These are in addition to
what our body of jurisprudence classifies as confidential55 and what our Constitution considers as Grave abuse of discretion means "such capricious and whimsical exercise of judgment as is equivalent to
belonging to the larger concept of executive privilege. Clearly, there is a recognized public interest in the lack of jurisdiction, or, in other words where the power is exercised in an arbitrary or despotic manner by
confidentiality of certain information. We find the information subject of this case belonging to such kind. reason of passion or personal hostility and it must be so patent and gross as to amount to an evasion of
positive duty or to a virtual refusal to perform the duty enjoined or to act at all in contemplation of law."60
More than anything else, though, the right of Congress or any of its Committees to obtain information in
aid of legislation cannot be equated with the people's right to public information. The former cannot claim It must be reiterated that when respondent Committees issued the show cause Letter dated November
that every legislative inquiry is an exercise of the people's right to information. The distinction between 22, 2007, petitioner replied immediately, manifesting that it was not his intention to ignore the Senate
such rights is laid down in Senate v. Ermita: hearing and that he thought the only remaining questions were the three (3) questions he claimed to be
covered by executive privilege. In addition thereto, he submitted Atty. Bautista's letter, stating that his non-

18
appearance was upon the order of the President and specifying the reasons why his conversations with So my suggestion, Mr. Chairman, is the Blue Ribbon Committee should not forget it's the lead committee
President Arroyo are covered by executive privilege. Both correspondences include an expression of here, and therefore, the will of the lead committee prevails over all the other, you, know reservations that
his willingness to testify again, provided he "be furnished in advance" copies of the questions. other committees might have who are only secondary or even tertiary committees, Mr. Chairman.
Without responding to his request for advance list of questions, respondent Committees issued the Order
dated January 30, 2008, citing him in contempt of respondent Committees and ordering his arrest and THE CHAIRMAN (SEN. CAYETANO, A.) Thank you very much to the Minority Leader. And I agree with
detention at the Office of the Senate Sergeant-At-Arms until such time that he would appear and give his the wisdom of his statements. I was merely mentioning that under Section 6 of the Rules of the
testimony. Thereupon, petitioner filed a motion for reconsideration, informing respondent Committees that Committee and under Section 6, "The Committee by a vote of a majority of all its members may punish for
he had filed the present petition for certiorari. contempt any witness before it who disobeys any order of the Committee."

Respondent Committees committed grave abuse of discretion in issuing the contempt Order in view of So the Blue Ribbon Committee is more than willing to take that responsibility. But we only have six
five (5) reasons. members here today, I am the seventh as chair and so we have not met that number. So I am
merely stating that, sir, that when we will prepare the documentation, if a majority of all members sign and
First, there being a legitimate claim of executive privilege, the issuance of the contempt Order suffers I am following the Sabio v. Gordon rule wherein I do believe, if I am not mistaken, Chairman Gordon
from constitutional infirmity. prepared the documentation and then either in caucus or in session asked the other members to sign.
And once the signatures are obtained, solely for the purpose that Secretary Neri or Mr. Lozada will not be
Second, respondent Committees did not comply with the requirement laid down in Senate v. Ermita that able to legally question our subpoena as being insufficient in accordance with law.
the invitations should contain the "possible needed statute which prompted the need for the inquiry," along
with "the usual indication of the subject of inquiry and the questions relative to and in furtherance SEN. PIMENTEL. Mr. Chairman, the caution that the chair is suggesting is very well-taken. But I'd like to
thereof." Compliance with this requirement is imperative, both under Sections 21 and 22 of Article VI of advert to the fact that the quorum of the committee is only two as far as I remember. Any two-member
the Constitution. This must be so to ensure that the rights of both persons appearing in or affected by senators attending a Senate committee hearing provide that quorum, and therefore there is more than a
such inquiry are respected as mandated by said Section 21 and by virtue of the express language of quorum demanded by our Rules as far as we are concerned now, and acting as Blue Ribbon Committee,
Section 22. Unfortunately, despite petitioner's repeated demands, respondent Committees did not send as Senator Enrile pointed out. In any event, the signatures that will follow by the additional members will
him an advance list of questions. only tend to strengthen the determination of this Committee to put its foot forward – put down on what is
happening in this country, Mr. Chairman, because it really looks terrible if the primary Committee of the
Third, a reading of the transcript of respondent Committees' January 30, 2008 proceeding reveals that Senate, which is the Blue Ribbon Committee, cannot even sanction people who openly defy, you know,
only a minority of the members of the Senate Blue Ribbon Committee was present during the the summons of this Committee. I know that the Chair is going through an agonizing moment here. I know
deliberation. 61 Section 18 of the Rules of Procedure Governing Inquiries in Aid of Legislation provides that. But nonetheless, I think we have to uphold, you know, the institution that we are representing
that: because the alternative will be a disaster for all of us, Mr. Chairman. So having said that, I'd like to
reiterate my point.
"The Committee, by a vote of majority of all its members, may punish for contempt any witness before it
who disobeys any order of the Committee or refuses to be sworn or to testify or to answer proper THE CHAIRMAN (SEN. CAYETANO, A.) First of all, I agree 100 percent with the intentions of the
questions by the Committee or any of its members." Minority Leader. But let me very respectfully disagree with the legal requirements. Because, yes, we
can have a hearing if we are only two but both under Section 18 of the Rules of the Senate and
under Section 6 of the Rules of the Blue Ribbon Committee, there is a need for a majority of all
Clearly, the needed vote is a majority of all the members of the Committee. Apparently, members who members if it is a case of contempt and arrest. So, I am simply trying to avoid the court rebuking the
did not actually participate in the deliberation were made to sign the contempt Order. Thus, there is a Committee, which will instead of strengthening will weaken us. But I do agree, Mr. Minority Leader, that
cloud of doubt as to the validity of the contempt Order dated January 30, 2008. We quote the pertinent we should push for this and show the executive branch that the well-decided – the issue has been
portion of the transcript, thus: decided upon the Sabio versus Gordon case. And it's very clear that we are all allowed to call witnesses.
And if they refure or they disobey not only can we cite them in contempt and have them arrested. x x x 62
THE CHAIRMAN (SEN. CAYETANO, A). For clarification. x x x The Chair will call either a caucus or
will ask the Committee on Rules if there is a problem. Meaning, if we do not have the sufficient Fourth, we find merit in the argument of the OSG that respondent Committees likewise violated Section
numbers. But if we have a sufficient number, we will just hold a caucus to be able to implement 21 of Article VI of the Constitution, requiring that the inquiry be in accordance with the "duly published
that right away because…Again, our Rules provide that any one held in contempt and ordered rules of procedure." We quote the OSG's explanation:
arrested, need the concurrence of a majority of all members of the said committee and we have
three committees conducting this.SEN. PIMENTEL. Mr. Chairman.
The phrase 'duly published rules of procedure' requires the Senate of every Congress to publish its rules
of procedure governing inquiries in aid of legislation because every Senate is distinct from the one before
THE CHAIRMAN (SEN. CAYETANO,A). May I recognize the Minority Leader and give him the floor, it or after it. Since Senatorial elections are held every three (3) years for one-half of the Senate's
Senator Pimentel. membership, the composition of the Senate also changes by the end of each term. Each Senate may thus
enact a different set of rules as it may deem fit. Not having published its Rules of Procedure, the
SEN. PIMENTEL. Mr. Chairman, there is no problem, I think, with consulting the other committees. subject hearings in aid of legislation conducted by the 14th Senate, are therefore, procedurally
But I am of the opinion that the Blue Ribbon Committee is the lead committee, and therefore, it infirm.
should have preference in enforcing its own decisions. Meaning to say, it is not something that is
subject to consultation with other committees. I am not sure that is the right interpretation. I think And fifth, respondent Committees' issuance of the contempt Order is arbitrary and precipitate. It must be
that once we decide here, we enforce what we decide, because otherwise, before we know it, our pointed out that respondent Committees did not first pass upon the claim of executive privilege and
determination is watered down by delay and, you know, the so-called "consultation" that inform petitioner of their ruling. Instead, they curtly dismissed his explanation as "unsatisfactory" and
inevitably will have to take place if we follow the premise that has been explained. simultaneously issued the Order citing him in contempt and ordering his immediate arrest and detention.

A fact worth highlighting is that petitioner is not an unwilling witness. He manifested several times his
readiness to testify before respondent Committees. He refused to answer the three (3) questions because

19
he was ordered by the President to claim executive privilege. It behooves respondent Committees to first WHEREFORE, the petition is hereby GRANTED. The subject Order dated January 30, 2008, citing
rule on the claim of executive privilege and inform petitioner of their finding thereon, instead of petitioner Romulo L. Neri in contempt of the Senate Committees and directing his arrest and detention, is
peremptorily dismissing his explanation as "unsatisfactory." Undoubtedly, respondent Committees' actions hereby nullified. SO ORDERED.
constitute grave abuse of discretion for being arbitrary and for denying petitioner due process of law. The
same quality afflicted their conduct when they (a) disregarded petitioner's motion for reconsideration G.R. No. 170338             December 23, 2008
alleging that he had filed the present petition before this Court and (b) ignored petitioner's repeated
request for an advance list of questions, if there be any aside from the three (3) questions as to which he
claimed to be covered by executive privilege. VIRGILIO O. GARCILLANO,  vs.
THE HOUSE OF REPRESENTATIVES
Even the courts are repeatedly advised to exercise the power of contempt judiciously and sparingly with
utmost self-restraint with the end in view of utilizing the same for correction and preservation of the dignity More than three years ago, tapes ostensibly containing a wiretapped conversation purportedly between
of the court, not for retaliation or vindication.63 Respondent Committees should have exercised the same the President of the Philippines and a high-ranking official of the Commission on Elections (COMELEC)
restraint, after all petitioner is not even an ordinary witness. He holds a high position in a co-equal branch surfaced. They captured unprecedented public attention and thrust the country into a controversy that
of government. placed the legitimacy of the present administration on the line, and resulted in the near-collapse of the
Arroyo government. The tapes, notoriously referred to as the "Hello Garci" tapes, allegedly contained the
President’s instructions to COMELEC Commissioner Virgilio Garcillano to manipulate in her favor results
In this regard, it is important to mention that many incidents of judicial review could have been avoided if of the 2004 presidential elections. These recordings were to become the subject of heated legislative
powers are discharged with circumspection and deference. Concomitant with the doctrine of separation of hearings conducted separately by committees of both Houses of Congress.1
powers is the mandate to observe respect to a co-equal branch of the government. One last word.
In the House of Representatives (House), on June 8, 2005, then Minority Floor Leader Francis G.
The Court was accused of attempting to abandon its constitutional duty when it required the parties to Escudero delivered a privilege speech, "Tale of Two Tapes," and set in motion a congressional
consider a proposal that would lead to a possible compromise. The accusation is far from the truth. The investigation jointly conducted by the Committees on Public Information, Public Order and Safety,
Court did so, only to test a tool that other jurisdictions find to be effective in settling similar cases, to avoid National Defense and Security, Information and Communications Technology, and Suffrage and Electoral
a piecemeal consideration of the questions for review and to avert a constitutional crisis between the Reforms (respondent House Committees). During the inquiry, several versions of the wiretapped
executive and legislative branches of government. conversation emerged. But on July 5, 2005, National Bureau of Investigation (NBI) Director Reynaldo
Wycoco, Atty. Alan Paguia and the lawyer of former NBI Deputy Director Samuel Ong submitted to the
In United States v. American Tel. & Tel Co.,64 the court refrained from deciding the case because of its respondent House Committees seven alleged "original" tape recordings of the supposed three-hour taped
desire to avoid a resolution that might disturb the balance of power between the two branches and conversation. After prolonged and impassioned debate by the committee members on the admissibility
inaccurately reflect their true needs. Instead, it remanded the record to the District Court for further and authenticity of the recordings, the tapes were eventually played in the chambers of the House.2
proceedings during which the parties are required to negotiate a settlement. In the subsequent case
of United States v. American Tel. &Tel Co.,65 it was held that "much of this spirit of compromise is On August 3, 2005, the respondent House Committees decided to suspend the hearings indefinitely.
reflected in the generality of language found in the Constitution." It proceeded to state: Nevertheless, they decided to prepare committee reports based on the said recordings and the
testimonies of the resource persons.3
Under this view, the coordinate branches do not exist in an exclusively adversary relationship to one
another when a conflict in authority arises. Rather each branch should take cognizance of an implicit Alarmed by these developments, petitioner Virgilio O. Garcillano (Garcillano) filed with this Court a
constitutional mandate to seek optimal accommodation through a realistic evaluation of the needs of the Petition for Prohibition and Injunction, with Prayer for Temporary Restraining Order and/or Writ of
conflicting branches in the particular fact situation. Preliminary Injunction4 docketed as G.R. No. 170338. He prayed that the respondent House Committees
be restrained from using these tape recordings of the "illegally obtained" wiretapped conversations in their
It thereafter concluded that: "The Separation of Powers often impairs efficiency, in terms of dispatch committee reports and for any other purpose. He further implored that the said recordings and any
and the immediate functioning of government. It is the long-term staying power of government reference thereto be ordered stricken off the records of the inquiry, and the respondent House
that is enhanced by the mutual accommodation required by the separation of powers." Committees directed to desist from further using the recordings in any of the House proceedings.5

In rendering this decision, the Court emphasizes once more that the basic principles of constitutional law Without reaching its denouement, the House discussion and debates on the "Garci tapes" abruptly
cannot be subordinated to the needs of a particular situation. As magistrates, our mandate is to rule stopped.
objectively and dispassionately, always mindful of Mr. Justice Holmes' warning on the dangers inherent in
cases of this nature, thus: After more than two years of quiescence, Senator Panfilo Lacson roused the slumbering issue with a
privilege speech, "The Lighthouse That Brought Darkness." In his discourse, Senator Lacson promised to
"some accident of immediate and overwhelming interest…appeals to the feelings and distorts the provide the public "the whole unvarnished truth – the what’s, when’s, where’s, who’s and why’s" of the
judgment. These immediate interests exercise a kind of hydraulic pressure which makes what previously alleged wiretap, and sought an inquiry into the perceived willingness of telecommunications providers to
was clear seem doubtful, and before which even well settled principles of law will bend."66 participate in nefarious wiretapping activities.

In this present crusade to "search for truth," we should turn to the fundamental constitutional principles On motion of Senator Francis Pangilinan, Senator Lacson’s speech was referred to the Senate
which underlie our tripartite system of government, where the Legislature enacts the law, the Judiciary Committee on National Defense and Security, chaired by Senator Rodolfo Biazon, who had previously
interprets it and the Executive implements it. They are considered separate, co-equal, coordinate and filed two bills6 seeking to regulate the sale, purchase and use of wiretapping equipment and to prohibit
supreme within their respective spheres but, imbued with a system of checks and balances to prevent the Armed Forces of the Philippines (AFP) from performing electoral duties.7
unwarranted exercise of power. The Court's mandate is to preserve these constitutional principles at all
times to keep the political branches of government within constitutional bounds in the exercise of their In the Senate’s plenary session the following day, a lengthy debate ensued when Senator Richard Gordon
respective powers and prerogatives, even if it be in the search for truth. This is the only way we can aired his concern on the possible transgression of Republic Act (R.A.) No. 42008 if the body were to
preserve the stability of our democratic institutions and uphold the Rule of Law. conduct a legislative inquiry on the matter. On August 28, 2007, Senator Miriam Defensor-Santiago

20
delivered a privilege speech, articulating her considered view that the Constitution absolutely bans the the National Telecommunications Commission. The majority, in the said case, echoed the current policy
use, possession, replay or communication of the contents of the "Hello Garci" tapes. However, she that "this Court has repeatedly and consistently refused to wield procedural barriers as impediments to its
recommended a legislative investigation into the role of the Intelligence Service of the AFP (ISAFP), the addressing and resolving serious legal questions that greatly impact on public interest, in keeping with the
Philippine National Police or other government entities in the alleged illegal wiretapping of public officials.9 Court’s duty under the 1987 Constitution to determine whether or not other branches of government have
kept themselves within the limits of the Constitution and the laws, and that they have not abused the
On September 6, 2007, petitioners Santiago Ranada and Oswaldo Agcaoili, retired justices of the Court of discretion given to them."26
Appeals, filed before this Court a Petition for Prohibition with Prayer for the Issuance of a Temporary
Restraining Order and/or Writ of Preliminary Injunction,10 docketed as G.R. No. 179275, seeking to bar In G.R. No. 170338, petitioner Garcillano justifies his standing to initiate the petition by alleging that he is
the Senate from conducting its scheduled legislative inquiry. They argued in the main that the intended the person alluded to in the "Hello Garci" tapes. Further, his was publicly identified by the members of the
legislative inquiry violates R.A. No. 4200 and Section 3, Article III of the Constitution.11 respondent committees as one of the voices in the recordings.27 Obviously, therefore, petitioner
Garcillano stands to be directly injured by the House committees’ actions and charges of electoral fraud.
As the Court did not issue an injunctive writ, the Senate proceeded with its public hearings on the "Hello The Court recognizes his standing to institute the petition for prohibition.
Garci" tapes on September 7,12 1713 and October 1,14 2007.
In G.R. No. 179275, petitioners Ranada and Agcaoili justify their standing by alleging that they are
Intervening as respondents,15 Senators Aquilino Q. Pimentel, Jr., Benigno Noynoy C. Aquino, Rodolfo G. concerned citizens, taxpayers, and members of the IBP. They are of the firm conviction that any attempt
Biazon, Panfilo M. Lacson, Loren B. Legarda, M.A. Jamby A.S. Madrigal and Antonio F. Trillanes filed to use the "Hello Garci" tapes will further divide the country. They wish to see the legal and proper use of
their Comment16 on the petition on September 25, 2007. public funds that will necessarily be defrayed in the ensuing public hearings. They are worried by the
continuous violation of the laws and individual rights, and the blatant attempt to abuse constitutional
processes through the conduct of legislative inquiries purportedly in aid of legislation.28
The Court subsequently heard the case on oral argument.17
Intervenor Sagge alleges violation of his right to due process considering that he is summoned to attend
On October 26, 2007, Maj. Lindsay Rex Sagge, a member of the ISAFP and one of the resource persons the Senate hearings without being apprised not only of his rights therein through the publication of the
summoned by the Senate to appear and testify at its hearings, moved to intervene as petitioner in G.R. Senate Rules of Procedure Governing Inquiries in Aid of Legislation, but also of the intended legislation
No. 179275.18 which underpins the investigation. He further intervenes as a taxpayer bewailing the useless and wasteful
expenditure of public funds involved in the conduct of the questioned hearings.29
On November 20, 2007, the Court resolved to consolidate G.R. Nos. 170338 and 179275.19
Given that petitioners Ranada and Agcaoili allege an interest in the execution of the laws and that
It may be noted that while both petitions involve the "Hello Garci" recordings, they have different intervenor Sagge asserts his constitutional right to due process,30 they satisfy the requisite personal
objectives–the first is poised at preventing the playing of the tapes in the House and their subsequent stake in the outcome of the controversy by merely being citizens of the Republic.
inclusion in the committee reports, and the second seeks to prohibit and stop the conduct of the Senate
inquiry on the wiretapped conversation. Following the Court’s ruling in Francisco, Jr. v. The House of Representatives,31 we find sufficient
petitioners Ranada’s and Agcaoili’s and intervenor Sagge’s allegation that the continuous conduct by the
The Court dismisses the first petition, G.R. No. 170338, and grants the second, G.R. No. 179275. Senate of the questioned legislative inquiry will necessarily involve the expenditure of public funds.32 It
should be noted that in Francisco, rights personal to then Chief Justice Hilario G. Davide, Jr. had been
-I- injured by the alleged unconstitutional acts of the House of Representatives, yet the Court granted
standing to the petitioners therein for, as in this case, they invariably invoked the vindication of their own
rights–as taxpayers, members of Congress, citizens, individually or in a class suit, and members of the
Before delving into the merits of the case, the Court shall first resolve the issue on the parties’ standing,
bar and of the legal profession–which were also supposedly violated by the therein assailed
argued at length in their pleadings.
unconstitutional acts.33

In Tolentino v. COMELEC,20 we explained that "‘[l]egal standing’ or locus standi refers to a personal and
Likewise, a reading of the petition in G.R. No. 179275 shows that the petitioners and intervenor Sagge
substantial interest in a case such that the party has sustained or will sustain direct injury because of the
advance constitutional issues which deserve the attention of this Court in view of their seriousness,
challenged governmental act x x x," thus,
novelty and weight as precedents. The issues are of transcendental and paramount importance not only
to the public but also to the Bench and the Bar, and should be resolved for the guidance of all.34
generally, a party will be allowed to litigate only when (1) he can show that he has personally suffered
some actual or threatened injury because of the allegedly illegal conduct of the government; (2) the injury
Thus, in the exercise of its sound discretion and given the liberal attitude it has shown in prior cases
is fairly traceable to the challenged action; and (3) the injury is likely to be redressed by a favorable
climaxing in the more recent case of Chavez, the Court recognizes the legal standing of petitioners
action.21
Ranada and Agcaoili and intervenor Sagge.

The gist of the question of standing is whether a party has "alleged such a personal stake in the outcome
- II -
of the controversy as to assure that concrete adverseness which sharpens the presentation of issues
upon which the court so largely depends for illumination of difficult constitutional questions."22
The Court, however, dismisses G.R. No. 170338 for being moot and academic. Repeatedly stressed in
our prior decisions is the principle that the exercise by this Court of judicial power is limited to the
However, considering that locus standi is a mere procedural technicality, the Court, in recent cases, has
determination and resolution of actual cases and controversies.35 By actual cases, we mean existing
relaxed the stringent direct injury test. David v. Macapagal-Arroyo23 articulates that a "liberal policy has
conflicts appropriate or ripe for judicial determination, not conjectural or anticipatory, for otherwise the
been observed, allowing ordinary citizens, members of Congress, and civic organizations to prosecute
decision of the Court will amount to an advisory opinion. The power of judicial inquiry does not extend to
actions involving the constitutionality or validity of laws, regulations and rulings."24 The fairly
hypothetical questions because any attempt at abstraction could only lead to dialectics and barren legal
recent Chavez v. Gonzales25 even permitted a non-member of the broadcast media, who failed to allege
questions and to sterile conclusions unrelated to actualities.36 Neither will the Court determine a moot
a personal stake in the outcome of the controversy, to challenge the acts of the Secretary of Justice and
question in a case in which no practical relief can be granted. A case becomes moot when its purpose has
21
become stale.37 It is unnecessary to indulge in academic discussion of a case presenting a moot requires a majority of Senators to "constitute a quorum to do business." Applying the same reasoning
question as a judgment thereon cannot have any practical legal effect or, in the nature of things, cannot in Arnault v. Nazareno, the Senate under the 1987 Constitution is not a continuing body because less
be enforced.38 than majority of the Senators continue into the next Congress. The consequence is that the Rules of
Procedure must be republished by the Senate after every expiry of the term of twelve Senators.47
In G.R. No. 170338, petitioner Garcillano implores from the Court, as aforementioned, the issuance of an
injunctive writ to prohibit the respondent House Committees from playing the tape recordings and from The subject was explained with greater lucidity in our Resolution48 (On the Motion for Reconsideration) in
including the same in their committee report. He likewise prays that the said tapes be stricken off the the same case, viz.:
records of the House proceedings. But the Court notes that the recordings were already played in the
House and heard by its members.39 There is also the widely publicized fact that the committee reports on On the nature of the Senate as a "continuing body," this Court sees fit to issue a clarification. Certainly,
the "Hello Garci" inquiry were completed and submitted to the House in plenary by the respondent there is no debate that the Senate as an institution is "continuing," as it is not dissolved as an entity with
committees.40 Having been overtaken by these events, the Garcillano petition has to be dismissed for each national election or change in the composition of its members. However, in the conduct of its day-to-
being moot and academic. After all, prohibition is a preventive remedy to restrain the doing of an act about day business the Senate of each Congress acts separately and independently of the Senate of the
to be done, and not intended to provide a remedy for an act already accomplished.41 Congress before it. The Rules of the Senate itself confirms this when it states:

- III - RULE XLIV UNFINISHED BUSINESS

As to the petition in G.R. No. 179275, the Court grants the same. The Senate cannot be allowed to SEC. 123. Unfinished business at the end of the session shall be taken up at the next session in the same
continue with the conduct of the questioned legislative inquiry without duly published rules of procedure, in status.
clear derogation of the constitutional requirement.
All pending matters and proceedings shall terminate upon the expiration of one (1) Congress, but
Section 21, Article VI of the 1987 Constitution explicitly provides that "[t]he Senate or the House of may be taken by the succeeding Congress as if present for the first time.
Representatives, or any of its respective committees may conduct inquiries in aid of legislation in
accordance with its duly published rules of procedure." The requisite of publication of the rules is intended
to satisfy the basic requirements of due process.42 Publication is indeed imperative, for it will be the Undeniably from the foregoing, all pending matters and proceedings, i.e., unpassed bills and even
height of injustice to punish or otherwise burden a citizen for the transgression of a law or rule of which he legislative investigations, of the Senate of a particular Congress are considered terminated upon the
had no notice whatsoever, not even a constructive one.43 What constitutes publication is set forth in expiration of that Congress and it is merely optional on the Senate of the succeeding Congress to take up
Article 2 of the Civil Code, which provides that "[l]aws shall take effect after 15 days following the such unfinished matters, not in the same status, but as if presented for the first time. The logic and
completion of their publication either in the Official Gazette, or in a newspaper of general circulation in the practicality of such a rule is readily apparent considering that the Senate of the succeeding Congress
Philippines."44 (which will typically have a different composition as that of the previous Congress) should not be bound by
the acts and deliberations of the Senate of which they had no part. If the Senate is a continuing body even
with respect to the conduct of its business, then pending matters will not be deemed terminated with the
The respondents in G.R. No. 179275 admit in their pleadings and even on oral argument that the Senate expiration of one Congress but will, as a matter of course, continue into the next Congress with the same
Rules of Procedure Governing Inquiries in Aid of Legislation had been published in newspapers of general status.
circulation only in 1995 and in 2006.45 With respect to the present Senate of the 14th Congress, however,
of which the term of half of its members commenced on June 30, 2007, no effort was undertaken for the
publication of these rules when they first opened their session. This dichotomy of the continuity of the Senate as an institution and of the opposite nature of the conduct
of its business is reflected in its Rules. The Rules of the Senate (i.e. the Senate’s main rules of procedure)
states:
Recently, the Court had occasion to rule on this very same question. In Neri v. Senate Committee on
Accountability of Public Officers and Investigations,46 we said:
RULE LI - AMENDMENTS TO, OR REVISIONS OF, THE RULES
Fourth, we find merit in the argument of the OSG that respondent Committees likewise violated Section
21 of Article VI of the Constitution, requiring that the inquiry be in accordance with the "duly published SEC. 136. At the start of each session in which the Senators elected in the preceding elections shall begin
rules of procedure." We quote the OSG’s explanation: their term of office, the President may endorse the Rules to the appropriate committee for amendment or
revision.
The phrase "duly published rules of procedure" requires the Senate of every Congress to publish its rules
of procedure governing inquiries in aid of legislation because every Senate is distinct from the one before The Rules may also be amended by means of a motion which should be presented at least one day
it or after it. Since Senatorial elections are held every three (3) years for one-half of the Senate’s before its consideration, and the vote of the majority of the Senators present in the session shall be
membership, the composition of the Senate also changes by the end of each term. Each Senate may thus required for its approval.
enact a different set of rules as it may deem fit. Not having published its Rules of Procedure, the
subject hearings in aid of legislation conducted by the 14th Senate, are therefore, procedurally RULE LII - DATE OF TAKING EFFECT
infirm.
SEC. 137. These Rules shall take effect on the date of their adoption and shall remain in force until they
Justice Antonio T. Carpio, in his Dissenting and Concurring Opinion, reinforces this ruling with the are amended or repealed.
following rationalization:
Section 136 of the Senate Rules quoted above takes into account the new composition of the Senate after
The present Senate under the 1987 Constitution is no longer a continuing legislative body. The present an election and the possibility of the amendment or revision of the Rules at the start of each session in
Senate has twenty-four members, twelve of whom are elected every three years for a term of six years which the newly elected Senators shall begin their term.
each. Thus, the term of twelve Senators expires every three years, leaving less than a majority of
Senators to continue into the next Congress. The 1987 Constitution, like the 1935 Constitution,

22
However, it is evident that the Senate has determined that its main rules are intended to be valid from the rules governed it, in clear contravention of the Constitution. With the foregoing disquisition, the Court finds
date of their adoption until they are amended or repealed. Such language is conspicuously absent from it unnecessary to discuss the other issues raised in the consolidated petitions.
the Rules. The Rules simply state "(t)hese Rules shall take effect seven (7) days after publication in two
(2) newspapers of general circulation." The latter does not explicitly provide for the continued effectivity of WHEREFORE, the petition in G.R. No. 170338 is DISMISSED, and the petition in G.R. No. 179275 is
such rules until they are amended or repealed. In view of the difference in the language of the two sets of GRANTED. Let a writ of prohibition be issued enjoining the Senate of the Republic of the Philippines
Senate rules, it cannot be presumed that the Rules (on legislative inquiries) would continue into the next and/or any of its committees from conducting any inquiry in aid of legislation centered on the "Hello Garci"
Congress. The Senate of the next Congress may easily adopt different rules for its legislative inquiries tapes. SO ORDERED.
which come within the rule on unfinished business.
G.R. No. 167173               December 27, 2007
The language of Section 21, Article VI of the Constitution requiring that the inquiry be conducted in
accordance with the duly published rules of procedure is categorical. It is incumbent upon the Senate to
publish the rules for its legislative inquiries in each Congress or otherwise make the published rules STANDARD CHARTERED BANK vs. SENATE COMMITTEE ON BANKS
clearly state that the same shall be effective in subsequent Congresses or until they are amended or
repealed to sufficiently put public on notice. Before us is a Petition for Prohibition (With Prayer for Issuance of Temporary Restraining Order and/or
Injunction) dated and filed on March 11, 2005 by petitioners against respondent Senate Committee on
If it was the intention of the Senate for its present rules on legislative inquiries to be effective even in the Banks, Financial Institutions and Currencies, as represented by its Chairperson Edgardo J. Angara
next Congress, it could have easily adopted the same language it had used in its main rules regarding (respondent).
effectivity.
Petitioner Standard Chartered Bank (SCB)-Philippines is an institution incorporated in England with
Respondents justify their non-observance of the constitutionally mandated publication by arguing that the limited liability and is licensed to engage in banking, trust, and other related operations in the Philippines.
rules have never been amended since 1995 and, despite that, they are published in booklet form available Petitioners Paul Simon Morris, Sundara Ramesh, Owen Belman, Sanjay Aggarwal, Rajamani
to anyone for free, and accessible to the public at the Senate’s internet web page.49 Chandrashekar, Marivel Gonzales, Ma. Ellen Victor, Chona G. Reyes, Zenaida Iglesias, Ramona Bernad,
Michaelangelo Aguilar, and Fernand Tansingco are the Chief Executive Officer, Chief Operations Officer,
Country Head of Consumer Banking, General Manager for Credit Card and Personal Loans, Chief
The Court does not agree. The absence of any amendment to the rules cannot justify the Senate’s Financial Officer, Legal and Compliance Officer, former Trust and Investment Services Head, Country Tax
defiance of the clear and unambiguous language of Section 21, Article VI of the Constitution. The organic Officer, Head of Corporate Affairs, Head of Banking Services, Head of Client Relationships, and the Head
law instructs, without more, that the Senate or its committees may conduct inquiries in aid of of Global Markets of SCB-Philippines, respectively. Respondent, on the other hand, is one of the
legislation only in accordance with duly published rules of procedure, and does not make any distinction permanent committees of the Senate of the Philippines.
whether or not these rules have undergone amendments or revision. The constitutional mandate to
publish the said rules prevails over any custom, practice or tradition followed by the Senate.
The petition seeks the issuance of a temporary restraining order (TRO) to enjoin respondent from (1)
proceeding with its inquiry pursuant to Philippine Senate (P.S.) Resolution No. 166; (2) compelling
Justice Carpio’s response to the same argument raised by the respondents is illuminating: petitioners who are officers of petitioner SCB-Philippines to attend and testify before any further hearing to
be conducted by respondent, particularly that set on March 15, 2005; and (3) enforcing any hold-
The publication of the Rules of Procedure in the website of the Senate, or in pamphlet form available at departure order (HDO) and/or putting the petitioners on the Watch List. It also prays that judgment be
the Senate, is not sufficient under the Tañada v. Tuvera ruling which requires publication either in the rendered (1) annulling the subpoenae ad testificandum and duces tecum issued to petitioners, and (2)
Official Gazette or in a newspaper of general circulation. The Rules of Procedure even provide that the prohibiting the respondent from compelling petitioners to appear and testify in the inquiry being conducted
rules "shall take effect seven (7) days after publication in two (2) newspapers of general circulation," pursuant to P.S. Resolution No. 166.
precluding any other form of publication. Publication in accordance with Tañada is mandatory to comply
with the due process requirement because the Rules of Procedure put a person’s liberty at risk. A person The facts are as follows:
who violates the Rules of Procedure could be arrested and detained by the Senate.
On February 1, 2005, Senator Juan Ponce Enrile, Vice Chairperson of respondent, delivered a privilege
The invocation by the respondents of the provisions of R.A. No. 8792,50 otherwise known as the speech entitled "Arrogance of Wealth"1 before the Senate based on a letter from Atty. Mark R. Bocobo
Electronic Commerce Act of 2000, to support their claim of valid publication through the internet is all the denouncing SCB-Philippines for selling unregistered foreign securities in violation of the Securities
more incorrect. R.A. 8792 considers an electronic data message or an electronic document as the Regulation Code (R.A. No. 8799) and urging the Senate to immediately conduct an inquiry, in aid of
functional equivalent of a written document only for evidentiary purposes.51 In other words, the law legislation, to prevent the occurrence of a similar fraudulent activity in the future. Upon motion of Senator
merely recognizes the admissibility in evidence (for their being the original) of electronic data messages Francis Pangilinan, the speech was referred to respondent. Prior to the privilege speech, Senator Enrile
and/or electronic documents.52 It does not make the internet a medium for publishing laws, rules and had introduced P.S. Resolution No. 166,2 to wit:
regulations.
RESOLUTION
Given this discussion, the respondent Senate Committees, therefore, could not, in violation of the
Constitution, use its unpublished rules in the legislative inquiry subject of these consolidated cases. The
conduct of inquiries in aid of legislation by the Senate has to be deferred until it shall have caused the DIRECTING THE COMMITTEE ON BANKS, FINANCIAL INSTITUTIONS AND CURRENCIES, TO
publication of the rules, because it can do so only "in accordance with its duly published rules of CONDUCT AN INQUIRY, IN AID OF LEGISLATION, INTO THE ILLEGAL SALE OF UNREGISTERED
procedure." AND HIGH-RISK SECURITIES BY STANDARD CHARTERED BANK, WHICH RESULTED IN
BILLIONS OF PESOS OF LOSSES TO THE INVESTING PUBLIC
Very recently, the Senate caused the publication of the Senate Rules of Procedure Governing Inquiries in
Aid of Legislation in the October 31, 2008 issues of Manila Bulletin and Malaya. While we take judicial WHEREAS, Republic Act No. 7721, otherwise known as the "Law Liberalizing the Entry and Scope of
notice of this fact, the recent publication does not cure the infirmity of the inquiry sought to be prohibited Operations of Foreign Banks in the Philippines," was approved on May 18, 1994 to promote greater
by the instant petitions. Insofar as the consolidated cases are concerned, the legislative investigation participation of foreign banks in the Philippine Banking Industry that will stimulate economic growth and
subject thereof still could not be undertaken by the respondent Senate Committees, because no published serve as a channel for the flow of funds into the economy;

23
WHEREAS, to promote greater competition in the Philippine Banking Industry, foreign banks were Now, therefore, BE IT RESOLVED, AS IT IS HEREBY RESOLVED, to direct the Committee on Banks,
accorded the same privileges, allowed to perform the same functions and subjected to the same Currencies, and Financial Institutions, to conduct an inquiry, in aid of legislation, into the reported sale of
limitations under relevant banking laws imposed upon domestic banks; unregistered and high-risk securities by Standard Chartered Bank which resulted in billions of losses to
the investing public.
WHEREAS, Standard Chartered Bank was among the foreign banks granted the privilege to do business
in our country under Republic Act No. 7721; Acting on the referral, respondent, through its Chairperson, Senator Edgardo J. Angara, set the initial
hearing on February 28, 2005 to investigate, in aid of legislation, the subject matter of the speech and
WHEREAS, there are complaints against Standard Chartered Bank whose actions have reportedly resolution filed by Senator Enrile.
defrauded hundreds of Filipino investors of billions of pesos through the sale of unregistered securities in
the form of high-risk mutual funds falsely advertised and marketed as safe investment havens; Respondent invited petitioners, among others, to attend the hearing, requesting them to submit their
written position paper. Petitioners, through counsel, submitted to respondent a letter3 dated February 24,
WHEREAS, there are reports that Standard Chartered Bank clearly knew that its actions were violative of 2005 presenting their position, particularly stressing that there were cases pending in court allegedly
Philippine banking and securities laws but cleverly disguised its illegal acts through the use of pro-forma involving the same issues subject of the legislative inquiry, thereby posing a challenge to the jurisdiction of
agreements containing waivers of liability in favor of the bank; respondent to continue with the inquiry.

WHEREAS, there are reports that in the early stages of conducting these questionable activities, the On February 28, 2005, respondent commenced the investigation. Senator Enrile inquired who among
Bangko Sentral ng Pilipinas warned and eventually fined Standard Chartered Bank a measly ₱30,000 for those invited as resource persons were present and who were absent. Thereafter, Senator Enrile moved
violating Philippine banking laws; that subpoenae be issued to those who did not attend the hearing and that the Senate request the
Department of Justice, through the Bureau of Immigration and Deportation, to issue an HDO against them
and/or include them in the Bureau’s Watch List. Senator Juan Flavier seconded the motion and the motion
WHEREAS, the particular operations of Standard Chartered Bank may constitute "conducting business in was approved.
an unsafe and unsound manner," punishable under Section 37 of Republic Act No. 7653 and should have
drawn the higher penalty of revocation of its quasi-banking license;
Respondent then proceeded with the investigation proper. Towards the end of the hearing, petitioners,
through counsel, made an Opening Statement4 that brought to the attention of respondent the lack of
WHEREAS, Republic Act No. 8791 or the "General Banking Act of 2000" deems a particular act or proper authorization from affected clients for the bank to make disclosures of their accounts and the lack
omission as conducting business in an unsafe and unsound manner as follows: of copies of the accusing documents mentioned in Senator Enrile's privilege speech, and reiterated that
there were pending court cases regarding the alleged sale in the Philippines by SCB-Philippines of
"Section 56.2 The act or omission has resulted or may result in material loss or damage or abnormal risk unregistered foreign securities.
to the institution's depositors, creditors, investors, stockholders or to the Bangko Sentral or to the public in
general." The February 28, 2005 hearing was adjourned without the setting of the next hearing date. However,
petitioners were later served by respondent with subpoenae ad testificandum and duces tecum to compel
WHEREAS, the sale of unregistered securities is also a clear violation of Republic Act No. 8799 or "The them to attend and testify at the hearing set on March 15, 2005. Hence, this petition.
Securities Regulation Code of 2000" which states:
The grounds relied upon by petitioners are as follows:
"Section 8.1 Securities shall not be sold or offered for sale or distribution within the Philippines, without a
registration statement duly filed with and approved by the Commission. Prior to such sale, information on I.
the securities, in such form and with such substance as the Commission may prescribe, shall be made
available to each prospective purchaser."
THE COMMITTEE ACTED WITHOUT JURISDICTION AND/OR ACTED WITH GRAVE ABUSE OF
DISCRETION AMOUNTING TO LACK OF JURISDICTION IN CONDUCTING AN INVESTIGATION,
WHEREAS, the Securities and Exchange Commission (SEC) reportedly issued a Cease-and-Desist PURPORTEDLY IN AID OF LEGISLATION, BUT IN REALITY PROBING INTO THE ISSUE OF
Order (CDO) against Standard Chartered Bank for the sale of these unregistered securities but the case WHETHER THE STANDARD CHARTERED BANK HAD SOLD UNREGISTERED FOREIGN
was reportedly settled administratively and dismissed after Standard Chartered Bank paid a fine of ₱7 SECURITIES IN THE PHILIPPINES. SAID ISSUE HAS LONG BEEN THE SUBJECT OF CRIMINAL AND
Million; CIVIL ACTIONS NOW PENDING BEFORE THE COURT OF APPEALS, REGIONAL TRIAL COURT OF
PASIG CITY, METROPOLITAN TRIAL COURT OF MAKATI CITY AND THE PROSECUTOR'S OFFICE
WHEREAS, the SEC reportedly made an official finding that Standard Chartered Bank actively engaged OF MAKATI CITY.
in promoting and marketing the so-called "Global Third Party Mutual Funds" to the investing public and
even set revenue quotas for the sale of these funds; II.

WHEREAS, existing laws including the Securities Regulation Code seem to be inadequate in preventing THE COMMITTEE ACTED IN GRAVE ABUSE OF DISCRETION AMOUNTING TO LACK OF
the sale of unregistered securities and in effectively enforcing the registration rules intended to protect the JURISDICTION BY CONDUCTING AN INVESTIGATION, PURPORTEDLY "IN AID OF LEGISLATION,"
investing public from fraudulent practices; BUT IN REALITY IN "AID OF COLLECTION" BY A HANDFUL OF TWO (2) CLIENTS OF STANDARD
CHARTERED BANK OF LOSSES WHICH WERE FOR THEIR ACCOUNT AND RISK. AT ANY RATE,
WHEREAS, the regulatory intervention by the SEC and BSP likewise appears inadequate in preventing SUCH COLLECTION IS WITHIN THE PROVINCE OF THE COURT RATHER THAN OF THE
the conduct of proscribed activities in a manner that would protect the investing public; LEGISLATURE.

WHEREAS, there is a need for remedial legislation to address the situation, having in mind the imposition III.
of proportionate penalties to offending entities and their directors, officers and representatives among
other additional regulatory measures;

24
THE COMMITTEE ACTED WITHOUT JURISDICTION AND/OR ACTED WITH GRAVE ABUSE OF The argument is misplaced. Bengzon does not apply squarely to petitioners’ case.
DISCRETION AMOUNTING TO LACK OF JURISDICTION IN COMPELLING PETITIONERS, SOME OF
WHOM ARE RESPONDENTS IN THE PENDING CRIMINAL AND CIVIL ACTIONS BROUGHT BY SAID It is true that in Bengzon, the Court declared that the issue to be investigated was one over which
CLIENTS, IN VIOLATION OF PETITIONERS’ RIGHT AGAINST SELF-INCRIMINATION AND RIGHT TO jurisdiction had already been acquired by the Sandiganbayan, and to allow the [Senate Blue Ribbon]
PURSUE AND DEFEND THEIR CAUSE IN COURT RATHER THAN ENGAGE IN TRIAL BY PUBLICITY Committee to investigate the matter would create the possibility of conflicting judgments; and that the
– A CLEAR VIOLATION OF DUE PROCESS, RIGHT TO PRIVACY AND TO TRAVEL. inquiry into the same justiciable controversy would be an encroachment on the exclusive domain of
judicial jurisdiction that had set in much earlier.
IV.
To the extent that, in the case at bench, there are a number of cases already pending in various courts
THE COMMITTEE ACTED IN GRAVE ABUSE OF DISCRETION AMOUNTING TO LACK OF and administrative bodies involving the petitioners, relative to the alleged sale of unregistered foreign
JURISDICTION BY DISREGARDING ITS OWN RULES.5 securities, there is a resemblance between this case and Bengzon. However, the similarity ends there.

Petitioners argue that respondent has no jurisdiction to conduct the inquiry because its subject matter is Central to the Court’s ruling in Bengzon -- that the Senate Blue Ribbon Committee was without any
the very same subject matter of the following cases, to wit: constitutional mooring to conduct the legislative investigation -- was the Court’s determination that the
intended inquiry was not in aid of legislation. The Court found that the speech of Senator Enrile, which
(a) CA-G.R. SP No. 85078, entitled "Manuel V. Baviera vs. Hon. Esperanza P. Rosario, et al., pending sought such investigation contained no suggestion of any contemplated legislation; it merely called upon
before the 9th Division of the Court of Appeals. In the petition, Mr. Baviera seeks to annul and set aside the Senate to look into possible violations of Section 5, Republic Act No. 3019. Thus, the Court held that
the dismissal by the Department of Justice of his complaint against Standard Chartered Bank and its the requested probe failed to comply with a fundamental requirement of Section 21, Article VI of the
officers accusing them of SELLING UNREGISTERED FOREIGN SECURITIES IN VIOLATION OF P.D. Constitution, which states:
NO. 1869 (SYNDICATED ESTAFA) AND ARTICLE 315 OF THE REVISED PENAL CODE.
The Senate or the House of Representatives or any of its respective committees may conduct inquiries in
(b) CA-G.R. SP No. 86200, entitled "Manuel V. Baviera vs. Hon. Rafael Buenaventura, et al.", pending aid of legislation in accordance with its duly published rules of procedure. The rights of persons appearing
before the 15th Division of the Court of Appeals. In the petition, Mr. Baviera seeks to annul and set aside in or affected by such inquiries shall be respected.
the termination for lack of probable cause by the Anti-Money Laundering Council ("AMLC") of the
investigation of Standard Chartered Bank for money laundering activities BY SELLING UNREGISTERED Accordingly, we stopped the Senate Blue Ribbon Committee from proceeding with the legislative
FOREIGN SECURITIES. investigation in that case.

(c) CA-G.R. SP No. 87328, entitled "Manuel V. Baviera vs. Hon. Esperanza Paglinawan Rozario, et Unfortunately for the petitioners, this distinguishing factual milieu in Bengzon does not obtain in the instant
al.," pending before the 16th Division of the Court of Appeals. The petition seeks to annul and set aside case. P.S. Resolution No. 166 is explicit on the subject and nature of the inquiry to be (and already being)
the dismissal by the Department of Justice of Mr. Baviera's complaint accusing SCB and its officers of conducted by the respondent Committee, as found in the last three Whereas clauses thereof, viz.:
violation of the Securities Regulation Code by SELLING UNREGISTERED FOREIGN SECURITIES.
WHEREAS, existing laws including the Securities Regulation Code seem to be inadequate in preventing
(d) Civil Case No. 70173, entitled "Mr. Noel G. Sanchez, et al. vs. Standard Chartered Bank," pending the sale of unregistered securities and in effectively enforcing the registration rules intended to protect the
before Branch 155 of the Regional Trial Court of Pasig City. Plaintiff seeks damages and recovery of their investing public from fraudulent practices;
investment accusing the bank of SELLING UNREGISTERED FOREIGN SECURITIES.
WHEREAS, the regulatory intervention by the SEC and BSP likewise appears inadequate in preventing
(e) Criminal Case No. 332034, entitled "People of the Philippines vs. Manuel V. Baviera," pending before the conduct of proscribed activities in a manner that would protect the investing public;
Branch 64 of the Metropolitan Trial Court of Makati City. Petitioner Morris is the private complainant in this
information for extortion or blackmail against Mr. Baviera for demanding the payment of US$2 Million with WHEREAS, there is a need for remedial legislation to address the situation, having in mind the imposition
the threat to EXPOSE THE BANK'S "LARGE SCALE SCAM" CONSISTING [OF] ILLEGAL SELLING of proportionate penalties to offending entities and their directors, officers and representatives among
OF UNREGISTERED FOREIGN SECURITIES BY THE BANK, before various government offices, such other additional regulatory measures; (emphasis supplied)
as the Department of Justice, the BIR, Bangko Sentral ng Pilipinas, Regional Trial Courts, and both
houses of Congress.
The unmistakable objective of the investigation, as set forth in the said resolution, exposes the error in
petitioners’ allegation that the inquiry, as initiated in a privilege speech by the very same Senator Enrile,
(f) Criminal Case No. 331395, entitled "People of the Philippines vs. Manuel V. Baviera," pending before was simply "to denounce the illegal practice committed by a foreign bank in selling unregistered foreign
Branch 64 of the Metropolitan Trial Court of Makati City. Petitioners Victor and Chona Reyes are the securities x x x." This fallacy is made more glaring when we consider that, at the conclusion of his
private complainants in this information for perjury committed by Mr. Baviera in securing a hold departure privilege speech, Senator Enrile urged the Senate "to immediately conduct an inquiry, in aid of legislation,
order against the petitioners herein from the Department of Justice for their alleged involvement in so as to prevent the occurrence of a similar fraudulent activity in the future."
syndicated estafa and swindling BY SELLING UNREGISTERED FOREIGN SECURITIES.
Indeed, the mere filing of a criminal or an administrative complaint before a court or a quasi-judicial body
(g) I.S. No. 2004-B-2279-80, entitled "Aurelio Litonjua III and Aurelio Litonjua, Jr. vs. Antonette de los should not automatically bar the conduct of legislative investigation. Otherwise, it would be extremely easy
Reyes, et al.," pending before the Office of the Prosecutor, Makati City. This is a criminal complaint to subvert any intended inquiry by Congress through the convenient ploy of instituting a criminal or an
accusing SCB and its officers of estafa for SELLING UNREGISTERED FOREIGN SECURITIES.6 administrative complaint. Surely, the exercise of sovereign legislative authority, of which the power of
legislative inquiry is an essential component, cannot be made subordinate to a criminal or an
Citing Bengzon, Jr. v. Senate Blue Ribbon Committee,7 the petitioners claim that since the issue of administrative investigation.
whether or not SCB-Philippines illegally sold unregistered foreign securities is already preempted by the
courts that took cognizance of the foregoing cases, the respondent, by this investigation, would encroach As succinctly stated in the landmark case Arnault v. Nazareno8 –
upon the judicial powers vested solely in these courts.

25
[T]he power of inquiry – with process to enforce it – is an essential and appropriate auxiliary to the sovereign character of the legislature as one of the three independent and coordinate branches of
legislative function. A legislative body cannot legislate wisely or effectively in the absence of information government.12
respecting the conditions which the legislation is intended to affect or change; and where the legislative
body does not itself possess the requisite information – which is not infrequently true – recourse must be In this case, petitioners’ imputation that the investigation was "in aid of collection" is a direct challenge
had to others who possess it. against the authority of the Senate Committee, as it ascribes ill motive to the latter. In this light, we find the
contempt citation against the petitioners reasonable and justified.
Neither can the petitioners claim that they were singled out by the respondent Committee. The Court
notes that among those invited as resource persons were officials of the Securities and Exchange Furthermore, it is axiomatic that the power of legislative investigation includes the power to compel the
Commission (SEC) and the Bangko Sentral ng Pilipinas (BSP). These officials were subjected to the attendance of witnesses. Corollary to the power to compel the attendance of witnesses is the power to
same critical scrutiny by the respondent relative to their separate findings on the illegal sale of ensure that said witnesses would be available to testify in the legislative investigation. In the case at
unregistered foreign securities by SCB-Philippines. It is obvious that the objective of the investigation was bench, considering that most of the officers of SCB-Philippines are not Filipino nationals who may easily
the quest for remedies, in terms of legislation, to prevent the recurrence of the allegedly fraudulent evade the compulsive character of respondent’s summons by leaving the country, it was reasonable for
activity. the respondent to request the assistance of the Bureau of Immigration and Deportation to prevent said
witnesses from evading the inquiry and defeating its purpose. In any event, no HDO was issued by a
Still, petitioners insist that the inquiry conducted by respondent was, in fact, "in aid of collection." They court. The BID instead included them only in the Watch List, which had the effect of merely delaying
claim that Atty. Bocobo and Manuel Baviera, the latter a party to the pending court cases cited by petitioners’ intended travel abroad for five (5) days, provided no HDO is issued against them.13
petitioners, were only seeking a friendly forum so that they could recover their investments from SCB-
Philippines; and that the respondent has allowed itself to be used as the conveniently available vehicle to With respect to the right of privacy which petitioners claim respondent has violated, suffice it to state that
effect this purpose. privacy is not an absolute right. While it is true that Section 21, Article VI of the Constitution, guarantees
respect for the rights of persons affected by the legislative investigation, not every invocation of the right
However, as correctly pointed out by respondent in its Comment on the petition, Atty. Bocobo did not file a to privacy should be allowed to thwart a legitimate congressional inquiry. In Sabio v. Gordon,14 we have
complaint before the Senate for the purpose of recovering his investment. On the contrary, and as held that the right of the people to access information on matters of public concern generally prevails over
confirmed during the initial hearing on February 28, 2005, his letter-complaint humbly requested the the right to privacy of ordinary financial transactions. In that case, we declared that the right to privacy is
Senate to conduct an inquiry into the purportedly illegal activities of SCB-Philippines, with the end view of not absolute where there is an overriding compelling state interest. Employing the rational basis
preventing the future occurrence of any similar fraudulent activity by the banks in general.9 Baviera, on relationship test, as laid down in Morfe v. Mutuc,15 there is no infringement of the individual’s right to
the other hand, was not a "complainant" but merely a witness in the investigation, invited to testify on the privacy as the requirement to disclosure information is for a valid purpose, in this case, to ensure that the
alleged illegal sale of unregistered foreign securities by SCB-Philippines, being one of the supposed government agencies involved in regulating banking transactions adequately protect the public who invest
victims thereof. in foreign securities. Suffice it to state that this purpose constitutes a reason compelling enough to
proceed with the assailed legislative investigation.16
The Court further notes that when it denied petitioners’ prayer for the issuance of a TRO to restrain the
hearing set on March 15, 2005,10 respondent proceeded with the investigation. On the said date, As regards the issue of self-incrimination, the petitioners, officers of SCB-Philippines, are not being
outraged by petitioners’ imputation that it was conducting the investigation "in aid of collection," indicted as accused in a criminal proceeding. They were summoned by respondent merely as resource
respondent held petitioners, together with their counsel, Atty. Reynaldo Geronimo, in contempt and persons, or as witnesses, in a legislative inquiry. As distinguished by this Court –
ordered their detention for six hours.
[An] accused occupies a different tier of protection from an ordinary witness. Whereas an ordinary witness
Petitioners filed a Motion for Partial Reconsideration of this Court’s Resolution dated March 14, 2005 only may be compelled to take the witness stand and claim the privilege as each question requiring an
with respect to the denial of the prayer for the issuance of a TRO and/or writ of preliminary injunction, incriminating answer is shot at him, an accused may altogether refuse to take the witness stand and
alleging that their being held in contempt was without legal basis, as the phrase "in aid of collection" refuse to answer any and all questions.17
partakes of an absolutely privileged allegation in the petition.
Concededly, this right of the accused against self-incrimination is extended to respondents in
We do not agree. The Court has already expounded on the essence of the contempt power of Congress administrative investigations that partake of the nature of or are analogous to criminal proceedings. The
and its committees in this wise – privilege has consistently been held to extend to all proceedings sanctioned by law; and to all cases in
which punishment is sought to be visited upon a witness, whether a party or not.18
The principle that Congress or any of its bodies has the power to punish recalcitrant witnesses is founded
upon reason and policy. Said power must be considered implied or incidental to the exercise of legislative However, in this case, petitioners neither stand as accused in a criminal case nor will they be subjected by
power. How could a legislative body obtain the knowledge and information on which to base intended the respondent to any penalty by reason of their testimonies. Hence, they cannot altogether decline
legislation if it cannot require and compel the disclosure of such knowledge and information, if it is appearing before respondent, although they may invoke the privilege when a question calling for an
impotent to punish a defiance of its power and authority? When the framers of the Constitution adopted incriminating answer is propounded.19
the principle of separation of powers, making each branch supreme within the realm of its respective
authority, it must have intended each department’s authority to be full and complete, independently of Petitioners’ argument, that the investigation before respondent may result in a recommendation for their
each other’s authority or power. And how could the authority and power become complete if for every act prosecution by the appropriate government agencies, such as the Department of Justice or the Office of
of refusal, every act of defiance, every act of contumacy against it, the legislative body must resort to the the Ombudsman, does not persuade.
judicial department for the appropriate remedy, because it is impotent by itself to punish or deal therewith,
with affronts committed against its authority or dignity.11
As held in Sinclair v. United States20 --
The exercise by Congress or by any of its committees of the power to punish contempt is based on the
principle of self-preservation. As the branch of the government vested with the legislative power, It may be conceded that Congress is without authority to compel disclosures for the purpose of aiding the
independently of the judicial branch, it can assert its authority and punish contumacious acts against it. prosecution of pending suits; but the authority of that body, directly or through its Committees, to require
Such power is sui generis, as it attaches not to the discharge of legislative functions per se, but to the pertinent disclosures in aid of its own constitutional power is not abridged because the information sought

26
to be elicited may also be of use in such suits. x x x It is plain that investigation of the matters involved in The solicited writs of certiorari and prohibition do not avail; the petition must be denied.
suits brought or to be commenced under the Senate resolution directing the institution of suits for the
cancellation of the leases might directly aid in respect of legislative action. I.

The prosecution of offenders by the prosecutorial agencies and the trial before the courts is for the The petitioners are high-ranking officers of the Armed Forces of the Philippines (AFP). Both petitioners,
punishment of persons who transgress the law. The intent of legislative inquiries, on the other hand, is to Brigadier General Francisco Gudani (Gen. Gudani) and Lieutenant Colonel Alexander Balutan (Col.
arrive at a policy determination, which may or may not be enacted into law. Balutan), belonged to the Philippine Marines. At the time of the subject incidents, both Gen. Gudani and
Col. Balutan were assigned to the Philippine Military Academy (PMA) in Baguio City, the former as the
Except only when it exercises the power to punish for contempt, the respondent, as with the other PMA Assistant Superintendent, and the latter as the Assistant Commandant of Cadets.2
Committees of the Senate or of the House of Representatives, cannot penalize violators even if there is
overwhelming evidence of criminal culpability. Other than proposing or initiating amendatory or remedial On 22 September 2005, Senator Rodolfo Biazon (Sen. Biazon) invited several senior officers of the AFP
legislation, respondent can only recommend measures to address or remedy whatever irregularities may to appear at a public hearing before the Senate Committee on National Defense and Security (Senate
be unearthed during the investigation, although it may include in its Report a recommendation for the Committee) scheduled on 28 September 2005. The hearing was scheduled after topics concerning the
criminal indictment of persons who may appear liable. At best, the recommendation, along with the conduct of the 2004 elections emerged in the public eye, particularly allegations of massive cheating and
evidence, contained in such a Report would be persuasive, but it is still up to the prosecutorial agencies the surfacing of copies of an audio excerpt purportedly of a phone conversation between President Gloria
and the courts to determine the liabilities of the offender. Macapagal Arroyo and an official of the Commission on Elections (COMELEC) widely reputed as then
COMELEC Commissioner Virgilio Garcillano. At the time of the 2004 elections, Gen. Gudani had been
Finally, petitioners sought anew, in their Manifestation and Motion21 dated June 21, 2006, the issuance designated as commander, and Col. Balutan a member, of "Joint Task Force Ranao" by the AFP
by this Court of a TRO and/or writ of preliminary injunction to prevent respondent from submitting its Southern Command. "Joint Task Force Ranao" was tasked with the maintenance of peace and order
Committee Report No. 75 to the Senate in plenary for approval. However, 16 days prior to the filing of the during the 2004 elections in the provinces of Lanao del Norte and Lanao del Sur.3 `
Manifestation and Motion, or on June 5, 2006, respondent had already submitted the report to the Senate
in plenary. While there is no showing that the said report has been approved by the Senate, the subject of Gen. Gudani, Col. Balutan, and AFP Chief of Staff Lieutenant General Generoso Senga (Gen. Senga)
the Manifestation and Motion has inescapably become moot and academic. were among the several AFP officers who received a letter invitation from Sen. Biazon to attend the 28
September 2005 hearing. On 23 September 2005, Gen. Senga replied through a letter to Sen. Biazon that
WHEREFORE, the Petition for Prohibition is DENIED for lack of merit. The Manifestation and Motion he would be unable to attend the hearing due to a previous commitment in Brunei, but he nonetheless
dated June 21, 2006 is, likewise, DENIED for being moot and academic. SO ORDERED "directed other officers from the AFP who were invited to attend the hearing."4

G.R. No. 170165               August 15, 2006 On 26 September 2005, the Office of the Chief of Staff of the AFP issued a Memorandum addressed to
the Superintendent of the PMA Gen. Cristolito P. Baloing (Gen. Baloing). It was signed by Lt. Col.
B/GEN. (RET.) FRANCISCO V. GUDANI vs.LT./GEN. GENEROSO S. Hernando DCA Iriberri in behalf of Gen. Senga.5 Noting that Gen. Gudani and Col. Balutan had been
invited to attend the Senate Committee hearing on 28 September 2005, the Memorandum directed the
two officers to attend the hearing.6 Conformably, Gen. Gudani and Col. Balutan filed their respective
A most dangerous general proposition is foisted on the Court — that soldiers who defy orders of their requests for travel authority addressed to the PMA Superintendent.
superior officers are exempt
On 27 September 2005, Gen. Senga wrote a letter to Sen. Biazon, requesting the postponement of the
from the strictures of military law and discipline if such defiance is predicated on an act otherwise valid hearing scheduled for the following day, since the AFP Chief of Staff was himself unable to attend said
under civilian law. Obedience and deference to the military chain of command and the President as hearing, and that some of the invited officers also could not attend as they were "attending to other urgent
commander-in-chief are the cornerstones of a professional military in the firm cusp of civilian control. operational matters." By this time, both Gen. Gudani and Col. Balutan had already departed Baguio for
These values of obedience and deference expected of military officers are content-neutral, beyond the Manila to attend the hearing.
sway of the officer’s own sense of what is prudent or rash, or more elementally, of right or wrong. A self-
righteous military invites itself as the scoundrel’s activist solution to the "ills" of participatory democracy.
Then on the evening of 27 September 2005, at around 10:10 p.m., a message was transmitted to the
PMA Superintendent from the office of Gen. Senga, stating as follows:
Petitioners seek the annulment of a directive from President Gloria Macapagal-Arroyo1 enjoining them
and other military officers from testifying before Congress without the President’s consent. Petitioners also
pray for injunctive relief against a pending preliminary investigation against them, in preparation for PER INSTRUCTION OF HER EXCELLENCY PGMA, NO AFP PERSONNEL SHALL APPEAR BEFORE
possible court-martial proceedings, initiated within the military justice system in connection with ANY CONGRESSIONAL OR SENATE HEARING WITHOUT HER APPROVAL. INFORM BGEN
petitioners’ violation of the aforementioned directive. FRANCISCO F GUDANI AFP AND LTC ALEXANDER BALUTAN PA (GSC) ACCORDINGLY.7

The Court is cognizant that petitioners, in their defense, invoke weighty constitutional principles that The following day, Gen. Senga sent another letter to Sen. Biazon, this time informing the senator that "no
center on fundamental freedoms enshrined in the Bill of Rights. Although these concerns will not be approval has been granted by the President to any AFP officer to appear" before the hearing scheduled
on that day. Nonetheless, both Gen. Gudani and Col. Balutan were present as the hearing started, and
they both testified as to the conduct of the 2004 elections.
addressed to the satisfaction of petitioners, the Court recognizes these values as of paramount
importance to our civil society, even if not determinative of the resolution of this petition. Had the relevant
issue before us been the right of the Senate to compel the testimony of petitioners, the constitutional The Office of the Solicitor General (OSG), representing the respondents before this Court, has offered
questions raised by them would have come to fore. Such a scenario could have very well been presented additional information surrounding the testimony of Gen. Gudani and Col. Balutan. The OSG manifests
to the Court in such manner, without the petitioners having had to violate a direct order from their that the couriers of the AFP Command Center had attempted to deliver the radio message to Gen.
commanding officer. Instead, the Court has to resolve whether petitioners may be subjected to military Gudani’s residence in a subdivision in Parañaque City late in the night of 27 September 2005, but they
discipline on account of their defiance of a direct order of the AFP Chief of Staff. were not permitted entry by the subdivision guards. The next day, 28 September 2005, shortly before the
start of the hearing, a copy of Gen. Senga’s letter to Sen. Biazon sent earlier that day was handed at the

27
Senate by Commodore Amable B. Tolentino of the AFP Office for Legislative Affairs to Gen. Gudani, who Articles of War defines persons subject to military law as "all officers and soldiers in the active service" of
replied that he already had a copy. Further, Gen. Senga called Commodore Tolentino on the latter’s cell the AFP.
phone and asked to talk to Gen. Gudani, but Gen. Gudani refused. In response, Gen. Senga instructed
Commodore Tolentino to inform Gen. Gudani that "it was an order," yet Gen. Gudani still refused to take II.
Gen. Senga’s call.8
We first proceed to define the proper litigable issues. Notably, the guilt or innocence of petitioners in
A few hours after Gen. Gudani and Col. Balutan had concluded their testimony, the office of Gen. Senga violating Articles 65 and 97 of the Articles of War is not an issue before this Court, especially considering
issued a statement which noted that the two had appeared before the Senate Committee "in spite of the that per records, petitioners have not yet been subjected to court martial proceedings. Owing to the
fact that a guidance has been given that a Presidential approval should be sought prior to such an absence of such proceedings, the correct inquiry should be limited to whether respondents could properly
appearance;" that such directive was "in keeping with the time[-]honored principle of the Chain of initiate such proceedings preparatory to a formal court-martial, such as the aforementioned preliminary
Command;" and that the two officers "disobeyed a legal order, in violation of A[rticles of] W[ar] 65 (Willfully investigation, on the basis of petitioners’ acts surrounding their testimony before the Senate on 28
Disobeying Superior Officer), hence they will be subjected to General Court Martial proceedings x x x" September 2005. Yet this Court, consistent with the principle that it is not a trier of facts at first
Both Gen. Gudani and Col. Balutan were likewise relieved of their assignments then.9 instance,21 is averse to making any authoritative findings of fact, for that function is first for the court-
martial court to fulfill.
On the very day of the hearing, 28 September 2005, President Gloria-Macapagal-Arroyo issued Executive
Order No. 464 (E.O. 464). The OSG notes that the E.O. "enjoined officials of the executive department Thus, we limit ourselves to those facts that are not controverted before the Court, having been commonly
including the military establishment from appearing in any legislative inquiry without her approval."10 This alleged by petitioners and the OSG (for respondents). Petitioners were called by the Senate Committee to
Court subsequently ruled on the constitutionality of the said executive order in Senate v. Ermita.11 The testify in its 28 September 2005 hearing. Petitioners attended such hearing and testified before the
relevance of E.O. 464 and Senate to the present petition shall be discussed forthwith. Committee, despite the fact that the day before, there was an order from Gen. Senga (which in turn was
sourced "per instruction" from President Arroyo) prohibiting them from testifying without the prior approval
In the meantime, on 30 September 2005, petitioners were directed by General Senga, through Col. Henry of the President. Petitioners do not precisely admit before this Court that they had learned of such order
A. Galarpe of the AFP Provost Marshal General, to appear before the Office of the Provost Marshal prior to their testimony, although the OSG asserts that at the very least, Gen. Gudani already knew of
General (OPMG) on 3 October 2005 for investigation. During their appearance before Col. Galarpe, both such order before he testified.22 Yet while this fact may be ultimately material in the court-martial
petitioners invoked their right to remain silent.12 The following day, Gen. Gudani was compulsorily retired proceedings, it is not determinative of this petition, which as stated earlier, does not proffer as an issue
from military service, having reached the age of 56.13 whether petitioners are guilty of violating the Articles of War.

In an Investigation Report dated 6 October 2005, the OPMG recommended that petitioners be charged What the Court has to consider though is whether the violation of the aforementioned order of Gen.
with violation of Article of War 65, on willfully disobeying a superior officer, in relation to Article of War 97, Senga, which emanated from the President, could lead to any investigation for court-martial of petitioners.
on conduct prejudicial to the good order and military discipline.14 As recommended, the case was It has to be acknowledged as a general principle23 that AFP personnel of whatever rank are liable under
referred to a Pre-Trial Investigation Officer (PTIO) preparatory to trial by the General Court Martial military law for violating a direct order of an officer superior in rank. Whether petitioners did violate such
(GCM).15 Consequently, on 24 October 2005, petitioners were separately served with Orders respectively an order is not for the Court to decide, but it will be necessary to assume, for the purposes of this petition,
addressed to them and signed by respondent Col. Gilbert Jose C. Roa, the Pre-Trial Investigating Officer that petitioners did so.
of the PTIO. The Orders directed petitioners to appear in person before Col. Roa at the Pre-Trial
Investigation of the Charges for violation of Articles 6516 and 9717 of Commonwealth Act No. 408,18 and III.
to submit their counter-affidavits and affidavits of witnesses at the Office of the Judge Advocate
General.19 The Orders were accompanied by respective charge sheets against petitioners, accusing
them of violating Articles of War 65 and 97. Preliminarily, we must discuss the effect of E.O. 464 and the Court’s ruling in Senate on the present
petition. Notably, it is not alleged that petitioners were in any way called to task for violating E.O.
464, but instead, they were charged for violating the direct order of Gen. Senga not to appear
It was from these premises that the present petition for certiorari and prohibition was filed, particularly before the Senate Committee, an order that stands independent of the executive order. Distinctions
seeking that (1) the order of President Arroyo coursed through Gen. Senga preventing petitioners from are called for, since Section 2(b) of E.O. 464 listed "generals and flag officers of the Armed Forces of the
testifying before Congress without her prior approval be declared unconstitutional; (2) the charges stated Philippines and such other officers who in the judgment of the Chief of Staff are covered by the executive
in the charge sheets against petitioners be quashed; and (3) Gen. Senga, Col. Galarpe, Col. Roa, and privilege," as among those public officials required in Section 3 of E.O. 464 "to secure prior consent of the
their successors-in-interest or persons acting for and on their behalf or orders, be permanently enjoined President prior to appearing before either House of Congress." The Court in Senate declared both Section
from proceeding against petitioners, as a consequence of their having testified before the Senate on 28 2(b) and Section 3 void,24 and the impression may have been left following Senate that it settled as
September 2005.20 doctrine, that the President is prohibited from requiring military personnel from attending congressional
hearings without having first secured prior presidential consent. That impression is wrong.
Petitioners characterize the directive from President Arroyo requiring her prior approval before any AFP
personnel appear before Congress as a "gag order," which violates the principle of separation of powers Senate turned on the nature of executive privilege, a presidential prerogative which is encumbered by
in government as it interferes with the investigation of the Senate Committee conducted in aid of significant limitations. Insofar as E.O. 464 compelled officials of the executive branch to seek prior
legislation. They also equate the "gag order" with culpable violation of the Constitution, particularly in presidential approval before appearing before Congress, the notion of executive control also comes into
relation to the public’s constitutional right to information and transparency in matters of public concern. consideration.25 However, the ability of the President to require a military official to secure prior consent
Plaintively, petitioners claim that "the Filipino people have every right to hear the [petitioners’] before appearing before Congress pertains to a wholly different and independent specie of presidential
testimonies," and even if the "gag order" were unconstitutional, it still was tantamount to "the crime of authority—the commander-in-chief powers of the President. By tradition and jurisprudence, the
obstruction of justice." Petitioners further argue that there was no law prohibiting them from testifying commander-in-chief powers of the President are not encumbered by the same degree of restriction as
before the Senate, and in fact, they were appearing in obeisance to the authority of Congress to conduct that which may attach to executive privilege or executive control.
inquiries in aid of legislation.
During the deliberations in Senate, the Court was very well aware of the pendency of this petition as well
Finally, it is stressed in the petition that Gen. Gudani was no longer subject to military jurisdiction on as the issues raised herein. The decision in Senate was rendered with the comfort that the nullification of
account of his compulsory retirement on 4 October 2005. It is pointed out that Article 2, Title I of the portions of E.O. 464 would bear no impact on the present petition since petitioners herein were not called

28
to task for violating the executive order. Moreover, the Court was then cognizant that Senate and this issue entails the examination of the basis and authority of the President to issue such an order in the first
case would ultimately hinge on disparate legal issues. Relevantly, Senate purposely did not touch upon or place to members of the AFP and the determination of whether such an order is subject to any limitations.
rule on the faculty of the President, under the aegis of the commander-in-chief powers26 to require
military officials from securing prior consent before appearing before Congress. The pertinent factors in The vitality of the tenet that the President is the commander-in-chief of the Armed Forces is most crucial
considering that question are markedly outside of those which did become relevant in adjudicating the to the democratic way of life, to civilian supremacy over the military, and to the general stability of our
issues raised in Senate. It is in this petition that those factors come into play. representative system of government. The Constitution reposes final authority, control and supervision of
the AFP to the President, a civilian who is not a member of the armed forces, and whose duties as
At this point, we wish to dispose of another peripheral issue before we strike at the heart of the matter. commander-in-chief represent only a part of the organic duties imposed upon the office, the other
General Gudani argues that he can no longer fall within the jurisdiction of the court-martial, considering functions being clearly civil in nature.31 Civilian supremacy over the military also countermands the notion
his retirement last 4 October 2005. He cites Article 2, Title I of Commonwealth Act No. 408, which defines that the military may bypass civilian authorities, such as civil courts, on matters such as conducting
persons subject to military law as, among others, "all officers and soldiers in the active service of the warrantless searches and seizures.32
[AFP]," and points out that he is no longer in the active service.
Pursuant to the maintenance of civilian supremacy over the military, the Constitution has allocated
This point was settled against Gen. Gudani’s position in Abadilla v. Ramos,27 where the Court declared specific roles to the legislative and executive branches of government in relation to military affairs. Military
that an officer whose name was dropped from the roll of officers cannot be considered to be outside the appropriations, as with all other appropriations, are determined by Congress, as is the power to declare
jurisdiction of military authorities when military justice proceedings were initiated against him before the the existence of a state of war.33 Congress is also empowered to revoke a proclamation of martial law or
termination of his service. Once jurisdiction has been acquired over the officer, it continues until his case the suspension of the writ of habeas corpus.34 The approval of the Commission on Appointments is also
is terminated. Thus, the Court held: required before the President can promote military officers from the rank of colonel or naval
captain.35 Otherwise, on the particulars of civilian dominance and administration over the military, the
The military authorities had jurisdiction over the person of Colonel Abadilla at the time of the alleged Constitution is silent, except for the commander-in-chief clause which is fertile in meaning and
offenses. This jurisdiction having been vested in the military authorities, it is retained up to the end of the
proceedings against Colonel Abadilla. Well-settled is the rule that jurisdiction once acquired is not lost implication as to whatever inherent martial authority the President may possess.36
upon the instance of the parties but continues until the case is terminated.28
The commander-in-chief provision in the Constitution is denominated as Section 18, Article VII, which
Citing Colonel Winthrop’s treatise on Military Law, the Court further stated: begins with the simple declaration that "[t]he President shall be the Commander-in-Chief of all armed
forces of the Philippines x x x"37 Outside explicit constitutional limitations, such as those found in Section
We have gone through the treatise of Colonel Winthrop and We find the following passage which goes 5, Article XVI, the commander-in-chief clause vests on the President, as commander-in-chief, absolute
against the contention of the petitioners, viz — authority over the persons and actions of the members of the armed forces. Such authority includes the
ability of the President to restrict the travel, movement and speech of military officers, activities which may
otherwise be sanctioned under civilian law.
3. Offenders in general — Attaching of jurisdiction. It has further been held, and is now settled law, in
regard to military offenders in general, that if the military jurisdiction has once duly attached to them
previous to the date of the termination of their legal period of service, they may be brought to trial by Reference to Kapunan, Jr. v. De Villa38 is useful in this regard. Lt. Col. Kapunan was ordered confined
court-martial after that date, their discharge being meanwhile withheld. This principle has mostly been under "house arrest" by then Chief of Staff (later President) Gen. Fidel Ramos. Kapunan was also
applied to cases where the offense was committed just prior to the end of the term. In such cases the ordered, as a condition for his house arrest, that he may not issue any press statements or give any press
interests of discipline clearly forbid that the offender should go unpunished. It is held therefore that if conference during his period of detention. The Court unanimously upheld such restrictions, noting:
before the day on which his service legally terminates and his right to a discharge is complete,
proceedings with a view to trial are commenced against him — as by arrest or the service of [T]he Court is of the view that such is justified by the requirements of military discipline. It cannot be
charges, — the military jurisdiction will fully attach and once attached may be continued by a trial gainsaid that certain liberties of persons in the military service, including the freedom of speech,
by court-martial ordered and held after the end of the term of the enlistment of the accused x x may be circumscribed by rules of military discipline. Thus, to a certain degree, individual rights
x 29 may be curtailed, because the effectiveness of the military in fulfilling its duties under the law
depends to a large extent on the maintenance of discipline within its ranks. Hence, lawful orders
Thus, military jurisdiction has fully attached to Gen. Gudani inasmuch as both the acts complained of and must be followed without question and rules must be faithfully complied with, irrespective of a
the initiation of the proceedings against him occurred before he compulsorily retired on 4 October 2005. soldier's personal views on the matter. It is from this viewpoint that the restrictions imposed on
We see no reason to unsettle the Abadilla doctrine. The OSG also points out that under Section 28 of petitioner Kapunan, an officer in the AFP, have to be considered.39
Presidential Decree No. 1638, as amended, "[a]n officer or enlisted man carried in the retired list [of the
Armed Forces of the Philippines] shall be subject to the Articles of War x x x"30 To this citation, petitioners Any good soldier, or indeed any ROTC cadet, can attest to the fact that the military way of life
do not offer any response, and in fact have excluded the matter of Gen. Gudani’s retirement as an issue circumscribes several of the cherished freedoms of civilian life. It is part and parcel of the military
in their subsequent memorandum. package. Those who cannot abide by these limitations normally do not pursue a military career and
instead find satisfaction in other fields; and in fact many of those discharged from the service are inspired
IV. in their later careers precisely by their rebellion against the regimentation of military life. Inability or
unwillingness to cope with military discipline is not a stain on character, for the military mode is a highly
idiosyncratic path which persons are not generally conscripted into, but volunteer themselves to be part
We now turn to the central issues. of. But for those who do make the choice to be a soldier, significant concessions to personal freedoms are
expected. After all, if need be, the men and women of the armed forces may be commanded upon to die
Petitioners wish to see annulled the "gag order" that required them to secure presidential consent prior to for country, even against their personal inclinations.
their appearance before the Senate, claiming that it violates the constitutional right to information and
transparency in matters of public concern; or if not, is tantamount at least to the criminal acts of It may be so that military culture is a remnant of a less democratic era, yet it has been fully integrated into
obstruction of justice and grave coercion. However, the proper perspective from which to consider this the democratic system of governance. The constitutional role of the armed forces is as protector of the
people and of the State.40 Towards this end, the military must insist upon a respect for duty and a

29
discipline without counterpart in civilian life.41 The laws and traditions governing that discipline have a birth of his first-born, or to attend the funeral of a parent. Yet again, military life calls for considerable
long history; but they are founded on unique military exigencies as powerful now as in the past.42 In the personal sacrifices during the period of conscription, wherein the higher duty is not to self but to country.
end, it must be borne in mind that the armed forces has a distinct subculture with unique needs, a
specialized society separate from civilian society. 43 In the elegant prose of the eminent British military Indeed, the military practice is to require a soldier to obtain permission from the commanding officer
historian, John Keegan: before he/she may leave his destination. A soldier who goes from the properly appointed place of duty or
absents from his/her command, guard, quarters, station, or camp without proper leave is subject to
[Warriors who fight wars have] values and skills [which] are not those of politicians and diplomats. They punishment by court-martial.48 It is even clear from the record that petitioners had actually requested for
are those of a world apart, a very ancient world, which exists in parallel with the everyday world but does travel authority from the PMA in Baguio City to Manila, to attend the Senate Hearing.49 Even petitioners
not belong to it. Both worlds change over time, and the warrior world adopts in step to the civilian. It are well aware that it was necessary for them to obtain permission from their superiors before they could
follows it, however, at a distance. The distance can never be closed, for the culture of the warrior can travel to Manila to attend the Senate Hearing.
never be that of civilization itself….44
It is clear that the basic position of petitioners impinges on these fundamental principles we have
Critical to military discipline is obeisance to the military chain of command. Willful disobedience of a discussed. They seek to be exempted from military justice for having traveled to the Senate to testify
superior officer is punishable by court-martial under Article 65 of the Articles of War.45 "An individual before the Senate Committee against the express orders of Gen. Senga, the AFP Chief of Staff. If
soldier is not free to ignore the lawful orders or duties assigned by his immediate superiors. For there petitioners’ position is affirmed, a considerable exception would be carved from the unimpeachable right
would be an end of all discipline if the seaman and marines on board a ship of war [or soldiers deployed in of military officers to restrict the speech and movement of their juniors. The ruinous consequences to the
the field], on a distant service, were permitted to act upon their own opinion of their rights [or their opinion chain of command and military discipline simply cannot warrant the Court’s imprimatur on petitioner’s
of the position.

President’s intent], and to throw off the authority of the commander whenever they supposed it to be V.
unlawfully exercised."46
Still, it would be highly myopic on our part to resolve the issue solely on generalities surrounding military
Further traditional restrictions on members of the armed forces are those imposed on free speech and discipline. After all, petitioners seek to impress on us that their acts are justified as they were responding
mobility.1âwphi1 Kapunan is ample precedent in justifying that a soldier may be restrained by a superior to an invitation from the Philippine Senate, a component of the legislative branch of government. At the
officer from speaking out on certain matters. As a general rule, the discretion of a military officer to same time, the order for them not to testify ultimately came from the President, the head of the executive
restrain the speech of a soldier under his/her command will be accorded deference, with minimal regard if branch of government and the commander-in-chief of the armed forces.
at all to the reason for such restraint. It is integral to military discipline that the soldier’s speech be with the
consent and approval of the military commander. Thus, we have to consider the question: may the President prevent a member of the armed forces from
testifying before a legislative inquiry? We hold that the President has constitutional authority to do so, by
The necessity of upholding the ability to restrain speech becomes even more imperative if the soldier virtue of her power as commander-in-chief, and that as a consequence a military officer who defies such
desires to speak freely on political matters. The Constitution requires that "[t]he armed forces shall be injunction is liable under military justice. At the same time, we also hold that any chamber of Congress
insulated from partisan politics," and that ‘[n]o member of the military shall engage directly or indirectly in which seeks the appearance before it of a military officer against the consent of the President has
any partisan political activity, except to vote."47 Certainly, no constitutional provision or military adequate remedies under law to compel such attendance. Any military official whom Congress summons
indoctrination will eliminate a soldier’s ability to form a personal political opinion, yet it is vital that such to testify before it may be compelled to do so by the President. If the President is not so inclined, the
opinions be kept out of the public eye. For one, political belief is a potential source of discord among President may be commanded by judicial order to compel the attendance of the military officer. Final
people, and a military torn by political strife is incapable of fulfilling its constitutional function as protectors judicial orders have the force of the law of the land which the President has the duty to faithfully
of the people and of the State. For another, it is ruinous to military discipline to foment an atmosphere that execute.50
promotes an active dislike of or dissent against the President, the commander-in-chief of the armed
forces. Soldiers are constitutionally obliged to obey a President they may dislike or distrust. This Explication of these principles is in order.
fundamental principle averts the country from going the way of banana republics.
As earlier noted, we ruled in Senate that the President may not issue a blanket requirement of prior
Parenthetically, it must be said that the Court is well aware that our country’s recent past is marked by consent on executive officials summoned by the legislature to attend a congressional hearing. In doing so,
regime changes wherein active military dissent from the chain of command formed a key, though not the Court recognized the considerable limitations on executive privilege, and affirmed that the privilege
exclusive, element. The Court is not blind to history, yet it is a judge not of history but of the Constitution. must be formally invoked on specified grounds. However, the ability of the President to prevent
The Constitution, and indeed our modern democratic order, frown in no uncertain terms on a politicized military officers from testifying before Congress does not turn on executive privilege, but on the
military, informed as they are on the trauma of absolute martial rule. Our history might imply that a political Chief Executive’s power as commander-in-chief to control the actions and speech of members of
military is part of the natural order, but this view cannot be affirmed by the legal order. The evolutionary the armed forces. The President’s prerogatives as commander-in-chief are not hampered by the
path of our young democracy necessitates a reorientation from this view, reliant as our socio-political same limitations as in executive privilege.
culture has become on it. At the same time, evolution mandates a similar demand that our system of
governance be more responsive to the needs and aspirations of the citizenry, so as to avoid an
environment vulnerable to a military apparatus able at will to exert an undue influence in our polity. Our ruling that the President could, as a general rule, require military officers to seek presidential approval
before appearing before Congress is based foremost on the notion that a contrary rule unduly diminishes
the prerogatives of the President as commander-in-chief. Congress holds significant control over the
Of possibly less gravitas, but of equal importance, is the principle that mobility of travel is another armed forces in matters such as budget appropriations and the approval of higher-rank promotions,51 yet
necessary restriction on members of the military. A soldier cannot leave his/her post without the consent it is on the President that the Constitution vests the title as commander-in-chief and all the prerogatives
of the commanding officer. The reasons are self-evident. The commanding officer has to be aware at all and functions appertaining to the position. Again, the exigencies of military discipline and the chain of
times of the location of the troops under command, so as to be able to appropriately respond to any command mandate that the President’s ability to control the individual members of the armed forces be
exigencies. For the same reason, commanding officers have to be able to restrict the movement or travel accorded the utmost respect. Where a military officer is torn between obeying the President and obeying
of their soldiers, if in their judgment, their presence at place of call of duty is necessary. At times, this may the Senate, the Court will without hesitation affirm that the officer has to choose the President. After all,
lead to unsentimental, painful consequences, such as a soldier being denied permission to witness the

30
the Constitution prescribes that it is the President, and not the Senate, who is the commander-in-chief of Section 21, Article VI likewise establishes critical safeguards that proscribe the legislative power of
the armed forces.52 inquiry. The provision requires that the inquiry be done in accordance with the Senate or House’s duly
published rules of procedure, necessarily implying the constitutional infirmity of an inquiry conducted
At the same time, the refusal of the President to allow members of the military to appear before Congress without duly published rules of procedure. Section 21 also mandates that the rights of persons appearing
is still subject to judicial relief. The Constitution itself recognizes as one of the legislature’s functions is the in or affected by such inquiries be respected, an imposition that obligates Congress to adhere to the
conduct of inquiries in aid of legislation.53 Inasmuch as it is ill-advised for Congress to interfere with the guarantees in the Bill of Rights.
President’s power as commander-in-chief, it is similarly detrimental for the President to unduly interfere
with Congress’s right to conduct legislative inquiries. The impasse did not come to pass in this petition, These abuses are, of course, remediable before the courts, upon the proper suit filed by the persons
since petitioners testified anyway despite the presidential prohibition. Yet the Court is aware that with its affected, even if they belong to the executive branch. Nonetheless, there may be exceptional
pronouncement today that the President has the right to require prior consent from members of the armed circumstances… wherein a clear pattern of abuse of the legislative power of inquiry might be established,
forces, the clash may soon loom or actualize. resulting in palpable violations of the rights guaranteed to members of the executive department under the
Bill of Rights. In such instances, depending on the particulars of each case, attempts by the Executive
We believe and hold that our constitutional and legal order sanctions a modality by which members of the Branch to forestall these abuses may be accorded judicial sanction59 .
military may be compelled to attend legislative inquiries even if the President desires otherwise, a
modality which does not offend the Chief Executive’s prerogatives as commander-in-chief. The remedy In Senate, the Court ruled that the President could not impose a blanket prohibition barring executive
lies with the courts. officials from testifying before Congress without the President’s consent notwithstanding the invocation of
executive privilege to justify such prohibition. The Court did not rule that the power to conduct legislative
The fact that the executive branch is an equal, coordinate branch of government to the legislative creates inquiry ipso facto superseded the claim of executive privilege, acknowledging instead that the viability of
a wrinkle to any basic rule that persons summoned to testify before Congress must do so. There is executive privilege stood on a case to case basis. Should neither branch yield to the other branch’s
considerable interplay between the legislative and executive branches, informed by due deference and assertion, the constitutional recourse is to the courts, as the final arbiter if the dispute. It is only the courts
respect as to their various constitutional functions. Reciprocal courtesy idealizes this relationship; hence, it that can compel, with conclusiveness, attendance or non-attendance in legislative inquiries.
is only as a last resort that one branch seeks to compel the other to a particular mode of behavior. The
judiciary, the third coordinate branch of government, does not enjoy a similar dynamic with either the Following these principles, it is clear that if the President or the Chief of Staff refuses to allow a member of
legislative or executive branches. Whatever weakness inheres on judicial power due to its inability to the AFP to appear before Congress, the legislative body seeking such testimony may seek judicial relief
originate national policies and legislation, such is balanced by the fact that it is the branch empowered by to compel the attendance. Such judicial action should be directed at the heads of the executive branch or
the Constitution to compel obeisance to its rulings by the other branches of government. the armed forces, the persons who wield authority and control over the actions of the officers concerned.
The legislative purpose of such testimony, as well as any defenses against the same — whether
As evidenced by Arnault v. Nazareno54 and Bengzon v. Senate Blue Ribbon Committee,55 among grounded on executive privilege, national security or similar concerns — would be accorded due judicial
others, the Court has not shirked from reviewing the exercise by Congress of its power of legislative evaluation. All the constitutional considerations pertinent to either branch of government may be raised,
inquiry.56 Arnault recognized that the legislative power of inquiry and the process to enforce it, "is an assessed, and ultimately weighed against each other. And once the courts speak with finality, both
essential and appropriate auxiliary to the legislative function."57 On the other branches of government have no option but to comply with the decision of the courts, whether the effect of
hand, Bengzon acknowledged that the power of both houses of Congress to conduct inquiries in aid of the decision is to their liking or disfavor.
legislation is not "absolute or unlimited", and its exercise is circumscribed by Section 21, Article VI of the
Constitution.58 From these premises, the Court enjoined the Senate Blue Ribbon Committee from Courts are empowered, under the constitutional principle of judicial review, to arbitrate disputes between
requiring the petitioners in Bengzon from testifying and producing evidence before the committee, holding the legislative and executive branches of government on the proper constitutional parameters of
that the inquiry in question did not involve any intended legislation. power.60 This is the fair and workable solution implicit in the constitutional allocation of powers among the
three branches of government. The judicial filter helps assure that the particularities of each case would
Senate affirmed both the Arnault and Bengzon rulings. It elucidated on the constitutional scope and ultimately govern, rather than any overarching principle unduly inclined towards one branch of
limitations on the constitutional power of congressional inquiry. Thus: government at the expense of the other. The procedure may not move as expeditiously as some may
desire, yet it ensures thorough deliberation of all relevant and cognizable issues before one branch is
compelled to yield to the other. Moreover, judicial review does not preclude the legislative and executive
As discussed in Arnault, the power of inquiry, "with process to enforce it," is grounded on the necessity of branches from negotiating a mutually acceptable solution to the impasse. After all, the two branches,
information in the legislative process. If the information possessed by executive officials on the operation exercising as they do functions and responsibilities that are political in nature, are free to smooth over the
of their offices is necessary for wise legislation on that subject, by parity of reasoning, Congress has the thorns in their relationship with a salve of their own choosing.
right to that information and the power to compel the disclosure thereof.
And if emphasis be needed, if the courts so rule, the duty falls on the shoulders of the President,
As evidenced by the American experience during the so-called "McCarthy era", however, the right of as commander-in-chief, to authorize the appearance of the military officers before Congress. Even
Congress to conduct inquirites in aid of legislation is, in theory, no less susceptible to abuse than if the President has earlier disagreed with the notion of officers appearing before the legislature to
executive or judicial power. It may thus be subjected to judicial review pursuant to the Court’s certiorari testify, the Chief Executive is nonetheless obliged to comply with the final orders of the courts.
powers under Section 1, Article VIII of the Constitution.
Petitioners have presented several issues relating to the tenability or wisdom of the President’s order on
For one, as noted in Bengzon v. Senate Blue Ribbon Committee, the inquiry itself might not properly be in them and other military officers not to testify before Congress without the President’s consent. Yet these
aid of legislation, and thus beyond the constitutional power of Congress. Such inquiry could not usurp issues ultimately detract from the main point — that they testified before the Senate despite an order from
judicial functions. Parenthetically, one possible way for Congress to avoid such result as occurred their commanding officer and their commander-in-chief for them not to do so,61 in contravention of the
in Bengzon is to indicate in its invitations to the public officials concerned, or to any person for that matter, traditions of military discipline which we affirm today.1âwphi1 The issues raised by petitioners could have
the possible needed statute which prompted the need for the inquiry. Given such statement in its very well been raised and properly adjudicated if the proper procedure was observed. Petitioners could
invitations, along with the usual indication of the subject of inquiry and the questions relative to and in have been appropriately allowed to testify before the Senate without having to countermand their
furtherance thereof, there would be less room for speculation on the part of the person invited on whether Commander-in-chief and superior officer under the setup we have prescribed.
the inquiry is in aid of legislation.

31
We consider the other issues raised by petitioners unnecessary to the resolution of this petition.Petitioners value of the government's equity position in these corporations from any abuses of power done by their
may have been of the honest belief that they were defying a direct order of their Commander-in-Chief and respective board of directors;
Commanding General in obeisance to a paramount idea formed within their consciences, which could not
be lightly ignored. Still, the Court, in turn, is guided by the superlative principle that is the Constitution, the WHEREFORE, be it resolved that the proper Senate Committee shall conduct an inquiry in aid of
embodiment of the national conscience. The Constitution simply does not permit the infraction which legislation, on the anomalous losses incurred by the Philippine Overseas Telecommunications
petitioners have allegedly committed, and moreover, provides for an orderly manner by which the same Corporation (POTC), Philippine Communications Satellite Corporation (PHILCOMSAT), and
result could have been achieved without offending constitutional principles. Philcomsat Holdings Corporations (PHC) due to the alleged improprieties in the operations by
their respective board of directors.
WHEREFORE, the petition is DENIED. No pronouncement as to costs.SO ORDERED.
Adopted. (Sgd) MIRIAM DEFENSOR SANTIAGO
G.R. No. 174340             October 17, 2006
On the same date, February 20, 2006, Senate Res. No. 455 was submitted to the Senate and referred to
IN THE MATTER OF THE PETITION FOR ISSUANCE OF WRIT OF HABEAS CORPUS OF CAMILO L. the Committee on Accountability of Public Officers and Investigations and Committee on Public Services.
SABIO,  However, on March 28, 2006, upon motion of Senator Francis N. Pangilinan, it was transferred to
the Committee on Government Corporations and Public Enterprises.5
Two decades ago, on February 28, 1986, former President Corazon C. Aquino installed her regime by
issuing Executive Order (E.O.) No. 1,1 creating the Presidential Commission on Good Government On May 8, 2006, Chief of Staff Rio C. Inocencio, under the authority of Senator Richard J. Gordon, wrote
(PCGG). She entrusted upon this Commission the herculean task of recovering the ill-gotten wealth Chairman Camilo L. Sabio of the PCGG, one of the herein petitioners, inviting him to be one of the
accumulated by the deposed President Ferdinand E. Marcos, his family, relatives, subordinates and close resource persons in the public meeting jointly conducted by the Committee on Government
associates.2 Section 4 (b) of E.O. No. 1 provides that: "No member or staff of the Commission shall be Corporations and Public Enterprises and Committee on Public Services. The purpose of the public
required to testify or produce evidence in any judicial, legislative or administrative proceeding meeting was to deliberate on Senate Res. No. 455.6
concerning matters within its official cognizance." Apparently, the purpose is to ensure PCGG's
unhampered performance of its task.3 On May 9, 2006, Chairman Sabio declined the invitation because of prior commitment.7 At the same
time, he invoked Section 4(b) of E.O. No. 1 earlier quoted.
Today, the constitutionality of Section 4(b) is being questioned on the ground that it tramples upon the
Senate's power to conduct legislative inquiry under Article VI, Section 21 of the 1987 Constitution, which On August 10, 2006, Senator Gordon issued a Subpoena Ad Testificandum,8 approved by Senate
reads: President Manuel Villar, requiring Chairman Sabio and PCGG Commissioners Ricardo Abcede, Nicasio
Conti, Tereso Javier and Narciso Nario to appear in the public hearing scheduled on August 23, 2006
The Senate or the House of Representatives or any of its respective committees may conduct inquiries in and testify on what they know relative to the matters specified in Senate Res. No. 455. Similar subpoenae
aid of legislation in accordance with its duly published rules of procedure. The rights of persons appearing were issued against the directors and officers of Philcomsat Holdings Corporation, namely: Benito V.
in or affected by such inquiries shall be respected. Araneta, Philip J. Brodett, Enrique L. Locsin, Manuel D. Andal, Roberto L. Abad, Luis K. Lokin, Jr., Julio J.
Jalandoni, Roberto V. San Jose, Delfin P. Angcao, Alma Kristina Alloba and Johnny Tan.9
The facts are undisputed.
Again, Chairman Sabio refused to appear. In his letter to Senator Gordon dated August 18, 2006, he
On February 20, 2006, Senator Miriam Defensor Santiago introduced Philippine Senate Resolution No. reiterated his earlier position, invoking Section 4(b) of E.O. No. 1. On the other hand, the directors and
455 (Senate Res. No. 455),4 "directing an inquiry in aid of legislation on the anomalous losses incurred by officers of Philcomsat Holdings Corporation relied on the position paper they previously filed, which raised
the Philippines Overseas Telecommunications Corporation (POTC), Philippine Communications Satellite issues on the propriety of legislative inquiry.
Corporation (PHILCOMSAT), and PHILCOMSAT Holdings Corporation (PHC) due to the alleged
improprieties in their operations by their respective Board of Directors." Thereafter, Chief of Staff Ma. Carissa O. Coscolluela, under the authority of Senator Gordon, sent another
notice10 to Chairman Sabio requiring him to appear and testify on the same subject matter set on
The pertinent portions of the Resolution read: September 6, 2006. The notice was issued "under the same authority of the Subpoena Ad
Testificandum previously served upon (him) last 16 August 2006."
WHEREAS, in the last quarter of 2005, the representation and entertainment expense of the PHC
skyrocketed to P4.3 million, as compared to the previous year's mere P106 thousand; Once more, Chairman Sabio did not comply with the notice. He sent a letter11 dated September 4, 2006
to Senator Gordon reiterating his reason for declining to appear in the public hearing.
WHEREAS, some board members established wholly owned PHC subsidiary called Telecommunications
Center, Inc. (TCI), where PHC funds are allegedly siphoned; in 18 months, over P73 million had been This prompted Senator Gordon to issue an Order dated September 7, 2006 requiring Chairman Sabio and
allegedly advanced to TCI without any accountability report given to PHC and PHILCOMSAT; Commissioners Abcede, Conti, Javier and Nario to show cause why they should not be cited in contempt
of the Senate. On September 11, 2006, they submitted to the Senate their Compliance and
Explanation,12 which partly reads:
WHEREAS, the Philippine Star, in its 12 February 2002 issue reported that the executive committee of
Philcomsat has precipitately released P265 million and granted P125 million loan to a relative of an
executive committee member; to date there have been no payments given, subjecting the company to an Doubtless, there are laudable intentions of the subject inquiry in aid of legislation. But the rule of
estimated interest income loss of P11.25 million in 2004; law requires that even the best intentions must be carried out within the parameters of the Constitution
and the law. Verily, laudable purposes must be carried out by legal methods. (Brillantes, Jr., et al. v.
Commission on Elections, En Banc [G.R. No. 163193, June 15, 2004])
WHEREAS, there is an urgent need to protect the interest of the Republic of the Philippines in the PHC,
PHILCOMSAT, and POTC from any anomalous transaction, and to conserve or salvage any remaining
On this score, Section 4(b) of E.O. No. 1 should not be ignored as it explicitly provides:

32
No member or staff of the Commission shall be required to testify or produce evidence in any Hence, Chairman Sabio filed with this Court a petition for habeas corpus against the Senate Committee
judicial legislative or administrative proceeding concerning matters within its official cognizance. on Government Corporations and Public Enterprises and Committee on Public Services, their Chairmen,
Senators Richard Gordon and Joker P. Arroyo and Members. The case was docketed as G.R. No.
With all due respect, Section 4(b) of E.O. No. 1 constitutes a limitation on the power of legislative inquiry, 174340.
and a recognition by the State of the need to provide protection to the PCGG in order to ensure the
unhampered performance of its duties under its charter. E.O. No. 1 is a law, Section 4(b) of which had not Chairman Sabio, Commissioners Abcede, Conti, Nario, and Javier, and the PCGG's nominees to
been amended, repealed or revised in any way. Philcomsat Holdings Corporation, Manuel Andal and Julio Jalandoni, likewise filed a petition for certiorari
and prohibition against the same respondents, and also against Senate President Manuel Villar, Senator
To say the least, it would require both Houses of Congress and Presidential fiat to amend or repeal the Juan Ponce Enrile, the Sergeant-at-Arms, and the entire Senate. The case was docketed as G.R. No.
provision in controversy. Until then, it stands to be respected as part of the legal system in this jurisdiction. 174318.
(As held in People v. Veneracion, G.R. Nos. 119987-88, October 12, 1995: Obedience to the rule of law
forms the bedrock of our system of justice. If judges, under the guise of religious or political beliefs were Meanwhile, Philcomsat Holdings Corporation and its officers and directors, namely: Philip G. Brodett, Luis
allowed to roam unrestricted beyond boundaries within which they are required by law to exercise the K. Lokin, Jr., Roberto V. San Jose, Delfin P. Angcao, Roberto L. Abad, Alma Kristina Alobba and Johnny
duties of their office, then law becomes meaningless. A government of laws, not of men excludes the Tan filed a petition for certiorari and prohibition against the Senate Committees on Government
exercise of broad discretionary powers by those acting under its authority. Under this system, judges are Corporations and Public Enterprises and Public Services, their Chairmen, Senators Gordon and Arroyo,
guided by the Rule of Law, and ought to 'protect and enforce it without fear or favor,' 4 [Act of Athens and Members. The case was docketed as G.R. No. 174177.
(1955)] resist encroachments by governments, political parties, or even the interference of their own
personal beliefs.) In G.R. No. 174340 (for habeas corpus) and G.R. No. 174318 (for certiorari and prohibition) Chairman
Sabio, Commissioners Abcede, Conti, Nario, and Javier; and the PCGG's nominees Andal and Jalandoni
xxxxxx alleged: first, respondent Senate Committees disregarded Section 4(b) of E.O. No. 1 without any
justifiable reason; second, the inquiries conducted by respondent Senate Committees are not in aid of
Relevantly, Chairman Sabio's letter to Sen. Gordon dated August 19, 2006 pointed out that the legislation; third, the inquiries were conducted in the absence of duly published Senate Rules of
anomalous transactions referred to in the P.S. Resolution No. 455 are subject of pending cases before the Procedure Governing Inquiries in Aid of Legislation; and fourth, respondent Senate Committees are not
regular courts, the Sandiganbayan and the Supreme Court (Pending cases include: a. Samuel Divina v. vested with the power of contempt.
Manuel Nieto, Jr., et al., CA-G.R. No. 89102; b. Philippine Communications Satellite Corporation v.
Manuel Nieto, et al.; c. Philippine Communications Satellite Corporation v. Manuel D. Andal, Civil Case In G.R. No. 174177, petitioners Philcomsat Holdings Corporation and its directors and officers
No. 06-095, RTC, Branch 61, Makati City; d. Philippine Communications Satellite Corporation v. alleged: first, respondent Senate Committees have no jurisdiction over the subject matter stated in Senate
PHILCOMSAT Holdings Corporation, et al., Civil Case No. 04-1049) for which reason they may not be Res. No. 455; second, the same inquiry is not in accordance with the Senate's Rules of Procedure
able to testify thereon under the principle of sub judice. The laudable objectives of the PCGG's functions, Governing Inquiries in Aid of Legislation; third, the subpoenae against the individual petitioners are void
recognized in several cases decided by the Supreme Court, of the PCGG will be put to naught if its for having been issued without authority; fourth, the conduct of legislative inquiry pursuant to Senate Res.
recovery efforts will be unduly impeded by a legislative investigation of cases that are already pending No. 455 constitutes undue encroachment by respondents into justiciable controversies over which several
before the Sandiganbayan and trial courts. courts and tribunals have already acquired jurisdiction; and fifth, the subpoenae violated petitioners' rights
to privacy and against self-incrimination.
In Bengzon v. Senate Blue Ribbon Committee, (203 SCRA 767, 784 [1991]) the Honorable Supreme
Court held: In their Consolidated Comment, the above-named respondents countered: first, the issues raised in the
petitions involve political questions over which this Court has no jurisdiction; second, Section 4(b) has
"…[T]he issues sought to be investigated by the respondent Committee is one over which jurisdiction had been repealed by the Constitution; third, respondent Senate Committees are vested with contempt
been acquired by the Sandiganbayan. In short, the issue has been pre-empted by that court. To allow the power; fourth, Senate's Rules of Procedure Governing Inquiries in Aid of Legislation have been duly
respondent Committee to conduct its own investigation of an issue already before the Sandigabayan published; fifth, respondents have not violated any civil right of the individual petitioners, such as
would not only pose the possibility of conflicting judgments between a legislative committee and a judicial their (a) right to privacy; and (b) right against self-incrimination; and sixth, the inquiry does not constitute
tribunal, but if the Committee's judgment were to be reached before that of the Sandiganbayan, the undue encroachment into justiciable controversies.
possibility of its influence being made to bear on the ultimate judgment of the Sandiganbayan can not be
discounted. During the oral arguments held on September 21, 2006, the parties were directed to submit
simultaneously their respective memoranda within a non-extendible period of fifteen (15) days from date.
IT IS IN VIEW OF THE FOREGOING CONSIDERATIONS that the Commission decided not to attend the In the meantime, per agreement of the parties, petitioner Chairman Sabio was allowed to go home. Thus,
Senate inquiry to testify and produce evidence thereat. his petition for habeas corpus has become moot. The parties also agreed that the service of the arrest
warrants issued against all petitioners and the proceedings before the respondent Senate Committees are
suspended during the pendency of the instant cases.14
Unconvinced with the above Compliance and Explanation, the Committee on Government Corporations
and Public Enterprises and the Committee on Public Services issued an Order13 directing Major General
Jose Balajadia (Ret.), Senate Sergeant-At-Arms, to place Chairman Sabio and his Commissioners under Crucial to the resolution of the present petitions is the fundamental issue of whether Section 4(b) of E.O.
arrest for contempt of the Senate. The Order bears the approval of Senate President Villar and the No. 1 is repealed by the 1987 Constitution. On this lone issue hinges the merit of the contention of
majority of the Committees' members. Chairman Sabio and his Commissioners that their refusal to appear before respondent Senate
Committees is justified. With the resolution of this issue, all the other issues raised by the parties have
become inconsequential.
On September 12, 2006, at around 10:45 a.m., Major General Balajadia arrested Chairman Sabio in his
office at IRC Building, No. 82 EDSA, Mandaluyong City and brought him to the Senate premises where he
was detained. Perched on one arm of the scale of justice is Article VI, Section 21 of the 1987 Constitution granting
respondent Senate Committees the power of legislative inquiry. It reads:

33
The Senate or the House of Representatives or any of its respective committees may conduct It can be said that the Congress' power of inquiry has gained more solid existence and expansive
inquiries in aid of legislation in accordance with its duly published rules of procedure. The rights construal. The Court's high regard to such power is rendered more evident in Senate v. Ermita,21 where it
of persons appearing in or affected by such inquiries shall be respected. categorically ruled that "the power of inquiry is broad enough to cover officials of the executive
branch." Verily, the Court reinforced the doctrine in Arnault that "the operation of government, being a
On the other arm of the scale is Section 4(b) of E.O. No.1 limiting such power of legislative inquiry by legitimate subject for legislation, is a proper subject for investigation" and that "the power of
exempting all PCGG members or staff from testifying in any judicial, legislative or administrative inquiry is co-extensive with the power to legislate."
proceeding, thus:
Considering these jurisprudential instructions, we find Section 4(b) directly repugnant with Article VI,
No member or staff of the Commission shall be required to testify or produce evidence in any Section 21. Section 4(b) exempts the PCGG members and staff from the Congress' power of
judicial, legislative or administrative proceeding concerning matters within its official cognizance. inquiry. This cannot be countenanced. Nowhere in the Constitution is any provision granting such
exemption. The Congress' power of inquiry, being broad, encompasses everything that concerns the
administration of existing laws as well as proposed or possibly needed statutes.22 It even extends "to
To determine whether there exists a clear and unequivocal repugnancy between the two quoted government agencies created by Congress and officers whose positions are within the power of
provisions that warrants a declaration that Section 4(b) has been repealed by the 1987 Constitution, a Congress to regulate or even abolish."23 PCGG belongs to this class.
brief consideration of the Congress' power of inquiry is imperative.
Certainly, a mere provision of law cannot pose a limitation to the broad power of Congress, in the absence
The Congress' power of inquiry has been recognized in foreign jurisdictions long before it reached our of any constitutional basis.
shores through McGrain v. Daugherty,15 cited in Arnault v. Nazareno.16 In those earlier days, American
courts considered the power of inquiry as inherent in the power to legislate. The 1864 case of Briggs v.
MacKellar17 explains the breath and basis of the power, thus: Furthermore, Section 4(b) is also inconsistent with Article XI, Section 1 of the Constitution stating that:
"Public office is a public trust. Public officers and employees must at all times be accountable to the
people, serve them with utmost responsibility, integrity, loyalty, and efficiency, act with patriotism and
Where no constitutional limitation or restriction exists, it is competent for either of the two bodies justice, and lead modest lives."
composing the legislature to do, in their separate capacity, whatever may be essential to enable them to
legislate….It is well-established principle of this parliamentary law, that either house may institute any
investigation having reference to its own organization, the conduct or qualification of its members, its The provision presupposes that since an incumbent of a public office is invested with certain powers and
proceedings, rights, or privileges or any matter affecting the public interest upon which it may be charged with certain duties pertinent to sovereignty, the powers so delegated to the officer are held in
important that it should have exact information, and in respect to which it would be competent for trust for the people and are to be exercised in behalf of the government or of all citizens who may
it to legislate. The right to pass laws, necessarily implies the right to obtain information upon any need the intervention of the officers. Such trust extends to all matters within the range of duties
matter which may become the subject of a law. It is essential to the full and intelligent exercise of pertaining to the office. In other words, public officers are but the servants of the people, and not
the legislative function….In American legislatures the investigation of public matters before their rulers.24
committees, preliminary to legislation, or with the view of advising the house appointing the
committee is, as a parliamentary usage, well established as it is in England, and the right of either Section 4(b), being in the nature of an immunity, is inconsistent with the principle of public
house to compel witnesses to appear and testify before its committee, and to punish for disobedience has accountability. It places the PCGG members and staff beyond the reach of courts, Congress and other
been frequently enforced….The right of inquiry, I think, extends to other matters, in respect to which it administrative bodies. Instead of encouraging public accountability, the same provision only
may be necessary, or may be deemed advisable to apply for legislative aid. institutionalizes irresponsibility and non-accountability. In Presidential Commission on Good Government
v. Peña,25 Justice Florentino P. Feliciano characterized as "obiter" the portion of the majority opinion
Remarkably, in Arnault, this Court adhered to a similar theory. Citing McGrain, it recognized that the barring, on the basis of Sections 4(a) and (b) of E.O. No. 1, a civil case for damages filed against the
power of inquiry is "an essential and appropriate auxiliary to the legislative function," thus: PCGG and its Commissioners. He eloquently opined:

Although there is no provision in the "Constitution expressly investing either House of Congress with The above underscored portions are, it is respectfully submitted, clearly obiter. It is important to make
power to make investigations and exact testimony to the end that it may exercise its legislative functions clear that the Court is not here interpreting, much less upholding as valid and constitutional, the
advisedly and effectively, such power is so far incidental to the legislative function as to be implied. In literal terms of Section 4 (a), (b) of Executive Order No.1. If Section 4 (a) were given its literal import
other words, the power of inquiry – with process to enforce it – is an essential and appropriate as immunizing the PCGG or any member thereof from civil liability "for anything done or omitted in the
auxiliary to the legislative function. A legislative body cannot legislate wisely or effectively in the discharge of the task contemplated by this Order," the constitutionality of Section 4 (a) would, in my
absence of information respecting the conditions which the legislation is intended to affect or submission, be open to most serious doubt. For so viewed, Section 4 (a) would institutionalize the
change; and where the legislation body does not itself possess the requisite information – which irresponsibility and non-accountability of members and staff of the PCGG, a notion that is clearly
is not infrequently true – recourse must be had to others who possess it." repugnant to both the 1973 and 1987 Constitution and a privileged status not claimed by any other official
of the Republic under the 1987 Constitution. x x x.
Dispelling any doubt as to the Philippine Congress' power of inquiry, provisions on such power made their
maiden appearance in Article VIII, Section 12 of the 1973 Constitution.18 Then came the 1987 xxxxxx
Constitution incorporating the present Article VI, Section 12. What was therefore implicit under the 1935
Constitution, as influenced by American jurisprudence, became explicit under the 1973 and 1987 It would seem constitutionally offensive to suppose that a member or staff member of the PCGG
Constitutions.19 could not be required to testify before the Sandiganbayan or that such members were exempted
from complying with orders of this Court.
Notably, the 1987 Constitution recognizes the power of investigation, not just of Congress, but also of
"any of its committee." This is significant because it constitutes a direct conferral of investigatory power Chavez v. Sandiganbayan26 reiterates the same view. Indeed, Section 4(b) has been frowned upon by
upon the committees and it means that the mechanisms which the Houses can take in order to effectively this Court even before the filing of the present petitions.
perform its investigative function are also available to the committees.20

34
Corollarily, Section 4(b) also runs counter to the following constitutional provisions ensuring the people's Significantly, Article XVIII, Section 3 of the Constitution provides:
access to information:
All existing laws, decrees, executive orders, proclamations, letters of instructions, and other executive
Article II, Section 28 issuances not inconsistent with this Constitution shall remain operative until amended, repealed, or
revoked.
Subject to reasonable conditions prescribed by law, the State adopts and implements a policy of full public
disclosure of all its transactions involving public interest. The clear import of this provision is that all existing laws, executive orders, proclamations, letters of
instructions and other executive issuances inconsistent or repugnant to the Constitution are repealed.
Article III, Section 7
Jurisprudence is replete with decisions invalidating laws, decrees, executive orders, proclamations, letters
The right of the people to information on matters of public concern shall be recognized. Access to official of instructions and other executive issuances inconsistent with the Constitution. In Pelaez v. Auditor
records, and to documents, and papers pertaining to official acts, transactions, or decisions, as well as to General,33 the Court considered repealed Section 68 of the Revised Administrative Code of 1917
government research data used as basis for policy development, shall be afforded the citizen, subject to authorizing the Executive to change the seat of the government of any subdivision of local governments,
such limitations as may be provided by law. upon the approval of the 1935 Constitution. Section 68 was adjudged incompatible and inconsistent with
the Constitutional grant of limited executive supervision over local governments. In Islamic Da'wah
Council of the Philippines, Inc., v. Office of the Executive Secretary,34 the Court declared Executive Order
These twin provisions of the Constitution seek to promote transparency in policy-making and in the No. 46, entitled "Authorizing the Office on Muslim Affairs to Undertake Philippine Halal Certification," void
operations of the government, as well as provide the people sufficient information to enable them to for encroaching on the religious freedom of Muslims. In The Province of Batangas v. Romulo,35 the Court
exercise effectively their constitutional rights. Armed with the right information, citizens can participate in declared some provisions of the General Appropriations Acts of 1999, 2000 and 2001 unconstitutional for
public discussions leading to the formulation of government policies and their effective implementation. violating the Constitutional precept on local autonomy. And in Ople v. Torres,36 the Court likewise
In Valmonte v. Belmonte, Jr.27 the Court explained that an informed citizenry is essential to the existence declared unconstitutional Administrative Order No. 308, entitled "Adoption of a National Computerized
and proper functioning of any democracy, thus: Identification Reference System," for being violative of the right to privacy protected by the Constitution.

An essential element of these freedoms is to keep open a continuing dialogue or process of These Decisions, and many others, highlight that the Constitution is the highest law of the land. It is "the
communication between the government and the people. It is in the interest of the State that the channels basic and paramount law to which all other laws must conform and to which all persons, including
for free political discussion be maintained to the end that the government may perceive and be responsive the highest officials of the land, must defer. No act shall be valid, however noble its intentions, if it
to the people's will. Yet, this open dialogue can be effective only to the extent that the citizenry is informed conflicts with the Constitution."37 Consequently, this Court has no recourse but to declare Section 4(b)
and thus able to formulate its will intelligently. Only when the participants in the discussion are aware of of E.O. No. 1 repealed by the 1987 Constitution.
the issues and have access to information relating thereto can such bear fruit.
Significantly, during the oral arguments on September 21, 2006, Chairman Sabio admitted that should this
Consequently, the conduct of inquiries in aid of legislation is not only intended to benefit Congress but Court rule that Section 4(b) is unconstitutional or that it does not apply to the Senate, he will answer the
also the citizenry. The people are equally concerned with this proceeding and have the right to participate questions of the Senators, thus:
therein in order to protect their interests. The extent of their participation will largely depend on the
information gathered and made known to them. In other words, the right to information really goes hand-
in-hand with the constitutional policies of full public disclosure and honesty in the public service. It is CHIEF JUSTICE PANGANIBAN:
meant to enhance the widening role of the citizenry in governmental decision-making as well as in Okay. Now, if the Supreme Court rules that Sec. 4(b) is unconstitutional or that it does not apply to the
checking abuse in the government.28 The cases of Tañada v. Tuvera29 and Legaspi v. Civil Service Senate, will you answer the questions of the Senators?
Commission30 have recognized a citizen's interest and personality to enforce a public duty and to bring CHAIRMAN SABIO:
an action to compel public officials and employees to perform that duty. Your Honor, my father was a judge, died being a judge. I was here in the Supreme Court as Chief of Staff
of Justice Feria. I would definitely honor the Supreme Court and the rule of law.
CHIEF JUSTICE PANGANIBAN:
Section 4(b) limits or obstructs the power of Congress to secure from PCGG members and staff You will answer the questions of the Senators if we say that?
information and other data in aid of its power to legislate. Again, this must not be countenanced. CHAIRMAN SABIO:
In Senate v. Ermita,31 this Court stressed: Yes, Your Honor. That is the law already as far as I am concerned.
With his admission, Chairman Sabio is not fully convinced that he and his Commissioners are shielded
To the extent that investigations in aid of legislation are generally conducted in public, however, any from testifying before respondent Senate Committees by Section 4(b) of E.O. No. 1. In effect, his
executive issuance tending to unduly limit disclosures of information in such investigations argument that the said provision exempts him and his co-respondent Commissioners from testifying
necessarily deprives the people of information which, being presumed to be in aid of legislation, is before respondent Senate Committees concerning Senate Res. No. 455 utterly lacks merit.
presumed to be a matter of public concern. The citizens are thereby denied access to information Incidentally, an argument repeated by Chairman Sabio is that respondent Senate Committees have no
which they can use in formulating their own opinions on the matter before Congress – opinions which they power to punish him and his Commissioners for contempt of the Senate.
can then communicate to their representatives and other government officials through the various legal
means allowed by their freedom of expression. The argument is misleading.
Article VI, Section 21 provides:
A statute may be declared unconstitutional because it is not within the legislative power to enact; or it The Senate or the House of Representatives or any of its respective committees may conduct
creates or establishes methods or forms that infringe constitutional principles; or its purpose or effect inquiries in aid of legislation in accordance with its duly published rules of procedure. The rights
violates the Constitution or its basic principles.32 As shown in the above discussion, Section 4(b) is of persons appearing in or affected by such inquiries shall be respected.
inconsistent with Article VI, Section 21 (Congress' power of inquiry), Article XI, Section 1 (principle of
public accountability), Article II, Section 28 (policy of full disclosure) and Article III, Section 7 (right to It must be stressed that the Order of Arrest for "contempt of Senate Committees and the Philippine
public information). Senate" was approved by Senate President Villar and signed by fifteen (15) Senators. From this, it

35
can be concluded that the Order is under the authority, not only of the respondent Senate Committees, authority and punishes contempts thereof. The contempt power of the legislature is, therefore, sui
but of the entire Senate. generis x x x.

At any rate, Article VI, Section 21 grants the power of inquiry not only to the Senate and the House of Meanwhile, with respect to G.R. No. 174177, the petition of Philcomsat Holdings Corporation and its
Representatives, but also to any of their respective committees. Clearly, there is a direct conferral of directors and officers, this Court holds that the respondent Senate Committees' inquiry does not violate
power to the committees. Father Bernas, in his Commentary on the 1987 Constitution, correctly pointed their right to privacy and right against self-incrimination.
out its significance:
One important limitation on the Congress' power of inquiry is that "the rights of persons appearing in or
It should also be noted that the Constitution explicitly recognizes the power of investigation not just of affected by such inquiries shall be respected." This is just another way of saying that the power of
Congress but also of "any of its committees." This is significant because it constitutes a direct inquiry must be "subject to the limitations placed by the Constitution on government action." As held
conferral of investigatory power upon the committees and it means that the means which the in Barenblatt v. United States,45 "the Congress, in common with all the other branches of the
Houses can take in order to effectively perform its investigative function are also available to the Government, must exercise its powers subject to the limitations placed by the Constitution on
Committees.38 governmental action, more particularly in the context of this case, the relevant limitations of the
Bill of Rights."
This is a reasonable conclusion. The conferral of the legislative power of inquiry upon any committee of
Congress must carry with it all powers necessary and proper for its effective discharge. Otherwise, Article First is the right to privacy.
VI, Section 21 will be meaningless. The indispensability and usefulness of the power of contempt in a
legislative inquiry is underscored in a catena of cases, foreign and local. Zones of privacy are recognized and protected in our laws.46 Within these zones, any form of intrusion is
impermissible unless excused by law and in accordance with customary legal process. The meticulous
In the 1821 case of Anderson v. Dunn,39 the function of the Houses of Congress with respect to the regard we accord to these zones arises not only from our conviction that the right to privacy is a
contempt power was likened to that of a court, thus: "constitutional right" and "the right most valued by civilized men,"47 but also from our adherence to the
Universal Declaration of Human Rights which mandates that, "no one shall be subjected to arbitrary
…But the court in its reasoning goes beyond this, and though the grounds of the decision are not very interference with his privacy" and "everyone has the right to the protection of the law against such
clearly stated, we take them to be: that there is in some cases a power in each House of Congress to interference or attacks."48
punish for contempt; that this power is analogous to that exercised by courts of justice, and that it
being the well established doctrine that when it appears that a prisoner is held under the order of Our Bill of Rights, enshrined in Article III of the Constitution, provides at least two guarantees that
a court of general jurisdiction for a contempt of its authority, no other court will discharge the explicitly create zones of privacy. It highlights a person's "right to be let alone" or the "right to determine
prisoner or make further inquiry into the cause of his commitment. That this is the general rule…as what, how much, to whom and when information about himself shall be disclosed."49 Section
regards the relation of one court to another must be conceded. 2 guarantees "the right of the people to be secure in their persons, houses, papers and effects
against unreasonable searches and seizures of whatever nature and for any purpose." Section
In McGrain,40 the U.S. Supreme Court held: "Experience has shown that mere requests for such 3 renders inviolable the "privacy of communication and correspondence" and further cautions that
information are often unavailing, and also that information which is volunteered is not always "any evidence obtained in violation of this or the preceding section shall be inadmissible for any
accurate or complete; so some means of compulsion is essential to obtain what is needed." purpose in any proceeding."
The Court, in Arnault v. Nazareno,41 sustained the Congress' power of contempt on the basis of this
observation. In evaluating a claim for violation of the right to privacy, a court must determine whether a person has
exhibited a reasonable expectation of privacy and, if so, whether that expectation has been violated by
In Arnault v. Balagtas,42 the Court further explained that the contempt power of Congress is founded unreasonable government intrusion.50 Applying this determination to these cases, the important inquiries
upon reason and policy and that the power of inquiry will not be complete if for every contumacious act, are: first, did the directors and officers of Philcomsat Holdings Corporation exhibit a reasonable
Congress has to resort to judicial interference, thus: expectation of privacy?; and second, did the government violate such expectation?

The principle that Congress or any of its bodies has the power to punish recalcitrant witnesses is The answers are in the negative. Petitioners were invited in the Senate's public hearing to deliberate on
founded upon reason and policy. Said power must be considered implied or incidental to the exercise of Senate Res. No. 455, particularly "on the anomalous losses incurred by the Philippine Overseas
legislative power. How could a legislative body obtain the knowledge and information on which to Telecommunications Corporation (POTC), Philippine Communications Satellite Corporation
base intended legislation if it cannot require and compel the disclosure of such knowledge and (PHILCOMSAT), and Philcomsat Holdings Corporations (PHC) due to the alleged improprieties in
information if it is impotent to punish a defiance of its power and authority? When the framers of the operations by their respective board of directors." Obviously, the inquiry focus on petitioners' acts
the Constitution adopted the principle of separation of powers, making each branch supreme committed in the discharge of their duties as officers and directors of the said corporations, particularly
within the realm of its respective authority, it must have intended each department's authority to Philcomsat Holdings Corporation. Consequently, they have no reasonable expectation of privacy
be full and complete, independently of the other's authority or power. And how could the authority over matters involving their offices in a corporation where the government has interest. Certainly,
and power become complete if for every act of refusal, every act of defiance, every act of such matters are of public concern and over which the people have the right to information.
contumacy against it, the legislative body must resort to the judicial department for the
appropriate remedy, because it is impotent by itself to punish or deal therewith, with the affronts This goes to show that the right to privacy is not absolute where there is an overriding compelling state
committed against its authority or dignity.43 interest. In Morfe v. Mutuc,51 the Court, in line with Whalen v. Roe,52 employed the rational basis
relationship test when it held that there was no infringement of the individual's right to privacy as the
In Negros Oriental II Electric Cooperative, Inc. v. Sangguniang Panlungsod of Dumaguete,44 the Court requirement to disclosure information is for a valid purpose, i.e., to curtail and minimize the opportunities
characterized contempt power as a matter of self-preservation, thus: for official corruption, maintain a standard of honesty in public service, and promote morality in public
administration.53 In Valmonte v. Belmonte,54 the Court remarked that as public figures, the Members of
the former Batasang Pambansa enjoy a more limited right to privacy as compared to ordinary
The exercise by the legislature of the contempt power is a matter of self-preservation as that branch of individuals, and their actions are subject to closer scrutiny. Taking this into consideration, the Court ruled
the government vested with the legislative power, independently of the judicial branch, asserts its

36
that the right of the people to access information on matters of public concern prevails over the right to Section 4(b) of E.O. No. 1 is declared REPEALED by the 1987 Constitution. Respondent Senate
privacy of financial transactions. Committees' power of inquiry relative to Senate Resolution 455 is upheld. PCGG Chairman Camilo L.
Sabio and Commissioners Ricardo Abcede, Narciso Nario, Nicasio Conti and Tereso Javier; and Manuel
Under the present circumstances, the alleged anomalies in the PHILCOMSAT, PHC and POTC, ranging Andal and Julio Jalandoni, PCGG's nominees to Philcomsat Holdings Corporation, as well as its directors
in millions of pesos, and the conspiratorial participation of the PCGG and its officials are compelling and officers, petitioners in G.R. No. 174177, are ordered to comply with the Subpoenae Ad
reasons for the Senate to exact vital information from the directors and officers of Philcomsat Holdings Testificandum issued by respondent Senate Committees directing them to appear and testify in public
Corporations, as well as from Chairman Sabio and his Commissioners to aid it in crafting the necessary hearings relative to Senate Resolution No. 455. SO ORDERED.
legislation to prevent corruption and formulate remedial measures and policy determination regarding
PCGG's efficacy. There being no reasonable expectation of privacy on the part of those directors and G.R. No. 169777*             April 20, 2006
officers over the subject covered by Senate Res. No. 455, it follows that their right to privacy has not been
violated by respondent Senate Committees. SENATE OF THE PHILIPPINES, vs. EDUARDO R. ERMITA

Anent the right against self-incrimination, it must be emphasized that this right maybe invoked by the said A transparent government is one of the hallmarks of a truly republican state. Even in the early history of
directors and officers of Philcomsat Holdings Corporation only when the incriminating question is republican thought, however, it has been recognized that the head of government may keep certain
being asked, since they have no way of knowing in advance the nature or effect of the questions information confidential in pursuit of the public interest. Explaining the reason for vesting executive power
to be asked of them."55 That this right may possibly be violated or abused is no ground for denying in only one magistrate, a distinguished delegate to the U.S. Constitutional Convention said: "Decision,
respondent Senate Committees their power of inquiry. The consolation is that when this power is abused, activity, secrecy, and dispatch will generally characterize the proceedings of one man, in a much more
such issue may be presented before the courts. At this juncture, what is important is that respondent eminent degree than the proceedings of any greater number; and in proportion as the number is
Senate Committees have sufficient Rules to guide them when the right against self-incrimination is increased, these qualities will be diminished."1
invoked. Sec. 19 reads:
History has been witness, however, to the fact that the power to withhold information lends itself to abuse,
Sec. 19. Privilege Against Self-Incrimination hence, the necessity to guard it zealously.

A witness can invoke his right against self-incrimination only when a question tends to elicit an answer The present consolidated petitions for certiorari and prohibition proffer that the President has abused such
that will incriminate him is propounded to him. However, he may offer to answer any question in an power by issuing Executive Order No. 464 (E.O. 464) last September 28, 2005. They thus pray for its
executive session. declaration as null and void for being unconstitutional.

No person can refuse to testify or be placed under oath or affirmation or answer questions before an In resolving the controversy, this Court shall proceed with the recognition that the issuance under review
incriminatory question is asked. His invocation of such right does not by itself excuse him from his duty to has come from a co-equal branch of government, which thus entitles it to a strong presumption of
give testimony. constitutionality. Once the challenged order is found to be indeed violative of the Constitution, it is duty-
bound to declare it so. For the Constitution, being the highest expression of the sovereign will of the
In such a case, the Committee, by a majority vote of the members present there being a quorum, shall Filipino people, must prevail over any issuance of the government that contravenes its mandates.
determine whether the right has been properly invoked. If the Committee decides otherwise, it shall
resume its investigation and the question or questions previously refused to be answered shall be In the exercise of its legislative power, the Senate of the Philippines, through its various Senate
repeated to the witness. If the latter continues to refuse to answer the question, the Committee may Committees, conducts inquiries or investigations in aid of legislation which call for, inter alia, the
punish him for contempt for contumacious conduct. attendance of officials and employees of the executive department, bureaus, and offices including those
employed in Government Owned and Controlled Corporations, the Armed Forces of the Philippines
The same directors and officers contend that the Senate is barred from inquiring into the same issues (AFP), and the Philippine National Police (PNP).
being litigated before the Court of Appeals and the Sandiganbayan. Suffice it to state that the Senate
Rules of Procedure Governing Inquiries in Aid of Legislation provide that the filing or pendency of any On September 21 to 23, 2005, the Committee of the Senate as a whole issued invitations to various
prosecution of criminal or administrative action should not stop or abate any inquiry to carry out a officials of the Executive Department for them to appear on September 29, 2005 as resource speakers in
legislative purpose. a public hearing on the railway project of the North Luzon Railways Corporation with the China National
Machinery and Equipment Group (hereinafter North Rail Project). The public hearing was sparked by a
Let it be stressed at this point that so long as the constitutional rights of witnesses, like Chairman Sabio privilege speech of Senator Juan Ponce Enrile urging the Senate to investigate the alleged overpricing
and his Commissioners, will be respected by respondent Senate Committees, it their duty to cooperate and other unlawful provisions of the contract covering the North Rail Project.
with them in their efforts to obtain the facts needed for intelligent legislative action. The unremitting
obligation of every citizen is to respond to subpoenae, to respect the dignity of the Congress and its The Senate Committee on National Defense and Security likewise issued invitations2 dated September
Committees, and to testify fully with respect to matters within the realm of proper investigation. 22, 2005 to the following officials of the AFP: the Commanding General of the Philippine Army, Lt. Gen.
Hermogenes C. Esperon; Inspector General of the AFP Vice Admiral Mateo M. Mayuga; Deputy Chief of
In fine, PCGG Chairman Camilo Sabio and Commissioners Ricardo Abcede, Narciso Nario, Nicasio Conti, Staff for Intelligence of the AFP Rear Admiral Tirso R. Danga; Chief of the Intelligence Service of the AFP
and Tereso Javier; and Manuel Andal and Julio Jalandoni, PCGG's nominees to Philcomsat Holdings Brig. Gen. Marlu Q. Quevedo; Assistant Superintendent of the Philippine Military Academy (PMA) Brig.
Corporation, as well as its directors and officers, must comply with the Subpoenae Ad Gen. Francisco V. Gudani; and Assistant Commandant, Corps of Cadets of the PMA, Col. Alexander F.
Testificandum issued by respondent Senate Committees directing them to appear and testify in public Balutan, for them to attend as resource persons in a public hearing scheduled on September 28, 2005 on
hearings relative to Senate Resolution No. 455. the following: (1) Privilege Speech of Senator Aquilino Q. Pimentel Jr., delivered on June 6, 2005 entitled
"Bunye has Provided Smoking Gun or has Opened a Can of Worms that Show Massive Electoral Fraud in
WHEREFORE, the petition in G.R. No. 174340 for habeas corpus is DISMISSED, for being moot. The the Presidential Election of May 2005"; (2) Privilege Speech of Senator Jinggoy E. Estrada delivered on
petitions in G.R Nos. 174318 and 174177 are likewise DISMISSED. July 26, 2005 entitled "The Philippines as the Wire-Tapping Capital of the World"; (3) Privilege Speech of
Senator Rodolfo Biazon delivered on August 1, 2005 entitled "Clear and Present Danger"; (4) Senate
Resolution No. 285 filed by Senator Maria Ana Consuelo Madrigal – Resolution Directing the Committee
37
on National Defense and Security to Conduct an Inquiry, in Aid of Legislation, and in the National Interest, Information between inter-government agencies prior to the conclusion of treaties and executive
on the Role of the Military in the So-called "Gloriagate Scandal"; and (5) Senate Resolution No. 295 filed agreements (Chavez v. Presidential Commission on Good Government, G.R. No. 130716, 9 December
by Senator Biazon – Resolution Directing the Committee on National Defense and Security to Conduct an 1998);
Inquiry, in Aid of Legislation, on the Wire-Tapping of the President of the Philippines.
Discussion in close-door Cabinet meetings (Chavez v. Presidential Commission on Good Government,
Also invited to the above-said hearing scheduled on September 28 2005 was the AFP Chief of Staff, G.R. No. 130716, 9 December 1998);
General Generoso S. Senga who, by letter3 dated September 27, 2005, requested for its postponement
"due to a pressing operational situation that demands [his utmost personal attention" while "some of the Matters affecting national security and public order (Chavez v. Public Estates Authority, G.R. No. 133250,
invited AFP officers are currently attending to other urgent operational matters." 9 July 2002).

On September 28, 2005, Senate President Franklin M. Drilon received from Executive Secretary Eduardo (b) Who are covered. – The following are covered by this executive order:
R. Ermita a letter4 dated September 27, 2005 "respectfully request[ing] for the postponement of the
hearing [regarding the NorthRail project] to which various officials of the Executive Department have been
invited" in order to "afford said officials ample time and opportunity to study and prepare for the various Senior officials of executive departments who in the judgment of the department heads are covered by the
issues so that they may better enlighten the Senate Committee on its investigation." executive privilege;

Senate President Drilon, however, wrote5 Executive Secretary Ermita that the Senators "are unable to Generals and flag officers of the Armed Forces of the Philippines and such other officers who in the
accede to [his request]" as it "was sent belatedly" and "[a]ll preparations and arrangements as well as judgment of the Chief of Staff are covered by the executive privilege;
notices to all resource persons were completed [the previous] week."
Philippine National Police (PNP) officers with rank of chief superintendent or higher and such other
Senate President Drilon likewise received on September 28, 2005 a letter6 from the President of the officers who in the judgment of the Chief of the PNP are covered by the executive privilege;
North Luzon Railways Corporation Jose L. Cortes, Jr. requesting that the hearing on the NorthRail project
be postponed or cancelled until a copy of the report of the UP Law Center on the contract agreements Senior national security officials who in the judgment of the National Security Adviser are covered by the
relative to the project had been secured. executive privilege; and

On September 28, 2005, the President issued E.O. 464, "Ensuring Observance of the Principle of Such other officers as may be determined by the President.
Separation of Powers, Adherence to the Rule on Executive Privilege and Respect for the Rights of Public
Officials Appearing in Legislative Inquiries in Aid of Legislation Under the Constitution, and For Other SECTION 3. Appearance of Other Public Officials Before Congress. – All public officials enumerated in
Purposes,"7 which, pursuant to Section 6 thereof, took effect immediately. The salient provisions of the Section 2 (b) hereof shall secure prior consent of the President prior to appearing before either House of
Order are as follows: Congress to ensure the observance of the principle of separation of powers, adherence to the rule on
executive privilege and respect for the rights of public officials appearing in inquiries in aid of legislation.
SECTION 1. Appearance by Heads of Departments Before Congress. – In accordance with Article VI, (Emphasis and underscoring supplied)
Section 22 of the Constitution and to implement the Constitutional provisions on the separation of powers
between co-equal branches of the government, all heads of departments of the Executive Branch of the Also on September 28, 2005, Senate President Drilon received from Executive Secretary Ermita a copy of
government shall secure the consent of the President prior to appearing before either House of Congress. E.O. 464, and another letter8 informing him "that officials of the Executive Department invited to appear at
the meeting [regarding the NorthRail project] will not be able to attend the same without the consent of the
When the security of the State or the public interest so requires and the President so states in writing, the President, pursuant to [E.O. 464]" and that "said officials have not secured the required consent from the
appearance shall only be conducted in executive session. President." On even date which was also the scheduled date of the hearing on the alleged wiretapping,
Gen. Senga sent a letter9 to Senator Biazon, Chairperson of the Committee on National Defense and
SECTION. 2. Nature, Scope and Coverage of Executive Privilege. – Security, informing him "that per instruction of [President Arroyo], thru the Secretary of National Defense,
no officer of the [AFP] is authorized to appear before any Senate or Congressional hearings without
seeking a written approval from the President" and "that no approval has been granted by the President to
(a) Nature and Scope. - The rule of confidentiality based on executive privilege is fundamental to the any AFP officer to appear before the public hearing of the Senate Committee on National Defense and
operation of government and rooted in the separation of powers under the Constitution (Almonte vs. Security scheduled [on] 28 September 2005."
Vasquez, G.R. No. 95367, 23 May 1995). Further, Republic Act No. 6713 or the Code of Conduct and
Ethical Standards for Public Officials and Employees provides that Public Officials and Employees shall
not use or divulge confidential or classified information officially known to them by reason of their office Despite the communications received from Executive Secretary Ermita and Gen. Senga, the investigation
and not made available to the public to prejudice the public interest. scheduled by the Committee on National Defense and Security pushed through, with only Col. Balutan
and Brig. Gen. Gudani among all the AFP officials invited attending.
Executive privilege covers all confidential or classified information between the President and the public
officers covered by this executive order, including: For defying President Arroyo’s order barring military personnel from testifying before legislative inquiries
without her approval, Brig. Gen. Gudani and Col. Balutan were relieved from their military posts and were
made to face court martial proceedings.
Conversations and correspondence between the President and the public official covered by this
executive order (Almonte vs. Vasquez G.R. No. 95367, 23 May 1995; Chavez v. Public Estates Authority,
G.R. No. 133250, 9 July 2002); As to the NorthRail project hearing scheduled on September 29, 2005, Executive Secretary Ermita, citing
E.O. 464, sent letter of regrets, in response to the invitations sent to the following government officials:
Light Railway Transit Authority Administrator Melquiades Robles, Metro Rail Transit Authority
Military, diplomatic and other national security matters which in the interest of national security should not Administrator Roberto Lastimoso, Department of Justice (DOJ) Chief State Counsel Ricardo V. Perez,
be divulged (Almonte vs. Vasquez, G.R. No. 95367, 23 May 1995; Chavez v. Presidential Commission on then Presidential Legal Counsel Merceditas Gutierrez, Department of Transportation and Communication
Good Government, G.R. No. 130716, 9 December 1998).

38
(DOTC) Undersecretary Guiling Mamonding, DOTC Secretary Leandro Mendoza, Philippine National In another investigation conducted jointly by the Senate Committee on Agriculture and Food and the Blue
Railways General Manager Jose Serase II, Monetary Board Member Juanita Amatong, Bases Conversion Ribbon Committee on the alleged mismanagement and use of the fertilizer fund under the Ginintuang
Development Authority Chairperson Gen. Narciso Abaya and Secretary Romulo L. Neri.10 NorthRail Masaganang Ani program of the Department of Agriculture (DA), several Cabinet officials were invited to
President Cortes sent personal regrets likewise citing E.O. 464.11 the hearings scheduled on October 5 and 26, November 24 and December 12, 2005 but most of them
failed to attend, DA Undersecretary Belinda Gonzales, DA Assistant Secretary Felix Jose Montes,
On October 3, 2005, three petitions, docketed as G.R. Nos. 169659, 169660, and 169667, for certiorari Fertilizer and Pesticide Authority Executive Director Norlito R. Gicana,17 and those from the Department
and prohibition, were filed before this Court challenging the constitutionality of E.O. 464. of Budget and Management18 having invoked E.O. 464.

In G.R. No. 169659, petitioners party-list Bayan Muna, House of Representatives Members Satur In the budget hearings set by the Senate on February 8 and 13, 2006, Press Secretary and Presidential
Ocampo, Crispin Beltran, Rafael Mariano, Liza Maza, Joel Virador and Teodoro Casino, Courage, an Spokesperson Ignacio R. Bunye,19 DOJ Secretary Raul M. Gonzalez20 and Department of Interior and
organization of government employees, and Counsels for the Defense of Liberties (CODAL), a group of Local Government Undersecretary Marius P. Corpus21 communicated their inability to attend due to lack
lawyers dedicated to the promotion of justice, democracy and peace, all claiming to have standing to file of appropriate clearance from the President pursuant to E.O. 464. During the February 13, 2005 budget
the suit because of the transcendental importance of the issues they posed, pray, in their petition that hearing, however, Secretary Bunye was allowed to attend by Executive Secretary Ermita.
E.O. 464 be declared null and void for being unconstitutional; that respondent Executive Secretary Ermita,
in his capacity as Executive Secretary and alter-ego of President Arroyo, be prohibited from imposing, and On February 13, 2006, Jose Anselmo I. Cadiz and the incumbent members of the Board of Governors of
threatening to impose sanctions on officials who appear before Congress due to congressional summons. the Integrated Bar of the Philippines, as taxpayers, and the Integrated Bar of the Philippines as the official
Additionally, petitioners claim that E.O. 464 infringes on their rights and impedes them from fulfilling their organization of all Philippine lawyers, all invoking their constitutional right to be informed on matters of
respective obligations. Thus, Bayan Muna alleges that E.O. 464 infringes on its right as a political party public interest, filed their petition for certiorari and prohibition, docketed as G.R. No. 171246, and pray that
entitled to participate in governance; Satur Ocampo, et al. allege that E.O. 464 infringes on their rights E.O. 464 be declared null and void.
and duties as members of Congress to conduct investigation in aid of legislation and conduct oversight
functions in the implementation of laws; Courage alleges that the tenure of its members in public office is All the petitions pray for the issuance of a Temporary Restraining Order enjoining respondents from
predicated on, and threatened by, their submission to the requirements of E.O. 464 should they be implementing, enforcing, and observing E.O. 464.
summoned by Congress; and CODAL alleges that its members have a sworn duty to uphold the rule of
law, and their rights to information and to transparent governance are threatened by the imposition of E.O.
464. In the oral arguments on the petitions conducted on February 21, 2006, the following substantive issues
were ventilated: (1) whether respondents committed grave abuse of discretion in implementing E.O. 464
prior to its publication in the Official Gazette or in a newspaper of general circulation; and (2) whether E.O.
In G.R. No. 169660, petitioner Francisco I. Chavez, claiming that his constitutional rights as a citizen, 464 violates the following provisions of the Constitution: Art. II, Sec. 28, Art. III, Sec. 4, Art. III, Sec. 7, Art.
taxpayer and law practitioner, are affected by the enforcement of E.O. 464, prays in his petition that E.O. IV. Sec. 1, Art. VI, Sec. 21, Art. VI, Sec. 22, Art. XI, Sec. 1, and Art. XIII, Sec. 16. The procedural issue of
464 be declared null and void for being unconstitutional. whether there is an actual case or controversy that calls for judicial review was not taken up; instead, the
parties were instructed to discuss it in their respective memoranda.
In G.R. No. 169667, petitioner Alternative Law Groups, Inc.12 (ALG), alleging that as a coalition of 17
legal resource non-governmental organizations engaged in developmental lawyering and work with the After the conclusion of the oral arguments, the parties were directed to submit their respective
poor and marginalized sectors in different parts of the country, and as an organization of citizens of the memoranda, paying particular attention to the following propositions: (1) that E.O. 464 is, on its face,
Philippines and a part of the general public, it has legal standing to institute the petition to enforce its unconstitutional; and (2) assuming that it is not, it is unconstitutional as applied in four instances, namely:
constitutional right to information on matters of public concern, a right which was denied to the public by (a) the so called Fertilizer scam; (b) the NorthRail investigation (c) the Wiretapping activity of the ISAFP;
E.O. 464,13 prays, that said order be declared null and void for being unconstitutional and that and (d) the investigation on the Venable contract.22
respondent Executive Secretary Ermita be ordered to cease from implementing it.
Petitioners in G.R. No. 16966023 and G.R. No. 16977724 filed their memoranda on March 7, 2006, while
On October 11, 2005, Petitioner Senate of the Philippines, alleging that it has a vital interest in the those in G.R. No. 16966725 and G.R. No. 16983426 filed theirs the next day or on March 8, 2006.
resolution of the issue of the validity of E.O. 464 for it stands to suffer imminent and material injury, as it Petitioners in G.R. No. 171246 did not file any memorandum.
has already sustained the same with its continued enforcement since it directly interferes with and
impedes the valid exercise of the Senate’s powers and functions and conceals information of great public
interest and concern, filed its petition for certiorari and prohibition, docketed as G.R. No. 169777 and Petitioners Bayan Muna et al. in G.R. No. 169659, after their motion for extension to file
prays that E.O. 464 be declared unconstitutional. memorandum27 was granted, subsequently filed a manifestation28 dated March 14, 2006 that it would no
longer file its memorandum in the interest of having the issues resolved soonest, prompting this Court to
issue a Resolution reprimanding them.29
On October 14, 2005, PDP-Laban, a registered political party with members duly elected into the Petitioners submit that E.O. 464 violates the following constitutional provisions:
Philippine Senate and House of Representatives, filed a similar petition for certiorari and prohibition, Art. VI, Sec. 2130
docketed as G.R. No. 169834, alleging that it is affected by the challenged E.O. 464 because it hampers Art. VI, Sec. 2231
its legislative agenda to be implemented through its members in Congress, particularly in the conduct of Art. VI, Sec. 132
inquiries in aid of legislation and transcendental issues need to be resolved to avert a constitutional crisis Art. XI, Sec. 133
between the executive and legislative branches of the government. Art. III, Sec. 734
Art. III, Sec. 435
Meanwhile, by letter14 dated February 6, 2006, Senator Biazon reiterated his invitation to Gen. Senga for Art. XIII, Sec. 16 36
him and other military officers to attend the hearing on the alleged wiretapping scheduled on February 10, Art. II, Sec. 2837
2005. Gen. Senga replied, however, by letter15 dated February 8, 2006, that "[p]ursuant to Executive Respondents Executive Secretary Ermita et al., on the other hand, pray in their consolidated
Order No. 464, th[e] Headquarters requested for a clearance from the President to allow [them] to appear memorandum38 on March 13, 2006 for the dismissal of the petitions for lack of merit.
before the public hearing" and that "they will attend once [their] request is approved by the President." As
none of those invited appeared, the hearing on February 10, 2006 was cancelled.16 The Court synthesizes the issues to be resolved as follows:
1. Whether E.O. 464 contravenes the power of inquiry vested in Congress;

39
2. Whether E.O. 464 violates the right of the people to information on matters of public concern; and (Gabriela) are allowed to sue to question the constitutionality of E.O. 464, the absence of any claim that
3. Whether respondents have committed grave abuse of discretion when they implemented E.O. 464 prior an investigation called by the House of Representatives or any of its committees was aborted due to the
to its publication in a newspaper of general circulation. implementation of E.O. 464 notwithstanding, it being sufficient that a claim is made that E.O. 464 infringes
Essential requisites for judicial review on their constitutional rights and duties as members of Congress to conduct investigation in aid of
legislation and conduct oversight functions in the implementation of laws.
Before proceeding to resolve the issue of the constitutionality of E.O. 464, ascertainment of whether the
requisites for a valid exercise of the Court’s power of judicial review are present is in order. The national political party, Bayan Muna, likewise meets the standing requirement as it obtained three
seats in the House of Representatives in the 2004 elections and is, therefore, entitled to participate in the
Like almost all powers conferred by the Constitution, the power of judicial review is subject to limitations, legislative process consonant with the declared policy underlying the party list system of affording citizens
to wit: (1) there must be an actual case or controversy calling for the exercise of judicial power; (2) the belonging to marginalized and underrepresented sectors, organizations and parties who lack well-defined
person challenging the act must have standing to challenge the validity of the subject act or issuance; political constituencies to contribute to the formulation and enactment of legislation that will benefit the
otherwise stated, he must have a personal and substantial interest in the case such that he has sustained, nation.48
or will sustain, direct injury as a result of its enforcement; (3) the question of constitutionality must be
raised at the earliest opportunity; and (4) the issue of constitutionality must be the very lis mota of the As Bayan Muna and Representatives Ocampo et al. have the standing to file their petitions, passing on
case.39 the standing of their co-petitioners Courage and Codal is rendered unnecessary.49

Except with respect to the requisites of standing and existence of an actual case or controversy where the In filing their respective petitions, Chavez, the ALG which claims to be an organization of citizens, and the
disagreement between the parties lies, discussion of the rest of the requisites shall be omitted. incumbent members of the IBP Board of Governors and the IBP in behalf of its lawyer members,50 invoke
their constitutional right to information on matters of public concern, asserting that the right to information,
Standing curtailed and violated by E.O. 464, is essential to the effective exercise of other constitutional rights51 and
to the maintenance of the balance of power among the three branches of the government through the
principle of checks and balances.52
Respondents, through the Solicitor General, assert that the allegations in G.R. Nos. 169659, 169660 and
169667 make it clear that they, adverting to the non-appearance of several officials of the executive
department in the investigations called by the different committees of the Senate, were brought to It is well-settled that when suing as a citizen, the interest of the petitioner in assailing the constitutionality
vindicate the constitutional duty of the Senate or its different committees to conduct inquiry in aid of of laws, presidential decrees, orders, and other regulations, must be direct and personal. In Franciso v.
legislation or in the exercise of its oversight functions. They maintain that Representatives Ocampo et al. House of Representatives,53 this Court held that when the proceeding involves the assertion of a public
have not shown any specific prerogative, power, and privilege of the House of Representatives which had right, the mere fact that he is a citizen satisfies the requirement of personal interest.
been effectively impaired by E.O. 464, there being no mention of any investigation called by the House of
Representatives or any of its committees which was aborted due to the implementation of E.O. 464. As for petitioner PDP-Laban, it asseverates that it is clothed with legal standing in view of the
transcendental issues raised in its petition which this Court needs to resolve in order to avert a
As for Bayan Muna’s alleged interest as a party-list representing the marginalized and underrepresented, constitutional crisis. For it to be accorded standing on the ground of transcendental importance, however,
and that of the other petitioner groups and individuals who profess to have standing as advocates and it must establish (1) the character of the funds (that it is public) or other assets involved in the case, (2)
defenders of the Constitution, respondents contend that such interest falls short of that required to confer the presence of a clear case of disregard of a constitutional or statutory prohibition by the public
standing on them as parties "injured-in-fact."40 respondent agency or instrumentality of the government, and (3) the lack of any party with a more direct
and specific interest in raising the questions being raised.54 The first and last determinants not being
present as no public funds or assets are involved and petitioners in G.R. Nos. 169777 and 169659 have
Respecting petitioner Chavez, respondents contend that Chavez may not claim an interest as a taxpayer direct and specific interests in the resolution of the controversy, petitioner PDP-Laban is bereft of standing
for the implementation of E.O. 464 does not involve the exercise of taxing or spending power.41 to file its petition. Its allegation that E.O. 464 hampers its legislative agenda is vague and uncertain, and at
best is only a "generalized interest" which it shares with the rest of the political parties. Concrete injury,
With regard to the petition filed by the Senate, respondents argue that in the absence of a personal or whether actual or threatened, is that indispensable element of a dispute which serves in part to cast it in a
direct injury by reason of the issuance of E.O. 464, the Senate and its individual members are not the form traditionally capable of judicial resolution.55 In fine, PDP-Laban’s alleged interest as a political party
proper parties to assail the constitutionality of E.O. 464. does not suffice to clothe it with legal standing.

Invoking this Court’s ruling in National Economic Protectionism Association v. Ongpin42 and Valmonte v. Actual Case or Controversy
Philippine Charity Sweepstakes Office,43 respondents assert that to be considered a proper party, one
must have a personal and substantial interest in the case, such that he has sustained or will sustain direct Petitioners assert that an actual case exists, they citing the absence of the executive officials invited by
injury due to the enforcement of E.O. 464.44 the Senate to its hearings after the issuance of E.O. 464, particularly those on the NorthRail project and
the wiretapping controversy.
That the Senate of the Philippines has a fundamental right essential not only for intelligent public decision-
making in a democratic system, but more especially for sound legislation45 is not disputed. E.O. 464, Respondents counter that there is no case or controversy, there being no showing that President Arroyo
however, allegedly stifles the ability of the members of Congress to access information that is crucial to has actually withheld her consent or prohibited the appearance of the invited officials.56 These officials,
law-making.46 Verily, the Senate, including its individual members, has a substantial and direct interest they claim, merely communicated to the Senate that they have not yet secured the consent of the
over the outcome of the controversy and is the proper party to assail the constitutionality of E.O. 464. President, not that the President prohibited their attendance.57 Specifically with regard to the AFP officers
Indeed, legislators have standing to maintain inviolate the prerogative, powers and privileges vested by who did not attend the hearing on September 28, 2005, respondents claim that the instruction not to
the Constitution in their office and are allowed to sue to question the validity of any official action which attend without the President’s consent was based on its role as Commander-in-Chief of the Armed
they claim infringes their prerogatives as legislators.47 Forces, not on E.O. 464.

In the same vein, party-list representatives Satur Ocampo (Bayan Muna), Teodoro Casino (Bayan Muna),
Joel Virador (Bayan Muna), Crispin Beltran (Anakpawis), Rafael Mariano (Anakpawis), and Liza Maza

40
Respondents thus conclude that the petitions merely rest on an unfounded apprehension that the That this power of inquiry is broad enough to cover officials of the executive branch may be deduced from
President will abuse its power of preventing the appearance of officials before Congress, and that such the same case. The power of inquiry, the Court therein ruled, is co-extensive with the power to
apprehension is not sufficient for challenging the validity of E.O. 464. legislate.60 The matters which may be a proper subject of legislation and those which may be a proper
subject of investigation are one. It follows that the operation of government, being a legitimate subject for
The Court finds respondents’ assertion that the President has not withheld her consent or prohibited the legislation, is a proper subject for investigation.
appearance of the officials concerned immaterial in determining the existence of an actual case or
controversy insofar as E.O. 464 is concerned. For E.O. 464 does not require either a deliberate Thus, the Court found that the Senate investigation of the government transaction involved in Arnault was
withholding of consent or an express prohibition issuing from the President in order to bar officials from a proper exercise of the power of inquiry. Besides being related to the expenditure of public funds of
appearing before Congress. which Congress is the guardian, the transaction, the Court held, "also involved government agencies
created by Congress and officers whose positions it is within the power of Congress to regulate or even
As the implementation of the challenged order has already resulted in the absence of officials invited to abolish."
the hearings of petitioner Senate of the Philippines, it would make no sense to wait for any further event
before considering the present case ripe for adjudication. Indeed, it would be sheer abandonment of duty Since Congress has authority to inquire into the operations of the executive branch, it would be
if this Court would now refrain from passing on the constitutionality of E.O. 464. incongruous to hold that the power of inquiry does not extend to executive officials who are the most
familiar with and informed on executive operations.
Constitutionality of E.O. 464
As discussed in Arnault, the power of inquiry, "with process to enforce it," is grounded on the necessity of
E.O. 464, to the extent that it bars the appearance of executive officials before Congress, deprives information in the legislative process. If the information possessed by executive officials on the operation
Congress of the information in the possession of these officials. To resolve the question of whether such of their offices is necessary for wise legislation on that subject, by parity of reasoning, Congress has the
withholding of information violates the Constitution, consideration of the general power of Congress to right to that information and the power to compel the disclosure thereof.
obtain information, otherwise known as the power of inquiry, is in order.
As evidenced by the American experience during the so-called "McCarthy era," however, the right of
The power of inquiry Congress to conduct inquiries in aid of legislation is, in theory, no less susceptible to abuse than executive
or judicial power. It may thus be subjected to judicial review pursuant to the Court’s certiorari powers
under Section 1, Article VIII of the Constitution.
The Congress power of inquiry is expressly recognized in Section 21 of Article VI of the Constitution which
reads:
For one, as noted in Bengzon v. Senate Blue Ribbon Committee,61 the inquiry itself might not properly be
in aid of legislation, and thus beyond the constitutional power of Congress. Such inquiry could not usurp
SECTION 21. The Senate or the House of Representatives or any of its respective committees may judicial functions. Parenthetically, one possible way for Congress to avoid such a result as occurred in
conduct inquiries in aid of legislation in accordance with its duly published rules of procedure. The rights Bengzon is to indicate in its invitations to the public officials concerned, or to any person for that matter,
of persons appearing in or affected by such inquiries shall be respected. (Underscoring supplied) the possible needed statute which prompted the need for the inquiry. Given such statement in its
invitations, along with the usual indication of the subject of inquiry and the questions relative to and in
This provision is worded exactly as Section 8 of Article VIII of the 1973 Constitution except that, in the furtherance thereof, there would be less room for speculation on the part of the person invited on whether
latter, it vests the power of inquiry in the unicameral legislature established therein – the Batasang the inquiry is in aid of legislation.
Pambansa – and its committees.
Section 21, Article VI likewise establishes crucial safeguards that proscribe the legislative power of
The 1935 Constitution did not contain a similar provision. Nonetheless, in Arnault v. Nazareno,58 a case inquiry. The provision requires that the inquiry be done in accordance with the Senate or House’s duly
decided in 1950 under that Constitution, the Court already recognized that the power of inquiry is inherent published rules of procedure, necessarily implying the constitutional infirmity of an inquiry conducted
in the power to legislate. without duly published rules of procedure. Section 21 also mandates that the rights of persons appearing
in or affected by such inquiries be respected, an imposition that obligates Congress to adhere to the
Arnault involved a Senate investigation of the reportedly anomalous purchase of the Buenavista and guarantees in the Bill of Rights.
Tambobong Estates by the Rural Progress Administration. Arnault, who was considered a leading witness
in the controversy, was called to testify thereon by the Senate. On account of his refusal to answer the These abuses are, of course, remediable before the courts, upon the proper suit filed by the persons
questions of the senators on an important point, he was, by resolution of the Senate, detained for affected, even if they belong to the executive branch. Nonetheless, there may be exceptional
contempt. Upholding the Senate’s power to punish Arnault for contempt, this Court held: circumstances, none appearing to obtain at present, wherein a clear pattern of abuse of the legislative
power of inquiry might be established, resulting in palpable violations of the rights guaranteed to members
Although there is no provision in the Constitution expressly investing either House of Congress with power of the executive department under the Bill of Rights. In such instances, depending on the particulars of
to make investigations and exact testimony to the end that it may exercise its legislative functions each case, attempts by the Executive Branch to forestall these abuses may be accorded judicial sanction.
advisedly and effectively, such power is so far incidental to the legislative function as to be implied. In
other words, the power of inquiry – with process to enforce it – is an essential and appropriate auxiliary to Even where the inquiry is in aid of legislation, there are still recognized exemptions to the power of inquiry,
the legislative function. A legislative body cannot legislate wisely or effectively in the absence of which exemptions fall under the rubric of "executive privilege." Since this term figures prominently in the
information respecting the conditions which the legislation is intended to affect or change; and where the challenged order, it being mentioned in its provisions, its preambular clauses,62 and in its very title, a
legislative body does not itself possess the requisite information – which is not infrequently true – discussion of executive privilege is crucial for determining the constitutionality of E.O. 464.
recourse must be had to others who do possess it. Experience has shown that mere requests for such
information are often unavailing, and also that information which is volunteered is not always accurate or Executive privilege
complete; so some means of compulsion is essential to obtain what is needed.59 . . . (Emphasis and
underscoring supplied)

41
The phrase "executive privilege" is not new in this jurisdiction. It has been used even prior to the criminal justice. Notably, the Court was careful to clarify that it was not there addressing the issue of
promulgation of the 1986 Constitution.63 Being of American origin, it is best understood in light of how it claims of privilege in a civil litigation or against congressional demands for information.
has been defined and used in the legal literature of the United States.
Cases in the U.S. which involve claims of executive privilege against Congress are rare.73 Despite
Schwartz defines executive privilege as "the power of the Government to withhold information from the frequent assertion of the privilege to deny information to Congress, beginning with President
public, the courts, and the Congress."64 Similarly, Rozell defines it as "the right of the President and high- Washington’s refusal to turn over treaty negotiation records to the House of Representatives, the U.S.
level executive branch officers to withhold information from Congress, the courts, and ultimately the Supreme Court has never adjudicated the issue.74 However, the U.S. Court of Appeals for the District of
public."65 Columbia Circuit, in a case decided earlier in the same year as Nixon, recognized the President’s privilege
over his conversations against a congressional subpoena.75 Anticipating the balancing approach adopted
Executive privilege is, nonetheless, not a clear or unitary concept. 66 It has encompassed claims of by the U.S. Supreme Court in Nixon, the Court of Appeals weighed the public interest protected by the
varying kinds.67 Tribe, in fact, comments that while it is customary to employ the phrase "executive claim of privilege against the interest that would be served by disclosure to the Committee. Ruling that the
privilege," it may be more accurate to speak of executive privileges "since presidential refusals to furnish balance favored the President, the Court declined to enforce the subpoena. 
information may be actuated by any of at least three distinct kinds of considerations, and may be
asserted, with differing degrees of success, in the context of either judicial or legislative investigations." In this jurisdiction, the doctrine of executive privilege was recognized by this Court in Almonte v.
Vasquez.77 Almonte used the term in reference to the same privilege subject of Nixon. It quoted the
One variety of the privilege, Tribe explains, is the state secrets privilege invoked by U.S. Presidents, following portion of the Nixon decision which explains the basis for the privilege:
beginning with Washington, on the ground that the information is of such nature that its disclosure would
subvert crucial military or diplomatic objectives. Another variety is the informer’s privilege, or the privilege "The expectation of a President to the confidentiality of his conversations and correspondences, like the
of the Government not to disclose the identity of persons who furnish information of violations of law to claim of confidentiality of judicial deliberations, for example, has all the values to which we accord
officers charged with the enforcement of that law. Finally, a generic privilege for internal deliberations has deference for the privacy of all citizens and, added to those values, is the necessity for protection of the
been said to attach to intragovernmental documents reflecting advisory opinions, recommendations and public interest in candid, objective, and even blunt or harsh opinions in Presidential decision-making. A
deliberations comprising part of a process by which governmental decisions and policies are President and those who assist him must be free to explore alternatives in the process of shaping policies
formulated. 68 and making decisions and to do so in a way many would be unwilling to express except privately. These
are the considerations justifying a presumptive privilege for Presidential communications. The privilege is
Tribe’s comment is supported by the ruling in In re Sealed Case, thus: fundamental to the operation of government and inextricably rooted in the separation of powers under the
Constitution x x x " (Emphasis and underscoring supplied)
Since the beginnings of our nation, executive officials have claimed a variety of privileges to resist
disclosure of information the confidentiality of which they felt was crucial to fulfillment of the unique role Almonte involved a subpoena duces tecum issued by the Ombudsman against the therein petitioners. It
and responsibilities of the executive branch of our government. Courts ruled early that the executive had a did not involve, as expressly stated in the decision, the right of the people to information.78 Nonetheless,
right to withhold documents that might reveal military or state secrets. The courts have also granted the the Court recognized that there are certain types of information which the government may withhold from
executive a right to withhold the identity of government informers in some circumstances and a qualified the public, thus acknowledging, in substance if not in name, that executive privilege may be claimed
right to withhold information related to pending investigations. x x x"69 (Emphasis and underscoring against citizens’ demands for information.
supplied)
In Chavez v. PCGG,79 the Court held that this jurisdiction recognizes the common law holding that there
The entry in Black’s Law Dictionary on "executive privilege" is similarly instructive regarding the scope of is a "governmental privilege against public disclosure with respect to state secrets regarding military,
the doctrine. diplomatic and other national security matters."80 The same case held that closed-door Cabinet meetings
are also a recognized limitation on the right to information.
This privilege, based on the constitutional doctrine of separation of powers, exempts the executive from
disclosure requirements applicable to the ordinary citizen or organization where such exemption is Similarly, in Chavez v. Public Estates Authority,81 the Court ruled that the right to information does not
necessary to the discharge of highly important executive responsibilities involved in maintaining extend to matters recognized as "privileged information under the separation of powers,"82 by which the
governmental operations, and extends not only to military and diplomatic secrets but also to documents Court meant Presidential conversations, correspondences, and discussions in closed-door Cabinet
integral to an appropriate exercise of the executive’ domestic decisional and policy making functions, that meetings. It also held that information on military and diplomatic secrets and those affecting national
is, those documents reflecting the frank expression necessary in intra-governmental advisory and security, and information on investigations of crimes by law enforcement agencies before the prosecution
deliberative communications.70 (Emphasis and underscoring supplied) of the accused were exempted from the right to information.

That a type of information is recognized as privileged does not, however, necessarily mean that it would From the above discussion on the meaning and scope of executive privilege, both in the United States
be considered privileged in all instances. For in determining the validity of a claim of privilege, the and in this jurisdiction, a clear principle emerges. Executive privilege, whether asserted against Congress,
question that must be asked is not only whether the requested information falls within one of the the courts, or the public, is recognized only in relation to certain types of information of a sensitive
traditional privileges, but also whether that privilege should be honored in a given procedural setting.71 character. While executive privilege is a constitutional concept, a claim thereof may be valid or not
depending on the ground invoked to justify it and the context in which it is made. Noticeably absent is any
recognition that executive officials are exempt from the duty to disclose information by the mere fact of
The leading case on executive privilege in the United States is U.S. v. Nixon, 72 decided in 1974. In issue being executive officials. Indeed, the extraordinary character of the exemptions indicates that the
in that case was the validity of President Nixon’s claim of executive privilege against a subpoena issued presumption inclines heavily against executive secrecy and in favor of disclosure.
by a district court requiring the production of certain tapes and documents relating to the Watergate
investigations. The claim of privilege was based on the President’s general interest in the confidentiality of
his conversations and correspondence. The U.S. Court held that while there is no explicit reference to a Validity of Section 1
privilege of confidentiality in the U.S. Constitution, it is constitutionally based to the extent that it relates to
the effective discharge of a President’s powers. The Court, nonetheless, rejected the President’s claim of Section 1 is similar to Section 3 in that both require the officials covered by them to secure the consent of
privilege, ruling that the privilege must be balanced against the public interest in the fair administration of the President prior to appearing before Congress. There are significant differences between the two
provisions, however, which constrain this Court to discuss the validity of these provisions separately.
42
Section 1 specifically applies to department heads. It does not, unlike Section 3, require a prior MR. DAVIDE. Thank you, Mr. Presiding Officer. I have only one reaction to the Question Hour. I propose
determination by any official whether they are covered by E.O. 464. The President herself has, through that instead of putting it as Section 31, it should follow Legislative Inquiries.
the challenged order, made the determination that they are. Further, unlike also Section 3, the coverage THE PRESIDING OFFICER. What does the committee say?
of department heads under Section 1 is not made to depend on the department heads’ possession of any MR. GUINGONA. I ask Commissioner Maambong to reply, Mr. Presiding Officer.
information which might be covered by executive privilege. In fact, in marked contrast to Section 3 vis-à- MR. MAAMBONG. Actually, we considered that previously when we sequenced this but we reasoned that
vis Section 2, there is no reference to executive privilege at all. Rather, the required prior consent under in Section 21, which is Legislative Inquiry, it is actually a power of Congress in terms of its own
Section 1 is grounded on Article VI, Section 22 of the Constitution on what has been referred to as the lawmaking; whereas, a Question Hour is not actually a power in terms of its own lawmaking power
question hour. because in Legislative Inquiry, it is in aid of legislation. And so we put Question Hour as Section 31. I
hope Commissioner Davide will consider this.
SECTION 22. The heads of departments may upon their own initiative, with the consent of the President, MR. DAVIDE. The Question Hour is closely related with the legislative power, and it is precisely as a
or upon the request of either House, as the rules of each House shall provide, appear before and be complement to or a supplement of the Legislative Inquiry. The appearance of the members of Cabinet
heard by such House on any matter pertaining to their departments. Written questions shall be submitted would be very, very essential not only in the application of check and balance but also, in effect, in aid of
to the President of the Senate or the Speaker of the House of Representatives at least three days before legislation.
their scheduled appearance. Interpellations shall not be limited to written questions, but may cover MR. MAAMBONG. After conferring with the committee, we find merit in the suggestion of Commissioner
matters related thereto. When the security of the State or the public interest so requires and the President Davide. In other words, we are accepting that and so this Section 31 would now become Section 22.
so states in writing, the appearance shall be conducted in executive session. Would it be, Commissioner Davide?
MR. DAVIDE. Yes.84 (Emphasis and underscoring supplied)
Consistent with their statements earlier in the deliberations, Commissioners Davide and Maambong
Determining the validity of Section 1 thus requires an examination of the meaning of Section 22 of Article proceeded from the same assumption that these provisions pertained to two different functions of the
VI. Section 22 which provides for the question hour must be interpreted vis-à-vis Section 21 which legislature. Both Commissioners understood that the power to conduct inquiries in aid of legislation is
provides for the power of either House of Congress to "conduct inquiries in aid of legislation." As the different from the power to conduct inquiries during the question hour. Commissioner Davide’s only
following excerpt of the deliberations of the Constitutional Commission shows, the framers were aware concern was that the two provisions on these distinct powers be placed closely together, they being
that these two provisions involved distinct functions of Congress. complementary to each other. Neither Commissioner considered them as identical functions of Congress.

MR. MAAMBONG. x x x When we amended Section 20 [now Section 22 on the Question Hour] yesterday, The foregoing opinion was not the two Commissioners’ alone. From the above-quoted exchange,
I noticed that members of the Cabinet cannot be compelled anymore to appear before the House of Commissioner Maambong’s committee – the Committee on Style – shared the view that the two
Representatives or before the Senate. I have a particular problem in this regard, Madam President, provisions reflected distinct functions of Congress. Commissioner Davide, on the other hand, was
because in our experience in the Regular Batasang Pambansa – as the Gentleman himself has speaking in his capacity as Chairman of the Committee on the Legislative Department. His views may
experienced in the interim Batasang Pambansa – one of the most competent inputs that we can put in our thus be presumed as representing that of his Committee.
committee deliberations, either in aid of legislation or in congressional investigations, is the testimonies of
Cabinet ministers. We usually invite them, but if they do not come and it is a congressional investigation,
we usually issue subpoenas. In the context of a parliamentary system of government, the "question hour" has a definite meaning. It is a
period of confrontation initiated by Parliament to hold the Prime Minister and the other ministers
accountable for their acts and the operation of the government,85 corresponding to what is known in
I want to be clarified on a statement made by Commissioner Suarez when he said that the fact that the Britain as the question period. There was a specific provision for a question hour in the 1973
Cabinet ministers may refuse to come to the House of Representatives or the Senate [when requested Constitution86 which made the appearance of ministers mandatory. The same perfectly conformed to the
under Section 22] does not mean that they need not come when they are invited or subpoenaed by the parliamentary system established by that Constitution, where the ministers are also members of the
committee of either House when it comes to inquiries in aid of legislation or congressional investigation. legislature and are directly accountable to it.
According to Commissioner Suarez, that is allowed and their presence can be had under Section 21.
Does the gentleman confirm this, Madam President?
An essential feature of the parliamentary system of government is the immediate accountability of the
Prime Minister and the Cabinet to the National Assembly. They shall be responsible to the National
MR. DAVIDE. We confirm that, Madam President, because Section 20 refers only to what was originally Assembly for the program of government and shall determine the guidelines of national policy. Unlike in
the Question Hour, whereas, Section 21 would refer specifically to inquiries in aid of legislation, under the presidential system where the tenure of office of all elected officials cannot be terminated before their
which anybody for that matter, may be summoned and if he refuses, he can be held in contempt of the term expired, the Prime Minister and the Cabinet remain in office only as long as they enjoy the
House.83 (Emphasis and underscoring supplied) confidence of the National Assembly. The moment this confidence is lost the Prime Minister and the
Cabinet may be changed.87
A distinction was thus made between inquiries in aid of legislation and the question hour. While
attendance was meant to be discretionary in the question hour, it was compulsory in inquiries in aid of The framers of the 1987 Constitution removed the mandatory nature of such appearance during the
legislation. The reference to Commissioner Suarez bears noting, he being one of the proponents of the question hour in the present Constitution so as to conform more fully to a system of separation of
amendment to make the appearance of department heads discretionary in the question hour. powers.88 To that extent, the question hour, as it is presently understood in this jurisdiction, departs from
the question period of the parliamentary system. That department heads may not be required to appear in
So clearly was this distinction conveyed to the members of the Commission that the Committee on Style, a question hour does not, however, mean that the legislature is rendered powerless to elicit information
precisely in recognition of this distinction, later moved the provision on question hour from its original from them in all circumstances. In fact, in light of the absence of a mandatory question period, the need to
position as Section 20 in the original draft down to Section 31, far from the provision on inquiries in aid of enforce Congress’ right to executive information in the performance of its legislative function becomes
legislation. This gave rise to the following exchange during the deliberations: more imperative. As Schwartz observes:

MR. GUINGONA. [speaking in his capacity as Chairman of the Committee on Style] We now go, Mr. Indeed, if the separation of powers has anything to tell us on the subject under discussion, it is that the
Presiding Officer, to the Article on Legislative and may I request the chairperson of the Legislative Congress has the right to obtain information from any source – even from officials of departments and
Department, Commissioner Davide, to give his reaction. agencies in the executive branch. In the United States there is, unlike the situation which prevails in a
parliamentary system such as that in Britain, a clear separation between the legislative and executive
THE PRESIDING OFFICER (Mr. Jamir). Commissioner Davide is recognized.|avvphi|.net branches. It is this very separation that makes the congressional right to obtain information from the

43
executive so essential, if the functions of the Congress as the elected representatives of the people are Section 1 cannot, however, be applied to appearances of department heads in inquiries in aid of
adequately to be carried out. The absence of close rapport between the legislative and executive legislation. Congress is not bound in such instances to respect the refusal of the department head to
branches in this country, comparable to those which exist under a parliamentary system, and the appear in such inquiry, unless a valid claim of privilege is subsequently made, either by the President
nonexistence in the Congress of an institution such as the British question period have perforce made herself or by the Executive Secretary.
reliance by the Congress upon its right to obtain information from the executive essential, if it is
intelligently to perform its legislative tasks. Unless the Congress possesses the right to obtain executive Validity of Sections 2 and 3
information, its power of oversight of administration in a system such as ours becomes a power devoid of
most of its practical content, since it depends for its effectiveness solely upon information parceled out ex
gratia by the executive.89 (Emphasis and underscoring supplied) Section 3 of E.O. 464 requires all the public officials enumerated in Section 2(b) to secure the consent of
the President prior to appearing before either house of Congress. The enumeration is broad. It covers all
senior officials of executive departments, all officers of the AFP and the PNP, and all senior national
Sections 21 and 22, therefore, while closely related and complementary to each other, should not be security officials who, in the judgment of the heads of offices designated in the same section (i.e.
considered as pertaining to the same power of Congress. One specifically relates to the power to conduct department heads, Chief of Staff of the AFP, Chief of the PNP, and the National Security Adviser), are
inquiries in aid of legislation, the aim of which is to elicit information that may be used for legislation, while "covered by the executive privilege."
the other pertains to the power to conduct a question hour, the objective of which is to obtain information
in pursuit of Congress’ oversight function.
The enumeration also includes such other officers as may be determined by the President. Given the title
of Section 2 — "Nature, Scope and Coverage of Executive Privilege" —, it is evident that under the rule of
When Congress merely seeks to be informed on how department heads are implementing the statutes ejusdem generis, the determination by the President under this provision is intended to be based on a
which it has issued, its right to such information is not as imperative as that of the President to whom, as similar finding of coverage under executive privilege.
Chief Executive, such department heads must give a report of their performance as a matter of duty. In
such instances, Section 22, in keeping with the separation of powers, states that Congress may only
request their appearance. Nonetheless, when the inquiry in which Congress requires their appearance is En passant, the Court notes that Section 2(b) of E.O. 464 virtually states that executive privilege actually
"in aid of legislation" under Section 21, the appearance is mandatory for the same reasons stated in covers persons. Such is a misuse of the doctrine. Executive privilege, as discussed above, is properly
Arnault.90 invoked in relation to specific categories of information and not to categories of persons.

In fine, the oversight function of Congress may be facilitated by compulsory process only to the extent that In light, however, of Sec 2(a) of E.O. 464 which deals with the nature, scope and coverage of executive
it is performed in pursuit of legislation. This is consistent with the intent discerned from the deliberations of privilege, the reference to persons being "covered by the executive privilege" may be read as an
the Constitutional Commission. abbreviated way of saying that the person is in possession of information which is, in the judgment of the
head of office concerned, privileged as defined in Section 2(a). The Court shall thus proceed on the
assumption that this is the intention of the challenged order.
Ultimately, the power of Congress to compel the appearance of executive officials under Section 21 and
the lack of it under Section 22 find their basis in the principle of separation of powers. While the executive
branch is a co-equal branch of the legislature, it cannot frustrate the power of Congress to legislate by Upon a determination by the designated head of office or by the President that an official is "covered by
refusing to comply with its demands for information. the executive privilege," such official is subjected to the requirement that he first secure the consent of the
President prior to appearing before Congress. This requirement effectively bars the appearance of the
official concerned unless the same is permitted by the President. The proviso allowing the President to
When Congress exercises its power of inquiry, the only way for department heads to exempt themselves give its consent means nothing more than that the President may reverse a prohibition which already
therefrom is by a valid claim of privilege. They are not exempt by the mere fact that they are department exists by virtue of E.O. 464.
heads. Only one executive official may be exempted from this power — the President on whom executive
power is vested, hence, beyond the reach of Congress except through the power of impeachment. It is
based on her being the highest official of the executive branch, and the due respect accorded to a co- Thus, underlying this requirement of prior consent is the determination by a head of office, authorized by
equal branch of government which is sanctioned by a long-standing custom. the President under E.O. 464, or by the President herself, that such official is in possession of information
that is covered by executive privilege. This determination then becomes the basis for the official’s not
showing up in the legislative investigation.
By the same token, members of the Supreme Court are also exempt from this power of inquiry. Unlike the
Presidency, judicial power is vested in a collegial body; hence, each member thereof is exempt on the
basis not only of separation of powers but also on the fiscal autonomy and the constitutional In view thereof, whenever an official invokes E.O. 464 to justify his failure to be present, such invocation
independence of the judiciary. This point is not in dispute, as even counsel for the Senate, Sen. Joker must be construed as a declaration to Congress that the President, or a head of office authorized by the
Arroyo, admitted it during the oral argument upon interpellation of the Chief Justice. President, has determined that the requested information is privileged, and that the President has not
reversed such determination. Such declaration, however, even without mentioning the term "executive
privilege," amounts to an implied claim that the information is being withheld by the executive branch, by
Having established the proper interpretation of Section 22, Article VI of the Constitution, the Court now authority of the President, on the basis of executive privilege. Verily, there is an implied claim of privilege.
proceeds to pass on the constitutionality of Section 1 of E.O. 464.
The letter dated September 28, 2005 of respondent Executive Secretary Ermita to Senate President
Section 1, in view of its specific reference to Section 22 of Article VI of the Constitution and the absence of Drilon illustrates the implied nature of the claim of privilege authorized by E.O. 464. It reads:
any reference to inquiries in aid of legislation, must be construed as limited in its application to
appearances of department heads in the question hour contemplated in the provision of said Section 22 of
Article VI. The reading is dictated by the basic rule of construction that issuances must be interpreted, as In connection with the inquiry to be conducted by the Committee of the Whole regarding the Northrail
much as possible, in a way that will render it constitutional. Project of the North Luzon Railways Corporation on 29 September 2005 at 10:00 a.m., please be
informed that officials of the Executive Department invited to appear at the meeting will not be able to
attend the same without the consent of the President, pursuant to Executive Order No. 464 (s. 2005),
The requirement then to secure presidential consent under Section 1, limited as it is only to appearances entitled "Ensuring Observance Of The Principle Of Separation Of Powers, Adherence To The Rule On
in the question hour, is valid on its face. For under Section 22, Article VI of the Constitution, the Executive Privilege And Respect For The Rights Of Public Officials Appearing In Legislative Inquiries In
appearance of department heads in the question hour is discretionary on their part.

44
Aid Of Legislation Under The Constitution, And For Other Purposes". Said officials have not secured the A claim of privilege, being a claim of exemption from an obligation to disclose information, must, therefore,
required consent from the President. (Underscoring supplied) be clearly asserted. As U.S. v. Reynolds teaches:

The letter does not explicitly invoke executive privilege or that the matter on which these officials are The privilege belongs to the government and must be asserted by it; it can neither be claimed nor waived
being requested to be resource persons falls under the recognized grounds of the privilege to justify their by a private party. It is not to be lightly invoked. There must be a formal claim of privilege, lodged by the
absence. Nor does it expressly state that in view of the lack of consent from the President under E.O. 464, head of the department which has control over the matter, after actual personal consideration by that
they cannot attend the hearing. officer. The court itself must determine whether the circumstances are appropriate for the claim of
privilege, and yet do so without forcing a disclosure of the very thing the privilege is designed to
Significant premises in this letter, however, are left unstated, deliberately or not. The letter assumes that protect.92 (Underscoring supplied)
the invited officials are covered by E.O. 464. As explained earlier, however, to be covered by the order
means that a determination has been made, by the designated head of office or the President, that the Absent then a statement of the specific basis of a claim of executive privilege, there is no way of
invited official possesses information that is covered by executive privilege. Thus, although it is not stated determining whether it falls under one of the traditional privileges, or whether, given the circumstances in
in the letter that such determination has been made, the same must be deemed implied. Respecting the which it is made, it should be respected.93 These, in substance, were the same criteria in assessing the
statement that the invited officials have not secured the consent of the President, it only means that the claim of privilege asserted against the Ombudsman in Almonte v. Vasquez94 and, more in point, against
President has not reversed the standing prohibition against their appearance before Congress. a committee of the Senate in Senate Select Committee on Presidential Campaign Activities v. Nixon.95

Inevitably, Executive Secretary Ermita’s letter leads to the conclusion that the executive branch, either A.O. Smith v. Federal Trade Commission is enlightening:
through the President or the heads of offices authorized under E.O. 464, has made a determination that
the information required by the Senate is privileged, and that, at the time of writing, there has been no [T]he lack of specificity renders an assessment of the potential harm resulting from disclosure impossible,
contrary pronouncement from the President. In fine, an implied claim of privilege has been made by the thereby preventing the Court from balancing such harm against plaintiffs’ needs to determine whether to
executive. override any claims of privilege.96 (Underscoring supplied)

While there is no Philippine case that directly addresses the issue of whether executive privilege may be And so is U.S. v. Article of Drug:97
invoked against Congress, it is gathered from Chavez v. PEA that certain information in the possession of
the executive may validly be claimed as privileged even against Congress. Thus, the case holds:
On the present state of the record, this Court is not called upon to perform this balancing operation. In
stating its objection to claimant’s interrogatories, government asserts, and nothing more, that the
There is no claim by PEA that the information demanded by petitioner is privileged information rooted in disclosures sought by claimant would inhibit the free expression of opinion that non-disclosure is designed
the separation of powers. The information does not cover Presidential conversations, correspondences, or to protect. The government has not shown – nor even alleged – that those who evaluated claimant’s
discussions during closed-door Cabinet meetings which, like internal-deliberations of the Supreme Court product were involved in internal policymaking, generally, or in this particular instance. Privilege cannot be
and other collegiate courts, or executive sessions of either house of Congress, are recognized as set up by an unsupported claim. The facts upon which the privilege is based must be established. To find
confidential. This kind of information cannot be pried open by a co-equal branch of government. A frank these interrogatories objectionable, this Court would have to assume that the evaluation and classification
exchange of exploratory ideas and assessments, free from the glare of publicity and pressure by of claimant’s products was a matter of internal policy formulation, an assumption in which this Court is
interested parties, is essential to protect the independence of decision-making of those tasked to exercise unwilling to indulge sua sponte.98 (Emphasis and underscoring supplied)
Presidential, Legislative and Judicial power. This is not the situation in the instant case.91 (Emphasis and
underscoring supplied)
Mobil Oil Corp. v. Department of Energy99 similarly emphasizes that "an agency must provide ‘precise
and certain’ reasons for preserving the confidentiality of requested information."
Section 3 of E.O. 464, therefore, cannot be dismissed outright as invalid by the mere fact that it sanctions
claims of executive privilege. This Court must look further and assess the claim of privilege authorized by
the Order to determine whether it is valid. Black v. Sheraton Corp. of America100 amplifies, thus:

While the validity of claims of privilege must be assessed on a case to case basis, examining the ground A formal and proper claim of executive privilege requires a specific designation and description of the
invoked therefor and the particular circumstances surrounding it, there is, in an implied claim of privilege, documents within its scope as well as precise and certain reasons for preserving their confidentiality.
a defect that renders it invalid per se. By its very nature, and as demonstrated by the letter of respondent Without this specificity, it is impossible for a court to analyze the claim short of disclosure of the very thing
Executive Secretary quoted above, the implied claim authorized by Section 3 of E.O. 464 is not sought to be protected. As the affidavit now stands, the Court has little more than its sua sponte
accompanied by any specific allegation of the basis thereof (e.g., whether the information demanded speculation with which to weigh the applicability of the claim. An improperly asserted claim of privilege is
involves military or diplomatic secrets, closed-door Cabinet meetings, etc.). While Section 2(a) no claim of privilege. Therefore, despite the fact that a claim was made by the proper executive as
enumerates the types of information that are covered by the privilege under the challenged order, Reynolds requires, the Court can not recognize the claim in the instant case because it is legally
Congress is left to speculate as to which among them is being referred to by the executive. The insufficient to allow the Court to make a just and reasonable determination as to its applicability. To
enumeration is not even intended to be comprehensive, but a mere statement of what is included in the recognize such a broad claim in which the Defendant has given no precise or compelling reasons to
phrase "confidential or classified information between the President and the public officers covered by this shield these documents from outside scrutiny, would make a farce of the whole procedure.101 (Emphasis
executive order." and underscoring supplied)

Certainly, Congress has the right to know why the executive considers the requested information Due respect for a co-equal branch of government, moreover, demands no less than a claim of privilege
privileged. It does not suffice to merely declare that the President, or an authorized head of office, has clearly stating the grounds therefor. Apropos is the following ruling in McPhaul v. U.S:102
determined that it is so, and that the President has not overturned that determination. Such declaration
leaves Congress in the dark on how the requested information could be classified as privileged. That the We think the Court’s decision in United States v. Bryan, 339 U.S. 323, 70 S. Ct. 724, is highly relevant to
message is couched in terms that, on first impression, do not seem like a claim of privilege only makes it these questions. For it is as true here as it was there, that ‘if (petitioner) had legitimate reasons for failing
more pernicious. It threatens to make Congress doubly blind to the question of why the executive branch to produce the records of the association, a decent respect for the House of Representatives, by whose
is not providing it with the information that it has requested. authority the subpoenas issued, would have required that (he) state (his) reasons for noncompliance upon

45
the return of the writ. Such a statement would have given the Subcommittee an opportunity to avoid the the President," which means that he personally consulted with her. The privilege being an extraordinary
blocking of its inquiry by taking other appropriate steps to obtain the records. ‘To deny the Committee the power, it must be wielded only by the highest official in the executive hierarchy. In other words, the
opportunity to consider the objection or remedy is in itself a contempt of its authority and an obstruction of President may not authorize her subordinates to exercise such power. There is even less reason to
its processes. His failure to make any such statement was "a patent evasion of the duty of one summoned uphold such authorization in the instant case where the authorization is not explicit but by mere silence.
to produce papers before a congressional committee[, and] cannot be condoned." (Emphasis and Section 3, in relation to Section 2(b), is further invalid on this score.
underscoring supplied; citations omitted)
It follows, therefore, that when an official is being summoned by Congress on a matter which, in his own
Upon the other hand, Congress must not require the executive to state the reasons for the claim with such judgment, might be covered by executive privilege, he must be afforded reasonable time to inform the
particularity as to compel disclosure of the information which the privilege is meant to protect.103 A useful President or the Executive Secretary of the possible need for invoking the privilege. This is necessary in
analogy in determining the requisite degree of particularity would be the privilege against self- order to provide the President or the Executive Secretary with fair opportunity to consider whether the
incrimination. Thus, Hoffman v. U.S.104 declares: matter indeed calls for a claim of executive privilege. If, after the lapse of that reasonable time, neither the
President nor the Executive Secretary invokes the privilege, Congress is no longer bound to respect the
The witness is not exonerated from answering merely because he declares that in so doing he would failure of the official to appear before Congress and may then opt to avail of the necessary legal means to
incriminate himself – his say-so does not of itself establish the hazard of incrimination. It is for the court to compel his appearance.
say whether his silence is justified, and to require him to answer if ‘it clearly appears to the court that he is
mistaken.’ However, if the witness, upon interposing his claim, were required to prove the hazard in the The Court notes that one of the expressed purposes for requiring officials to secure the consent of the
sense in which a claim is usually required to be established in court, he would be compelled to surrender President under Section 3 of E.O. 464 is to ensure "respect for the rights of public officials appearing in
the very protection which the privilege is designed to guarantee. To sustain the privilege, it need only be inquiries in aid of legislation." That such rights must indeed be respected by Congress is an echo from
evident from the implications of the question, in the setting in which it is asked, that a responsive answer Article VI Section 21 of the Constitution mandating that "[t]he rights of persons appearing in or affected by
to the question or an explanation of why it cannot be answered might be dangerous because injurious such inquiries shall be respected."
disclosure could result." x x x (Emphasis and underscoring supplied)
In light of the above discussion of Section 3, it is clear that it is essentially an authorization for implied
The claim of privilege under Section 3 of E.O. 464 in relation to Section 2(b) is thus invalid per se. It is not claims of executive privilege, for which reason it must be invalidated. That such authorization is partly
asserted. It is merely implied. Instead of providing precise and certain reasons for the claim, it merely motivated by the need to ensure respect for such officials does not change the infirm nature of the
invokes E.O. 464, coupled with an announcement that the President has not given her consent. It is authorization itself.
woefully insufficient for Congress to determine whether the withholding of information is justified under the
circumstances of each case. It severely frustrates the power of inquiry of Congress. Right to Information

In fine, Section 3 and Section 2(b) of E.O. 464 must be invalidated. E.O 464 is concerned only with the demands of Congress for the appearance of executive officials in the
hearings conducted by it, and not with the demands of citizens for information pursuant to their right to
No infirmity, however, can be imputed to Section 2(a) as it merely provides guidelines, binding only on the information on matters of public concern. Petitioners are not amiss in claiming, however, that what is
heads of office mentioned in Section 2(b), on what is covered by executive privilege. It does not purport to involved in the present controversy is not merely the legislative power of inquiry, but the right of the
be conclusive on the other branches of government. It may thus be construed as a mere expression of people to information.
opinion by the President regarding the nature and scope of executive privilege.
There are, it bears noting, clear distinctions between the right of Congress to information which underlies
Petitioners, however, assert as another ground for invalidating the challenged order the alleged unlawful the power of inquiry and the right of the people to information on matters of public concern. For one, the
delegation of authority to the heads of offices in Section 2(b). Petitioner Senate of the Philippines, in demand of a citizen for the production of documents pursuant to his right to information does not have the
particular, cites the case of the United States where, so it claims, only the President can assert executive same obligatory force as a subpoena duces tecum issued by Congress. Neither does the right to
privilege to withhold information from Congress. information grant a citizen the power to exact testimony from government officials. These powers belong
only to Congress and not to an individual citizen.
Section 2(b) in relation to Section 3 virtually provides that, once the head of office determines that a
certain information is privileged, such determination is presumed to bear the President’s authority and has Thus, while Congress is composed of representatives elected by the people, it does not follow, except in a
the effect of prohibiting the official from appearing before Congress, subject only to the express highly qualified sense, that in every exercise of its power of inquiry, the people are exercising their right to
pronouncement of the President that it is allowing the appearance of such official. These provisions thus information.
allow the President to authorize claims of privilege by mere silence.
To the extent that investigations in aid of legislation are generally conducted in public, however, any
Such presumptive authorization, however, is contrary to the exceptional nature of the privilege. Executive executive issuance tending to unduly limit disclosures of information in such investigations necessarily
privilege, as already discussed, is recognized with respect to information the confidential nature of which deprives the people of information which, being presumed to be in aid of legislation, is presumed to be a
is crucial to the fulfillment of the unique role and responsibilities of the executive branch,105 or in those matter of public concern. The citizens are thereby denied access to information which they can use in
instances where exemption from disclosure is necessary to the discharge of highly important executive formulating their own opinions on the matter before Congress — opinions which they can then
responsibilities.106 The doctrine of executive privilege is thus premised on the fact that certain communicate to their representatives and other government officials through the various legal means
informations must, as a matter of necessity, be kept confidential in pursuit of the public interest. The allowed by their freedom of expression. Thus holds Valmonte v. Belmonte:
privilege being, by definition, an exemption from the obligation to disclose information, in this case to
Congress, the necessity must be of such high degree as to outweigh the public interest in enforcing that It is in the interest of the State that the channels for free political discussion be maintained to the end that
obligation in a particular case. the government may perceive and be responsive to the people’s will. Yet, this open dialogue can be
effective only to the extent that the citizenry is informed and thus able to formulate its will intelligently.
In light of this highly exceptional nature of the privilege, the Court finds it essential to limit to the President Only when the participants in the discussion are aware of the issues and have access to information
the power to invoke the privilege. She may of course authorize the Executive Secretary to invoke the relating thereto can such bear fruit.107 (Emphasis and underscoring supplied)
privilege on her behalf, in which case the Executive Secretary must state that the authority is "By order of
46
The impairment of the right of the people to information as a consequence of E.O. 464 is, therefore, in the These are motions seeking reconsideration of our decision dismissing the petitions filed in these cases for
sense explained above, just as direct as its violation of the legislature’s power of inquiry. the declaration of unconstitutionality of R.A. No. 7716, otherwise known as the Expanded Value-Added
Tax Law. The motions, of which there are 10 in all, have been filed by the several petitioners in these
Implementation of E.O. 464 prior to its publication cases, with the exception of the Philippine Educational Publishers Association, Inc. and the Association of
Philippine Booksellers, petitioners in G.R. No. 115931.
While E.O. 464 applies only to officials of the executive branch, it does not follow that the same is exempt
from the need for publication. On the need for publishing even those statutes that do not directly apply to The Solicitor General, representing the respondents, filed a consolidated comment, to which the
people in general, Tañada v. Tuvera states: Philippine Airlines, Inc., petitioner in G.R. No. 115852, and the Philippine Press Institute, Inc., petitioner in
G.R. No. 115544, and Juan T. David, petitioner in G.R. No. 115525, each filed a reply. In turn the Solicitor
General filed on June 1, 1995 a rejoinder to the PPI's reply.
The term "laws" should refer to all laws and not only to those of general application, for strictly speaking
all laws relate to the people in general albeit there are some that do not apply to them directly. An
example is a law granting citizenship to a particular individual, like a relative of President Marcos who was On June 27, 1995 the matter was submitted for resolution.
decreed instant naturalization. It surely cannot be said that such a law does not affect the public although
it unquestionably does not apply directly to all the people. The subject of such law is a matter of public I. Power of the Senate to propose amendments to revenue bills. Some of the petitioners (Tolentino,
interest which any member of the body politic may question in the political forums or, if he is a proper Kilosbayan, Inc., Philippine Airlines (PAL), Roco, and Chamber of Real Estate and Builders Association
party, even in courts of justice.108 (Emphasis and underscoring supplied) (CREBA)) reiterate previous claims made by them that R.A. No. 7716 did not "originate exclusively" in the
House of Representatives as required by Art. VI, §24 of the Constitution. Although they admit that H. No.
Although the above statement was made in reference to statutes, logic dictates that the challenged order 11197 was filed in the House of Representatives where it passed three readings and that afterward it was
must be covered by the publication requirement. As explained above, E.O. 464 has a direct effect on the sent to the Senate where after first reading it was referred to the Senate Ways and Means Committee,
right of the people to information on matters of public concern. It is, therefore, a matter of public interest they complain that the Senate did not pass it on second and third readings. Instead what the Senate did
which members of the body politic may question before this Court. Due process thus requires that the was to pass its own version (S. No. 1630) which it approved on May 24, 1994. Petitioner Tolentino adds
people should have been apprised of this issuance before it was implemented. that what the Senate committee should have done was to amend H. No. 11197 by striking out the text of
the bill and substituting it with the text of S. No. 1630. That way, it is said, "the bill remains a House bill
and the Senate version just becomes the text (only the text) of the House bill."
Conclusion
The contention has no merit.
Congress undoubtedly has a right to information from the executive branch whenever it is sought in aid of
legislation. If the executive branch withholds such information on the ground that it is privileged, it must so
assert it and state the reason therefor and why it must be respected. The enactment of S. No. 1630 is not the only instance in which the Senate proposed an amendment to a
House revenue bill by enacting its own version of a revenue bill. On at least two occasions during
the Eighth Congress, the Senate passed its own version of revenue bills, which, in consolidation with
The infirm provisions of E.O. 464, however, allow the executive branch to evade congressional requests House bills earlier passed, became the enrolled bills. These were:
for information without need of clearly asserting a right to do so and/or proffering its reasons therefor. By
the mere expedient of invoking said provisions, the power of Congress to conduct inquiries in aid of
legislation is frustrated. That is impermissible. For R.A. No. 7369 (AN ACT TO AMEND THE OMNIBUS INVESTMENTS CODE OF 1987 BY EXTENDING
FROM FIVE (5) YEARS TO TEN YEARS THE PERIOD FOR TAX AND DUTY EXEMPTION AND TAX
CREDIT ON CAPITAL EQUIPMENT) which was approved by the President on April 10, 1992. This Act is
[w]hat republican theory did accomplish…was to reverse the old presumption in favor of secrecy, based actually a consolidation of H. No. 34254, which was approved by the House on January 29, 1992, and S.
on the divine right of kings and nobles, and replace it with a presumption in favor of publicity, based on the No. 1920, which was approved by the Senate on February 3, 1992.
doctrine of popular sovereignty. (Underscoring supplied)109
R.A. No. 7549 (AN ACT GRANTING TAX EXEMPTIONS TO WHOEVER SHALL GIVE REWARD TO
Resort to any means then by which officials of the executive branch could refuse to divulge information ANY FILIPINO ATHLETE WINNING A MEDAL IN OLYMPIC GAMES) which was approved by the
cannot be presumed valid. Otherwise, we shall not have merely nullified the power of our legislature to President on May 22, 1992. This Act is a consolidation of H. No. 22232, which was approved by the
inquire into the operations of government, but we shall have given up something of much greater value – House of Representatives on August 2, 1989, and S. No. 807, which was approved by the Senate on
our right as a people to take part in government. October 21, 1991.

WHEREFORE, the petitions are PARTLY GRANTED. Sections 2(b) and 3 of Executive Order No. 464 On the other hand, the Ninth Congress passed revenue laws which were also the result of the
(series of 2005), "Ensuring Observance of the Principle of Separation of Powers, Adherence to the Rule consolidation of House and Senate bills. These are the following, with indications of the dates on which
on Executive the laws were approved by the President and dates the separate bills of the two chambers of Congress
were respectively passed:
Privilege and Respect for the Rights of Public Officials Appearing in Legislative Inquiries in Aid of
Legislation Under the Constitution, and For Other Purposes," are declared VOID. Sections 1 and 2(a) are, 1. R.A. NO. 7642
however, VALID. SO ORDERED AN ACT INCREASING THE PENALTIES FOR TAX EVASION, AMENDING FOR THIS PURPOSE THE
PERTINENT SECTIONS OF THE NATIONAL INTERNAL REVENUE CODE (December 28, 1992).
G.R. No. 115455 October 30, 1995 House Bill No. 2165, October 5, 1992
Senate Bill No. 32, December 7, 1992
ARTURO M. TOLENTINO, vs. 2. R.A. NO. 7643
THE SECRETARY OF FINANCE AN ACT TO EMPOWER THE COMMISSIONER OF INTERNAL REVENUE TO REQUIRE THE
PAYMENT OF THE VALUE-ADDED TAX EVERY MONTH AND TO ALLOW LOCAL GOVERNMENT
UNITS TO SHARE IN VAT REVENUE, AMENDING FOR THIS PURPOSE CERTAIN SECTIONS OF
THE NATIONAL INTERNAL REVENUE CODE (December 28, 1992)
47
House Bill No. 1503, September 3, 1992 No amendment by substitution shall be entertained unless the text thereof is submitted in writing.
Senate Bill No. 968, December 7, 1992
3. R.A. NO. 7646 Any of said amendments may be withdrawn before a vote is taken thereon.
AN ACT AUTHORIZING THE COMMISSIONER OF INTERNAL REVENUE TO PRESCRIBE THE PLACE
FOR PAYMENT OF INTERNAL REVENUE TAXES BY LARGE TAXPAYERS, AMENDING FOR THIS
PURPOSE CERTAIN PROVISIONS OF THE NATIONAL INTERNAL REVENUE CODE, AS AMENDED §69. No amendment which seeks the inclusion of a legislative provision foreign to the subject matter of a
(February 24, 1993) bill (rider) shall be entertained.

House Bill No. 1470, October 20, 1992 §70-A. A bill or resolution shall not be amended by substituting it with another which covers a subject
Senate Bill No. 35, November 19, 1992 distinct from that proposed in the original bill or resolution. (emphasis added).
4. R.A. NO. 7649
AN ACT REQUIRING THE GOVERNMENT OR ANY OF ITS POLITICAL SUBDIVISIONS, Nor is there merit in petitioners' contention that, with regard to revenue bills, the Philippine Senate
INSTRUMENTALITIES OR AGENCIES INCLUDING GOVERNMENT-OWNED OR CONTROLLED possesses less power than the U.S. Senate because of textual differences between constitutional
CORPORATIONS (GOCCS) TO DEDUCT AND WITHHOLD THE VALUE-ADDED TAX DUE AT THE provisions giving them the power to propose or concur with amendments.
RATE OF THREE PERCENT (3%) ON GROSS PAYMENT FOR THE PURCHASE OF GOODS AND SIX
PERCENT (6%) ON GROSS RECEIPTS FOR SERVICES RENDERED BY CONTRACTORS (April 6, Art. I, §7, cl. 1 of the U.S. Constitution reads:
1993)
All Bills for raising Revenue shall originate in the House of Representatives; but the Senate may propose
House Bill No. 5260, January 26, 1993 or concur with amendments as on other Bills.
Senate Bill No. 1141, March 30, 1993
5. R.A. NO. 7656
Art. VI, §24 of our Constitution reads:
AN ACT REQUIRING GOVERNMENT-OWNED OR CONTROLLED CORPORATIONS TO DECLARE
DIVIDENDS UNDER CERTAIN CONDITIONS TO THE NATIONAL GOVERNMENT, AND FOR OTHER
PURPOSES (November 9, 1993) All appropriation, revenue or tariff bills, bills authorizing increase of the public debt, bills of local
application, and private bills shall originate exclusively in the House of Representatives, but the Senate
may propose or concur with amendments.
House Bill No. 11024, November 3, 1993
Senate Bill No. 1168, November 3, 1993
6. R.A. NO. 7660 The addition of the word "exclusively" in the Philippine Constitution and the decision to drop the phrase
AN ACT RATIONALIZING FURTHER THE STRUCTURE AND ADMINISTRATION OF THE "as on other Bills" in the American version, according to petitioners, shows the intention of the framers of
DOCUMENTARY STAMP TAX, AMENDING FOR THE PURPOSE CERTAIN PROVISIONS OF THE our Constitution to restrict the Senate's power to propose amendments to revenue bills. Petitioner
NATIONAL INTERNAL REVENUE CODE, AS AMENDED, ALLOCATING FUNDS FOR SPECIFIC Tolentino contends that the word "exclusively" was inserted to modify "originate" and "the words 'as in
PROGRAMS, AND FOR OTHER PURPOSES (December 23, 1993) any other bills' (sic) were eliminated so as to show that these bills were not to be like other bills but must
be treated as a special kind."
House Bill No. 7789, May 31, 1993
Senate Bill No. 1330, November 18, 1993 The history of this provision does not support this contention. The supposed indicia of constitutional intent
7. R.A. NO. 7717 are nothing but the relics of an unsuccessful attempt to limit the power of the Senate. It will be recalled
AN ACT IMPOSING A TAX ON THE SALE, BARTER OR EXCHANGE OF SHARES OF STOCK LISTED that the 1935 Constitution originally provided for a unicameral National Assembly. When it was decided in
AND TRADED THROUGH THE LOCAL STOCK EXCHANGE OR THROUGH INITIAL PUBLIC 1939 to change to a bicameral legislature, it became necessary to provide for the procedure for
OFFERING, AMENDING FOR THE PURPOSE THE NATIONAL INTERNAL REVENUE CODE, AS lawmaking by the Senate and the House of Representatives. The work of proposing amendments to the
AMENDED, BY INSERTING A NEW SECTION AND REPEALING CERTAIN SUBSECTIONS THEREOF Constitution was done by the National Assembly, acting as a constituent assembly, some of whose
(May 5, 1994) members, jealous of preserving the Assembly's lawmaking powers, sought to curtail the powers of the
House Bill No. 9187, November 3, 1993 proposed Senate. Accordingly they proposed the following provision:
Senate Bill No. 1127, March 23, 1994
Thus, the enactment of S. No. 1630 is not the only instance in which the Senate, in the exercise of its All bills appropriating public funds, revenue or tariff bills, bills of local application, and private bills shall
power to propose amendments to bills required to originate in the House, passed its own version of a originate exclusively in the Assembly, but the Senate may propose or concur with amendments. In case of
House revenue measure. It is noteworthy that, in the particular case of S. No. 1630, petitioners Tolentino disapproval by the Senate of any such bills, the Assembly may repass the same by a two-thirds vote of all
and Roco, as members of the Senate, voted to approve it on second and third readings. its members, and thereupon, the bill so repassed shall be deemed enacted and may be submitted to the
President for corresponding action. In the event that the Senate should fail to finally act on any such bills,
On the other hand, amendment by substitution, in the manner urged by petitioner Tolentino, concerns a the Assembly may, after thirty days from the opening of the next regular session of the same legislative
mere matter of form. Petitioner has not shown what substantial difference it would make if, as the Senate term, reapprove the same with a vote of two-thirds of all the members of the Assembly. And upon such
actually did in this case, a separate bill like S. No. 1630 is instead enacted as a substitute measure, reapproval, the bill shall be deemed enacted and may be submitted to the President for corresponding
"taking into Consideration . . . H.B. 11197." action.

Indeed, so far as pertinent, the Rules of the Senate only provide: The special committee on the revision of laws of the Second National Assembly vetoed the proposal. It
deleted everything after the first sentence. As rewritten, the proposal was approved by the National
Assembly and embodied in Resolution No. 38, as amended by Resolution No. 73. (J. ARUEGO, KNOW
RULE XXIX - AMENDMENTS
YOUR CONSTITUTION 65-66 (1950)). The proposed amendment was submitted to the people and
ratified by them in the elections held on June 18, 1940.
§68. Not more than one amendment to the original amendment shall be considered.

48
This is the history of Art. VI, §18 (2) of the 1935 Constitution, from which Art. VI, §24 of the present and H.B. No. 11197," implying that there is something substantially different between the reference to S.
Constitution was derived. It explains why the word "exclusively" was added to the American text from No. 1129 and the reference to H. No. 11197. From this premise, they conclude that R.A. No. 7716
which the framers of the Philippine Constitution borrowed and why the phrase "as on other Bills" was not originated both in the House and in the Senate and that it is the product of two "half-baked bills because
copied. Considering the defeat of the proposal, the power of the Senate to propose amendments must be neither H. No. 11197 nor S. No. 1630 was passed by both houses of Congress."
understood to be full, plenary and complete "as on other Bills." Thus, because revenue bills are required
to originate exclusively in the House of Representatives, the Senate cannot enact revenue measures of its In point of fact, in several instances the provisions of S. No. 1630, clearly appear to be mere amendments
own without such bills. After a revenue bill is passed and sent over to it by the House, however, the of the corresponding provisions of H. No. 11197. The very tabular comparison of the provisions of H. No.
Senate certainly can pass its own version on the same subject matter. This follows from the coequality of 11197 and S. No. 1630 attached as Supplement A to the basic petition of petitioner Tolentino, while
the two chambers of Congress. showing differences between the two bills, at the same time indicates that the provisions of the Senate bill
were precisely intended to be amendments to the House bill.
That this is also the understanding of book authors of the scope of the Senate's power to concur is clear
from the following commentaries: Without H. No. 11197, the Senate could not have enacted S. No. 1630. Because the Senate bill was a
mere amendment of the House bill, H. No. 11197 in its original form did not have to pass the Senate on
The power of the Senate to propose or concur with amendments is apparently without restriction. It would second and three readings. It was enough that after it was passed on first reading it was referred to the
seem that by virtue of this power, the Senate can practically re-write a bill required to come from the Senate Committee on Ways and Means. Neither was it required that S. No. 1630 be passed by the House
House and leave only a trace of the original bill. For example, a general revenue bill passed by the lower of Representatives before the two bills could be referred to the Conference Committee.
house of the United States Congress contained provisions for the imposition of an inheritance tax . This
was changed by the Senate into a corporation tax. The amending authority of the Senate was declared by There is legislative precedent for what was done in the case of H. No. 11197 and S. No. 1630. When the
the United States Supreme Court to be sufficiently broad to enable it to make the alteration. [Flint v. Stone House bill and Senate bill, which became R.A. No. 1405 (Act prohibiting the disclosure of bank deposits),
Tracy Company, 220 U.S. 107, 55 L. ed. 389]. were referred to a conference committee, the question was raised whether the two bills could be the
subject of such conference, considering that the bill from one house had not been passed by the other
(L. TAÑADA AND F. CARREON, POLITICAL LAW OF THE PHILIPPINES 247 (1961)) and vice versa. As Congressman Duran put the question:

The above-mentioned bills are supposed to be initiated by the House of Representatives because it is MR. DURAN. Therefore, I raise this question of order as to procedure: If a House bill is passed by the
more numerous in membership and therefore also more representative of the people. Moreover, its House but not passed by the Senate, and a Senate bill of a similar nature is passed in the Senate but
members are presumed to be more familiar with the needs of the country in regard to the enactment of never passed in the House, can the two bills be the subject of a conference, and can a law be enacted
the legislation involved. from these two bills? I understand that the Senate bill in this particular instance does not refer to
investments in government securities, whereas the bill in the House, which was introduced by the
The Senate is, however, allowed much leeway in the exercise of its power to propose or concur with Speaker, covers two subject matters: not only investigation of deposits in banks but also investigation of
amendments to the bills initiated by the House of Representatives. Thus, in one case, a bill introduced in investments in government securities. Now, since the two bills differ in their subject matter, I believe that
the U.S. House of Representatives was changed by the Senate to make a proposed inheritance tax a no law can be enacted.
corporation tax. It is also accepted practice for the Senate to introduce what is known as an amendment
by substitution, which may entirely replace the bill initiated in the House of Representatives. Ruling on the point of order raised, the chair (Speaker Jose B. Laurel, Jr.) said:

(I. CRUZ, PHILIPPINE POLITICAL LAW 144-145 (1993)). THE SPEAKER. The report of the conference committee is in order. It is precisely in cases like this where
a conference should be had. If the House bill had been approved by the Senate, there would have been
In sum, while Art. VI, §24 provides that all appropriation, revenue or tariff bills, bills authorizing increase of no need of a conference; but precisely because the Senate passed another bill on the same subject
the public debt, bills of local application, and private bills must "originate exclusively in the House of matter, the conference committee had to be created, and we are now considering the report of that
Representatives," it also adds, "but the Senate may propose or concur with amendments." In the exercise committee.
of this power, the Senate may propose an entirely new bill as a substitute measure. As petitioner
Tolentino states in a high school text, a committee to which a bill is referred may do any of the following: (2 CONG. REC. NO. 13, July 27, 1955, pp. 3841-42 (emphasis added))

(1) to endorse the bill without changes; (2) to make changes in the bill omitting or adding sections or III. The President's certification. The fallacy in thinking that H. No. 11197 and S. No. 1630 are distinct and
altering its language; (3) to make and endorse an entirely new bill as a substitute, in which case it will be unrelated measures also accounts for the petitioners' (Kilosbayan's and PAL's) contention that because
known as a committee bill; or (4) to make no report at all. the President separately certified to the need for the immediate enactment of these measures, his
certification was ineffectual and void. The certification had to be made of the version of the same revenue
(A. TOLENTINO, THE GOVERNMENT OF THE PHILIPPINES 258 (1950)) bill which at the moment was being considered. Otherwise, to follow petitioners' theory, it would be
necessary for the President to certify as many bills as are presented in a house of Congress even though
the bills are merely versions of the bill he has already certified. It is enough that he certifies the bill which,
To except from this procedure the amendment of bills which are required to originate in the House by at the time he makes the certification, is under consideration. Since on March 22, 1994 the Senate was
prescribing that the number of the House bill and its other parts up to the enacting clause must be considering S. No. 1630, it was that bill which had to be certified. For that matter on June 1, 1993 the
preserved although the text of the Senate amendment may be incorporated in place of the original body of President had earlier certified H. No. 9210 for immediate enactment because it was the one which at that
the bill is to insist on a mere technicality. At any rate there is no rule prescribing this form. S. No. 1630, as time was being considered by the House. This bill was later substituted, together with other bills, by H. No.
a substitute measure, is therefore as much an amendment of H. No. 11197 as any which the Senate 11197.
could have made.
As to what Presidential certification can accomplish, we have already explained in the main decision that
II. S. No. 1630 a mere amendment of H. No. 11197. Petitioners' basic error is that they assume that S. the phrase "except when the President certifies to the necessity of its immediate enactment, etc." in Art.
No. 1630 is an independent and distinct bill. Hence their repeated references to its certification that it was VI, §26 (2) qualifies not only the requirement that "printed copies [of a bill] in its final form [must be]
passed by the Senate "in substitution of S.B. No. 1129, taking into consideration P.S. Res. No. 734 distributed to the members three days before its passage" but also the requirement that before a bill can
49
become a law it must have passed "three readings on separate days." There is not only textual support for IV. Power of Conference Committee. It is contended (principally by Kilosbayan, Inc. and the Movement of
such construction but historical basis as well. Attorneys for Brotherhood, Integrity and Nationalism, Inc. (MABINI)) that in violation of the constitutional
policy of full public disclosure and the people's right to know (Art. II, §28 and Art. III, §7) the Conference
Art. VI, §21 (2) of the 1935 Constitution originally provided: Committee met for two days in executive session with only the conferees present.

(2) No bill shall be passed by either House unless it shall have been printed and copies thereof in its final As pointed out in our main decision, even in the United States it was customary to hold such sessions with
form furnished its Members at least three calendar days prior to its passage, except when the President only the conferees and their staffs in attendance and it was only in 1975 when a new rule was adopted
shall have certified to the necessity of its immediate enactment. Upon the last reading of a bill, no requiring open sessions. Unlike its American counterpart, the Philippine Congress has not adopted a rule
amendment thereof shall be allowed and the question upon its passage shall be taken immediately prescribing open hearings for conference committees.
thereafter, and the yeas and nays entered on the Journal.
It is nevertheless claimed that in the United States, before the adoption of the rule in 1975, at least staff
When the 1973 Constitution was adopted, it was provided in Art. VIII, §19 (2): members were present. These were staff members of the Senators and Congressmen, however, who
may be presumed to be their confidential men, not stenographers as in this case who on the last two days
of the conference were excluded. There is no showing that the conferees themselves did not take notes of
(2) No bill shall become a law unless it has passed three readings on separate days, and printed copies their proceedings so as to give petitioner Kilosbayan basis for claiming that even in secret diplomatic
thereof in its final form have been distributed to the Members three days before its passage, except when negotiations involving state interests, conferees keep notes of their meetings. Above all, the public's right
the Prime Minister certifies to the necessity of its immediate enactment to meet a public calamity or to know was fully served because the Conference Committee in this case submitted a report showing the
emergency. Upon the last reading of a bill, no amendment thereto shall be allowed, and the vote thereon changes made on the differing versions of the House and the Senate.
shall be taken immediately thereafter, and the yeas and nays entered in the Journal.
Petitioners cite the rules of both houses which provide that conference committee reports must contain "a
This provision of the 1973 document, with slight modification, was adopted in Art. VI, §26 (2) of the detailed, sufficiently explicit statement of the changes in or other amendments." These changes are
present Constitution, thus: shown in the bill attached to the Conference Committee Report. The members of both houses could thus
ascertain what changes had been made in the original bills without the need of a statement detailing the
(2) No bill passed by either House shall become a law unless it has passed three readings on separate changes.
days, and printed copies thereof in its final form have been distributed to its Members three days before
its passage, except when the President certifies to the necessity of its immediate enactment to meet a The same question now presented was raised when the bill which became R.A. No. 1400 (Land Reform
public calamity or emergency. Upon the last reading of a bill, no amendment thereto shall be allowed, and Act of 1955) was reported by the Conference Committee. Congressman Bengzon raised a point of order.
the vote thereon shall be taken immediately thereafter, and the yeas and nays entered in the Journal. He said:

The exception is based on the prudential consideration that if in all cases three readings on separate days MR. BENGZON. My point of order is that it is out of order to consider the report of the conference
are required and a bill has to be printed in final form before it can be passed, the need for a law may be committee regarding House Bill No. 2557 by reason of the provision of Section 11, Article XII, of the Rules
rendered academic by the occurrence of the very emergency or public calamity which it is meant to of this House which provides specifically that the conference report must be accompanied by a detailed
address. statement of the effects of the amendment on the bill of the House. This conference committee report is
not accompanied by that detailed statement, Mr. Speaker. Therefore it is out of order to consider it.
Petitioners further contend that a "growing budget deficit" is not an emergency, especially in a country like
the Philippines where budget deficit is a chronic condition. Even if this were the case, an enormous Petitioner Tolentino, then the Majority Floor Leader, answered:
budget deficit does not make the need for R.A. No. 7716 any less urgent or the situation calling for its
enactment any less an emergency.
MR. TOLENTINO. Mr. Speaker, I should just like to say a few words in connection with the point of order
raised by the gentleman from Pangasinan.
Apparently, the members of the Senate (including some of the petitioners in these cases) believed that
there was an urgent need for consideration of S. No. 1630, because they responded to the call of the
President by voting on the bill on second and third readings on the same day. While the judicial There is no question about the provision of the Rule cited by the gentleman from Pangasinan, but this
department is not bound by the Senate's acceptance of the President's certification, the respect due provision applies to those cases where only portions of the bill have been amended. In this case before
coequal departments of the government in matters committed to them by the Constitution and the us an entire bill is presented; therefore, it can be easily seen from the reading of the bill what the
absence of a clear showing of grave abuse of discretion caution a stay of the judicial hand. provisions are. Besides, this procedure has been an established practice.

At any rate, we are satisfied that S. No. 1630 received thorough consideration in the Senate where it was After some interruption, he continued:
discussed for six days. Only its distribution in advance in its final printed form was actually dispensed with
by holding the voting on second and third readings on the same day (March 24, 1994). Otherwise, MR. TOLENTINO. As I was saying, Mr. Speaker, we have to look into the reason for the provisions of the
sufficient time between the submission of the bill on February 8, 1994 on second reading and its approval Rules, and the reason for the requirement in the provision cited by the gentleman from Pangasinan is
on March 24, 1994 elapsed before it was finally voted on by the Senate on third reading. when there are only certain words or phrases inserted in or deleted from the provisions of the bill included
in the conference report, and we cannot understand what those words and phrases mean and their
The purpose for which three readings on separate days is required is said to be two-fold: (1) to inform the relation to the bill. In that case, it is necessary to make a detailed statement on how those words and
members of Congress of what they must vote on and (2) to give them notice that a measure is phrases will affect the bill as a whole; but when the entire bill itself is copied verbatim in the conference
progressing through the enacting process, thus enabling them and others interested in the measure to report, that is not necessary. So when the reason for the Rule does not exist, the Rule does not exist.
prepare their positions with reference to it. (1 J. G. SUTHERLAND, STATUTES AND STATUTORY
CONSTRUCTION §10.04, p. 282 (1972)). These purposes were substantially achieved in the case of R.A. (2 CONG. REC. NO. 2, p. 4056. (emphasis added))
No. 7716.

50
Congressman Tolentino was sustained by the chair. The record shows that when the ruling was appealed, PAL was exempted from the payment of the VAT along with other entities by §103 of the National Internal
it was upheld by viva voce and when a division of the House was called, it was sustained by a vote of 48 Revenue Code, which provides as follows:
to 5. (Id.,
p. 4058) §103. Exempt transactions. — The following shall be exempt from the value-added tax:

Nor is there any doubt about the power of a conference committee to insert new provisions as long as (q) Transactions which are exempt under special laws or international agreements to which the
these are germane to the subject of the conference. As this Court held in Philippine Judges Association Philippines is a signatory.
v. Prado, 227 SCRA 703 (1993), in an opinion written by then Justice Cruz, the jurisdiction of the
conference committee is not limited to resolving differences between the Senate and the House. It may
propose an entirely new provision. What is important is that its report is subsequently approved by the R.A. No. 7716 seeks to withdraw certain exemptions, including that granted to PAL, by amending §103,
respective houses of Congress. This Court ruled that it would not entertain allegations that, because new as follows:
provisions had been added by the conference committee, there was thereby a violation of the
constitutional injunction that "upon the last reading of a bill, no amendment thereto shall be allowed." §103. Exempt transactions. — The following shall be exempt from the value-added tax:

Applying these principles, we shall decline to look into the petitioners' charges that an amendment was (q) Transactions which are exempt under special laws, except those granted under Presidential Decree
made upon the last reading of the bill that eventually became R.A. No. 7354 and that copies thereof in its Nos. 66, 529, 972, 1491, 1590. . . .
final form were not distributed among the members of each House. Both the enrolled bill and the
legislative journals certify that the measure was duly enacted i.e., in accordance with Article VI, Sec. 26 The amendment of §103 is expressed in the title of R.A. No. 7716 which reads:
(2) of the Constitution. We are bound by such official assurances from a coordinate department of the
government, to which we owe, at the very least, a becoming courtesy.
AN ACT RESTRUCTURING THE VALUE-ADDED TAX (VAT) SYSTEM, WIDENING ITS TAX BASE AND
ENHANCING ITS ADMINISTRATION, AND FOR THESE PURPOSES AMENDING AND REPEALING
(Id. at 710. (emphasis added)) THE RELEVANT PROVISIONS OF THE NATIONAL INTERNAL REVENUE CODE, AS AMENDED, AND
FOR OTHER PURPOSES.
It is interesting to note the following description of conference committees in the Philippines in a 1979
study: By stating that R.A. No. 7716 seeks to "[RESTRUCTURE] THE VALUE-ADDED TAX (VAT) SYSTEM
[BY] WIDENING ITS TAX BASE AND ENHANCING ITS ADMINISTRATION, AND FOR THESE
Conference committees may be of two types: free or instructed. These committees may be given PURPOSES AMENDING AND REPEALING THE RELEVANT PROVISIONS OF THE NATIONAL
instructions by their parent bodies or they may be left without instructions. Normally the conference INTERNAL REVENUE CODE, AS AMENDED AND FOR OTHER PURPOSES," Congress thereby clearly
committees are without instructions, and this is why they are often critically referred to as "the little expresses its intention to amend any provision of the NIRC which stands in the way of accomplishing the
legislatures." Once bills have been sent to them, the conferees have almost unlimited authority to change purpose of the law.
the clauses of the bills and in fact sometimes introduce new measures that were not in the original
legislation. No minutes are kept, and members' activities on conference committees are difficult to PAL asserts that the amendment of its franchise must be reflected in the title of the law by specific
determine. One congressman known for his idealism put it this way: "I killed a bill on export incentives for reference to P.D. No. 1590. It is unnecessary to do this in order to comply with the constitutional
my interest group [copra] in the conference committee but I could not have done so anywhere else." The requirement, since it is already stated in the title that the law seeks to amend the pertinent provisions of
conference committee submits a report to both houses, and usually it is accepted. If the report is not the NIRC, among which is §103(q), in order to widen the base of the VAT. Actually, it is the bill which
accepted, then the committee is discharged and new members are appointed. becomes a law that is required to express in its title the subject of legislation. The titles of H. No. 11197
and S. No. 1630 in fact specifically referred to §103 of the NIRC as among the provisions sought to be
(R. Jackson, Committees in the Philippine Congress, in COMMITTEES AND LEGISLATURES: A amended. We are satisfied that sufficient notice had been given of the pendency of these bills in
COMPARATIVE ANALYSIS 163 (J. D. LEES AND M. SHAW, eds.)). Congress before they were enacted into what is now R.A.
No. 7716.
In citing this study, we pass no judgment on the methods of conference committees. We cite it only to say
that conference committees here are no different from their counterparts in the United States whose vast In Philippine Judges Association v. Prado, supra, a similar argument as that now made by PAL was
powers we noted in Philippine Judges Association v. Prado, supra. At all events, under Art. VI, §16(3) rejected. R.A. No. 7354 is entitled AN ACT CREATING THE PHILIPPINE POSTAL CORPORATION,
each house has the power "to determine the rules of its proceedings," including those of its committees. DEFINING ITS POWERS, FUNCTIONS AND RESPONSIBILITIES, PROVIDING FOR REGULATION OF
Any meaningful change in the method and procedures of Congress or its committees must therefore be THE INDUSTRY AND FOR OTHER PURPOSES CONNECTED THEREWITH. It contained a provision
sought in that body itself. repealing all franking privileges. It was contended that the withdrawal of franking privileges was not
expressed in the title of the law. In holding that there was sufficient description of the subject of the law in
V. The titles of S. No. 1630 and H. No. 11197. PAL maintains that R.A. No. 7716 violates Art. VI, §26 (1) its title, including the repeal of franking privileges, this Court held:
of the Constitution which provides that "Every bill passed by Congress shall embrace only one subject
which shall be expressed in the title thereof." PAL contends that the amendment of its franchise by the To require every end and means necessary for the accomplishment of the general objectives of the
withdrawal of its exemption from the VAT is not expressed in the title of the law. statute to be expressed in its title would not only be unreasonable but would actually render legislation
impossible. [Cooley, Constitutional Limitations, 8th Ed., p. 297] As has been correctly explained:
Pursuant to §13 of P.D. No. 1590, PAL pays a franchise tax of 2% on its gross revenue "in lieu of all other
taxes, duties, royalties, registration, license and other fees and charges of any kind, nature, or description, The details of a legislative act need not be specifically stated in its title, but matter germane to the subject
imposed, levied, established, assessed or collected by any municipal, city, provincial or national authority as expressed in the title, and adopted to the accomplishment of the object in view, may properly be
or government agency, now or in the future." included in the act. Thus, it is proper to create in the same act the machinery by which the act is to be
enforced, to prescribe the penalties for its infraction, and to remove obstacles in the way of its execution.
If such matters are properly connected with the subject as expressed in the title, it is unnecessary that
they should also have special mention in the title. (Southern Pac. Co. v. Bartine, 170 Fed. 725)

51
(227 SCRA at 707-708) (c) Goods subject to excise tax such as petroleum products or to be used for manufacture of petroleum
products subject to excise tax and services subject to percentage tax.
VI. Claims of press freedom and religious liberty. We have held that, as a general proposition, the press is
not exempt from the taxing power of the State and that what the constitutional guarantee of free press (d) Educational services, medical, dental, hospital and veterinary services, and services rendered under
prohibits are laws which single out the press or target a group belonging to the press for special treatment employer-employee relationship.
or which in any way discriminate against the press on the basis of the content of the publication, and R.A. (e) Works of art and similar creations sold by the artist himself.
No. 7716 is none of these. (f) Transactions exempted under special laws, or international agreements.
(g) Export-sales by persons not VAT-registered.
Now it is contended by the PPI that by removing the exemption of the press from the VAT while (h) Goods or services with gross annual sale or receipt not exceeding P500,000.00.
maintaining those granted to others, the law discriminates against the press. At any rate, it is averred, (Respondents' Consolidated Comment on the Motions for Reconsideration, pp. 58-60)
"even nondiscriminatory taxation of constitutionally guaranteed freedom is unconstitutional."
The PPI asserts that it does not really matter that the law does not discriminate against the press because
With respect to the first contention, it would suffice to say that since the law granted the press a privilege, "even nondiscriminatory taxation on constitutionally guaranteed freedom is unconstitutional." PPI cites in
the law could take back the privilege anytime without offense to the Constitution. The reason is simple: by support of this assertion the following statement in Murdock v. Pennsylvania, 319 U.S. 105, 87 L. Ed.
granting exemptions, the State does not forever waive the exercise of its sovereign prerogative. 1292 (1943):

Indeed, in withdrawing the exemption, the law merely subjects the press to the same tax burden to which The fact that the ordinance is "nondiscriminatory" is immaterial. The protection afforded by the First
other businesses have long ago been subject. It is thus different from the tax involved in the cases Amendment is not so restricted. A license tax certainly does not acquire constitutional validity because it
invoked by the PPI. The license tax in Grosjean v. American Press Co., 297 U.S. 233, 80 L. Ed. 660 classifies the privileges protected by the First Amendment along with the wares and merchandise of
(1936) was found to be discriminatory because it was laid on the gross advertising receipts only of hucksters and peddlers and treats them all alike. Such equality in treatment does not save the ordinance.
newspapers whose weekly circulation was over 20,000, with the result that the tax applied only to 13 out Freedom of press, freedom of speech, freedom of religion are in preferred position.
of 124 publishers in Louisiana. These large papers were critical of Senator Huey Long who controlled the
state legislature which enacted the license tax. The censorial motivation for the law was thus evident. The Court was speaking in that case of a license tax, which, unlike an ordinary tax, is mainly for
regulation. Its imposition on the press is unconstitutional because it lays a prior restraint on the exercise of
On the other hand, in Minneapolis Star & Tribune Co. v. Minnesota Comm'r of Revenue, 460 U.S. 575, 75 its right. Hence, although its application to others, such those selling goods, is valid, its application to the
L. Ed. 2d 295 (1983), the tax was found to be discriminatory because although it could have been made press or to religious groups, such as the Jehovah's Witnesses, in connection with the latter's sale of
liable for the sales tax or, in lieu thereof, for the use tax on the privilege of using, storing or consuming religious books and pamphlets, is unconstitutional. As the U.S. Supreme Court put it, "it is one thing to
tangible goods, the press was not. Instead, the press was exempted from both taxes. It was, however, impose a tax on income or property of a preacher. It is quite another thing to exact a tax on him for
later made to pay a special use tax on the cost of paper and ink which made these items "the only items delivering a sermon."
subject to the use tax that were component of goods to be sold at retail." The U.S. Supreme Court held
that the differential treatment of the press "suggests that the goal of regulation is not related to A similar ruling was made by this Court in American Bible Society v. City of Manila, 101 Phil. 386 (1957)
suppression of expression, and such goal is presumptively unconstitutional." It would therefore appear which invalidated a city ordinance requiring a business license fee on those engaged in the sale of
that even a law that favors the press is constitutionally suspect. (See the dissent of Rehnquist, J. in that general merchandise. It was held that the tax could not be imposed on the sale of bibles by the American
case) Bible Society without restraining the free exercise of its right to propagate.

Nor is it true that only two exemptions previously granted by E.O. No. 273 are withdrawn "absolutely and The VAT is, however, different. It is not a license tax. It is not a tax on the exercise of a privilege, much
unqualifiedly" by R.A. No. 7716. Other exemptions from the VAT, such as those previously granted to less a constitutional right. It is imposed on the sale, barter, lease or exchange of goods or properties or
PAL, petroleum concessionaires, enterprises registered with the Export Processing Zone Authority, and the sale or exchange of services and the lease of properties purely for revenue purposes. To subject the
many more are likewise totally withdrawn, in addition to exemptions which are partially withdrawn, in an press to its payment is not to burden the exercise of its right any more than to make the press pay income
effort to broaden the base of the tax. tax or subject it to general regulation is not to violate its freedom under the Constitution.

The PPI says that the discriminatory treatment of the press is highlighted by the fact that transactions, Additionally, the Philippine Bible Society, Inc. claims that although it sells bibles, the proceeds derived
which are profit oriented, continue to enjoy exemption under R.A. No. 7716. An enumeration of some of from the sales are used to subsidize the cost of printing copies which are given free to those who cannot
these transactions will suffice to show that by and large this is not so and that the exemptions are granted afford to pay so that to tax the sales would be to increase the price, while reducing the volume of sale.
for a purpose. As the Solicitor General says, such exemptions are granted, in some cases, to encourage Granting that to be the case, the resulting burden on the exercise of religious freedom is so incidental as
agricultural production and, in other cases, for the personal benefit of the end-user rather than for profit. to make it difficult to differentiate it from any other economic imposition that might make the right to
The exempt transactions are: disseminate religious doctrines costly. Otherwise, to follow the petitioner's argument, to increase the tax
on the sale of vestments would be to lay an impermissible burden on the right of the preacher to make a
(a) Goods for consumption or use which are in their original state (agricultural, marine and forest products, sermon.
cotton seeds in their original state, fertilizers, seeds, seedlings, fingerlings, fish, prawn livestock and
poultry feeds) and goods or services to enhance agriculture (milling of palay, corn, sugar cane and raw On the other hand the registration fee of P1,000.00 imposed by §107 of the NIRC, as amended by §7 of
sugar, livestock, poultry feeds, fertilizer, ingredients used for the manufacture of feeds). R.A. No. 7716, although fixed in amount, is really just to pay for the expenses of registration and
enforcement of provisions such as those relating to accounting in §108 of the NIRC. That the PBS
(b) Goods used for personal consumption or use (household and personal effects of citizens returning to distributes free bibles and therefore is not liable to pay the VAT does not excuse it from the payment of
the Philippines) or for professional use, like professional instruments and implements, by persons coming this fee because it also sells some copies. At any rate whether the PBS is liable for the VAT must be
to the Philippines to settle here. decided in concrete cases, in the event it is assessed this tax by the Commissioner of Internal Revenue.

VII. Alleged violations of the due process, equal protection and contract clauses and the rule on taxation.
CREBA asserts that R.A. No. 7716 (1) impairs the obligations of contracts, (2) classifies transactions as
52
covered or exempt without reasonable basis and (3) violates the rule that taxes should be uniform and The sales tax adopted in EO 273 is applied similarly on all goods and services sold to the public, which
equitable and that Congress shall "evolve a progressive system of taxation." are not exempt, at the constant rate of 0% or 10%.

With respect to the first contention, it is claimed that the application of the tax to existing contracts of the The disputed sales tax is also equitable. It is imposed only on sales of goods or services by persons
sale of real property by installment or on deferred payment basis would result in substantial increases in engaged in business with an aggregate gross annual sales exceeding P200,000.00. Small corner sari-sari
the monthly amortizations to be paid because of the 10% VAT. The additional amount, it is pointed out, is stores are consequently exempt from its application. Likewise exempt from the tax are sales of farm and
something that the buyer did not anticipate at the time he entered into the contract. marine products, so that the costs of basic food and other necessities, spared as they are from the
incidence of the VAT, are expected to be relatively lower and within the reach of the general public.
The short answer to this is the one given by this Court in an early case: "Authorities from numerous
sources are cited by the plaintiffs, but none of them show that a lawful tax on a new subject, or an (At 382-383)
increased tax on an old one, interferes with a contract or impairs its obligation, within the meaning of the
Constitution. Even though such taxation may affect particular contracts, as it may increase the debt of one The CREBA claims that the VAT is regressive. A similar claim is made by the Cooperative Union of the
person and lessen the security of another, or may impose additional burdens upon one class and release Philippines, Inc. (CUP), while petitioner Juan T. David argues that the law contravenes the mandate of
the burdens of another, still the tax must be paid unless prohibited by the Constitution, nor can it be said Congress to provide for a progressive system of taxation because the law imposes a flat rate of 10% and
that it impairs the obligation of any existing contract in its true legal sense." (La Insular v. Machuca Go- thus places the tax burden on all taxpayers without regard to their ability to pay.
Tauco and Nubla Co-Siong, 39 Phil. 567, 574 (1919)). Indeed not only existing laws but also "the
reservation of the essential attributes of sovereignty, is . . . read into contracts as a postulate of the legal
order." (Philippine-American Life Ins. Co. v. Auditor General, 22 SCRA 135, 147 (1968)) Contracts must The Constitution does not really prohibit the imposition of indirect taxes which, like the VAT, are
be understood as having been made in reference to the possible exercise of the rightful authority of the regressive. What it simply provides is that Congress shall "evolve a progressive system of taxation." The
government and no obligation of contract can extend to the defeat of that authority. (Norman v. Baltimore constitutional provision has been interpreted to mean simply that "direct taxes are . . . to be preferred
and Ohio R.R., 79 L. Ed. 885 (1935)). [and] as much as possible, indirect taxes should be minimized." (E. FERNANDO, THE CONSTITUTION
OF THE PHILIPPINES 221 (Second ed. (1977)). Indeed, the mandate to Congress is not to prescribe, but
to evolve, a progressive tax system. Otherwise, sales taxes, which perhaps are the oldest form of indirect
It is next pointed out that while §4 of R.A. No. 7716 exempts such transactions as the sale of agricultural taxes, would have been prohibited with the proclamation of Art. VIII, §17(1) of the 1973 Constitution from
products, food items, petroleum, and medical and veterinary services, it grants no exemption on the sale which the present Art. VI, §28(1) was taken. Sales taxes are also regressive.
of real property which is equally essential. The sale of real property for socialized and low-cost housing is
exempted from the tax, but CREBA claims that real estate transactions of "the less poor," i.e., the middle
class, who are equally homeless, should likewise be exempted. Resort to indirect taxes should be minimized but not avoided entirely because it is difficult, if not
impossible, to avoid them by imposing such taxes according to the taxpayers' ability to pay. In the case of
the VAT, the law minimizes the regressive effects of this imposition by providing for zero rating of certain
The sale of food items, petroleum, medical and veterinary services, etc., which are essential goods and transactions (R.A. No. 7716, §3, amending §102 (b) of the NIRC), while granting exemptions to other
services was already exempt under §103, pars. (b) (d) (1) of the NIRC before the enactment of R.A. No. transactions. (R.A. No. 7716, §4, amending §103 of the NIRC).
7716. Petitioner is in error in claiming that R.A. No. 7716 granted exemption to these transactions, while
subjecting those of petitioner to the payment of the VAT. Moreover, there is a difference between the
"homeless poor" and the "homeless less poor" in the example given by petitioner, because the second Thus, the following transactions involving basic and essential goods and services are exempted from the
group or middle class can afford to rent houses in the meantime that they cannot yet buy their own VAT:
homes. The two social classes are thus differently situated in life. "It is inherent in the power to tax that the
State be free to select the subjects of taxation, and it has been repeatedly held that 'inequalities which (a) Goods for consumption or use which are in their original state (agricultural, marine and forest products,
result from a singling out of one particular class for taxation, or exemption infringe no constitutional cotton seeds in their original state, fertilizers, seeds, seedlings, fingerlings, fish, prawn livestock and
limitation.'" (Lutz v. Araneta, 98 Phil. 148, 153 (1955). Accord, City of Baguio v. De Leon, 134 Phil. 912 poultry feeds) and goods or services to enhance agriculture (milling of palay, corn sugar cane and raw
(1968); Sison, Jr. v. Ancheta, 130 SCRA 654, 663 (1984); Kapatiran ng mga Naglilingkod sa Pamahalaan sugar, livestock, poultry feeds, fertilizer, ingredients used for the manufacture of feeds).
ng Pilipinas, Inc. v. Tan, 163 SCRA 371 (1988)).
(b) Goods used for personal consumption or use (household and personal effects of citizens returning to
Finally, it is contended, for the reasons already noted, that R.A. No. 7716 also violates Art. VI, §28(1) the Philippines) and or professional use, like professional instruments and implements, by persons
which provides that "The rule of taxation shall be uniform and equitable. The Congress shall evolve a coming to the Philippines to settle here.
progressive system of taxation."
(c) Goods subject to excise tax such as petroleum products or to be used for manufacture of petroleum
Equality and uniformity of taxation means that all taxable articles or kinds of property of the same class be products subject to excise tax and services subject to percentage tax.
taxed at the same rate. The taxing power has the authority to make reasonable and natural classifications
for purposes of taxation. To satisfy this requirement it is enough that the statute or ordinance applies (d) Educational services, medical, dental, hospital and veterinary services, and services rendered under
equally to all persons, forms and corporations placed in similar situation. (City of Baguio v. De employer-employee relationship.
Leon, supra; Sison, Jr. v. Ancheta, supra)
(e) Works of art and similar creations sold by the artist himself.
Indeed, the VAT was already provided in E.O. No. 273 long before R.A. No. 7716 was enacted. R.A. No. (f) Transactions exempted under special laws, or international agreements.
7716 merely expands the base of the tax. The validity of the original VAT Law was questioned (g) Export-sales by persons not VAT-registered.
in Kapatiran ng Naglilingkod sa Pamahalaan ng Pilipinas, Inc. v. Tan, 163 SCRA 383 (1988) on grounds (h) Goods or services with gross annual sale or receipt not exceeding P500,000.00.
similar to those made in these cases, namely, that the law was "oppressive, discriminatory, unjust and (Respondents' Consolidated Comment on the Motions for Reconsideration, pp. 58-60)
regressive in violation of Art. VI, §28(1) of the Constitution." (At 382) Rejecting the challenge to the law, On the other hand, the transactions which are subject to the VAT are those which involve goods and
this Court held: services which are used or availed of mainly by higher income groups. These include real properties held
primarily for sale to customers or for lease in the ordinary course of trade or business, the right or privilege
As the Court sees it, EO 273 satisfies all the requirements of a valid tax. It is uniform. . . . to use patent, copyright, and other similar property or right, the right or privilege to use industrial,

53
commercial or scientific equipment, motion picture films, tapes and discs, radio, television, satellite the tax exemption to cooperatives and instead upheld the policy of strengthening the cooperatives by way
transmission and cable television time, hotels, restaurants and similar places, securities, lending of the grant of tax exemptions," by providing the following in Art. XII:
investments, taxicabs, utility cars for rent, tourist buses, and other common carriers, services of franchise
grantees of telephone and telegraph. §1. The goals of the national economy are a more equitable distribution of opportunities, income, and
wealth; a sustained increase in the amount of goods and services produced by the nation for the benefit of
The problem with CREBA's petition is that it presents broad claims of constitutional violations by tendering the people; and an expanding productivity as the key to raising the quality of life for all, especially the
issues not at retail but at wholesale and in the abstract. There is no fully developed record which can underprivileged.
impart to adjudication the impact of actuality. There is no factual foundation to show in the concrete the
application of the law to actual contracts and exemplify its effect on property rights. For the fact is that The State shall promote industrialization and full employment based on sound agricultural development
petitioner's members have not even been assessed the VAT. Petitioner's case is not made concrete by a and agrarian reform, through industries that make full and efficient use of human and natural resources,
series of hypothetical questions asked which are no different from those dealt with in advisory opinions. and which are competitive in both domestic and foreign markets. However, the State shall protect Filipino
enterprises against unfair foreign competition and trade practices.
The difficulty confronting petitioner is thus apparent. He alleges arbitrariness. A mere allegation, as here,
does not suffice. There must be a factual foundation of such unconstitutional taint. Considering that In the pursuit of these goals, all sectors of the economy and all regions of the country shall be given
petitioner here would condemn such a provision as void on its face, he has not made out a case. This is optimum opportunity to develop. Private enterprises, including corporations, cooperatives, and similar
merely to adhere to the authoritative doctrine that where the due process and equal protection clauses are collective organizations, shall be encouraged to broaden the base of their ownership.
invoked, considering that they are not fixed rules but rather broad standards, there is a need for proof of
such persuasive character as would lead to such a conclusion. Absent such a showing, the presumption
of validity must prevail. §15. The Congress shall create an agency to promote the viability and growth of cooperatives as
instruments for social justice and economic development.
(Sison, Jr. v. Ancheta, 130 SCRA at 661)
Petitioner's contention has no merit. In the first place, it is not true that P.D. No. 1955 singled out
cooperatives by withdrawing their exemption from income and sales taxes under P.D. No. 175, §5. What
Adjudication of these broad claims must await the development of a concrete case. It may be that P.D. No. 1955, §1 did was to withdraw the exemptions and preferential treatments theretofore granted to
postponement of adjudication would result in a multiplicity of suits. This need not be the case, however. private business enterprises in general, in view of the economic crisis which then beset the nation. It is
Enforcement of the law may give rise to such a case. A test case, provided it is an actual case and not an true that after P.D. No. 2008, §2 had restored the tax exemptions of cooperatives in 1986, the exemption
abstract or hypothetical one, may thus be presented. was again repealed by E.O. No. 93, §1, but then again cooperatives were not the only ones whose
exemptions were withdrawn. The withdrawal of tax incentives applied to all, including government and
Nor is hardship to taxpayers alone an adequate justification for adjudicating abstract issues. Otherwise, private entities. In the second place, the Constitution does not really require that cooperatives be granted
adjudication would be no different from the giving of advisory opinion that does not really settle legal tax exemptions in order to promote their growth and viability. Hence, there is no basis for petitioner's
issues. assertion that the government's policy toward cooperatives had been one of vacillation, as far as the grant
of tax privileges was concerned, and that it was to put an end to this indecision that the constitutional
We are told that it is our duty under Art. VIII, §1, ¶2 to decide whenever a claim is made that "there has provisions cited were adopted. Perhaps as a matter of policy cooperatives should be granted tax
been a grave abuse of discretion amounting to lack or excess of jurisdiction on the part of any branch or exemptions, but that is left to the discretion of Congress. If Congress does not grant exemption and there
instrumentality of the government." This duty can only arise if an actual case or controversy is before us. is no discrimination to cooperatives, no violation of any constitutional policy can be charged.
Under Art . VIII, §5 our jurisdiction is defined in terms of "cases" and all that Art. VIII, §1, ¶2 can plausibly
mean is that in the exercise of that jurisdiction we have the judicial power to determine questions of grave Indeed, petitioner's theory amounts to saying that under the Constitution cooperatives are exempt from
abuse of discretion by any branch or instrumentality of the government. taxation. Such theory is contrary to the Constitution under which only the following are exempt from
taxation: charitable institutions, churches and parsonages, by reason of Art. VI, §28 (3), and non-stock,
Put in another way, what is granted in Art. VIII, §1, ¶2 is "judicial power," which is "the power of a court to non-profit educational institutions by reason of Art. XIV, §4 (3).
hear and decide cases pending between parties who have the right to sue and be sued in the courts of
law and equity" (Lamb v. Phipps, 22 Phil. 456, 559 (1912)), as distinguished from legislative and executive CUP's further ground for seeking the invalidation of R.A. No. 7716 is that it denies cooperatives the equal
power. This power cannot be directly appropriated until it is apportioned among several courts either by protection of the law because electric cooperatives are exempted from the VAT. The classification
the Constitution, as in the case of Art. VIII, §5, or by statute, as in the case of the Judiciary Act of 1948 between electric and other cooperatives (farmers cooperatives, producers cooperatives, marketing
(R.A. No. 296) and the Judiciary Reorganization Act of 1980 (B.P. Blg. 129). The power thus apportioned cooperatives, etc.) apparently rests on a congressional determination that there is greater need to provide
constitutes the court's "jurisdiction," defined as "the power conferred by law upon a court or judge to take cheaper electric power to as many people as possible, especially those living in the rural areas, than there
cognizance of a case, to the exclusion of all others." (United States v. Arceo, 6 Phil. 29 (1906)) Without an is to provide them with other necessities in life. We cannot say that such classification is unreasonable.
actual case coming within its jurisdiction, this Court cannot inquire into any allegation of grave abuse of
discretion by the other departments of the government. We have carefully read the various arguments raised against the constitutional validity of R.A. No. 7716.
We have in fact taken the extraordinary step of enjoining its enforcement pending resolution of these
VIII. Alleged violation of policy towards cooperatives. On the other hand, the Cooperative Union of the cases. We have now come to the conclusion that the law suffers from none of the infirmities attributed to it
Philippines (CUP), after briefly surveying the course of legislation, argues that it was to adopt a definite by petitioners and that its enactment by the other branches of the government does not constitute a grave
policy of granting tax exemption to cooperatives that the present Constitution embodies provisions on abuse of discretion. Any question as to its necessity, desirability or expediency must be addressed to
cooperatives. To subject cooperatives to the VAT would therefore be to infringe a constitutional policy. Congress as the body which is electorally responsible, remembering that, as Justice Holmes has said,
Petitioner claims that in 1973, P.D. No. 175 was promulgated exempting cooperatives from the payment "legislators are the ultimate guardians of the liberties and welfare of the people in quite as great a degree
of income taxes and sales taxes but in 1984, because of the crisis which menaced the national economy, as are the courts." (Missouri, Kansas & Texas Ry. Co. v. May, 194 U.S. 267, 270, 48 L. Ed. 971, 973
this exemption was withdrawn by P.D. No. 1955; that in 1986, P.D. No. 2008 again granted cooperatives (1904)). It is not right, as petitioner in G.R. No. 115543 does in arguing that we should enforce the public
exemption from income and sales taxes until December 31, 1991, but, in the same year, E.O. No. 93 accountability of legislators, that those who took part in passing the law in question by voting for it in
revoked the exemption; and that finally in 1987 the framers of the Constitution "repudiated the previous Congress should later thrust to the courts the burden of reviewing measures in the flush of enactment.
actions of the government adverse to the interests of the cooperatives, that is, the repeated revocation of

54
This Court does not sit as a third branch of the legislature, much less exercise a veto power over On March 3, 1994, Committee Report No. 378 was passed by the Senate on Second Reading and was
legislation. approved on Third Reading on March 14, 1994. On March 22, 1994, the House of Representatives, upon
being apprised of the action of the Senate, approved the amendments proposed by the Senate.
WHEREFORE, the motions for reconsideration are denied with finality and the temporary restraining order
previously issued is hereby lifted. The enrolled bill, submitted to the President on April 12, 1994, was signed by the Chief Executive on May
5, 1994 as Republic Act No. 7720. When a plebiscite on the Act was held on July 13, 1994, a great
SO ORDERED majority of the registered voters of Santiago voted in favor of the conversion of Santiago into a city.

G.R. No. 118303             January 31, 1996 The question as to the validity of Republic Act No. 7720 hinges on the following twin issues: (I) Whether or
not the Internal Revenue Allotments (IRAs) are to be included in the computation of the average annual
income of a municipality for purposes of its conversion into an independent component city, and (II)
SENATOR HEHERSON T. ALVAREZ, vs.HON. TEOFISTO T. GUINGONA, Whether or not, considering that the Senate passed SB No. 1243, its own version of HB No. 8817,
Republic Act No. 7720 can be said to have originated in the House of Representatives.
Of main concern to the petitioners is whether Republic Act No. 7720, just recently passed by Congress
and signed by the President into law, is constitutionally infirm. I

Indeed, in this Petition for Prohibition with prayer for Temporary Restraining Order and Preliminary The annual income of a local
Prohibitory Injunction, petitioners assail the validity of Republic Act No. 7720, entitled, "An Act Converting government unit includes the IRAs
the Municipality of Santiago, Isabela into an Independent Component City to be known as the City of
Santiago," mainly because the Act allegedly did not originate exclusively in the House of Representatives
as mandated by Section 24, Article VI of the 1987 Constitution. Petitioners claim that Santiago could not qualify into a component city because its average annual income
for the last two (2) consecutive years based on 1991 constant prices falls below the required annual
income of Twenty Million Pesos (P20,000,000.00) for its conversion into a city, petitioners having
Also, petitioners claim that the Municipality of Santiago has not met the minimum average annual income computed Santiago's average annual income in the following manner:
required under Section 450 of the Local Government Code of 1991 in order to be converted into a
component city.
Total income (at 1991 constant prices) for 1991 P
Undisputed is the following chronicle of the metamorphosis of House Bill No. 8817 into Republic Act No. 20,379,057.07
7720:
Total income (at 1991 constant prices) for 1992 P 21,570,106.87
On April 18, 1993, HB No. 8817, entitled "An Act Converting the Municipality of Santiago into an
Independent Component City to be known as the City of Santiago," was filed in the House of
Representatives with Representative Antonio Abaya as principal author. Other sponsors included Total income for 1991 and 1992 P 41,949,163.94
Representatives Ciriaco Alfelor, Rodolfo Albano, Santiago Respicio and Faustino Dy. The bill was referred
to the House Committee on Local Government and the House Committee on Appropriations on May 5,
1993. Minus:

On May 19, 1993, June 1, 1993, November 28, 1993, and December 1, 1993, public hearings on HB No.
IRAs for 1991 and 1992 P 15,730,043.00
8817 were conducted by the House Committee on Local Government. The committee submitted to the
House a favorable report, with amendments, on December 9, 1993.
Total income for 1991 and 1992 P 26,219,120.94
On December 13, 1993, HB No. 8817 was passed by the House of Representatives on Second Reading
and was approved on Third Reading on December 17, 1993. On January 28, 1994, HB No. 8817 was
transmitted to the Senate. Average Annual Income P 13,109,560.47
===============
Meanwhile, a counterpart of HB No. 8817, Senate Bill No. 1243, entitled, "An Act Converting the
Municipality of Santiago into an Independent Component City to be Known as the City of Santiago," was By dividing the total income of Santiago for calendar years 1991 and 1992, after deducting the IRAs, the
filed in the Senate. It was introduced by Senator Vicente Sotto III, as principal sponsor, on May 19, 1993. average annual income arrived at would only be P13,109,560.47 based on the 1991 constant prices.
This was just after the House of Representatives had conducted its first public hearing on HB No. 8817. Thus, petitioners claim that Santiago's income is far below the aforesaid Twenty Million Pesos average
annual income requirement.
On February 23, 1994, or a little less than a month after HB No. 8817 was transmitted to the Senate, the
Senate Committee on Local Government conducted public hearings on SB No. 1243. On March 1, 1994, The certification issued by the Bureau of Local Government Finance of the Department of Finance, which
the said committee submitted Committee Report No. 378 on HB No. 8817, with the recommendation that indicates Santiago's average annual income to be P20,974,581.97, is allegedly not accurate as the
it be approved without amendment, taking into consideration the reality that H.B. No. 8817 was on all Internal Revenue Allotments were not excluded from the computation. Petitioners asseverate that the
fours with SB No. 1243. Senator Heherson T. Alvarez, one of the herein petitioners, indicated his approval IRAs are not actually income but transfers and/or budgetary aid from the national government and that
thereto by signing said report as member of the Committee on Local Government. they fluctuate, increase or decrease, depending on factors like population, land and equal sharing.

In this regard, we hold that petitioners asseverations are untenable because Internal Revenue Allotments
form part of the income of Local Government Units.

55
It is true that for a municipality to be converted into a component city, it must, among others, have an and municipalities from regular sources of the Local General Fund including the internal revenue
average annual income of at least Twenty Million Pesos for the last two (2) consecutive years based on allotment and other shares provided for in Sections 284, 290 and 291 of the Code, but exclusive of non-
1991 constant prices.1 Such income must be duly certified by the Department of Finance. recurring receipts, such as other national aids, grants, financial assistance, loan proceeds, sales of fixed
assets, and similar others" (Emphasis ours).14 Such order, constituting executive or contemporaneous
Resolution of the controversy regarding compliance by the Municipality of Santiago with the aforecited construction of a statute by an administrative agency charged with the task of interpreting and applying
income requirement hinges on a correlative and contextual explication of the meaning of internal revenue the same, is entitled to full respect and should be accorded great weight by the courts, unless such
allotments (IRAs) vis-a-vis the notion of income of a local government unit and the principles of local construction is clearly shown to be in sharp conflict with the Constitution, the governing statute, or other
autonomy and decentralization underlying the institutionalization and intensified empowerment of the local laws.15
government system.
II
A Local Government Unit is a political subdivision of the State which is constituted by law and possessed
of substantial control over its own affairs.3 Remaining to be an intra sovereign subdivision of one In the enactment of RA No. 7720,
sovereign nation, but not intended, however, to be an imperium in imperio,4 the local government unit is there was compliance with Section 24,
autonomous in the sense that it is given more powers, authority, responsibilities and resources.5 Power Article VI of the 1987 Constitution
which used to be highly centralized in Manila, is thereby deconcentrated, enabling especially the
peripheral local government units to develop not only at their own pace and discretion but also with their Although a bill of local application like HB No. 8817 should, by constitutional prescription,16 originate
own resources and assets. exclusively in the House of Representatives, the claim of petitioners that Republic Act No. 7720 did not
originate exclusively in the House of Representatives because a bill of the same import, SB No. 1243,
The practical side to development through a decentralized local government system certainly concerns was passed in the Senate, is untenable because it cannot be denied that HB No. 8817 was filed in the
the matter of financial resources. With its broadened powers and increased responsibilities, a local House of Representatives first before SB No. 1243 was filed in the Senate. Petitioners themselves cannot
government unit must now operate on a much wider scale. More extensive operations, in turn, entail more disavow their own admission that HB No. 8817 was filed on April 18, 1993 while SB No. 1243 was filed on
expenses. Understandably, the vesting of duty, responsibility and accountability in every local government May 19, 1993. The filing of HB No. 8817 was thus precursive not only of the said Act in question but also
unit is accompanied with a provision for reasonably adequate resources to discharge its powers and of SB No. 1243. Thus, HB No. 8817, was the bill that initiated the legislative process that culminated in the
effectively carry out its functions.7 Availment of such resources is effectuated through the vesting in every enactment of Republic Act No. 7720. No violation of Section 24, Article VI, of the 1987 Constitution is
local government unit of (1) the right to create and broaden its own source of revenue; (2) the right to be perceptible under the circumstances attending the instant controversy.
allocated a just share in national taxes, such share being in the form of internal revenue allotments (IRAs);
and (3) the right to be given its equitable share in the proceeds of the utilization and development of the Furthermore, petitioners themselves acknowledge that HB No. 8817 was already approved on Third
national wealth, if any, within its territorial boundaries.8 Reading and duly transmitted to the Senate when the Senate Committee on Local Government conducted
its public hearing on HB No. 8817. HB No. 8817 was approved on the Third Reading on December 17,
The funds generated from local taxes, IRAs and national wealth utilization proceeds accrue to the general 1993 and transmitted to the Senate on January 28, 1994; a little less than a month thereafter, or on
fund of the local government and are used to finance its operations subject to specified modes of February 23, 1994, the Senate Committee on Local Government conducted public hearings on SB No.
spending the same as provided for in the Local Government Code and its implementing rules and 1243. Clearly, the Senate held in abeyance any action on SB No. 1243 until it received HB No. 8817,
regulations. For instance, not less than twenty percent (20%) of the IRAs must be set aside for local already approved on the Third Reading, from the House of Representatives. The filing in the Senate of a
development projects.9 As such, for purposes of budget preparation, which budget should reflect the substitute bill in anticipation of its receipt of the bill from the House, does not contravene the constitutional
estimates of the income of the local government unit, among others, the IRAs and the share in the requirement that a bill of local application should originate in the House of Representatives, for as long as
national wealth utilization proceeds are considered items of income. This is as it should be, since income the Senate does not act thereupon until it receives the House bill.
is defined in the Local Government Code to be all revenues and receipts collected or received forming the
gross accretions of funds of the local government unit.10 We have already addressed this issue in the case of Tolentino vs. Secretary of Finance.17 There, on the
matter of the Expanded Value Added Tax (EVAT) Law, which, as a revenue bill, is nonetheless
The IRAs are items of income because they form part of the gross accretion of the funds of the local constitutionally required to originate exclusively in the House of Representatives, we explained:
government unit. The IRAs regularly and automatically accrue to the local treasury without need of any
further action on the part of the local government unit.11 They thus constitute income which the local . . . To begin with, it is not the law — but the revenue bill — which is required by the Constitution to
government can invariably rely upon as the source of much needed funds. "originate exclusively" in the House of Representatives. It is important to emphasize this, because a bill
originating in the House may undergo such extensive changes in the Senate that the result may be a
For purposes of converting the Municipality of Santiago into a city, the Department of Finance certified, rewriting of the whole. . . . as a result of the Senate action, a distinct bill may be produced. To insist that a
among others, that the municipality had an average annual income of at least Twenty Million Pesos for the revenue statute — and not only the bill which initiated the legislative process culminating in the enactment
last two (2) consecutive years based on 1991 constant prices. This, the Department of Finance did after of the law — must substantially be the same as the House bill would be to deny the Senate's power not
including the IRAs in its computation of said average annual income. only to "concur with amendments" but also to "propose amendments." It would be to violate the coequality
of legislative power of the two houses of Congress and in fact make the House superior to the Senate.
Furthermore, Section 450 (c) of the Local Government Code provides that "the average annual income
shall include the income accruing to the general fund, exclusive of special funds, transfers, and non- xxx       xxx       xxx
recurring income." To reiterate, IRAs are a regular, recurring item of income; nil is there a basis, too, to
classify the same as a special fund or transfer, since IRAs have a technical definition and meaning all its It is insisted, however, that S. No. 1630 was passed not in substitution of H. No. 11197 but of another
own as used in the Local Government Code that unequivocally makes it distinct from special funds or Senate bill (S. No. 1129) earlier filed and that what the Senate did was merely to "take [H. No. 11197] into
transfers referred to when the Code speaks of "funding support from the national government, its consideration" in enacting S. No. 1630. There is really no difference between the Senate preserving H.
instrumentalities and government-owned-or-controlled corporations".12 No. 11197 up to the enacting clause and then writing its own version following the enacting clause (which,
it would seem petitioners admit is an amendment by substitution), and, on the other hand, separately
Thus, Department of Finance Order No. 35-9313 correctly encapsulizes the full import of the above presenting a bill of its own on the same subject matter. In either case the result are two bills on the same
disquisition when it defined ANNUAL INCOME to be "revenues and receipts realized by provinces, cities subject.

56
Indeed, what the Constitution simply means is that the initiative for filing revenue, tariff, or tax bills, bills Responding to Sen. Estrada’s revelation, Secretary Florencio Abad of the DBM issued a public statement
authorizing an increase of the public debt, private bills and bills of local application must come from the entitled Abad: Releases to Senators Part of Spending Acceleration Program,1 explaining that the funds
House of Representatives on the theory that, elected as they are from the districts, the members of the released to the Senators had been part of the DAP, a program designed by the DBM to ramp up spending
House can be expected to be more sensitive to the local needs and problems. On the other hand, the to accelerate economic expansion. He clarified that the funds had been released to the Senators based
senators, who are elected at large, are expected to approach the same problems from the national on their letters of request for funding; and that it was not the first time that releases from the DAP had
perspective. Both views are thereby made to bear on the enactment of such laws. been made because the DAP had already been instituted in 2011 to ramp up spending after sluggish
disbursements had caused the growth of the gross domestic product (GDP) to slow down. He explained
Nor does the Constitution prohibit the filing in the Senate of a substitute bill in anticipation of its receipt of that the funds under the DAP were usually taken from (1) unreleased appropriations under Personnel
the bill from the House, so long as action by the Senate as a body is withheld pending receipt of the Services;2 (2) unprogrammed funds; (3) carry-over appropriations unreleased from the previous year; and
House bill. . . .18 (4) budgets for slow-moving items or projects that had been realigned to support faster-disbursing
projects.
III
The DBM soon came out to claim in its website3 that the DAP releases had been sourced from savings
generated by the Government, and from unprogrammed funds; and that the savings had been derived
Every law, including RA No. 7720, from (1) the pooling of unreleased appropriations, like unreleased Personnel Services4 appropriations that
has in its favor the presumption would lapse at the end of the year, unreleased appropriations of slow-moving projects and discontinued
of constitutionality projects per zero based budgeting findings;5 and (2) the withdrawal of unobligated allotments also for
slow-moving programs and projects that had been earlier released to the agencies of the National
It is a well-entrenched jurisprudential rule that on the side of every law lies the presumption of Government.
constitutionality.19 Consequently, for RA No. 7720 to be nullified, it must be shown that there is a clear
and unequivocal breach of the Constitution, not merely a doubtful and equivocal one; in other words, the The DBM listed the following as the legal bases for the DAP’s use of savings,6 namely: (1) Section 25(5),
grounds for nullity must be clear and beyond reasonable doubt.20 Those who petition this court to declare Article VI of the 1987 Constitution, which granted to the President the authority to augment an item for his
a law to be unconstitutional must clearly and fully establish the basis that will justify such a declaration; office in the general appropriations law; (2) Section 49 (Authority to Use Savings for Certain Purposes)
otherwise, their petition must fail. Taking into consideration the justification of our stand on the and Section 38 (Suspension of Expenditure Appropriations), Chapter 5, Book VI of Executive Order (EO)
immediately preceding ground raised by petitioners to challenge the constitutionality of RA No. 7720, the No. 292 (Administrative Code of 1987); and (3) the General Appropriations Acts (GAAs) of 2011, 2012
Court stands on the holding that petitioners have failed to overcome the presumption. The dismissal of and 2013, particularly their provisions on the (a) use of savings; (b) meanings of savings and
this petition is, therefore, inevitable. augmentation; and (c) priority in the use of savings.

WHEREFORE, the instant petition is DISMISSED for lack of merit with costs against petitioners.SO As for the use of unprogrammed funds under the DAP, the DBM cited as legal bases the special
ORDERED. provisions on unprogrammed fund contained in the GAAs of 2011, 2012 and 2013.

G.R. No. 209135 The revelation of Sen. Estrada and the reactions of Sec. Abad and the DBM brought the DAP to the
consciousness of the Nation for the first time, and made this present controversy inevitable. That the
AUGUSTO L. SY JUCO JR., Ph.D., Petitioner,vs. FLORENCIO B. ABAD issues against the DAP came at a time when the Nation was still seething in anger over Congressional
pork barrel – "an appropriation of government spending meant for localized projects and secured solely or
For resolution are the consolidated petitions assailing the constitutionality of the Disbursement primarily to bring money to a representative’s district"7 – excited the Nation as heatedly as the pork barrel
Acceleration Program(DAP), National Budget Circular (NBC) No. 541, and related issuances of the controversy.
Department of Budget and Management (DBM) implementing the DAP.
Nine petitions assailing the constitutionality of the DAP and the issuances relating to the DAP were filed
At the core of the controversy is Section 29(1) of Article VI of the 1987 Constitution, a provision of the within days of each other, as follows: G.R. No. 209135 (Syjuco), on October 7, 2013; G.R. No. 209136
fundamental law that firmly ordains that "[n]o money shall be paid out of the Treasury except in pursuance (Luna), on October 7, 2013; G.R. No. 209155 (Villegas),8 on October 16, 2013; G.R. No. 209164
of an appropriation made by law." The tenor and context of the challenges posed by the petitioners (PHILCONSA), on October 8, 2013; G.R. No. 209260 (IBP), on October 16, 2013; G.R. No. 209287
against the DAP indicate that the DAP contravened this provision by allowing the Executive to allocate (Araullo), on October 17, 2013; G.R. No. 209442 (Belgica), on October 29, 2013; G.R. No. 209517
public money pooled from programmed and unprogrammed funds of its various agencies in the guise of (COURAGE), on November6, 2013; and G.R. No. 209569 (VACC), on November 8, 2013.
the President exercising his constitutional authority under Section 25(5) of the 1987 Constitution to
transfer funds out of savings to augment the appropriations of offices within the Executive Branch of the In G.R. No. 209287 (Araullo), the petitioners brought to the Court’s attention NBC No. 541 (Adoption of
Government. But the challenges are further complicated by the interjection of allegations of transfer of Operational Efficiency Measure – Withdrawal of Agencies’ Unobligated Allotments as of June 30, 2012),
funds to agencies or offices outside of the Executive. alleging that NBC No. 541, which was issued to implement the DAP, directed the withdrawal of
unobligated allotments as of June 30, 2012 of government agencies and offices with low levels of
Antecedents obligations, both for continuing and current allotments.

What has precipitated the controversy? In due time, the respondents filed their Consolidated Comment through the Office of the Solicitor General
(OSG).
On September 25, 2013, Sen. Jinggoy Ejercito Estrada delivered a privilege speech in the Senate of the
Philippines to reveal that some Senators, including himself, had been allotted an additional ₱50 Million The Court directed the holding of oral arguments on the significant issues raised and joined.
each as "incentive" for voting in favor of the impeachment of Chief Justice Renato C. Corona.
Issues

57
Under the Advisory issued on November 14, 2013, the presentations of the parties during the oral a. NBC No. 528 dated January 3, 2011 (Guidelines on the Release of Funds for FY 2011);
arguments were limited to the following, to wit:
b. NBC No. 535 dated December 29, 2011 (Guidelines on the Release of Funds for FY 2012);
Procedural Issue:
c. NBC No. 541 dated July 18, 2012 (Adoption of Operational Efficiency Measure – Withdrawal of
A. Whether or not certiorari, prohibition, and mandamus are proper remedies to assail the constitutionality Agencies’ Unobligated Allotments as of June 30, 2012);
and validity of the Disbursement Acceleration Program (DAP), National Budget Circular (NBC) No. 541,
and all other executive issuances allegedly implementing the DAP. Subsumed in this issue are whether d. NBC No. 545 dated January 2, 2013 (Guidelines on the Release of Funds for FY 2013);
there is a controversy ripe for judicial determination, and the standing of petitioners.
e. DBM Circular Letter No. 2004-2 dated January 26, 2004 (Budgetary Treatment of
Substantive Issues: Commitments/Obligations of the National Government);

B. Whether or not the DAP violates Sec. 29, Art. VI of the 1987 Constitution, which provides: "No money f. COA-DBM Joint Circular No. 2013-1 dated March 15, 2013 (Revised Guidelines on the Submission of
shall be paid out of the Treasury except in pursuance of an appropriation made by law." Quarterly Accountability Reports on Appropriations, Allotments, Obligations and Disbursements);

C. Whether or not the DAP, NBC No. 541, and all other executive issuances allegedly implementing the g. NBC No. 440 dated January 30, 1995 (Adoption of a Simplified Fund Release System in the
DAP violate Sec. 25(5), Art. VI of the 1987 Constitution insofar as: Government).

(a)They treat the unreleased appropriations and unobligated allotments withdrawn from government (3) A breakdown of the sources of savings, including savings from discontinued projects and unpaid
agencies as "savings" as the term is used in Sec. 25(5), in relation to the provisions of the GAAs of 2011, appropriations for compensation from 2011 to 2013
2012 and 2013;
On January 28, 2014, the OSG, to comply with the Resolution issued on January 21, 2014 directing the
(b)They authorize the disbursement of funds for projects or programs not provided in the GAAs for the respondents to submit the documents not yet submitted in compliance with the directives of the Court or
Executive Department; and its Members, submitted several evidence packets to aid the Court in understanding the factual bases of
the DAP, to wit:
(c)They "augment" discretionary lump sum appropriations in the GAAs.
(1) First Evidence Packet11 – containing seven memoranda issued by the DBM through Sec. Abad,
D. Whether or not the DAP violates: (1) the Equal Protection Clause, (2) the system of checks and inclusive of annexes, listing in detail the 116 DAP identified projects approved and duly signed by the
balances, and (3) the principle of public accountability enshrined in the 1987 Constitution considering that President, as follows:
it authorizes the release of funds upon the request of legislators.
a. Memorandum for the President dated October 12, 2011 (FY 2011 Proposed Disbursement Acceleration
E. Whether or not factual and legal justification exists to issue a temporary restraining order to restrain the Program (Projects and Sources of Funds);
implementation of the DAP, NBC No. 541, and all other executive issuances allegedly implementing the
DAP. b. Memorandum for the President dated December 12, 2011 (Omnibus Authority to Consolidate
Savings/Unutilized Balances and its Realignment);
In its Consolidated Comment, the OSG raised the matter of unprogrammed funds in order to support its
argument regarding the President’s power to spend. During the oral arguments, the propriety of releasing c. Memorandum for the President dated June 25, 2012 (Omnibus Authority to Consolidate
unprogrammed funds to support projects under the DAP was considerably discussed. The petitioners in Savings/Unutilized Balances and their Realignment);
G.R. No. 209287 (Araullo) and G.R. No. 209442 (Belgica) dwelled on unprogrammed funds in their
respective memoranda. Hence, an additional issue for the oral arguments is stated as follows:
d. Memorandum for the President dated September 4, 2012 (Release of funds for other priority projects
and expenditures of the Government);
F. Whether or not the release of unprogrammed funds under the DAP was in accord with the GAAs.
e. Memorandum for the President dated December 19, 2012 (Proposed Priority Projects and
During the oral arguments held on November 19, 2013, the Court directed Sec. Abad to submit a list of Expenditures of the Government);
savings brought under the DAP that had been sourced from (a) completed programs; (b) discontinued or
abandoned programs; (c) unpaid appropriations for compensation; (d) a certified copy of the President’s
directive dated June 27, 2012 referred to in NBC No. 541; and (e) all circulars or orders issued in relation f. Memorandum for the President dated May 20, 2013 (Omnibus Authority to Consolidate
to the DAP.9 Savings/Unutilized Balances and their Realignment to Fund the Quarterly Disbursement Acceleration
Program); and
In compliance, the OSG submitted several documents, as follows:
g. Memorandum for the President dated September 25, 2013 (Funding for the Task Force Pablo
Rehabilitation Plan).
(1) A certified copy of the Memorandum for the President dated June 25, 2012 (Omnibus Authority to
Consolidate Savings/Unutilized Balances and their Realignment);10
(2) Second Evidence Packet12 – consisting of 15 applications of the DAP, with their corresponding
Special Allotment Release Orders (SAROs) and appropriation covers;
(2) Circulars and orders, which the respondents identified as related to the DAP, namely:
(3) Third Evidence Packet13 – containing a list and descriptions of 12 projects under the DAP;

58
(4) Fourth Evidence Packet14 – identifying the DAP-related portions of the Annual Financial Report (AFR) of NBC No. 541 were not in the exercise of the taxing or spending power of Congress;20 and that even if
of the Commission on Audit for 2011 and 2012; the petitioners had suffered injury, there were plain, speedy and adequate remedies in the ordinary course
of law available to them, like assailing the regularity of the DAP and related issuances before the
(5) Fifth Evidence Packet15 – containing a letter of Department of Transportation and Commission on Audit (COA) or in the trial courts.21
Communications(DOTC) Sec. Joseph Abaya addressed to Sec. Abad recommending the withdrawal of
funds from his agency, inclusive of annexes; and The respondents aver that the special civil actions of certiorari and prohibition are not proper actions for
directly assailing the constitutionality and validity of the DAP, NBC No. 541, and the other executive
(6) Sixth Evidence Packet16 – a print-out of the Solicitor General’s visual presentation for the January 28, issuances implementing the DAP.22
2014 oral arguments.
In their memorandum, the respondents further contend that there is no authorized proceeding under the
On February 5, 2014,17 the OSG forwarded the Seventh Evidence Packet,18 which listed the sources of Constitution and the Rules of Court for questioning the validity of any law unless there is an actual case or
funds brought under the DAP, the uses of such funds per project or activity pursuant to DAP, and the legal controversy the resolution of which requires the determination of the constitutional question; that the
bases thereof. jurisdiction of the Court is largely appellate; that for a court of law to pass upon the constitutionality of a
law or any act of the Government when there is no case or controversy is for that court to set itself up as a
reviewer of the acts of Congress and of the President in violation of the principle of separation of powers;
On February 14, 2014, the OSG submitted another set of documents in further compliance with the and that, in the absence of a pending case or controversy involving the DAP and NBC No. 541, any
Resolution dated January 28, 2014, viz: decision herein could amount to a mere advisory opinion that no court can validly render.23

(1) Certified copies of the certifications issued by the Bureau of Treasury to the effect that the revenue The respondents argue that it is the application of the DAP to actual situations that the petitioners can
collections exceeded the original revenue targets for the years 2011, 2012 and 2013, including collections question either in the trial courts or in the COA; that if the petitioners are dissatisfied with the ruling either
arising from sources not considered in the original revenue targets, which certifications were required for of the trial courts or of the COA, they can appeal the decision of the trial courts by petition for review on
the release of the unprogrammed funds as provided in Special Provision No. 1 of Article XLV, Article XVI, certiorari, or assail the decision or final order of the COA by special civil action for certiorari under Rule 64
and Article XLV of the 2011, 2012 and 2013 GAAs; and (2) A report on releases of savings of the of the Rules of Court.24
Executive Department for the use of the Constitutional Commissions and other branches of the
Government, as well as the fund releases to the Senate and the Commission on Elections (COMELEC).
The respondents’ arguments and submissions on the procedural issue are bereft of merit.
RULING
Section 1, Article VIII of the 1987 Constitution expressly provides:
I.
Section 1. The judicial power shall be vested in one Supreme Court and in such lower courts as may be
established by law.
Procedural Issue:
Judicial power includes the duty of the courts of justice to settle actual controversies involving rights which
a) The petitions under Rule 65 are proper remedies are legally demandable and enforceable, and to determine whether or not there has been a grave abuse
of discretion amounting to lack or excess of jurisdiction on the part of any branch or instrumentality of the
All the petitions are filed under Rule 65 of the Rules of Court, and include applications for the issuance of Government.
writs of preliminary prohibitory injunction or temporary restraining orders. More specifically, the nature of
the petitions is individually set forth hereunder, to wit: Thus, the Constitution vests judicial power in the Court and in such lower courts as may be established by
law. In creating a lower court, Congress concomitantly determines the jurisdiction of that court, and that
G.R. No. 209135 (Syjuco) Certiorari, Prohibition and Mandamus court, upon its creation, becomes by operation of the Constitution one of the repositories of judicial
power.25 However, only the Court is a constitutionally created court, the rest being created by Congress
G.R. No. 209136 (Luna) Certiorariand Prohibition in its exercise of the legislative power.

G.R. No. 209155 (Villegas) Certiorariand Prohibition The Constitution states that judicial power includes the duty of the courts of justice not only "to settle
actual controversies involving rights which are legally demandable and enforceable" but also "to
G.R. No. 209164 (PHILCONSA)Certiorariand Prohibition
determine whether or not there has been a grave abuse of discretion amounting to lack or excess of
G.R. No. 209260 (IBP) Prohibition jurisdiction on the part of any branch or instrumentality of the Government." It has thereby expanded the
concept of judicial power, which up to then was confined to its traditional ambit of settling actual
G.R. No. 209287 (Araullo) Certiorariand Prohibition controversies involving rights that were legally demandable and enforceable.

G.R. No. 209442 (Belgica) Certiorari The background and rationale of the expansion of judicial power under the 1987 Constitution were laid out
G.R. No. 209517 (COURAGE) Certiorari and Prohibition during the deliberations of the 1986 Constitutional Commission by Commissioner Roberto R. Concepcion
(a former Chief Justice of the Philippines) in his sponsorship of the proposed provisions on the Judiciary,
G.R. No. 209569 (VACC) Certiorari and Prohibition where he said:–
The respondents submit that there is no actual controversy that is ripe for adjudication in the absence of
adverse claims between the parties;19 that the petitioners lacked legal standing to sue because no The Supreme Court, like all other courts, has one main function: to settle actual controversies involving
allegations were made to the effect that they had suffered any injury as a result of the adoption of the DAP conflicts of rights which are demandable and enforceable. There are rights which are guaranteed by law
and issuance of NBC No. 541; that their being taxpayers did not immediately confer upon the petitioners but cannot be enforced by a judicial party. In a decided case, a husband complained that his wife was
the legal standing to sue considering that the adoption and implementation of the DAP and the issuance unwilling to perform her duties as a wife. The Court said: "We can tell your wife what her duties as such

59
are and that she is bound to comply with them, but we cannot force her physically to discharge her main x x x In times of social disquietude or political excitement, the great landmarks of the Constitution are apt
marital duty to her husband. There are some rights guaranteed by law, but they are so personal that to to be forgotten or marred, if not entirely obliterated. In cases of conflict, the judicial department is the only
enforce them by actual compulsion would be highly derogatory to human dignity." This is why the first part constitutional organ which can be called upon to determine the proper allocation of powers between the
of the second paragraph of Section 1 provides that: Judicial power includes the duty of courts to settle several department and among the integral or constituent units thereof.
actual controversies involving rights which are legally demandable or enforceable…
xxxx
The courts, therefore, cannot entertain, much less decide, hypothetical questions. In a presidential system
of government, the Supreme Court has, also, another important function. The powers of government are The Constitution is a definition of the powers of government. Who is to determine the nature, scope and
generally considered divided into three branches: the Legislative, the Executive and the Judiciary. Each extent of such powers? The Constitution itself has provided for the instrumentality of the judiciary as the
one is supreme within its own sphere and independent of the others. Because of that supremacy power to rational way. And when the judiciary mediates to allocate constitutional boundaries, it does not assert any
determine whether a given law is valid or not is vested in courts of justice. superiority over the other department; it does not in reality nullify or invalidate an act of the legislature, but
only asserts the solemn and sacred obligation assigned to it by the Constitution to determine conflicting
Briefly stated, courts of justice determine the limits of power of the agencies and offices of the government claims of authority under the Constitution and to establish for the parties in an actual controversy the
as well as those of its officers. In other words, the judiciary is the final arbiter on the question whether or rights which that instrument secures and guarantees to them. This is in truth all that is involved in what is
not a branch of government or any of its officials has acted without jurisdiction or in excess of jurisdiction, termed "judicial supremacy" which properly is the power of judicial review under the Constitution. x x x29
or so capriciously as to constitute an abuse of discretion amounting to excess of jurisdiction or lack of
jurisdiction. This is not only a judicial power but a duty to pass judgmenton matters of this nature. What are the remedies by which the grave abuse of discretion amounting to lack or excess of jurisdiction
on the part of any branch or instrumentality of the Government may be determined under the
This is the background of paragraph 2 of Section 1, which means that the courts cannot hereafter evade Constitution?
the duty to settle matters of this nature, by claiming that such matters constitute a political question. (Bold
emphasis supplied)26 The present Rules of Court uses two special civil actions for determining and correcting grave abuse of
discretion amounting to lack or excess of jurisdiction. These are the special civil actions for certiorari and
Upon interpellation by Commissioner Nolledo, Commissioner Concepcion clarified the scope of judicial prohibition, and both are governed by Rule 65. A similar remedy of certiorari exists under Rule 64, but the
power in the following manner:– remedy is expressly applicable only to the judgments and final orders or resolutions of the Commission on
Elections and the Commission on Audit.
MR. NOLLEDO. x x x
The ordinary nature and function of the writ of certiorari in our present system are aptly explained in Delos
The second paragraph of Section 1 states: "Judicial power includes the duty of courts of justice to settle Santos v. Metropolitan Bank and Trust Company:30
actual controversies…" The term "actual controversies" according to the Commissioner should refer to
questions which are political in nature and, therefore, the courts should not refuse to decide those political In the common law, from which the remedy of certiorari evolved, the writ of certiorari was issued out of
questions. But do I understand it right that this is restrictive or only an example? I know there are cases Chancery, or the King’s Bench, commanding agents or officers of the inferior courts to return the record of
which are not actual yet the court can assume jurisdiction. An example is the petition for declaratory relief. a cause pending before them, so as to give the party more sure and speedy justice, for the writ would
enable the superior court to determine from an inspection of the record whether the inferior court’s
May I ask the Commissioner’s opinion about that? judgment was rendered without authority. The errors were of such a nature that, if allowed to stand, they
would result in a substantial injury to the petitioner to whom no other remedy was available. If the inferior
court acted without authority, the record was then revised and corrected in matters of law. The writ of
MR. CONCEPCION. The Supreme Court has no jurisdiction to grant declaratory judgments. certiorari was limited to cases in which the inferior court was said to be exceeding its jurisdiction or was
not proceeding according to essential requirements of law and would lie only to review judicial or quasi-
MR. NOLLEDO. The Gentleman used the term "judicial power" but judicial power is not vested in the judicial acts.
Supreme Court alone but also in other lower courts as may be created by law.
The concept of the remedy of certiorari in our judicial system remains much the same as it has been in the
MR. CONCEPCION. Yes. common law. In this jurisdiction, however, the exercise of the power to issue the writ of certiorari is largely
regulated by laying down the instances or situations in the Rules of Court in which a superior court may
MR. NOLLEDO. And so, is this only an example? issue the writ of certiorari to an inferior court or officer. Section 1, Rule 65 of the Rules of Court
compellingly provides the requirements for that purpose, viz:
MR. CONCEPCION. No, I know this is not. The Gentleman seems to identify political questions with
jurisdictional questions. But there is a difference. xxxx

MR. NOLLEDO. Because of the expression "judicial power"? The sole office of the writ of certiorari is the correction of errors of jurisdiction, which includes the
commission of grave abuse of discretion amounting to lack of jurisdiction. In this regard, mere abuse of
discretion is not enough to warrant the issuance of the writ. The abuse of discretion must be grave, which
MR. CONCEPCION. No. Judicial power, as I said, refers to ordinary cases but where there is a question
means either that the judicial or quasi-judicial power was exercised in an arbitrary or despotic manner by
as to whether the government had authority or had abused its authority to the extent of lacking jurisdiction
reason of passion or personal hostility, or that the respondent judge, tribunal or board evaded a positive
or excess of jurisdiction, that is not a political question. Therefore, the court has the duty to decide.27
duty, or virtually refused to perform the duty enjoined or to act in contemplation of law, such as when such
judge, tribunal or board exercising judicial or quasi-judicial powers acted in a capricious or whimsical
Our previous Constitutions equally recognized the extent of the power of judicial review and the great manner as to be equivalent to lack of jurisdiction.31
responsibility of the Judiciary in maintaining the allocation of powers among the three great branches of
Government. Speaking for the Court in Angara v. Electoral Commission,28 Justice Jose P. Laurel intoned:

60
Although similar to prohibition in that it will lie for want or excess of jurisdiction, certiorari is to be The first requisite demands that there be an actual case calling for the exercise of judicial power by the
distinguished from prohibition by the fact that it is a corrective remedy used for the re-examination of Court.37 An actual case or controversy, in the words of Belgica v. Executive Secretary Ochoa:38
some action of an inferior tribunal, and is directed to the cause or proceeding in the lower court and not to
the court itself, while prohibition is a preventative remedy issuing to restrain future action, and is directed x x x is one which involves a conflict of legal rights, an assertion of opposite legal claims, susceptible of
to the court itself.32 The Court expounded on the nature and function of the writ of prohibition in Holy judicial resolution as distinguished from a hypothetical or abstract difference or dispute. In other words,
Spirit Homeowners Association, Inc. v. Defensor:33 "[t]here must be a contrariety of legal rights that can be interpreted and enforced on the basis of existing
law and jurisprudence." Related to the requirement of an actual case or controversy is the requirement of
A petition for prohibition is also not the proper remedy to assail an IRR issued in the exercise of a quasi- "ripeness," meaning that the questions raised for constitutional scrutiny are already ripe for adjudication.
legislative function. Prohibition is an extraordinary writ directed against any tribunal, corporation, board, "A question is ripe for adjudication when the act being challenged has had a direct adverse effect on the
officer or person, whether exercising judicial, quasi-judicial or ministerial functions, ordering said entity or individual challenging it. It is a prerequisite that something had then been accomplished or performed by
person to desist from further proceedings when said proceedings are without or in excess of said entity’s either branch before a court may come into the picture, and the petitioner must allege the existence of an
or person’s jurisdiction, or are accompanied with grave abuse of discretion, and there is no appeal or any immediate or threatened injury to itself as a result of the challenged action." "Withal, courts will decline to
other plain, speedy and adequate remedy in the ordinary course of law. Prohibition lies against judicial or pass upon constitutional issues through advisory opinions, bereft as they are of authority to resolve
ministerial functions, but not against legislative or quasi-legislative functions. Generally, the purpose of a hypothetical or moot questions."
writ of prohibition is to keep a lower court within the limits of its jurisdiction in order to maintain the
administration of justice in orderly channels. Prohibition is the proper remedy to afford relief against An actual and justiciable controversy exists in these consolidated cases. The incompatibility of the
usurpation of jurisdiction or power by an inferior court, or when, in the exercise of jurisdiction in handling perspectives of the parties on the constitutionality of the DAP and its relevant issuances satisfy the
matters clearly within its cognizance the inferior court transgresses the bounds prescribed to it by the law, requirement for a conflict between legal rights. The issues being raised herein meet the requisite ripeness
or where there is no adequate remedy available in the ordinary course of law by which such relief can be considering that the challenged executive acts were already being implemented by the DBM, and there
obtained. Where the principal relief sought is to invalidate an IRR, petitioners’ remedy is an ordinary are averments by the petitioners that such implementation was repugnant to the letter and spirit of the
action for its nullification, an action which properly falls under the jurisdiction of the Regional Trial Court. In Constitution. Moreover, the implementation of the DAP entailed the allocation and expenditure of huge
any case, petitioners’ allegation that "respondents are performing or threatening to perform functions sums of public funds. The fact that public funds have been allocated, disbursed or utilized by reason or on
without or in excess of their jurisdiction" may appropriately be enjoined by the trial court through a writ of account of such challenged executive acts gave rise, therefore, to an actual controversy that is ripe for
injunction or a temporary restraining order. adjudication by the Court.

With respect to the Court, however, the remedies of certiorari and prohibition are necessarily broader in It is true that Sec. Abad manifested during the January 28, 2014 oral arguments that the DAP as a
scope and reach, and the writ of certiorari or prohibition may be issued to correct errors of jurisdiction program had been meanwhile discontinued because it had fully served its purpose, saying: "In conclusion,
committed not only by a tribunal, corporation, board or officer exercising judicial, quasi-judicial or Your Honors, may I inform the Court that because the DAP has already fully served its purpose, the
ministerial functions but also to set right, undo and restrain any act of grave abuse of discretion amounting Administration’s economic managers have recommended its termination to the President. x x x."39
to lack or excess of jurisdiction by any branch or instrumentality of the Government, even if the latter does
not exercise judicial, quasi-judicial or ministerial functions. This application is expressly authorized by the
text of the second paragraph of Section 1, supra. The Solicitor General then quickly confirmed the termination of the DAP as a program, and urged that its
termination had already mooted the challenges to the DAP’s constitutionality, viz:
Thus, petitions for certiorari and prohibition are appropriate remedies to raise constitutional issues and to
review and/or prohibit or nullify the acts of legislative and executive officials.34 DAP as a program, no longer exists, thereby mooting these present cases brought to challenge its
constitutionality. Any constitutional challenge should no longer be at the level of the program, which is
now extinct, but at the level of its prior applications or the specific disbursements under the now defunct
Necessarily, in discharging its duty under Section 1, supra, to set right and undo any act of grave abuse of policy. We challenge the petitioners to pick and choose which among the 116 DAP projects they wish to
discretion amounting to lack or excess of jurisdiction by any branch or instrumentality of the Government, nullify, the full details we will have provided by February 5. We urge this Court to be cautious in limiting
the Court is not at all precluded from making the inquiry provided the challenge was properly brought by the constitutional authority of the President and the Legislature to respond to the dynamic needs of the
interested or affected parties. The Court has been thereby entrusted expressly or by necessary country and the evolving demands of governance, lest we end up straight jacketing our elected
implication with both the duty and the obligation of determining, in appropriate cases, the validity of any representatives in ways not consistent with our constitutional structure and democratic principles.40
assailed legislative or executive action. This entrustment is consistent with the republican system of
checks and balances.35
A moot and academic case is one that ceases to present a justiciable controversy by virtue of
supervening events, so that a declaration thereon would be of no practical use or value.41
Following our recent dispositions concerning the congressional pork barrel, the Court has become more
alert to discharge its constitutional duty. We will not now refrain from exercising our expanded judicial
power in order to review and determine, with authority, the limitations on the Chief Executive’s spending The Court cannot agree that the termination of the DAP as a program was a supervening event that
power. effectively mooted these consolidated cases. Verily, the Court had in the past exercised its power of
judicial review despite the cases being rendered moot and academic by supervening events, like: (1)
when there was a grave violation of the Constitution; (2) when the case involved a situation of exceptional
b) Requisites for the exercise of the character and was of paramount public interest; (3) when the constitutional issue raised required the
power of judicial review were formulation of controlling principles to guide the Bench, the Bar and the public; and (4) when the case was
complied with capable of repetition yet evading review.42

The requisites for the exercise of the power of judicial review are the following, namely: (1) there must Assuming that the petitioners’ several submissions against the DAP were ultimately sustained by the
bean actual case or justiciable controversy before the Court; (2) the question before the Court must be Court here, these cases would definitely come under all the exceptions. Hence, the Court should not
ripe for adjudication; (3) the person challenging the act must be a proper party; and (4) the issue of abstain from exercising its power of judicial review.
constitutionality must be raised at the earliest opportunity and must be the very litis mota of the case.36
Did the petitioners have the legal standing to sue?

61
Legal standing, as a requisite for the exercise of judicial review, refers to "a right of appearance in a court remedied." With respect to taxpayer’s suits, Terr v. Jordan held that "the right of a citizen and a taxpayer
of justice on a given question."43 The concept of legal standing, or locus standi, was particularly to maintain an action in courts to restrain the unlawful use of public funds to his injury cannot be
discussed in De Castro v. Judicial and Bar Council,44 where the Court said: denied."45

In public or constitutional litigations, the Court is often burdened with the determination of the locus standi The Court has cogently observed in Agan, Jr. v. Philippine International Air Terminals Co., Inc.46 that
of the petitioners due to the ever-present need to regulate the invocation of the intervention of the Court to "[s]tanding is a peculiar concept in constitutional law because in some cases, suits are not brought by
correct any official action or policy in order to avoid obstructing the efficient functioning of public officials parties who have been personally injured by the operation of a law or any other government act but by
and offices involved in public service. It is required, therefore, that the petitioner must have a personal concerned citizens, taxpayers or voters who actually sue in the public interest."
stake in the outcome of the controversy, for, as indicated in Agan, Jr. v. Philippine International Air
Terminals Co., Inc.: Except for PHILCONSA, a petitioner in G.R. No. 209164, the petitioners have invoked their capacities as
taxpayers who, by averring that the issuance and implementation of the DAP and its relevant issuances
The question on legal standing is whether such parties have "alleged such a personal stake in the involved the illegal disbursements of public funds, have an interest in preventing the further dissipation of
outcome of the controversy as to assure that concrete adverseness which sharpens the presentation of public funds. The petitioners in G.R. No. 209287 (Araullo) and G.R. No. 209442 (Belgica) also assert their
issues upon which the court so largely depends for illumination of difficult constitutional questions." right as citizens to sue for the enforcement and observance of the constitutional limitations on the political
Accordingly, it has been held that the interest of a person assailing the constitutionality of a statute must branches of the Government.47
be direct and personal. He must be able to show, not only that the law or any government act is invalid,
but also that he sustained or is in imminent danger of sustaining some direct injury as a result of its On its part, PHILCONSA simply reminds that the Court has long recognized its legal standing to bring
enforcement, and not merely that he suffers thereby in some indefinite way. It must appear that the cases upon constitutional issues.48 Luna, the petitioner in G.R. No. 209136, cites his additional capacity
person complaining has been or is about to be denied some right or privilege to which he is lawfully as a lawyer. The IBP, the petitioner in G.R. No. 209260, stands by "its avowed duty to work for the rule of
entitled or that he is about to be subjected to some burdens or penalties by reason of the statute or act law and of paramount importance of the question in this action, not to mention its civic duty as the official
complained of. association of all lawyers in this country."49

It is true that as early as in 1937, in People v. Vera, the Court adopted the direct injury test for determining Under their respective circumstances, each of the petitioners has established sufficient interest in the
whether a petitioner in a public action had locus standi. There, the Court held that the person who would outcome of the controversy as to confer locus standi on each of them.
assail the validity of a statute must have "a personal and substantial interest in the case such that he has
sustained, or will sustain direct injury as a result." Vera was followed in Custodio v. President of the
Senate, Manila Race Horse Trainers’ Association v. De la Fuente, Anti-Chinese League of the Philippines In addition, considering that the issues center on the extent of the power of the Chief Executive to
v. Felix, and Pascual v. Secretary of Public Works. disburse and allocate public funds, whether appropriated by Congress or not, these cases pose issues
that are of transcendental importance to the entire Nation, the petitioners included. As such, the
determination of such important issues call for the Court’s exercise of its broad and wise discretion "to
Yet, the Court has also held that the requirement of locus standi, being a mere procedural technicality, waive the requirement and so remove the impediment to its addressing and resolving the serious
can be waived by the Court in the exercise of its discretion. For instance, in 1949, in Araneta v. Dinglasan, constitutional questions raised."50
the Court liberalized the approach when the cases had "transcendental importance." Some notable
controversies whose petitioners did not pass the direct injury test were allowed to be treated in the same
way as in Araneta v. Dinglasan. II. Substantive Issues

In the 1975 decision in Aquino v. Commission on Elections, this Court decided to resolve the issues 1.Overview of the Budget System
raised by the petition due to their "far reaching implications," even if the petitioner had no personality to
file the suit. The liberal approach of Aquino v. Commission on Elections has been adopted in several An understanding of the Budget System of the Philippines will aid the Court in properly appreciating and
notable cases, permitting ordinary citizens, legislators, and civic organizations to bring their suits involving justly resolving the substantive issues.
the constitutionality or validity of laws, regulations, and rulings.
a) Origin of the Budget System
However, the assertion of a public right as a predicate for challenging a supposedly illegal or
unconstitutional executive or legislative action rests on the theory that the petitioner represents the public The term "budget" originated from the Middle English word bouget that had derived from the Latin word
in general. Although such petitioner may not be as adversely affected by the action complained against as bulga (which means bag or purse).51
are others, it is enough that he sufficiently demonstrates in his petition that he is entitled to protection or
relief from the Court in the vindication of a public right.
In the Philippine setting, Commonwealth Act (CA) No. 246 (Budget Act) defined "budget" as the financial
program of the National Government for a designated fiscal year, consisting of the statements of
Quite often, as here, the petitioner in a public action sues as a citizen or taxpayer to gain locus standi. estimated receipts and expenditures for the fiscal year for which it was intended to be effective based on
That is not surprising, for even if the issue may appear to concern only the public in general, such the results of operations during the preceding fiscal years. The term was given a different meaning under
capacities nonetheless equip the petitioner with adequate interest to sue. In David v. Macapagal-Arroyo, Republic Act No. 992 (Revised Budget Act) by describing the budget as the delineation of the services
the Court aptly explains why: and products, or benefits that would accrue to the public together with the estimated unit cost of each type
of service, product or benefit.52 For a forthright definition, budget should simply be identified as the
Case law in most jurisdiction snow allows both "citizen" and "taxpayer" standing in public actions. The financial plan of the Government,53 or "the master plan of government."54
distinction was first laid down in Beauchamp v. Silk, where it was held that the plaintiff in a taxpayer’s suit
is in a different category from the plaintiff in a citizen’s suit. In the former, the plaintiff is affected by the The concept of budgeting has not been the product of recent economies. In reality, financing public goals
expenditure of public funds, while in the latter, he is but the mere instrument of the public concern. As held and activities was an idea that existed from the creation of the State.55 To protect the people, the territory
by the New York Supreme Court in People ex rel Case v. Collins: "In matter of mere public right, and sovereignty of the State, its government must perform vital functions that required public
however…the people are the real parties…It is at least the right, if not the duty, of every citizen to interfere expenditures. At the beginning, enormous public expenditures were spent for war activities, preservation
and see that a public offence be properly pursued and punished, and that a public grievance be of peace and order, security, administration of justice, religion, and supply of limited goods and

62
services.56 In order to finance those expenditures, the State raised revenues through taxes and which is addressed to all agencies, including state universities and colleges; and (2) a Corporate Budget
impositions.57 Thus, budgeting became necessary to allocate public revenues for specific government Call, which is addressed to all government-owned and -controlled corporations (GOCCs) and government
functions.58 The State’s budgeting mechanism eventually developed through the years with the growing financial institutions (GFIs).
functions of its government and changes in its market economy.
Following the issuance of the Budget Call, the various departments and agencies submit their respective
The Philippine Budget System has been greatly influenced by western public financial institutions. This is Agency Budget Proposals to the DBM. To boost citizen participation, the current administration has
because of the country’s past as a colony successively of Spain and the United States for a long period of tasked the various departments and agencies to partner with civil society organizations and other citizen-
time. Many aspects of the country’s public fiscal administration, including its Budget System, have been stakeholders in the preparation of the Agency Budget Proposals, which proposals are then presented
naturally patterned after the practices and experiences of the western public financial institutions. At any before a technical panel of the DBM in scheduled budget hearings wherein the various departments and
rate, the Philippine Budget System is presently guided by two principal objectives that are vital to the agencies are given the opportunity to defend their budget proposals. DBM bureaus thereafter review the
development of a progressive democratic government, namely: (1) to carry on all government activities Agency Budget Proposals and come up with recommendations for the Executive Review Board,
under a comprehensive fiscal plan developed, authorized and executed in accordance with the comprised by the DBM Secretary and the DBM’s senior officials. The discussions of the Executive Review
Constitution, prevailing statutes and the principles of sound public management; and (2) to provide for the Board cover the prioritization of programs and their corresponding support vis-à-vis the priority agenda of
periodic review and disclosure of the budgetary status of the Government in such detail so that persons the National Government, and their implementation.
entrusted by law with the responsibility as well as the enlightened citizenry can determine the adequacy of
the budget actions taken, authorized or proposed, as well as the true financial position of the The DBM next consolidates the recommended agency budgets into the National Expenditure Program
Government.59 (NEP)and a Budget of Expenditures and Sources of Financing (BESF). The NEP provides the details of
spending for each department and agency by program, activity or project (PAP), and is submitted in the
b) Evolution of the Philippine Budget System form of a proposed GAA. The Details of Selected Programs and Projects is the more detailed
disaggregation of key PAPs in the NEP, especially those in line with the National Government’s
The budget process in the Philippines evolved from the early years of the American Regime up to the development plan. The Staffing Summary provides the staffing complement of each department and
passage of the Jones Law in 1916. A Budget Office was created within the Department of Finance by the agency, including the number of positions and amounts allocated.
Jones Law to discharge the budgeting function, and was given the responsibility to assist in the
preparation of an executive budget for submission to the Philippine Legislature.60 The NEP and BESF are thereafter presented by the DBM and the DBCC to the President and the Cabinet
for further refinements or reprioritization. Once the NEP and the BESF are approved by the President and
As early as under the 1935 Constitution, a budget policy and a budget procedure were established, and the Cabinet, the DBM prepares the budget documents for submission to Congress. The budget
subsequently strengthened through the enactment of laws and executive acts.61 EO No. 25, issued by documents consist of: (1) the President’s Budget Message, through which the President explains the
President Manuel L. Quezon on April 25, 1936, created the Budget Commission to serve as the agency policy framework and budget priorities; (2) the BESF, mandated by Section 22, Article VII of the
that carried out the President’s responsibility of preparing the budget.62 CA No. 246, the first budget law, Constitution,68 which contains the macroeconomic assumptions, public sector context, breakdown of the
went into effect on January 1, 1938 and established the Philippine budget process. The law also provided expenditures and funding sources for the fiscal year and the two previous years; and (3) the NEP.
a line-item budget as the framework of the Government’s budgeting system,63 with emphasis on the
observance of a "balanced budget" to tie up proposed expenditures with existing revenues. Public or government expenditures are generally classified into two categories, specifically: (1) capital
expenditures or outlays; and (2) current operating expenditures. Capital expenditures are the expenses
CA No. 246 governed the budget process until the passage on June 4, 1954 of Republic Act (RA) No. whose usefulness lasts for more than one year, and which add to the assets of the Government, including
992,whereby Congress introduced performance-budgeting to give importance to functions, projects and investments in the capital of government-owned or controlled corporations and their
activities in terms of expected results.64 RA No. 992 also enhanced the role of the Budget Commission as subsidiaries.69 Current operating expenditures are the purchases of goods and services in current
the fiscal arm of the Government.65 consumption the benefit of which does not extend beyond the fiscal year.70 The two components of
current expenditures are those for personal services (PS), and those for maintenance and other operating
expenses(MOOE).
The 1973 Constitution and various presidential decrees directed a series of budgetary reforms that
culminated in the enactment of PD No. 1177 that President Marcos issued on July30, 1977, and of PD
No. 1405, issued on June 11, 1978. The latter decree converted the Budget Commission into the Ministry Public expenditures are also broadly grouped according to their functions into: (1) economic development
of Budget, and gave its head the rank of a Cabinet member. expenditures (i.e., expenditures on agriculture and natural resources, transportation and communications,
commerce and industry, and other economic development efforts);71 (2) social services or social
development expenditures (i.e., government outlay on education, public health and medicare, labor and
The Ministry of Budget was later renamed the Office of Budget and Management (OBM) under EO No. welfare and others);72 (3) general government or general public services expenditures (i.e., expenditures
711. The OBM became the DBM pursuant to EO No. 292 effective on November 24, 1989. for the general government, legislative services, the administration of justice, and for pensions and
gratuities);73 (4) national defense expenditures (i.e., sub-divided into national security expenditures and
c) The Philippine Budget Cycle66 expenditures for the maintenance of peace and order);74 and (5) public debt.75

Four phases comprise the Philippine budget process, specifically: (1) Budget Preparation; (2) Budget Public expenditures may further be classified according to the nature of funds, i.e., general fund, special
Legislation; (3) Budget Execution; and (4) Accountability. Each phase is distinctly separate from the fund or bond fund.76
others but they overlap in the implementation of the budget during the budget year.
On the other hand, public revenues complement public expenditures and cover all income or receipts of
c.1.Budget Preparation67 the government treasury used to support government expenditures.77

The budget preparation phase is commenced through the issuance of a Budget Call by the DBM. The Classical economist Adam Smith categorized public revenues based on two principal sources, stating:
Budget Call contains budget parameters earlier set by the Development Budget Coordination Committee "The revenue which must defray…the necessary expenses of government may be drawn either, first from
(DBCC) as well as policy guidelines and procedures to aid government agencies in the preparation and some fund which peculiarly belongs to the sovereign or commonwealth, and which is independent of the
submission of their budget proposals. The Budget Call is of two kinds, namely: (1) a National Budget Call, revenue of the people, or, secondly, from the revenue of the people."78 Adam Smith’s classification relied

63
on the two aspects of the nature of the State: first, the State as a juristic person with an artificial budget hearings to examine the PAPs of the departments and agencies. Thereafter, the House of
personality, and, second, the State as a sovereign or entity possessing supreme power. Under the first Representatives drafts the General Appropriations Bill (GAB).87
aspect, the State could hold property and engage in trade, thereby deriving what is called its quasi private
income or revenues, and which "peculiarly belonged to the sovereign." Under the second aspect, the The GABis sponsored, presented and defended by the House of Representatives’ Appropriations
State could collect by imposing charges on the revenues of its subjects in the form of taxes.79 Committee and Sub-Committees in plenary session. As with other laws, the GAB is approved on Third
Reading before the House of Representatives’ version is transmitted to the Senate.88
In the Philippines, public revenues are generally derived from the following sources, to wit: (1) tax
revenues(i.e., compulsory contributions to finance government activities); 80 (2) capital revenues(i.e., After transmission, the Senate conducts its own committee hearings on the GAB. To expedite
proceeds from sales of fixed capital assets or scrap thereof and public domain, and gains on such sales proceedings, the Senate may conduct its committee hearings simultaneously with the House of
like sale of public lands, buildings and other structures, equipment, and other properties recorded as fixed Representatives’ deliberations. The Senate’s Finance Committee and its Sub-Committees may submit the
assets); 81 (3) grants(i.e., voluntary contributions and aids given to the Government for its operation on proposed amendments to the GAB to the plenary of the Senate only after the House of Representatives
specific purposes in the form of money and/or materials, and do not require any monetary commitment on has formally transmitted its version to the Senate. The Senate version of the GAB is likewise approved on
the part of the recipient);82 (4) extraordinary income(i.e., repayment of loans and advances made by Third Reading.89
government corporations and local governments and the receipts and shares in income of the Banko
Sentral ng Pilipinas, and other receipts);83 and (5) public borrowings(i.e., proceeds of repayable
obligations generally with interest from domestic and foreign creditors of the Government in general, The House of Representatives and the Senate then constitute a panel each to sit in the Bicameral
including the National Government and its political subdivisions).84 Conference Committee for the purpose of discussing and harmonizing the conflicting provisions of their
versions of the GAB. The "harmonized" version of the GAB is next presented to the President for
approval.90 The President reviews the GAB, and prepares the Veto Message where budget items are
More specifically, public revenues are classified as follows:85 subjected to direct veto,91 or are identified for conditional implementation.

General Income Specific Income If, by the end of any fiscal year, the Congress shall have failed to pass the GAB for the ensuing fiscal
Subsidy Income from National 1. Income Taxes year, the GAA for the preceding fiscal year shall be deemed re-enacted and shall remain in force and
Government effect until the GAB is passed by the Congress.92
Property Taxes
Subsidy from Central Office
Taxes on Goods and Services c.3. Budget Execution93
Subsidy from Regional
Office/Staff Bureaus Taxes on International Trade and With the GAA now in full force and effect, the next step is the implementation of the budget. The Budget
Income from Government Transactions Execution Phase is primarily the function of the DBM, which is tasked to perform the following procedures,
Services Other Taxes 6.Fines and namely: (1) to issue the programs and guidelines for the release of funds; (2) to prepare an Allotment and
Income from Government Penalties-Tax Revenue Cash Release Program; (3) to release allotments; and (4) to issue disbursement authorities.
Business Operations Other Specific Income
Sales Revenue The implementation of the GAA is directed by the guidelines issued by the DBM. Prior to this, the various
Rent Income departments and agencies are required to submit Budget Execution Documents(BED) to outline their
plans and performance targets by laying down the physical and financial plan, the monthly cash program,
Insurance Income the estimate of monthly income, and the list of obligations that are not yet due and demandable.
Dividend Income
Interest Income Thereafter, the DBM prepares an Allotment Release Program (ARP)and a Cash Release Program
Sale of Confiscated Goods and (CRP).The ARP sets a limit for allotments issued in general and to a specific agency. The CRP fixes the
Properties monthly, quarterly and annual disbursement levels.
Foreign Exchange (FOREX)
Gains Allotments, which authorize an agency to enter into obligations, are issued by the DBM. Allotments are
lesser in scope than appropriations, in that the latter embrace the general legislative authority to spend.
Miscellaneous Operating and Allotments may be released in two forms – through a comprehensive Agency Budget Matrix (ABM),94 or,
Service Income individually, by SARO.95
Fines and Penalties-Government
Services and Business Operations Armed with either the ABM or the SARO, agencies become authorized to incur obligations96 on behalf of
Income from Grants and the Government in order to implement their PAPs. Obligations may be incurred in various ways, like hiring
Donations of personnel, entering into contracts for the supply of goods and services, and using utilities.
c.2. Budget Legislation86
In order to settle the obligations incurred by the agencies, the DBM issues a disbursement authority so
The Budget Legislation Phase covers the period commencing from the time Congress receives the that cash may be allocated in payment of the obligations. A cash or disbursement authority that is
President’s Budget, which is inclusive of the NEPand the BESF, up to the President’s approval of the periodically issued is referred to as a Notice of Cash Allocation (NCA),97 which issuance is based upon
GAA. This phase is also known as the Budget Authorization Phase, and involves the significant an agency’s submission of its Monthly Cash Program and other required documents. The NCA specifies
participation of the Legislative through its deliberations. the maximum amount of cash that can be withdrawn from a government servicing bank for the period
indicated. Apart from the NCA, the DBM may issue a Non-Cash Availment Authority(NCAA) to authorize
non-cash disbursements, or a Cash Disbursement Ceiling(CDC) for departments with overseas
Initially, the President’s Budget is assigned to the House of Representatives’ Appropriations Committee operations to allow the use of income collected by their foreign posts for their operating requirements.
on First Reading. The Appropriations Committee and its various Sub-Committees schedule and conduct

64
Actual disbursement or spending of government funds terminates the Budget Execution Phase and is b. History of the implementation of
usually accomplished through the Modified Disbursement Scheme under which disbursements the DAP, and sources of funds
chargeable against the National Treasury are coursed through the government servicing banks. under the DAP

c.4. Accountability98 How the Administration’s economic managers conceptualized and developed the DAP, and finally
presented it to the President remains unknown because the relevant documents appear to be scarce.
Accountability is a significant phase of the budget cycle because it ensures that the government funds
have been effectively and efficiently utilized to achieve the State’s socio-economic goals. It also allows the The earliest available document relating to the genesis of the DAP was the memorandum of October
DBM to assess the performance of agencies during the fiscal year for the purpose of implementing 12,2011 from Sec. Abad seeking the approval of the President to implement the proposed DAP. The
reforms and establishing new policies. memorandum, which contained a list of the funding sources for ₱72.11 billion and of the proposed priority
projects to be funded,115 reads:
An agency’s accountability may be examined and evaluated through (1) performance targets and
outcomes; (2) budget accountability reports; (3) review of agency performance; and (4) audit conducted MEMORANDUM FOR THE PRESIDENT
by the Commission on Audit(COA).
xxxx
2. Nature of the DAP as a fiscal plan
SUBJECT: FY 2011 PROPOSED DISBURSEMENT ACCELERATION PROGRAM (PROJECTS AND
a. DAP was a program designed to SOURCES OF FUNDS)
promote economic growth
DATE: OCTOBER 12, 2011
Policy is always a part of every budget and fiscal decision of any Administration.99 The national budget
the Executive prepares and presents to Congress represents the Administration’s "blueprint for public Mr. President, this is to formally confirm your approval of the Disbursement Acceleration Program totaling
policy" and reflects the Government’s goals and strategies.100 As such, the national budget becomes a ₱72.11 billion. We are already working with all the agencies concerned for the immediate execution of the
tangible representation of the programs of the Government in monetary terms, specifying therein the projects therein.
PAPs and services for which specific amounts of public funds are proposed and allocated.101 Embodied
in every national budget is government spending.102
A. Fund Sources for the Acceleration Program
When he assumed office in the middle of 2010, President Aquino made efficiency and transparency in
government spending a significant focus of his Administration. Yet, although such focus resulted in an Amount
improved fiscal deficit of 0.5% in the gross domestic product (GDP) from January to July of 2011, it also Action
Fund Sources (In million Description
unfortunately decelerated government project implementation and payment schedules.103 The World Requested
Php)
Bank observed that the Philippines’ economic growth could be reduced, and potential growth could be
weakened should the Government continue with its underspending and fail to address the large
deficiencies in infrastructure.104 The economic situation prevailing in the middle of 2011 thus paved the FY 2011 30,000 Unreleased Personnel Declare as
way for the development and implementation of the DAP as a stimulus package intended to fast-track Unreleased Services (PS) savings and
public spending and to push economic growth by investing on high-impact budgetary PAPs to be funded Personal appropriations which approve/
from the "savings" generated during the year as well as from unprogrammed funds.105 In that respect, Services (PS) will lapse at the end of authorize its use
the DAP was the product of "plain executive policy-making" to stimulate the economy by way of Appropriations FY 2011 but may be for the 2011
accelerated spending.106 The Administration would thereby accelerate government spending by: (1) pooled as savings and Disbursement
streamlining the implementation process through the clustering of infrastructure projects of the realigned for priority Acceleration
Department of Public Works and Highways (DPWH) and the Department of Education (DepEd),and (2) programs that require Program
front loading PPP-related projects107 due for implementation in the following year.108 immediate funding

Did the stimulus package work? FY 2011 482 Unreleased  


Unreleased appropriations (slow
Appropriations moving projects and
The March 2012 report of the World Bank,109 released after the initial implementation of the DAP, programs for
revealed that the DAP was partially successful. The disbursements under the DAP contributed 1.3 discontinuance)
percentage points to GDP growth by the fourth quarter of 2011.110 The continued implementation of the
DAP strengthened growth by 11.8% year on year while infrastructure spending rebounded from a 29%
contraction to a 34% growth as of September 2013.111 FY 2010 12,336 Supported by the GFI Approve and
Unprogrammed Dividends authorize its use
Fund for the 2011
The DAP thus proved to be a demonstration that expenditure was a policy instrument that the Disbursement
Government could use to direct the economies towards growth and development.112 The Government, Acceleration
by spending on public infrastructure, would signify its commitment of ensuring profitability for prospective Program
investors.113 The PAPs funded under the DAP were chosen for this reason based on their: (1) multiplier
impact on the economy and infrastructure development; (2) beneficial effect on the poor; and (3)
translation into disbursements.114

65
FY 2010 21,544 Unreleased With prior delay in the certification of the LGU
Carryover appropriations (slow approval from counterpart. Without it, the NG is obliged to
Appropriation moving projects and the President in pay the full amount.
programs for November 2010
discontinuance) and to declare as 8. Philpost: Purchase of foreclosed property. 644
savings from Zero-based savings and with Payment of Mandatory Obligations, (GSIS,
Budgeting authority to use PhilHealth, ECC), Franking Privilege
Initiative for priority
projects 9. BSP: First equity infusion out of Php 40B 10,000
capitalization under the BSP Law
FY 2011 Budget 7,748 FY 2011 Agency For information
10. PCMC: Capital and Equipment Renovation 280
items for Budget items that can
realignment be realigned within the
11. LCOP: 105
agency to fund new fast
a. Pediatric Pulmonary Program
disbursing projects
b. Bio-regenerative Technology Program 35
DPWH-3.981 Billion
(Stem-Cell Research – subject to legal
DA – 2.497 Billion 70
review and presentation)
DOT – 1.000 Billion
DepEd – 270 Million
12. TIDCORP: NG Equity infusion 570

TOTAL 72.110 TOTAL 26,945


B. Projects in the Disbursement Acceleration Program
NGAs/LGUs
(Descriptions of projects attached as Annex A)
Agency/Project Allotment
(SARO) Cash
GOCCs and GFIs (In Million Requirement
Php) (NCA)
Agency/Project Allotment
(SARO and NCA Release) (in Million Php) 13. DOF-BIR: NPSTAR
centralization of data    
1. LRTA: Rehabilitation of LRT 1 and 2 1,868 processing and others (To be    
synchronized with GFMIS    
2. NHA: 11,050 activities) 758 758
a. Resettlement of North Triangle residents to 450 14. COA: IT infrastructure
Camarin A7 program and hiring of    
b. Housing for BFP/BJMP 500 additional litigational experts 144 144
c. On-site development for families living 10,000
along dangerous 15. DND-PAF: On Base Housing
d. Relocation sites for informal settlers 100 Facilities and Communication    
along Iloilo River and its tributaries Equipment 30 30
3. PHIL. HEART CENTER: Upgrading of 357 16. DA: 2,959 2,223
ageing physical plant and medical equipment a. Irrigation, FMRs and
Integrated Community Based Multi-Species    
4. CREDIT INFO CORP: Establishment of 75 Hatchery and Aquasilvi    
centralized credit information system Farming 1,629 1,629
b. Mindanao Rural
5. PIDS: purchase of land to relocate the PIDS 100 Development Project 919 183
office and building construction
c. NIA Agno River Integrated
6. HGC: Equity infusion for credit insurance 400 Irrigation Project 411 411
and mortgage guaranty operations of HGC
17. DAR: 1,293 1,293
7. PHIC: Obligations incurred (premium 1,496 a. Agrarian Reform
subsidy for indigent families) in January-June Communities Project 2 1,293 132
2010, booked for payment in Jul[y] – Dec b. Landowners Compensation 5,432
2010. The delay in payment is due to the

66
18. DBM: Conduct of National and Development Intervention 8,592 8,592
Survey of    
Farmers/Fisherfolks/Ips 625 625 31. DOTC-MRT: Purchase of
additional MRT cars 4,500 -
19. DOJ: Operating requirements
of 50 investigation agents and     32. LGU Support Fund 6,500 6,500
15 state attorneys 11 11
33. Various Other Local Projects 6,500 6,500
20. DOT: Preservation of the Cine
Corregidor Complex 25 25 34. Development Assistance to the
Province of Quezon 750 750
21. OPAPP: Activities for Peace
Process (PAMANA- Project     TOTAL 45,165 44,000
details: budget breakdown,    
C. Summary
implementation plan, and    
conditions on fund release       Fund Sources
attached as Annex B) 1,819 1,819 Identified for Allotments Cash
Approval for Release Requirements for
22. DOST 425 425 (In Million Release in FY
a. Establishment of National Php) 2011
Meterological and Climate    
Center 275 275 Total 72,110 72,110 70,895
b. Enhancement of Doppler
Radar Network for National     GOCCs 26,895 26,895
Weather Watch, Accurate    
Forecasting and Flood Early     NGAs/LGUs 45,165 44,000
Warning 190 190
For His Excellency’s Consideration
23. DOF-BOC: To settle the
principal obligations with     (Sgd.) FLORENCIO B. ABAD
PDIC consistent with the    
agreement with the CISS and     [/] APPROVED
SGS 2,800 2,800

24. OEO-FDCP: Establishment of [ ] DISAPPROVED


the National Film Archive and    
local cinematheques, and other     (Sgd.) H.E. BENIGNO S. AQUINO, III
local activities 20 20
OCT 12, 2011
25. DPWH: Various infrastructure
projects 5,500 5,500
The memorandum of October 12, 2011 was followed by another memorandum for the President dated
December 12, 2011116 requesting omnibus authority to consolidate the savings and unutilized balances
26. DepEd/ERDT/DOST: Thin
for fiscal year 2011. Pertinent portions of the memorandum of December 12, 2011 read:
Client Cloud Computing    
Project 270 270
MEMORANDUM FOR THE PRESIDENT
27. DOH: Hiring of nurses and
midwives 294 294 xxxx

28. TESDA: Training Program in


SUBJECT: Omnibus Authority to Consolidate Savings/Unutilized Balances and its Realignment
partnership with BPO industry    
and other sectors 1,100 1,100
DATE: December 12, 2011
29. DILG: Performance Challenge
Fund (People Empowered     This is to respectfully request for the grant of Omnibus Authority to consolidate savings/unutilized
Community Driven     balances in FY 2011 corresponding to completed or discontinued projects which may be pooled to fund
Development with DSWD and     additional projects or expenditures.
NAPC) 250 50
In addition, Mr. President, this measure will allow us to undertake projects even if their implementation
30. ARMM: Comprehensive Peace
carries over to 2012 without necessarily impacting on our budget deficit cap next year.

67
BACKGROUND 8.1 Grant of omnibus authority to consolidate FY 2011 savings/unutilized balances and its realignment;
and
1.0 The DBM, during the course of performance reviews conducted on the agencies’ operations,
particularly on the implementation of their projects/activities, including expenses incurred in undertaking 8.2 The proposed additional projects identified for funding.
the same, have identified savings out of the 2011 General Appropriations Act. Said savings correspond to
completed or discontinued projects under certain departments/agencies which may be pooled, for the For His Excellency’s consideration and approval.
following:
(Sgd.)
1.1 to provide for new activities which have not been anticipated during preparation of the budget;
[/] APPROVED
1.2 to augment additional requirements of on-going priority projects; and
[ ] DISAPPROVED
1.3 to provide for deficiencies under the Special Purpose Funds, e.g., PDAF, Calamity Fund, Contingent
Fund
(Sgd.) H.E. BENIGNO S. AQUINO, III
1.4 to cover for the modifications of the original allotment class allocation as a result of on-going priority
projects and implementation of new activities DEC 21, 2011

2.0 x x x x Substantially identical requests for authority to pool savings and to fund proposed projects were contained
in various other memoranda from Sec. Abad dated June 25, 2012,117 September 4, 2012,118 December
19, 2012,119 May 20, 2013,120 and September 25, 2013.121 The President apparently approved all the
2.1 x x x requests, withholding approval only of the proposed projects contained in the June 25, 2012
memorandum, as borne out by his marginal note therein to the effect that the proposed projects should
2.2 x x x still be "subject to further discussions."122

ON THE UTILIZATION OF POOLED SAVINGS In order to implement the June25, 2012 memorandum, Sec. Abad issued NBC No. 541 (Adoption of
Operational Efficiency Measure – Withdrawal of Agencies’ Unobligated Allotments as of June 30,
3.0 It may be recalled that the President approved our request for omnibus authority to pool 2012),123 reproduced herein as follows:
savings/unutilized balances in FY 2010 last November 25, 2010.
NATIONAL BUDGET CIRCULAR No. 541
4.0 It is understood that in the utilization of the pooled savings, the DBM shall secure the corresponding
approval/confirmation of the President. Furthermore, it is assured that the proposed realignments shall be July 18, 2012
within the authorized Expenditure level.
TO: All Heads of Departments/Agencies/State Universities and Colleges and other Offices of the National
5.0 Relative thereto, we have identified some expenditure items that may be sourced from the said pooled Government, Budget and Planning Officers; Heads of Accounting Units and All Others Concerned
appropriations in FY 2010 that will expire on December 31, 2011 and appropriations in FY 2011 that may
be declared as savings to fund additional expenditures. SUBJECT : Adoption of Operational Efficiency Measure – Withdrawal of Agencies’ Unobligated
Allotments as of June 30, 2012
5.1 The 2010 Continuing Appropriations (pooled savings) is proposed to be spent for the projects that we
have identified to be immediate actual disbursements considering that this same fund source will expire 1.0 Rationale
on December 31, 2011.
The DBM, as mandated by Executive Order (EO) No. 292 (Administrative Code of 1987), periodically
5.2 With respect to the proposed expenditure items to be funded from the FY 2011 Unreleased reviews and evaluates the departments/agencies’ efficiency and effectiveness in utilizing budgeted funds
Appropriations, most of these are the same projects for which the DBM is directed by the Office of the for the delivery of services and production of goods, consistent with the government priorities.
President, thru the Executive Secretary, to source funds.
In the event that a measure is necessary to further improve the operational efficiency of the government,
6.0 Among others, the following are such proposed additional projects that have been chosen given their the President is authorized to suspend or stop further use of funds allotted for any agency or expenditure
multiplier impact on economy and infrastructure development, their beneficial effect on the poor, and their authorized in the General Appropriations Act. Withdrawal and pooling of unutilized allotment releases can
translation into disbursements. Please note that we have classified the list of proposed projects as follows: be effected by DBM based on authority of the President, as mandated under Sections 38 and 39, Chapter
5, Book VI of EO 292.
7.0 x x x
For the first five months of 2012, the National Government has not met its spending targets. In order to
FOR THE PRESIDENT’S APPROVAL accelerate spending and sustain the fiscal targets during the year, expenditure measures have to be
implemented to optimize the utilization of available resources.
8.0 Foregoing considered, may we respectfully request for the President’s approval for the following:
Departments/agencies have registered low spending levels, in terms of obligations and disbursements per
initial review of their 2012 performance. To enhance agencies’ performance, the DBM conducts

68
continuous consultation meetings and/or send call-up letters, requesting them to identify slow-moving 4.2.2 MOOE items earmarked for specific purposes or subject to realignment conditions per General
programs/projects and the factors/issues affecting their performance (both pertaining to internal systems Provisions of the GAA:
and those which are outside the agencies’ spheres of control). Also, they are asked to formulate
strategies and improvement plans for the rest of 2012. • Confidential and Intelligence Fund;

Notwithstanding these initiatives, some departments/agencies have continued to post low obligation levels • Savings from Traveling, Communication, Transportation and Delivery, Repair and Maintenance,
as of end of first semester, thus resulting to substantial unobligated allotments. Supplies and Materials and Utility which shall be used for the grant of Collective Negotiation Agreement
incentive benefit;
In line with this, the President, per directive dated June 27, 2012 authorized the withdrawal of unobligated
allotments of agencies with low levels of obligations as of June 30, 2012, both for continuing and current • Savings from mandatory expenditures which can be realigned only in the last quarter after taking into
allotments. This measure will allow the maximum utilization of available allotments to fund and undertake consideration the agency’s full year requirements, i.e., Petroleum, Oil and Lubricants, Water, Illumination,
other priority expenditures of the national government. Power Services, Telephone, other Communication Services and Rent.

2.0 Purpose 4.2.3 Foreign-Assisted Projects (loan proceeds and peso counterpart);

2.1 To provide the conditions and parameters on the withdrawal of unobligated allotments of agencies as 4.2.4 Special Purpose Funds such as: E-Government Fund, International Commitments Fund, PAMANA,
of June 30, 2012 to fund priority and/or fast-moving programs/projects of the national government; Priority Development Assistance Fund, Calamity Fund, Budgetary Support to GOCCs and Allocation to
LGUs, among others;
2.2 To prescribe the reports and documents to be used as bases on the withdrawal of said unobligated
allotments; and 4.2.5 Quick Response Funds; and

2.3 To provide guidelines in the utilization or reallocation of the withdrawn allotments. 4.2.6 Automatic Appropriations i.e., Retirement Life Insurance Premium and Special Accounts in the
General Fund.
3.0 Coverage
5.0 Guidelines
3.1 These guidelines shall cover the withdrawal of unobligated allotments as of June 30, 2012 of all
national government agencies (NGAs) charged against FY 2011 Continuing Appropriation (R.A. 5.1 National government agencies shall continue to undertake procurement activities notwithstanding the
No.10147) and FY 2012 Current Appropriation (R.A. No. 10155), pertaining to: implementation of the policy of withdrawal of unobligated allotments until the end of the third quarter, FY
2012. Even without the allotments, the agency shall proceed in undertaking the procurement processes
3.1.1 Capital Outlays (CO); (i.e., procurement planning up to the conduct of bidding but short of awarding of contract) pursuant to
GPPB Circular Nos. 02-2008 and 01-2009 and DBM Circular Letter No. 2010-9.
3.1.2 Maintenance and Other Operating Expenses (MOOE) related to the implementation of programs
and projects, as well as capitalized MOOE; and 5.2 For the purpose of determining the amount of unobligated allotments that shall be withdrawn, all
departments/agencies/operating units (OUs) shall submit to DBM not later than July 30, 2012, the
3.1.3 Personal Services corresponding to unutilized pension benefits declared as savings by the agencies following budget accountability reports as of June 30, 2012;
concerned based on their updated/validated list of pensioners.
• Statement of Allotments, Obligations and Balances (SAOB);
3.2 The withdrawal of unobligated allotments may cover the identified programs, projects and activities of
the departments/agencies reflected in the DBM list shown as Annex A or specific programs and projects • Financial Report of Operations (FRO); and
as may be identified by the agencies.
• Physical Report of Operations.
4.0 Exemption
5.3 In the absence of the June 30, 2012 reports cited under item 5.2 of this Circular, the agency’s latest
These guidelines shall not apply to the following: report available shall be used by DBM as basis for withdrawal of allotment. The DBM shall
compute/approximate the agency’s obligation level as of June 30 to derive its unobligated allotments as of
4.1 NGAs same period. Example: If the March 31 SAOB or FRO reflects actual obligations of P 800M then the June
30 obligation level shall approximate to ₱1,600 M (i.e., ₱800 M x 2 quarters).
4.1.1 Constitutional Offices/Fiscal Autonomy Group, granted fiscal autonomy under the Philippine
Constitution; and 5.4 All released allotments in FY 2011 charged against R.A. No. 10147 which remained unobligated as of
June 30, 2012 shall be immediately considered for withdrawal. This policy is based on the following
considerations:
4.1.2 State Universities and Colleges, adopting the Normative Funding allocation scheme i.e., distribution
of a predetermined budget ceiling.
5.4.1 The departments/agencies’ approved priority programs and projects are assumed to be
implementation-ready and doable during the given fiscal year; and
4.2 Fund Sources

4.2.1 Personal Services other than pension benefits;


69
5.4.2 The practice of having substantial carryover appropriations may imply that the agency has a slower- This circular shall take effect immediately.
than-programmed implementation capacity or agency tends to implement projects within a two-year
timeframe. (Sgd.) FLORENCIO B. ABAD
Secretary
5.5. Consistent with the President’s directive, the DBM shall, based on evaluation of the reports cited
above and results of consultations with the departments/agencies, withdraw the unobligated allotments as As can be seen, NBC No. 541 specified that the unobligated allotments of all agencies and departments
of June 30, 2012 through issuance of negative Special Allotment Release Orders (SAROs). as of June 30, 2012 that were charged against the continuing appropriations for fiscal year 2011 and the
2012 GAA (R.A. No. 10155) were subject to withdrawal through the issuance of negative SAROs, but
5.6 DBM shall prepare and submit to the President, a report on the magnitude of withdrawn allotments. such allotments could be either: (1) reissued for the original PAPs of the concerned agencies from which
The report shall highlight the agencies which failed to submit the June 30 reports required under this they were withdrawn; or (2) realigned to cover additional funding for other existing PAPs of the concerned
Circular. agencies; or (3) used to augment existing PAPs of any agency and to fund priority PAPs not considered in
the 2012 budget but expected to be started or implemented in 2012. Financing the other priority PAPs
5.7 The withdrawn allotments may be: was made subject to the approval of the President. Note here that NBC No. 541 used terminologies like
"realignment" and "augmentation" in the application of the withdrawn unobligated allotments.
5.7.1 Reissued for the original programs and projects of the agencies/OUs concerned, from which the
allotments were withdrawn; Taken together, all the issuances showed how the DAP was to be implemented and funded, that is — (1)
by declaring "savings" coming from the various departments and agencies derived from pooling
unobligated allotments and withdrawing unreleased appropriations; (2) releasing unprogrammed funds;
5.7.2 Realigned to cover additional funding for other existing programs and projects of the agency/OU; or and (3) applying the "savings" and unprogrammed funds to augment existing PAPs or to support other
priority PAPs.
5.7.3 Used to augment existing programs and projects of any agency and to fund priority programs and
projects not considered in the 2012 budget but expected to be started or implemented during the current c. DAP was not an appropriation
year. measure; hence, no appropriation
law was required to adopt or to
5.8 For items 5.7.1 and 5.7.2 above, agencies/OUs concerned may submit to DBM a Special Budget implement it
Request (SBR), supported with the following:
Petitioners Syjuco, Luna, Villegas and PHILCONSA state that Congress did not enact a law to establish
5.8.1 Physical and Financial Plan (PFP); the DAP, or to authorize the disbursement and release of public funds to implement the DAP. Villegas,
PHILCONSA, IBP, Araullo, and COURAGE observe that the appropriations funded under the DAP were
5.8.2 Monthly Cash Program (MCP); and not included in the 2011, 2012 and 2013 GAAs. To petitioners IBP, Araullo, and COURAGE, the DAP,
being actually an appropriation that set aside public funds for public use, should require an enabling law
for its validity. VACC maintains that the DAP, because it involved huge allocations that were separate and
5.8.3 Proof that the project/activity has started the procurement processes i.e., Proof of Posting and/or
distinct from the GAAs, circumvented and duplicated the GAAs without congressional authorization and
Advertisement of the Invitation to Bid.
control.

5.9 The deadline for submission of request/s pertaining to these categories shall be until the end of the
The petitioners contend in unison that based on how it was developed and implemented the DAP violated
third quarter i.e., September 30, 2012. After said cut-off date, the withdrawn allotments shall be pooled
the mandate of Section 29(1), Article VI of the 1987 Constitution that "[n]o money shall be paid out of the
and form part of the overall savings of the national government.
Treasury except in pursuance of an appropriation made by law."

5.10 Utilization of the consolidated withdrawn allotments for other priority programs and projects as cited
The OSG posits, however, that no law was necessary for the adoption and implementation of the DAP
under item 5.7.3 of this Circular, shall be subject to approval of the President. Based on the approval of
because of its being neither a fund nor an appropriation, but a program or an administrative system of
the President, DBM shall issue the SARO to cover the approved priority expenditures subject to
prioritizing spending; and that the adoption of the DAP was by virtue of the authority of the President as
submission by the agency/OU concerned of the SBR and supported with PFP and MCP.
the Chief Executive to ensure that laws were faithfully executed.

5.11 It is understood that all releases to be made out of the withdrawn allotments (both 2011 and 2012
We agree with the OSG’s position.
unobligated allotments) shall be within the approved Expenditure Program level of the national
government for the current year. The SAROs to be issued shall properly disclose the appropriation source
of the release to determine the extent of allotment validity, as follows: The DAP was a government policy or strategy designed to stimulate the economy through accelerated
spending. In the context of the DAP’s adoption and implementation being a function pertaining to the
Executive as the main actor during the Budget Execution Stage under its constitutional mandate to
• For charges under R.A. 10147 – allotments shall be valid up to December 31, 2012; and
faithfully execute the laws, including the GAAs, Congress did not need to legislate to adopt or to
implement the DAP. Congress could appropriate but would have nothing more to do during the Budget
• For charges under R.A. 10155 – allotments shall be valid up to December 31, 2013. Execution Stage. Indeed, appropriation was the act by which Congress "designates a particular fund, or
sets apart a specified portion of the public revenue or of the money in the public treasury, to be applied to
5.12 Timely compliance with the submission of existing BARs and other reportorial requirements is some general object of governmental expenditure, or to some individual purchase or expense."124 As
reiterated for monitoring purposes. pointed out in Gonzales v. Raquiza:125 ‘"In a strict sense, appropriation has been defined ‘as nothing
more than the legislative authorization prescribed by the Constitution that money may be paid out of the
Treasury,’ while appropriation made by law refers to ‘the act of the legislature setting apart or assigning to
6.0 Effectivity
a particular use a certain sum to be used in the payment of debt or dues from the State to its
creditors.’"126
70
On the other hand, the President, in keeping with his duty to faithfully execute the laws, had sufficient xxxx
discretion during the execution of the budget to adapt the budget to changes in the country’s economic
situation.127 He could adopt a plan like the DAP for the purpose. He could pool the savings and identify x x x The expenditure process, by its very nature, requires substantial discretion for administrators. They
the PAPs to be funded under the DAP. The pooling of savings pursuant to the DAP, and the identification need to exercise judgment and take responsibility for their actions, but those actions ought to be directed
of the PAPs to be funded under the DAP did not involve appropriation in the strict sense because the toward executing congressional, not administrative policy. Let there be discretion, but channel it and use it
money had been already set apart from the public treasury by Congress through the GAAs. In such to satisfy the programs and priorities established by Congress.
actions, the Executive did not usurp the power vested in Congress under Section 29(1), Article VI of the
Constitution.
In contrast, by allowing to the heads of offices some power to transfer funds within their respective offices,
the Constitution itself ensures the fiscal autonomy of their offices, and at the same time maintains the
3. separation of powers among the three main branches of the Government. The Court has recognized this,
Unreleased appropriations and withdrawn and emphasized so in Bengzon v. Drilon,133 viz:
unobligated allotments under the DAP
were not savings, and the use of such
appropriations contravened Section 25(5), The Judiciary, the Constitutional Commissions, and the Ombudsman must have the independence and
Article VI of the 1987 Constitution. flexibility needed in the discharge of their constitutional duties. The imposition of restrictions and
constraints on the manner the independent constitutional offices allocate and utilize the funds
appropriated for their operations is anathema to fiscal autonomy and violative not only of the express
Notwithstanding our appreciation of the DAP as a plan or strategy validly adopted by the Executive to mandate of the Constitution but especially as regards the Supreme Court, of the independence and
ramp up spending to accelerate economic growth, the challenges posed by the petitioners constrain us to separation of powers upon which the entire fabric of our constitutional system is based.
dissect the mechanics of the actual execution of the DAP. The management and utilization of the public
wealth inevitably demands a most careful scrutiny of whether the Executive’s implementation of the DAP
was consistent with the Constitution, the relevant GAAs and other existing laws. In the case of the President, the power to transfer funds from one item to another within the Executive has
not been the mere offshoot of established usage, but has emanated from law itself. It has existed since
the time of the American Governors-General.134 Act No. 1902 (An Act authorizing the Governor-General
a. Although executive discretion to direct any unexpended balances of appropriations be returned to the general fund of the Insular
and flexibility are necessary in Treasury and to transfer from the general fund moneys which have been returned thereto), passed on
the execution of the budget, any May 18, 1909 by the First Philippine Legislature,135 was the first enabling law that granted statutory
transfer of appropriated funds authority to the President to transfer funds. The authority was without any limitation, for the Act explicitly
should conform to Section 25(5), empowered the Governor-General to transfer any unexpended balance of appropriations for any bureau
Article VI of the Constitution or office to another, and to spend such balance as if it had originally been appropriated for that bureau or
office.
We begin this dissection by reiterating that Congress cannot anticipate all issues and needs that may
come into play once the budget reaches its execution stage. Executive discretion is necessary at that From 1916 until 1920, the appropriations laws set a cap on the amounts of funds that could be
stage to achieve a sound fiscal administration and assure effective budget implementation. The heads of transferred, thereby limiting the power to transfer funds. Only 10% of the amounts appropriated for
offices, particularly the President, require flexibility in their operations under performance budgeting to contingent or miscellaneous expenses could be transferred to a bureau or office, and the transferred
enable them to make whatever adjustments are needed to meet established work goals under changing funds were to be used to cover deficiencies in the appropriations also for miscellaneous expenses of said
conditions.128 In particular, the power to transfer funds can give the President the flexibility to meet bureau or office.
unforeseen events that may otherwise impede the efficient implementation of the PAPs set by Congress
in the GAA.
In 1921, the ceiling on the amounts of funds to be transferred from items under miscellaneous expenses
to any other item of a certain bureau or office was removed.
Congress has traditionally allowed much flexibility to the President in allocating funds pursuant to the
GAAs,129 particularly when the funds are grouped to form lump sum accounts.130 It is assumed that the
agencies of the Government enjoy more flexibility when the GAAs provide broader appropriation During the Commonwealth period, the power of the President to transfer funds continued to be governed
items.131 This flexibility comes in the form of policies that the Executive may adopt during the budget by the GAAs despite the enactment of the Constitution in 1935. It is notable that the 1935 Constitution did
execution phase. The DAP – as a strategy to improve the country’s economic position – was one policy not include a provision on the power to transfer funds. At any rate, a shift in the extent of the President’s
that the President decided to carry out in order to fulfill his mandate under the GAAs. power to transfer funds was again experienced during this era, with the President being given more
flexibility in implementing the budget. The GAAs provided that the power to transfer all or portions of the
appropriations in the Executive Department could be made in the "interest of the public, as the President
Denying to the Executive flexibility in the expenditure process would be counterproductive. In Presidential may determine."136
Spending Power,132 Prof. Louis Fisher, an American constitutional scholar whose specialties have
included budget policy, has justified extending discretionary authority to the Executive thusly:
In its time, the 1971 Constitutional Convention wanted to curtail the President’s seemingly unbounded
discretion in transferring funds.137 Its Committee on the Budget and Appropriation proposed to prohibit
[T]he impulse to deny discretionary authority altogether should be resisted. There are many number of the transfer of funds among the separate branches of the Government and the independent constitutional
reasons why obligations and outlays by administrators may have to differ from appropriations by bodies, but to allow instead their respective heads to augment items of appropriations from savings in
legislators. Appropriations are made many months, and sometimes years, in advance of expenditures. their respective budgets under certain limitations.138 The clear intention of the Convention was to further
Congress acts with imperfect knowledge in trying to legislate in fields that are highly technical and restrict, not to liberalize, the power to transfer appropriations.139 Thus, the Committee on the Budget and
constantly undergoing change. New circumstances will develop to make obsolete and mistaken the Appropriation initially considered setting stringent limitations on the power to augment, and suggested that
decisions reached by Congress at the appropriation stage. It is not practicable for Congress to adjust to the augmentation of an item of appropriation could be made "by not more than ten percent if the original
each new development by passing separate supplemental appropriation bills. Were Congress to control item of appropriation to be augmented does not exceed one million pesos, or by not more than five
expenditures by confining administrators to narrow statutory details, it would perhaps protect its power of percent if the original item of appropriation to be augmented exceeds one million pesos."140 But two
the purse but it would not protect the purse itself. The realities and complexities of public policy require members of the Committee objected to the ₱1,000,000.00 threshold, saying that the amount was arbitrary
executive discretion for the sound management of public funds.

71
and might not be reasonable in the future. The Committee agreed to eliminate the ₱1,000,000.00 The foregoing history makes it evident that the Constitutional Commission included Section 25(5), supra,
threshold, and settled on the ten percent limitation.141 to keep a tight rein on the exercise of the power to transfer funds appropriated by Congress by the
President and the other high officials of the Government named therein. The Court stated in Nazareth v.
In the end, the ten percent limitation was discarded during the plenary of the Convention, which adopted Villar:144
the following final version under Section 16, Article VIII of the 1973 Constitution, to wit:
In the funding of current activities, projects, and programs, the general rule should still be that the
(5) No law shall be passed authorizing any transfer of appropriations; however, the President, the Prime budgetary amount contained in the appropriations bill is the extent Congress will determine as sufficient
Minister, the Speaker, the Chief Justice of the Supreme Court, and the heads of Constitutional for the budgetary allocation for the proponent agency. The only exception is found in Section 25 (5),
Commissions may by law be authorized to augment any item in the general appropriations law for their Article VI of the Constitution, by which the President, the President of the Senate, the Speaker of the
respective offices from savings in other items of their respective appropriations. House of Representatives, the Chief Justice of the Supreme Court, and the heads of Constitutional
Commissions are authorized to transfer appropriations to augmentany item in the GAA for their respective
offices from the savings in other items of their respective appropriations. The plain language of the
The 1973 Constitution explicitly and categorically prohibited the transfer of funds from one item to another, constitutional restriction leaves no room for the petitioner’s posture, which we should now dispose of as
unless Congress enacted a law authorizing the President, the Prime Minister, the Speaker, the Chief untenable.
Justice of the Supreme Court, and the heads of the Constitutional omissions to transfer funds for the
purpose of augmenting any item from savings in another item in the GAA of their respective offices. The
leeway was limited to augmentation only, and was further constricted by the condition that the funds to be It bears emphasizing that the exception in favor of the high officials named in Section 25(5), Article VI of
transferred should come from savings from another item in the appropriation of the office.142 the Constitution limiting the authority to transfer savings only to augment another item in the GAA is
strictly but reasonably construed as exclusive. As the Court has expounded in Lokin, Jr. v. Commission on
Elections:
On July 30, 1977, President Marcos issued PD No. 1177, providing in its Section 44 that:
When the statute itself enumerates the exceptions to the application of the general rule, the exceptions
Section 44. Authority to Approve Fund Transfers. The President shall have the authority to transfer any are strictly but reasonably construed. The exceptions extend only as far as their language fairly warrants,
fund appropriated for the different departments, bureaus, offices and agencies of the Executive and all doubts should be resolved in favor of the general provision rather than the exceptions. Where the
Department which are included in the General Appropriations Act, to any program, project, or activity of general rule is established by a statute with exceptions, none but the enacting authority can curtail the
any department, bureau or office included in the General Appropriations Act or approved after its former. Not even the courts may add to the latter by implication, and it is a rule that an express exception
enactment. excludes all others, although it is always proper in determining the applicability of the rule to inquire
whether, in a particular case, it accords with reason and justice.
The President shall, likewise, have the authority to augment any appropriation of the Executive
Department in the General Appropriations Act, from savings in the appropriations of another department, The appropriate and natural office of the exception is to exempt something from the scope of the general
bureau, office or agency within the Executive Branch, pursuant to the provisions of Article VIII, Section 16 words of a statute, which is otherwise within the scope and meaning of such general words.
(5) of the Constitution. Consequently, the existence of an exception in a statute clarifies the intent that the statute shall apply to
all cases not excepted. Exceptions are subject to the rule of strict construction; hence, any doubt will be
In Demetria v. Alba, however, the Court struck down the first paragraph of Section 44 for contravening resolved in favor of the general provision and against the exception. Indeed, the liberal construction of a
Section 16(5)of the 1973 Constitution, ruling: statute will seem to require in many circumstances that the exception, by which the operation of the
statute is limited or abridged, should receive a restricted construction.
Paragraph 1 of Section 44 of P.D. No. 1177 unduly over-extends the privilege granted under said Section
16. It empowers the President to indiscriminately transfer funds from one department, bureau, office or Accordingly, we should interpret Section 25(5), supra, in the context of a limitation on the President’s
agency of the Executive Department to any program, project or activity of any department, bureau or discretion over the appropriations during the Budget Execution Phase.
office included in the General Appropriations Act or approved after its enactment, without regard as to
whether or not the funds to be transferred are actually savings in the item from which the same are to be b. Requisites for the valid transfer of
taken, or whether or not the transfer is for the purpose of augmenting the item to which said transfer is to appropriated funds under Section
be made. It does not only completely disregard the standards set in the fundamental law, thereby 25(5), Article VI of the 1987
amounting to an undue delegation of legislative powers, but likewise goes beyond the tenor thereof. Constitution
Indeed, such constitutional infirmities render the provision in question null and void.143
The transfer of appropriated funds, to be valid under Section 25(5), supra, must be made upon a
It is significant that Demetria was promulgated 25 days after the ratification by the people of the 1987 concurrence of the following requisites, namely:
Constitution, whose Section 25(5) of Article VI is identical to Section 16(5), Article VIII of the 1973
Constitution, to wit:
(1) There is a law authorizing the President, the President of the Senate, the Speaker of the House of
Representatives, the Chief Justice of the Supreme Court, and the heads of the Constitutional
Section 25. x x x Commissions to transfer funds within their respective offices;

5) No law shall be passed authorizing any transfer of appropriations; however, the President, the (2) The funds to be transferred are savings generated from the appropriations for their respective offices;
President of the Senate, the Speaker of the House of Representatives, the Chief Justice of the Supreme and (3) The purpose of the transfer is to augment an item in the general appropriations law for their
Court, and the heads of Constitutional Commissions may, by law, be authorized to augment any item in respective offices.
the general appropriations law for their respective offices from savings in other items of their respective
appropriations.
b.1. First Requisite–GAAs of 2011 and
2012 lacked valid provisions to
xxxx authorize transfers of funds under

72
the DAP; hence, transfers under the The petitioners claim that the funds used in the DAP — the unreleased appropriations and withdrawn
DAP were unconstitutional unobligated allotments — were not actual savings within the context of Section 25(5), supra, and the
relevant provisions of the GAAs. Belgica argues that "savings" should be understood to refer to the
Section 25(5), supra, not being a self-executing provision of the Constitution, must have an implementing excess money after the items that needed to be funded have been funded, or those that needed to be
law for it to be operative. That law, generally, is the GAA of a given fiscal year. To comply with the first paid have been paid pursuant to the budget.146 The petitioners posit that there could be savings only
requisite, the GAAs should expressly authorize the transfer of funds. when the PAPs for which the funds had been appropriated were actually implemented and completed, or
finally discontinued or abandoned. They insist that savings could not be realized with certainty in the
middle of the fiscal year; and that the funds for "slow-moving" PAPs could not be considered as savings
Did the GAAs expressly authorize the transfer of funds? because such PAPs had not actually been abandoned or discontinued yet.147 They stress that NBC No.
541, by allowing the withdrawn funds to be reissued to the "original program or project from which it was
In the 2011 GAA, the provision that gave the President and the other high officials the authority to transfer withdrawn," conceded that the PAPs from which the supposed savings were taken had not been
funds was Section 59, as follows: completed, abandoned or discontinued.148

Section 59. Use of Savings. The President of the Philippines, the Senate President, the Speaker of the The OSG represents that "savings" were "appropriations balances," being the difference between the
House of Representatives, the Chief Justice of the Supreme Court, the Heads of Constitutional appropriation authorized by Congress and the actual amount allotted for the appropriation; that the
Commissions enjoying fiscal autonomy, and the Ombudsman are hereby authorized to augment any item definition of "savings" in the GAAs set only the parameters for determining when savings occurred; that it
in this Act from savings in other items of their respective appropriations. was still the President (as well as the other officers vested by the Constitution with the authority to
augment) who ultimately determined when savings actually existed because savings could be determined
In the 2012 GAA, the empowering provision was Section 53, to wit: only during the stage of budget execution; that the President must be given a wide discretion to
accomplish his tasks; and that the withdrawn unobligated allotments were savings inasmuch as they were
clearly "portions or balances of any programmed appropriation…free from any obligation or
Section 53. Use of Savings. The President of the Philippines, the Senate President, the Speaker of the
encumbrances which are (i) still available after the completion or final discontinuance or abandonment of
House of Representatives, the Chief Justice of the Supreme Court, the Heads of Constitutional
the work, activity or purpose for which the appropriation is authorized…"
Commissions enjoying fiscal autonomy, and the Ombudsman are hereby authorized to augment any item
in this Act from savings in other items of their respective appropriations.
We partially find for the petitioners.
In fact, the foregoing provisions of the 2011 and 2012 GAAs were cited by the DBM as justification for the
use of savings under the DAP.145 In ascertaining the meaning of savings, certain principles should be borne in mind. The first principle is
that Congress wields the power of the purse. Congress decides how the budget will be spent; what PAPs
to fund; and the amounts of money to be spent for each PAP. The second principle is that the Executive,
A reading shows, however, that the aforequoted provisions of the GAAs of 2011 and 2012 were textually
as the department of the Government tasked to enforce the laws, is expected to faithfully execute the
unfaithful to the Constitution for not carrying the phrase "for their respective offices" contained in Section
GAA and to spend the budget in accordance with the provisions of the GAA.149 The Executive is
25(5), supra. The impact of the phrase "for their respective offices" was to authorize only transfers of
expected to faithfully implement the PAPs for which Congress allocated funds, and to limit the
funds within their offices (i.e., in the case of the President, the transfer was to an item of appropriation
expenditures within the allocations, unless exigencies result to deficiencies for which augmentation is
within the Executive). The provisions carried a different phrase ("to augment any item in this Act"), and the
authorized, subject to the conditions provided by law. The third principle is that in making the President’s
effect was that the 2011 and 2012 GAAs thereby literally allowed the transfer of funds from savings to
power to augment operative under the GAA, Congress recognizes the need for flexibility in budget
augment any item in the GAAs even if the item belonged to an office outside the Executive. To that extent
execution. In so doing, Congress diminishes its own power of the purse, for it delegates a fraction of its
did the 2011 and 2012 GAAs contravene the Constitution. At the very least, the aforequoted provisions
power to the Executive. But Congress does not thereby allow the Executive to override its authority over
cannot be used to claim authority to transfer appropriations from the Executive to another branch, or to a
the purse as to let the Executive exceed its delegated authority. And the fourth principle is that savings
constitutional commission.
should be actual. "Actual" denotes something that is real or substantial, or something that exists presently
in fact, as opposed to something that is merely theoretical, possible, potential or hypothetical.150
Apparently realizing the problem, Congress inserted the omitted phrase in the counterpart provision in the
2013 GAA, to wit:
The foregoing principles caution us to construe savings strictly against expanding the scope of the power
to augment. It is then indubitable that the power to augment was to be used only when the purpose for
Section 52. Use of Savings. The President of the Philippines, the Senate President, the Speaker of the which the funds had been allocated were already satisfied, or the need for such funds had ceased to
House of Representatives, the Chief Justice of the Supreme Court, the Heads of Constitutional exist, for only then could savings be properly realized. This interpretation prevents the Executive from
Commissions enjoying fiscal autonomy, and the Ombudsman are hereby authorized to use savings in unduly transgressing Congress’ power of the purse.
their respective appropriations to augment actual deficiencies incurred for the current year in any item of
their respective appropriations.
The definition of "savings" in the GAAs, particularly for 2011, 2012 and 2013, reflected this interpretation
and made it operational, viz:
Even had a valid law authorizing the transfer of funds pursuant to Section 25(5), supra, existed, there still
remained two other requisites to be met, namely: that the source of funds to be transferred were savings
Savings refer to portions or balances of any programmed appropriation in this Act free from any obligation
from appropriations within the respective offices; and that the transfer must be for the purpose of
or encumbrance which are: (i) still available after the completion or final discontinuance or abandonment
augmenting an item of appropriation within the respective offices.
of the work, activity or purpose for which the appropriation is authorized; (ii) from appropriations balances
arising from unpaid compensation and related costs pertaining to vacant positions and leaves of absence
b.2. Second Requisite – There were without pay; and (iii) from appropriations balances realized from the implementation of measures resulting
no savings from which funds in improved systems and efficiencies and thus enabled agencies to meet and deliver the required or
could be sourced for the DAP planned targets, programs and services approved in this Act at a lesser cost.
Were the funds used in the DAP actually savings?

73
The three instances listed in the GAAs’ aforequoted definition were a sure indication that savings could be ON THE AUTHORITY TO WITHDRAW UNOBLIGATED ALLOTMENTS
generated only upon the purpose of the appropriation being fulfilled, or upon the need for the
appropriation being no longer existent. 5.0 The DBM, during the course of performance reviews conducted on the agencies’ operations,
particularly on the implementation of their projects/activities, including expenses incurred in undertaking
The phrase "free from any obligation or encumbrance" in the definition of savings in the GAAs conveyed the same, have been continuously calling the attention of all National Government agencies (NGAs) with
the notion that the appropriation was at that stage when the appropriation was already obligated and the low levels of obligations as of end of the first quarter to speedup the implementation of their programs and
appropriation was already released. This interpretation was reinforced by the enumeration of the three projects in the second quarter.
instances for savings to arise, which showed that the appropriation referred to had reached the agency
level. It could not be otherwise, considering that only when the appropriation had reached the agency 6.0 Said reminders were made in a series of consultation meetings with the concerned agencies and with
level could it be determined whether (a) the PAP for which the appropriation had been authorized was call-up letters sent.
completed, finally discontinued, or abandoned; or (b) there were vacant positions and leaves of absence
without pay; or (c) the required or planned targets, programs and services were realized at a lesser cost
because of the implementation of measures resulting in improved systems and efficiencies. 7.0 Despite said reminders and the availability of funds at the department’s disposal, the level of financial
performance of some departments registered below program, with the targeted obligations/disbursements
for the first semester still not being met.
The DBM declares that part of the savings brought under the DAP came from "pooling of unreleased
appropriations such as unreleased Personnel Services appropriations which will lapse at the end of the
year, unreleased appropriations of slow moving projects and discontinued projects per Zero-Based 8.0 In order to maximize the use of the available allotment, all unobligated balances as of June 30, 2012,
Budgeting findings." both for continuing and current allotments shall be withdrawn and pooled to fund fast moving
programs/projects.
The declaration of the DBM by itself does not state the clear legal basis for the treatment of unreleased or
unalloted appropriations as savings. 9.0 It may be emphasized that the allotments to be withdrawn will be based on the list of slow moving
projects to be identified by the agencies and their catch up plans to be evaluated by the DBM.
The fact alone that the appropriations are unreleased or unalloted is a mere description of the status of
the items as unalloted or unreleased. They have not yet ripened into categories of items from which It is apparent from the foregoing text that the withdrawal of unobligated allotments would be based on
savings can be generated. Appropriations have been considered "released" if there has already been an whether the allotments pertained to slow-moving projects, or not. However, NBC No. 541 did not set in
allotment or authorization to incur obligations and disbursement authority. This means that the DBM has clear terms the criteria for the withdrawal of unobligated allotments, viz:
issued either an ABM (for those not needing clearance), or a SARO (for those needing clearance), and
consequently an NCA, NCAA or CDC, as the case may be. Appropriations remain unreleased, for 3.1. These guidelines shall cover the withdrawal of unobligated allotments as of June 30, 2012 ofall
instance, because of noncompliance with documentary requirements (like the Special Budget Request), national government agencies (NGAs) charged against FY 2011 Continuing Appropriation (R.A. No.
or simply because of the unavailability of funds. But the appropriations do not actually reach the agencies 10147) and FY 2012 Current Appropriation (R.A. No. 10155), pertaining to:
to which they were allocated under the GAAs, and have remained with the DBM technically speaking.
Ergo, unreleased appropriations refer to appropriations with allotments but without disbursement 3.1.1 Capital Outlays (CO);
authority.
3.1.2 Maintenance and Other Operating Expenses (MOOE) related to the implementation of programs
For us to consider unreleased appropriations as savings, unless these met the statutory definition of and projects, as well as capitalized MOOE; and
savings, would seriously undercut the congressional power of the purse, because such appropriations had
not even reached and been used by the agency concerned vis-à-vis the PAPs for which Congress had
3.1.3 Personal Services corresponding to unutilized pension benefits declared as savings by the agencies
allocated them. However, if an agency has unfilled positions in its plantilla and did not receive an
concerned based on their undated/validated list of pensioners.
allotment and NCA for such vacancies, appropriations for such positions, although unreleased, may
already constitute savings for that agency under the second instance.
A perusal of its various provisions reveals that NBC No. 541 targeted the "withdrawal of unobligated
allotments of agencies with low levels of obligations"151 "to fund priority and/or fast-moving
Unobligated allotments, on the other hand, were encompassed by the first part of the definition of
programs/projects."152 But the fact that the withdrawn allotments could be "[r]eissued for the original
"savings" in the GAA, that is, as "portions or balances of any programmed appropriation in this Act free
programs and projects of the agencies/OUs concerned, from which the allotments were
from any obligation or encumbrance." But the first part of the definition was further qualified by the three
withdrawn"153 supported the conclusion that the PAPs had not yet been finally discontinued or
enumerated instances of when savings would be realized. As such, unobligated allotments could not be
abandoned. Thus, the purpose for which the withdrawn funds had been appropriated was not yet fulfilled,
indiscriminately declared as savings without first determining whether any of the three instances existed.
or did not yet cease to exist, rendering the declaration of the funds as savings impossible.
This signified that the DBM’s withdrawal of unobligated allotments had disregarded the definition of
savings under the GAAs.
Worse, NBC No. 541 immediately considered for withdrawal all released allotments in 2011 charged
against the 2011 GAA that had remained unobligated based on the following considerations, to wit:
Justice Carpio has validly observed in his Separate Concurring Opinion that MOOE appropriations are
deemed divided into twelve monthly allocations within the fiscal year; hence, savings could be generated
monthly from the excess or unused MOOE appropriations other than the Mandatory Expenditures and 5.4.1 The departments/agencies’ approved priority programs and projects are assumed to be
Expenditures for Business-type Activities because of the physical impossibility to obligate and spend such implementation-ready and doable during the given fiscal year; and
funds as MOOE for a period that already lapsed. Following this observation, MOOE for future months are
not savings and cannot be transferred. 5.4.2 The practice of having substantial carryover appropriations may imply that the agency has a slower-
than-programmed implementation capacity or agency tends to implement projects within a two-year
The DBM’s Memorandum for the President dated June 25, 2012 (which became the basis of NBC No. timeframe.
541) stated:

74
Such withdrawals pursuant to NBC No. 541, the circular that affected the unobligated allotments for The respondents belie the accusation, insisting that the unobligated allotments were being withdrawn
continuing and current appropriations as of June 30, 2012, disregarded the 2-year period of availability of upon the instance of the implementing agencies based on their own assessment that they could not
the appropriations for MOOE and capital outlay extended under Section 65, General Provisions of the obligate those allotments pursuant to the President’s directive for them to spend their appropriations as
2011 GAA, viz: quickly as they could in order to ramp up the economy.156

Section 65. Availability of Appropriations. — Appropriations for MOOE and capital outlays authorized in We agree with the petitioners.
this Act shall be available for release and obligation for the purpose specified, and under the same special
provisions applicable thereto, for a period extending to one fiscal year after the end of the year in which Contrary to the respondents’ insistence, the withdrawals were upon the initiative of the DBM itself. The
such items were appropriated: PROVIDED, That appropriations for MOOE and capital outlays under R.A. text of NBC No. 541 bears this out, to wit:
No. 9970 shall be made available up to the end of FY 2011: PROVIDED, FURTHER, That a report on
these releases and obligations shall be submitted to the Senate Committee on Finance and the House
Committee on Appropriations. 5.2 For the purpose of determining the amount of unobligated allotments that shall be withdrawn, all
departments/agencies/operating units (OUs) shall submit to DBM not later than July 30, 2012, the
following budget accountability reports as of June 30, 2012;
and Section 63 General Provisions of the 2012 GAA, viz:
• Statement of Allotments, Obligation and Balances (SAOB);
Section 63. Availability of Appropriations. — Appropriations for MOOE and capital outlays authorized in
this Act shall be available for release and obligation for the purpose specified, and under the same special
provisions applicable thereto, for a period extending to one fiscal year after the end of the year in which • Financial Report of Operations (FRO); and
such items were appropriated: PROVIDED, That a report on these releases and obligations shall be
submitted to the Senate Committee on Finance and the House Committee on Appropriations, either in • Physical Report of Operations.
printed form or by way of electronic document.154
5.3 In the absence of the June 30, 2012 reports cited under item 5.2 of this Circular, the agency’s latest
Thus, another alleged area of constitutional infirmity was that the DAP and its relevant issuances report available shall be used by DBM as basis for withdrawal of allotment. The DBM shall
shortened the period of availability of the appropriations for MOOE and capital outlays. compute/approximate the agency’s obligation level as of June 30 to derive its unobligated allotments as of
same period. Example: If the March 31 SAOB or FRO reflects actual obligations of P 800M then the June
Congress provided a one-year period of availability of the funds for all allotment classes in the 2013 GAA 30 obligation level shall approximate to ₱1,600 M (i.e., ₱800 M x 2 quarters).
(R.A. No. 10352), to wit:
The petitioners assert that no law had authorized the withdrawal and transfer of unobligated allotments
Section 63. Availability of Appropriations.— All appropriations authorized in this Act shall be available for and the pooling of unreleased appropriations; and that the unbridled withdrawal of unobligated allotments
release and obligation for the purposes specified, and under the same special provisions applicable and the retention of appropriated funds were akin to the impoundment of appropriations that could be
thereto, until the end of FY 2013: PROVIDED, That a report on these releases and obligations shall be allowed only in case of "unmanageable national government budget deficit" under the GAAs,157 thus
submitted to the Senate Committee on Finance and House Committee on Appropriations, either in printed violating the provisions of the GAAs of 2011, 2012 and 2013 prohibiting the retention or deduction of
form or by way of electronic document. allotments.158

Yet, in his memorandum for the President dated May 20, 2013, Sec. Abad sought omnibus authority to In contrast, the respondents emphasize that NBC No. 541 adopted a spending, not saving, policy as a
consolidate savings and unutilized balances to fund the DAP on a quarterly basis, viz: last-ditch effort of the Executive to push agencies into actually spending their appropriations; that such
policy did not amount to an impoundment scheme, because impoundment referred to the decision of the
Executive to refuse to spend funds for political or ideological reasons; and that the withdrawal of
7.0 If the level of financial performance of some department will register below program, even with the allotments under NBC No. 541 was made pursuant to Section 38, Chapter 5, Book VI of the
availability of funds at their disposal, the targeted obligations/disbursements for each quarter will not be Administrative Code, by which the President was granted the authority to suspend or otherwise stop
met. It is important to note that these funds will lapse at the end of the fiscal year if these remain further expenditure of funds allotted to any agency whenever in his judgment the public interest so
unobligated. required.

8.0 To maximize the use of the available allotment, all unobligated balances at the end of every quarter, The assertions of the petitioners are upheld. The withdrawal and transfer of unobligated allotments and
both for continuing and current allotments shall be withdrawn and pooled to fund fast moving the pooling of unreleased appropriations were invalid for being bereft of legal support. Nonetheless, such
programs/projects. withdrawal of unobligated allotments and the retention of appropriated funds cannot be considered as
impoundment.
9.0 It may be emphasized that the allotments to be withdrawn will be based on the list of slow moving
projects to be identified by the agencies and their catch up plans to be evaluated by the DBM. According to Philippine Constitution Association v. Enriquez:159 "Impoundment refers to a refusal by the
President, for whatever reason, to spend funds made available by Congress. It is the failure to spend or
The validity period of the affected appropriations, already given the brief Lifes pan of one year, was further obligate budget authority of any type." Impoundment under the GAA is understood to mean the retention
shortened to only a quarter of a year under the DBM’s memorandum dated May 20, 2013. or deduction of appropriations. The 2011 GAA authorized impoundment only in case of unmanageable
National Government budget deficit, to wit:
The petitioners accuse the respondents of forcing the generation of savings in order to have a larger fund
available for discretionary spending. They aver that the respondents, by withdrawing unobligated Section 66. Prohibition Against Impoundment of Appropriations. No appropriations authorized under this
allotments in the middle of the fiscal year, in effect deprived funding for PAPs with existing appropriations Act shall be impounded through retention or deduction, unless in accordance with the rules and
under the GAAs.155 regulations to be issued by the DBM: PROVIDED, That all the funds appropriated for the purposes,
programs, projects and activities authorized under this Act, except those covered under the
Unprogrammed Fund, shall be released pursuant to Section 33 (3), Chapter 5, Book VI of E.O. No. 292.

75
Section 67. Unmanageable National Government Budget Deficit. Retention or deduction of appropriations The balances of continuing appropriations shall be reviewed as part of the annual budget preparation
authorized in this Act shall be effected only in cases where there is an unmanageable national process and the preparation process and the President may approve upon recommendation of the
government budget deficit. Secretary, the reversion of funds no longer needed in connection with the activities funded by said
continuing appropriations.
Unmanageable national government budget deficit as used in this section shall be construed to mean that
(i) the actual national government budget deficit has exceeded the quarterly budget deficit targets The Executive could not circumvent this provision by declaring unreleased appropriations and unobligated
consistent with the full-year target deficit as indicated in the FY 2011 Budget of allotments as savings prior to the end of the fiscal year.

Expenditures and Sources of Financing submitted by the President and approved by Congress pursuant b.3. Third Requisite – No funds from
to Section 22, Article VII of the Constitution, or (ii) there are clear economic indications of an impending savings could be transferred under
occurrence of such condition, as determined by the Development Budget Coordinating Committee and the DAP to augment deficient items
approved by the President. not provided in the GAA

The 2012 and 2013 GAAs contained similar provisions. The third requisite for a valid transfer of funds is that the purpose of the transfer should be "to augment an
item in the general appropriations law for the respective offices." The term "augment" means to enlarge or
The withdrawal of unobligated allotments under the DAP should not be regarded as impoundment increase in size, amount, or degree.160
because it entailed only the transfer of funds, not the retention or deduction of appropriations.
The GAAs for 2011, 2012 and 2013 set as a condition for augmentation that the appropriation for the PAP
Nor could Section 68 of the 2011 GAA (and the similar provisions of the 2012 and 2013 GAAs) be item to be augmented must be deficient, to wit: –
applicable. They uniformly stated:
x x x Augmentation implies the existence in this Act of a program, activity, or project with an appropriation,
Section 68. Prohibition Against Retention/Deduction of Allotment. Fund releases from appropriations which upon implementation, or subsequent evaluation of needed resources, is determined to be deficient.
provided in this Act shall be transmitted intact or in full to the office or agency concerned. No retention or In no case shall a non-existent program, activity, or project, be funded by augmentation from savings or
deduction as reserves or overhead shall be made, except as authorized by law, or upon direction of the by the use of appropriations otherwise authorized in this Act.
President of the Philippines. The COA shall ensure compliance with this provision to the extent that sub-
allotments by agencies to their subordinate offices are in conformity with the release documents issued by In other words, an appropriation for any PAP must first be determined to be deficient before it could be
the DBM. augmented from savings. Note is taken of the fact that the 2013 GAA already made this quite clear, thus:

The provision obviously pertained to the retention or deduction of allotments upon their release from the Section 52. Use of Savings. The President of the Philippines, the Senate President, the Speaker of the
DBM, which was a different matter altogether. The Court should not expand the meaning of the provision House of Representatives, the Chief Justice of the Supreme Court, the Heads of Constitutional
by applying it to the withdrawal of allotments. Commissions enjoying fiscal autonomy, and the Ombudsman are hereby authorized to use savings in
their respective appropriations to augment actual deficiencies incurred for the current year in any item of
The respondents rely on Section 38, Chapter 5, Book VI of the Administrative Code of 1987 to justify the their respective appropriations.
withdrawal of unobligated allotments. But the provision authorized only the suspension or stoppage of
further expenditures, not the withdrawal of unobligated allotments, to wit: As of 2013, a total of ₱144.4 billion worth of PAPs were implemented through the DAP.161

Section 38. Suspension of Expenditure of Appropriations.- Except as otherwise provided in the General Of this amount ₱82.5 billion were released in 2011 and ₱54.8 billion in 2012.162 Sec. Abad has reported
Appropriations Act and whenever in his judgment the public interest so requires, the President, upon that 9% of the total DAP releases were applied to the PAPs identified by the legislators.163
notice to the head of office concerned, is authorized to suspend or otherwise stop further expenditure of
funds allotted for any agency, or any other expenditure authorized in the General Appropriations Act, The petitioners disagree, however, and insist that the DAP supported the following PAPs that had not
except for personal services appropriations used for permanent officials and employees. been covered with appropriations in the respective GAAs, namely:

Moreover, the DBM did not suspend or stop further expenditures in accordance with Section 38, supra, (i) ₱1.5 billion for the Cordillera People’s Liberation Army;
but instead transferred the funds to other PAPs.
(ii) ₱1.8 billion for the Moro National Liberation Front;
It is relevant to remind at this juncture that the balances of appropriations that remained unexpended at
the end of the fiscal year were to be reverted to the General Fund.1âwphi1 This was the mandate of
Section 28, Chapter IV, Book VI of the Administrative Code, to wit: (iii) ₱700 million for assistance to Quezon Province;164

Section 28. Reversion of Unexpended Balances of Appropriations, Continuing Appropriations.- (iv) ₱50 million to ₱100 (million) each to certain senators;165
Unexpended balances of appropriations authorized in the General Appropriation Act shall revert to the
unappropriated surplus of the General Fund at the end of the fiscal year and shall not thereafter be (v) ₱10 billion for the relocation of families living along dangerous zones under the National Housing
available for expenditure except by subsequent legislative enactment: Provided, that appropriations for Authority;
capital outlays shall remain valid until fully spent or reverted: provided, further, that continuing
appropriations for current operating expenditures may be specifically recommended and approved as (vi) ₱10 billion and ₱20 billion equity infusion under the Bangko Sentral;
such in support of projects whose effective implementation calls for multi-year expenditure commitments:
provided, finally, that the President may authorize the use of savings realized by an agency during given
(vii) ₱5.4 billion landowners’ compensation under the Department of Agrarian Reform;
year to meet non-recurring expenditures in a subsequent year.

76
(viii) ₱8.6 billion for the ARMM comprehensive peace and development program; . Science 00
and Technology
(ix) ₱6.5 billion augmentation of LGU internal revenue allotments Activities

(x) ₱5 billion for crucial projects like tourism road construction under the Department of Tourism and the
Department of Public Works and Highways; 1 Central Office 1,554,238,0 1,554,238,0
. 00 00
(xi) ₱1.8 billion for the DAR-DPWH Tulay ng Pangulo;

a.
(xii) ₱1.96 billion for the DOH-DPWH rehabilitation of regional health units; and Generation
of new
(xiii) ₱4 billion for the DepEd-PPP school infrastructure projects.166 knowledge
and
In refutation, the OSG argues that a total of 116 DAP-financed PAPs were implemented, had technologi
appropriation covers, and could properly be accounted for because the funds were released following and es and
pursuant to the standard practices adopted by the DBM.167 In support of its argument, the OSG has research
submitted seven evidence packets containing memoranda, SAROs, and other pertinent documents capability
relative to the implementation and fund transfers under the DAP.168 building in
priority
areas
Upon careful review of the documents contained in the seven evidence packets, we conclude that the
identified
"savings" pooled under the DAP were allocated to PAPs that were not covered by any appropriations in
as
the pertinent GAAs.
strategic to
National
For example, the SARO issued on December 22, 2011 for the highly vaunted Disaster Risk, Exposure, Developm 537,910,00 537,910,00
Assessment and Mitigation (DREAM) project under the Department of Science and Technology (DOST) ent 0 0
covered the amount of ₱1.6 Billion,169 broken down as follows:

Aside from this transfer under the DAP to the DREAM project exceeding by almost 300% the
APPROPRIATION PARTICULARS AMOUNT appropriation by Congress for the program Generation of new knowledge and technologies and research
CODE AUTHORIZED capability building in priority areas identified as strategic to National Development, the Executive allotted
funds for personnel services and capital outlays. The Executive thereby substituted its will to that of
Congress. Worse, the Executive had not earlier proposed any amount for personnel services and capital
A.03.a.01.a Generation of new knowledge and outlays in the NEP that became the basis of the 2011 GAA.170
technologies and research capability
building in priority areas identified as It is worth stressing in this connection that the failure of the GAAs to set aside any amounts for an
strategic to National Development expense category sufficiently indicated that Congress purposely did not see fit to fund, much less
Personnel Services P 43,504,024 implement, the PAP concerned. This indication becomes clearer when even the President himself did not
Maintenance and Other Operating 1,164,517,589 recommend in the NEP to fund the PAP. The consequence was that any PAP requiring expenditure that
Expenses 391,978,387 did not receive any appropriation under the GAAs could only be a new PAP, any funding for which would
Capital Outlays P 1,600,000,000 go beyond the authority laid down by Congress in enacting the GAAs. That happened in some instances
under the DAP.

the pertinent provision of the 2011 GAA (R.A. No. 10147) showed that Congress had appropriated only
In relation to the December 22, 2011 SARO issued to the Philippine Council for Industry, Energy and
₱537,910,000 for MOOE, but nothing for personnel services and capital outlays, to wit:
Emerging Technology Research and Development (DOST-PCIEETRD)171 for Establishment of the
Advanced Failure Analysis Laboratory, which reads:
Personnel Maintenanc Capital TOTAL
Services e Outlays
APPROPRIATION PARTICULARS AMOUNT
and Other
CODE AUTHORIZED
Operating
Expenditure
s
Development, integration and coordination of the National Research
A.02.a System for Industry, Energy and Emerging Technology and Related Fields
Capital Outlays P 300,000,000
III Operations
.
the appropriation code and the particulars appearing in the SARO did not correspond to the program
a Funding 177,406,0 1,887,365,0 49,090,0 2,113,861,0 specified in the GAA, whose particulars were Research and Management Services(inclusive of the
Assistance to 00 00 00 following activities: (1) Technological and Economic Assessment for Industry, Energy and Utilities; (2)

77
Dissemination of Science and Technology Information; and (3) Management of PCIERD Information During the oral arguments on January 28, 2014, Sec. Abad admitted making some cross-border
System for Industry, Energy and Utilities. Even assuming that Development, integration and coordination augmentations, to wit:
of the National Research System for Industry, Energy and Emerging Technology and Related Fields– the
particulars stated in the SARO – could fall under the broad program description of Research and JUSTICE BERSAMIN:
Management Services– as appearing in the SARO, it would nonetheless remain a new activity by reason
of its not being specifically stated in the GAA. As such, the DBM, sans legislative authorization, could not
validly fund and implement such PAP under the DAP. Alright, the whole time that you have been Secretary of Department of Budget and Management, did the
Executive Department ever redirect any part of savings of the National Government under your control
cross border to another department?
In defending the disbursements, however, the OSG contends that the Executive enjoyed sound discretion
in implementing the budget given the generality in the language and the broad policy objectives identified
under the GAAs;172 and that the President enjoyed unlimited authority to spend the initial appropriations SECRETARY ABAD:
under his authority to declare and utilize savings,173 and in keeping with his duty to faithfully execute the
laws. Well, in the Memos that we submitted to you, such an instance, Your Honor

Although the OSG rightly contends that the Executive was authorized to spend in line with its mandate to JUSTICE BERSAMIN:
faithfully execute the laws (which included the GAAs), such authority did not translate to unfettered
discretion that allowed the President to substitute his own will for that of Congress. He was still required to Can you tell me two instances? I don’t recall having read your material.
remain faithful to the provisions of the GAAs, given that his power to spend pursuant to the GAAs was but
a delegation to him from Congress. Verily, the power to spend the public wealth resided in Congress, not
SECRETARY ABAD:
in the Executive.174 Moreover, leaving the spending power of the Executive unrestricted would threaten
to undo the principle of separation of powers.175
Well, the first instance had to do with a request from the House of Representatives. They started building
their e-library in 2010 and they had a budget for about 207 Million but they lack about 43 Million to
Congress acts as the guardian of the public treasury in faithful discharge of its power of the purse
complete its 250 Million requirements. Prior to that, the COA, in an audit observation informed the
whenever it deliberates and acts on the budget proposal submitted by the Executive.176 Its power of the
Speaker that they had to continue with that construction otherwise the whole building, as well as the
purse is touted as the very foundation of its institutional strength,177 and underpins "all other legislative
equipments therein may suffer from serious deterioration. And at that time, since the budget of the House
decisions and regulating the balance of influence between the legislative and executive branches of
of Representatives was not enough to complete 250 Million, they wrote to the President requesting for an
government."178 Such enormous power encompasses the capacity to generate money for the
augmentation of that particular item, which was granted, Your Honor. The second instance in the Memos
Government, to appropriate public funds, and to spend the money.179 Pertinently, when it exercises its
is a request from the Commission on Audit. At the time they were pushing very strongly the good
power of the purse, Congress wields control by specifying the PAPs for which public money should be
governance programs of the government and therefore, part of that is a requirement to conduct audits as
spent.
well as review financial reports of many agencies. And in the performance of that function, the
Commission on Audit needed information technology equipment as well as hire consultants and litigators
It is the President who proposes the budget but it is Congress that has the final say on matters of to help them with their audit work and for that they requested funds from the Executive and the President
appropriations.180 For this purpose, appropriation involves two governing principles, namely: (1) "a saw that it was important for the Commission to be provided with those IT equipments and litigators and
Principle of the Public Fisc, asserting that all monies received from whatever source by any part of the consultants and the request was granted, Your Honor.
government are public funds;" and (2) "a Principle of Appropriations Control, prohibiting expenditure of
any public money without legislative authorization."181 To conform with the governing principles, the
JUSTICE BERSAMIN:
Executive cannot circumvent the prohibition by Congress of an expenditure for a PAP by resorting to
either public or private funds.182 Nor could the Executive transfer appropriated funds resulting in an
increase in the budget for one PAP, for by so doing the appropriation for another PAP is necessarily These cross border examples, cross border augmentations were not supported by appropriations…
decreased. The terms of both appropriations will thereby be violated.
SECRETARY ABAD:
b.4 Third Requisite – Cross-border
augmentations from savings were They were, we were augmenting existing items within their… (interrupted)
prohibited by the Constitution
JUSTICE BERSAMIN:
By providing that the President, the President of the Senate, the Speaker of the House of
Representatives, the Chief Justice of the Supreme Court, and the Heads of the Constitutional No, appropriations before you augmented because this is a cross border and the tenor or text of the
Commissions may be authorized to augment any item in the GAA "for their respective offices," Section Constitution is quite clear as far as I am concerned. It says here, "The power to augment may only be
25(5), supra, has delineated borders between their offices, such that funds appropriated for one office are made to increase any item in the General Appropriations Law for their respective offices." Did you not feel
prohibited from crossing over to another office even in the guise of augmentation of a deficient item or constricted by this provision?
items. Thus, we call such transfers of funds cross-border transfers or cross-border augmentations.

SECRETARY ABAD:
To be sure, the phrase "respective offices" used in Section 25(5), supra, refers to the entire Executive,
with respect to the President; the Senate, with respect to the Senate President; the House of
Representatives, with respect to the Speaker; the Judiciary, with respect to the Chief Justice; the Well, as the Constitution provides, the prohibition we felt was on the transfer of appropriations, Your
Constitutional Commissions, with respect to their respective Chairpersons. Honor. What we thought we did was to transfer savings which was needed by the Commission to address
deficiency in an existing item in both the Commission as well as in the House of Representatives; that’s
how we saw…(interrupted)
Did any cross-border transfers or augmentations transpire?

78
JUSTICE BERSAMIN: HONORABLE MENDOZA:

So your position as Secretary of Budget is that you could do that? The cross-border transfers, if Your Honors please, is not an application of the DAP. What were these
cross-border transfers? They are transfers of savings as defined in the various General Appropriations
SECRETARY ABAD: Act. So, that makes it similar to the DAP, the use of savings. There was a cross-border which appears to
be in violation of Section 25, paragraph 5 of Article VI, in the sense that the border was crossed. But
never has it been claimed that the purpose was to augment a deficient item in another department of the
In an extreme instances because…(interrupted) government or agency of the government. The cross-border transfers, if Your Honors please, were in the
nature of [aid] rather than augmentations. Here is a government entity separate and independent from the
JUSTICE BERSAMIN: Executive Department solely in need of public funds. The President is there 24 hours a day, 7 days a
week. He’s in charge of the whole operation although six or seven heads of government offices are given
No, no, in all instances, extreme or not extreme, you could do that, that’s your feeling. the power to augment. Only the President stationed there and in effect in-charge and has the
responsibility for the failure of any part of the government. You have election, for one reason or another,
the money is not enough to hold election. There would be chaos if no money is given as an aid, not to
SECRETARY ABAD:
augment, but as an aid to a department like COA. The President is responsible in a way that the other
heads, given the power to augment, are not. So, he cannot very well allow this, if Your Honor please.189
Well, in that particular situation when the request was made by the Commission and the House of
Representatives, we felt that we needed to respond because we felt…(interrupted).183
JUSTICE LEONEN:

The records show, indeed, that funds amounting to ₱143,700,000.00 and ₱250,000,000.00 were
May I move to another point, maybe just briefly. I am curious that the position now, I think, of government
transferred under the DAP respectively to the COA184 and the House of Representatives.185 Those
is that some transfers of savings is now considered to be, if I’m not mistaken, aid not augmentation. Am I
transfers of funds, which constituted cross-border augmentations for being from the Executive to the COA
correct in my hearing of your argument?
and the House of Representatives, are graphed as follows:186

HONORABLE MENDOZA:

That’s our submission, if Your Honor, please.


AMOUNT
(In thousand pesos) JUSTICE LEONEN:
DATE
PURPOSE
RELEASED
May I know, Justice, where can we situate this in the text of the Constitution? Where do we actually derive
Reserve Releases
the concepts that transfers of appropriation from one branch to the other or what happened in DAP can be
Imposed
considered a said? What particular text in the Constitution can we situate this?

on
IT Infrastructure Program and hiring of additional litigation experts 11/11/11 HONORABLE MENDOZA:
143,700

There is no particular provision or statutory provision for that matter, if Your Honor please. It is drawn from
the fact that the Executive is the executive in-charge of the success of the government.
Completion of the construction of the Legislative Library and Archives 07/23/12 207,034 250,000
Building/Congressional e-library (Savings of HOR) JUSTICE LEONEN:
ves
So, the residual powers labelled in Marcos v. Manglapus would be the basis for this theory of the
The respondents further stated in their memorandum that the President "made available" to the government?
"Commission on Elections the savings of his department upon [its] request for funds…"187 This was
another instance of a cross-border augmentation. HONORABLE MENDOZA:

The respondents justified all the cross-border transfers thusly: Yes, if Your Honor, please.

99. The Constitution does not prevent the President from transferring savings of his department to another JUSTICE LEONEN:
department upon the latter’s request, provided it is the recipient department that uses such funds to
augment its own appropriation. In such a case, the President merely gives the other department access to A while ago, Justice Carpio mentioned that the remedy is might be to go to Congress. That there are
public funds but he cannot dictate how they shall be applied by that department whose fiscal autonomy is opportunities and there have been opportunities of the President to actually go to Congress and ask for
guaranteed by the Constitution.188 supplemental budgets?

In the oral arguments held on February 18, 2014, Justice Vicente V. Mendoza, representing Congress, HONORABLE MENDOZA:
announced a different characterization of the cross-border transfers of funds as in the nature of "aid"
instead of "augmentation," viz:
If there is time to do that, I would say yes.

79
JUSTICE LEONEN: the revenue collections should exceed the revenue targets must still be complied with in order to justify
the release of the unprogrammed funds.
So, the theory of aid rather than augmentation applies in extra-ordinary situation?
The view that there were only two instances when the unprogrammed funds could be released was
HONORABLE MENDOZA: bolstered by the following texts of the Special Provisions of the 2011 and 2012 GAAs, to wit:

Very extra-ordinary situations. 2011 GAA

JUSTICE LEONEN: 1. Release of Fund. The amounts authorized herein shall be released only when the revenue collections
exceed the original revenue targets submitted by the President of the Philippines to Congress pursuant to
Section 22, Article VII of the Constitution, including savings generated from programmed appropriations
But Counsel, this would be new doctrine, in case? for the year: PROVIDED, That collections arising from sources not considered in the aforesaid original
revenue targets may be used to cover releases from appropriations in this Fund: PROVIDED, FURTHER,
HONORABLE MENDOZA: That in case of newly approved loans for foreign-assisted projects, the existence of a perfected loan
agreement for the purpose shall be sufficient basis for the issuance of a SARO covering the loan
Yes, if Your Honor please.190 proceeds: PROVIDED, FURTHERMORE, That if there are savings generated from the programmed
appropriations for the first two quarters of the year, the DBM may, subject to the approval of the President,
release the pertinent appropriations under the Unprogrammed Fund corresponding to only fifty percent
Regardless of the variant characterizations of the cross-border transfers of funds, the plain text of Section
(50%) of the said savings net of revenue shortfall: PROVIDED, FINALLY, That the release of the balance
25(5), supra, disallowing cross border transfers was disobeyed. Cross-border transfers, whether as
of the total savings from programmed appropriations for the year shall be subject to fiscal programming
augmentation, or as aid, were prohibited under Section 25(5), supra.
and approval of the President.

4.
2012 GAA
Sourcing the DAP from unprogrammed
funds despite the original revenue targets
not having been exceeded was invalid 1. Release of the Fund. The amounts authorized herein shall be released only when the revenue
collections exceed the original revenue targets submitted by the President of the Philippines to Congress
pursuant to Section 22, Article VII of the Constitution: PROVIDED, That collections arising from sources
Funding under the DAP were also sourced from unprogrammed funds provided in the GAAs for 2011,
not considered in the aforesaid original revenue targets may be used to cover releases from
2012,and 2013. The respondents stress, however, that the unprogrammed funds were not brought under
appropriations in this Fund: PROVIDED, FURTHER, That in case of newly approved loans for foreign-
the DAP as savings, but as separate sources of funds; and that, consequently, the release and use of
assisted projects, the existence of a perfected loan agreement for the purpose shall be sufficient basis for
unprogrammed funds were not subject to the restrictions under Section 25(5), supra.
the issuance of a SARO covering the loan proceeds.

The documents contained in the Evidence Packets by the OSG have confirmed that the unprogrammed
As can be noted, the provisos in both provisions to the effect that "collections arising from sources not
funds were treated as separate sources of funds. Even so, the release and use of the unprogrammed
considered in the aforesaid original revenue targets may be used to cover releases from appropriations in
funds were still subject to restrictions, for, to start with, the GAAs precisely specified the instances when
this Fund" gave the authority to use such additional revenues for appropriations funded from the
the unprogrammed funds could be released and the purposes for which they could be used.
unprogrammed funds. They did not at all waive compliance with the basic requirement that revenue
collections must still exceed the original revenue targets.
The petitioners point out that a condition for the release of the unprogrammed funds was that the revenue
collections must exceed revenue targets; and that the release of the unprogrammed funds was illegal
In contrast, the texts of the provisos with regard to additional revenues generated from newly-approved
because such condition was not met.191
foreign loans were clear to the effect that the perfected loan agreement would be in itself "sufficient basis"
for the issuance of a SARO to release the funds but only to the extent of the amount of the loan. In such
The respondents disagree, holding that the release and use of the unprogrammed funds under the DAP instance, the revenue collections need not exceed the revenue targets to warrant the release of the loan
were in accordance with the pertinent provisions of the GAAs. In particular, the DBM avers that the proceeds, and the mere perfection of the loan agreement would suffice.
unprogrammed funds could be availed of when any of the following three instances occur, to wit: (1) the
revenue collections exceeded the original revenue targets proposed in the BESFs submitted by the
It can be inferred from the foregoing that under these provisions of the GAAs the additional revenues from
President to Congress; (2) new revenues were collected or realized from sources not originally considered
sources not considered in the BESFs must be taken into account in determining if the revenue collections
in the BESFs; or(3) newly-approved loans for foreign assisted projects were secured, or when conditions
exceeded the revenue targets. The text of the relevant provision of the 2013 GAA, which was substantially
were triggered for other sources of funds, such as perfected loan agreements for foreign-assisted
similar to those of the GAAs for 2011 and 2012, already made this explicit, thus:
projects.192 This view of the DBM was adopted by all the respondents in their Consolidated
Comment.193
1. Release of the Fund. The amounts authorized herein shall be released only when the revenue
collections exceed the original revenue targets submitted by the President of the Philippines to Congress
The BESFs for 2011, 2012 and 2013 uniformly defined "unprogrammed appropriations" as appropriations
pursuant to Section 22, Article VII of the Constitution, including collections arising from sources not
that provided standby authority to incur additional agency obligations for priority PAPs when revenue
considered in the aforesaid original revenue target, as certified by the BTr: PROVIDED, That in case of
collections exceeded targets, and when additional foreign funds are generated.194 Contrary to the DBM’s
newly approved loans for foreign-assisted projects, the existence of a perfected loan agreement for the
averment that there were three instances when unprogrammed funds could be released, the BESFs
purpose shall be sufficient basis for the issuance of a SARO covering the loan proceeds.
envisioned only two instances. The third mentioned by the DBM – the collection of new revenues from
sources not originally considered in the BESFs – was not included. This meant that the collection of
additional revenues from new sources did not warrant the release of the unprogrammed funds. Hence,
even if the revenues not considered in the BESFs were collected or generated, the basic condition that

80
Consequently, that there were additional revenues from sources not considered in the revenue target To justify the release of the unprogrammed funds for 2011, the OSG presented the certification dated
would not be enough. The total revenue collections must still exceed the original revenue targets to justify March 4, 2011 issued by DOF Undersecretary Gil S. Beltran, as follows:
the release of the unprogrammed funds (other than those from newly-approved foreign loans).
This is to certify that under the Budget for Expenditures and Sources of Financing for 2011, the
The present controversy on the unprogrammed funds was rooted in the correct interpretation of the programmed income from dividends from shares of stock in government-owned and controlled
phrase "revenue collections should exceed the original revenue targets." The petitioners take the phrase corporations is 5.5 billion.
to mean that the total revenue collections must exceed the total revenue target stated in the BESF, but
the respondents understand the phrase to refer only to the collections for each source of revenue as This is to certify further that based on the records of the Bureau of Treasury, the National Government has
enumerated in the BESF, with the condition being deemed complied with once the revenue collections recorded dividend income amounting to ₱23.8 billion as of 31 January 2011.196
from a particular source already exceeded the stated target.
For 2012, the OSG submitted the certification dated April 26, 2012 issued by National Treasurer Roberto
The BESF provided for the following sources of revenue, with the corresponding revenue target stated for B. Tan, viz:
each source of revenue, to wit:
This is to certify that the actual dividend collections remitted to the National Government for the period
TAX REVENUES January to March 2012 amounted to ₱19.419 billion compared to the full year program of ₱5.5 billion for
2012.197
Taxes on Net Income and Profits
Taxes on Property And, finally, for 2013, the OSG presented the certification dated July 3, 2013 issued by National Treasurer
Taxes on Domestic Goods and Services Rosalia V. De Leon, to wit:

General Sales, Turnover or VAT This is to certify that the actual dividend collections remitted to the National Government for the period
Selected Excises on Goods January to May 2013 amounted to ₱12.438 billion compared to the full year program of ₱10.0198 billion
for 2013.
Selected Taxes on Services
Taxes on the Use of Goods or Property or Permission to Perform Activities Moreover, the National Government accounted for the sale of the right to build and operate the NAIA
Other Taxes expressway amounting to ₱11.0 billion in June 2013.199
Taxes on International Trade and Transactions
The certifications reflected that by collecting dividends amounting to ₱23.8 billion in 2011, ₱19.419 billion
NON-TAX REVENUES in 2012, and ₱12.438 billion in 2013 the BTr had exceeded only the ₱5.5 billion in target revenues in the
form of dividends from stocks in each of 2011 and 2012, and only the ₱10 billion in target revenues in the
Fees and Charges form of dividends from stocks in 2013.
BTR Income
However, the requirement that revenue collections exceed the original revenue targets was to be
Government Services construed in light of the purpose for which the unprogrammed funds were incorporated in the GAAs as
Interest on NG Deposits standby appropriations to support additional expenditures for certain priority PAPs should the revenue
Interest on Advances to Government Corporations collections exceed the resource targets assumed in the budget or when additional foreign project loan
Income from Investments proceeds were realized. The unprogrammed funds were included in the GAAs to provide ready cover so
as not to delay the implementation of the PAPs should new or additional revenue sources be realized
Interest on Bond Holdings during the year.200 Given the tenor of the certifications, the unprogrammed funds were thus not yet
supported by the corresponding resources.201
Guarantee Fee
Gain on Foreign Exchange The revenue targets stated in the BESF were intended to address the funding requirements of the
NG Income Collected by BTr proposed programmed appropriations. In contrast, the unprogrammed funds, as standby appropriations,
were to be released only when there were revenues in excess of what the programmed appropriations
required. As such, the revenue targets should be considered as a whole, not individually; otherwise, we
Dividends on Stocks would be dealing with artificial revenue surpluses. The requirement that revenue collections must exceed
NG Share from Airport Terminal Fee revenue target should be understood to mean that the revenue collections must exceed the total of the
NG Share from PAGCOR Income revenue targets stated in the BESF. Moreover, to release the unprogrammed funds simply because there
NG Share from MIAA Profit was an excess revenue as to one source of revenue would be an unsound fiscal management measure
because it would disregard the budget plan and foster budget deficits, in contravention of the
Privatization Government’s surplus budget policy.202
Foreign Grants
We cannot, therefore, subscribe to the respondents’ view.
Thus, when the Court required the respondents to submit a certification from the Bureau of Treasury (BTr)
to the effect that the revenue collections had exceeded the original revenue targets,195 they complied by 5.
submitting certifications from the BTr and Department of Finance (DOF) pertaining to only one identified Equal protection, checks and balances,
source of revenue – the dividends from the shares of stock held by the Government in government-owned and public accountability challenges
and controlled corporations.

81
The DAP is further challenged as violative of the Equal Protection Clause, the system of checks and constitutional system. Our earlier expositions on the DAP and its implementing issuances infringing the
balances, and the principle of public accountability. doctrine of separation of powers effectively addressed this particular concern.

With respect to the challenge against the DAP under the Equal Protection Clause,203 Luna argues that Anent the principle of public accountability being transgressed because the adoption and implementation
the implementation of the DAP was "unfair as it [was] selective" because the funds released under the of the DAP constituted an assumption by the Executive of Congress’ power of appropriation, we have
DAP was not made available to all the legislators, with some of them refusing to avail themselves of the already held that the DAP and its implementing issuances were policies and acts that the Executive could
DAP funds, and others being unaware of the availability of such funds. Thus, the DAP practised "undue properly adopt and do in the execution of the GAAs to the extent that they sought to implement strategies
favoritism" in favor of select legislators in contravention of the Equal Protection Clause. to ramp up or accelerate the economy of the country.

Similarly, COURAGE contends that the DAP violated the Equal Protection Clause because no reasonable 6.
classification was used in distributing the funds under the DAP; and that the Senators who supposedly Doctrine of operative fact was applicable
availed themselves of said funds were differently treated as to the amounts they respectively received.
After declaring the DAP and its implementing issuances constitutionally infirm, we must now deal with the
Anent the petitioners’ theory that the DAP violated the system of checks and balances, Luna submits that consequences of the declaration.
the grant of the funds under the DAP to some legislators forced their silence about the issues and
anomalies surrounding the DAP. Meanwhile, Belgica stresses that the DAP, by allowing the legislators to Article 7 of the Civil Code provides:
identify PAPs, authorized them to take part in the implementation and execution of the GAAs, a function
that exclusively belonged to the Executive; that such situation constituted undue and unjustified legislative
encroachment in the functions of the Executive; and that the President arrogated unto himself the power Article 7. Laws are repealed only by subsequent ones, and their violation or non-observance shall not be
of appropriation vested in Congress because NBC No. 541 authorized the use of the funds under the DAP excused by disuse, or custom or practice to the contrary.
for PAPs not considered in the 2012 budget.
When the courts declared a law to be inconsistent with the Constitution, the former shall be void and the
Finally, the petitioners insist that the DAP was repugnant to the principle of public accountability enshrined latter shall govern.
in the Constitution,204 because the legislators relinquished the power of appropriation to the Executive,
and exhibited a reluctance to inquire into the legality of the DAP. Administrative or executive acts, orders and regulations shall be valid only when they are not contrary to
the laws or the Constitution.
The OSG counters the challenges, stating that the supposed discrimination in the release of funds under
the DAP could be raised only by the affected Members of Congress themselves, and if the challenge A legislative or executive act that is declared void for being unconstitutional cannot give rise to any right or
based on the violation of the Equal Protection Clause was really against the constitutionality of the DAP, obligation.206 However, the generality of the rule makes us ponder whether rigidly applying the rule may
the arguments of the petitioners should be directed to the entitlement of the legislators to the funds, not to at times be impracticable or wasteful. Should we not recognize the need to except from the rigid
the proposition that all of the legislators should have been given such entitlement. application of the rule the instances in which the void law or executive act produced an almost irreversible
result?
The challenge based on the contravention of the Equal Protection Clause, which focuses on the release
of funds under the DAP to legislators, lacks factual and legal basis. The allegations about Senators and The need is answered by the doctrine of operative fact. The doctrine, definitely not a novel one, has been
Congressmen being unaware of the existence and implementation of the DAP, and about some of them exhaustively explained in De Agbayani v. Philippine National Bank:207
having refused to accept such funds were unsupported with relevant data. Also, the claim that the
Executive discriminated against some legislators on the ground alone of their receiving less than the The decision now on appeal reflects the orthodox view that an unconstitutional act, for that matter an
others could not of itself warrant a finding of contravention of the Equal Protection Clause. The denial of executive order or a municipal ordinance likewise suffering from that infirmity, cannot be the source of any
equal protection of any law should be an issue to be raised only by parties who supposedly suffer it, and, legal rights or duties. Nor can it justify any official act taken under it. Its repugnancy to the fundamental
in these cases, such parties would be the few legislators claimed to have been discriminated against in law once judicially declared results in its being to all intents and purposes a mere scrap of paper. As the
the releases of funds under the DAP. The reason for the requirement is that only such affected legislators new Civil Code puts it: ‘When the courts declare a law to be inconsistent with the Constitution, the former
could properly and fully bring to the fore when and how the denial of equal protection occurred, and shall be void and the latter shall govern.’ Administrative or executive acts, orders and regulations shall be
explain why there was a denial in their situation. The requirement was not met here. Consequently, the valid only when they are not contrary to the laws of the Constitution. It is understandable why it should be
Court was not put in the position to determine if there was a denial of equal protection. To have the Court so, the Constitution being supreme and paramount. Any legislative or executive act contrary to its terms
do so despite the inadequacy of the showing of factual and legal support would be to compel it to cannot survive.
speculate, and the outcome would not do justice to those for whose supposed benefit the claim of denial
of equal protection has been made.
Such a view has support in logic and possesses the merit of simplicity. It may not however be sufficiently
realistic. It does not admit of doubt that prior to the declaration of nullity such challenged legislative or
The argument that the release of funds under the DAP effectively stayed the hands of the legislators from executive act must have been in force and had to be complied with. This is so as until after the judiciary,
conducting congressional inquiries into the legality and propriety of the DAP is speculative. That in an appropriate case, declares its invalidity, it is entitled to obedience and respect. Parties may have
deficiency eliminated any need to consider and resolve the argument, for it is fundamental that acted under it and may have changed their positions. What could be more fitting than that in a subsequent
speculation would not support any proper judicial determination of an issue simply because nothing litigation regard be had to what has been done while such legislative or executive act was in operation
concrete can thereby be gained. In order to sustain their constitutional challenges against official acts of and presumed to be valid in all respects. It is now accepted as a doctrine that prior to its being nullified, its
the Government, the petitioners must discharge the basic burden of proving that the constitutional existence as a fact must be reckoned with. This is merely to reflect awareness that precisely because the
infirmities actually existed.205 Simply put, guesswork and speculation cannot overcome the presumption judiciary is the governmental organ which has the final say on whether or not a legislative or executive
of the constitutionality of the assailed executive act. measure is valid, a period of time may have elapsed before it can exercise the power of judicial review
that may lead to a declaration of nullity. It would be to deprive the law of its quality of fairness and justice
We do not need to discuss whether or not the DAP and its implementation through the various circulars then, if there be no recognition of what had transpired prior to such adjudication.
and memoranda of the DBM transgressed the system of checks and balances in place in our

82
In the language of an American Supreme Court decision: ‘The actual existence of a statute, prior to such Evidently, the operative fact doctrine is not confined to statutes and rules and regulations issued by the
a determination [of unconstitutionality], is an operative fact and may have consequences which cannot executive department that are accorded the same status as that of a statute or those which are quasi-
justly be ignored. The past cannot always be erased by a new judicial declaration. The effect of the legislative in nature.
subsequent ruling as to invalidity may have to be considered in various aspects, with respect to particular
relations, individual and corporate, and particular conduct, private and official.’" Even assuming that De Agbayani initially applied the operative fact doctrine only to executive issuances
like orders and rules and regulations, said principle can nonetheless be applied, by analogy, to decisions
The doctrine of operative fact recognizes the existence of the law or executive act prior to the made by the President or the agencies under the executive department. This doctrine, in the interest of
determination of its unconstitutionality as an operative fact that produced consequences that cannot justice and equity, can be applied liberally and in a broad sense to encompass said decisions of the
always be erased, ignored or disregarded. In short, it nullifies the void law or executive act but sustains its executive branch. In keeping with the demands of equity, the Court can apply the operative fact doctrine
effects. It provides an exception to the general rule that a void or unconstitutional law produces no to acts and consequences that resulted from the reliance not only on a law or executive act which is
effect.208 But its use must be subjected to great scrutiny and circumspection, and it cannot be invoked to quasi-legislative in nature but also on decisions or orders of the executive branch which were later
validate an unconstitutional law or executive act, but is resorted to only as a matter of equity and fair nullified. This Court is not unmindful that such acts and consequences must be recognized in the higher
play.209 It applies only to cases where extraordinary circumstances exist, and only when the interest of justice, equity and fairness.
extraordinary circumstances have met the stringent conditions that will permit its application.
Significantly, a decision made by the President or the administrative agencies has to be complied with
We find the doctrine of operative fact applicable to the adoption and implementation of the DAP. Its because it has the force and effect of law, springing from the powers of the President under the
application to the DAP proceeds from equity and fair play. The consequences resulting from the DAP and Constitution and existing laws. Prior to the nullification or recall of said decision, it may have produced
its related issuances could not be ignored or could no longer be undone. acts and consequences in conformity to and in reliance of said decision, which must be respected. It is on
this score that the operative fact doctrine should be applied to acts and consequences that resulted from
To be clear, the doctrine of operative fact extends to a void or unconstitutional executive act. The term the implementation of the PARC Resolution approving the SDP of HLI. (Bold underscoring supplied for
executive act is broad enough to include any and all acts of the Executive, including those that are quasi emphasis)
legislative and quasi-judicial in nature. The Court held so in Hacienda Luisita, Inc. v. Presidential Agrarian
Reform Council:210 In Commissioner of Internal Revenue v. San Roque Power Corporation,211 the Court likewise declared
that "for the operative fact doctrine to apply, there must be a ‘legislative or executive measure,’ meaning a
Nonetheless, the minority is of the persistent view that the applicability of the operative fact doctrine law or executive issuance." Thus, the Court opined there that the operative fact doctrine did not apply to a
should be limited to statutes and rules and regulations issued by the executive department that are mere administrative practice of the Bureau of Internal Revenue, viz:
accorded the same status as that of a statute or those which are quasi-legislative in nature. Thus, the
minority concludes that the phrase ‘executive act’ used in the case of De Agbayani v. Philippine National Under Section 246, taxpayers may rely upon a rule or ruling issued by the Commissioner from the time
Bank refers only to acts, orders, and rules and regulations that have the force and effect of law. The the rule or ruling is issued up to its reversal by the Commissioner or this Court. The reversal is not given
minority also made mention of the Concurring Opinion of Justice Enrique Fernando in Municipality of retroactive effect. This, in essence, is the doctrine of operative fact. There must, however, be a rule or
Malabang v. Benito, where it was supposedly made explicit that the operative fact doctrine applies to ruling issued by the Commissioner that is relied upon by the taxpayer in good faith. A mere administrative
executive acts, which are ultimately quasi-legislative in nature. practice, not formalized into a rule or ruling, will not suffice because such a mere administrative practice
may not be uniformly and consistently applied. An administrative practice, if not formalized as a rule or
We disagree. For one, neither the De Agbayani case nor the Municipality of Malabang case elaborates ruling, will not be known to the general public and can be availed of only by those with informal contacts
what ‘executive act’ mean. Moreover, while orders, rules and regulations issued by the President or the with the government agency.
executive branch have fixed definitions and meaning in the Administrative Code and jurisprudence, the
phrase ‘executive act’ does not have such specific definition under existing laws. It should be noted that in It is clear from the foregoing that the adoption and the implementation of the DAP and its related
the cases cited by the minority, nowhere can it be found that the term ‘executive act’ is confined to the issuances were executive acts.1avvphi1 The DAP itself, as a policy, transcended a merely administrative
foregoing. Contrarily, the term ‘executive act’ is broad enough to encompass decisions of administrative practice especially after the Executive, through the DBM, implemented it by issuing various memoranda
bodies and agencies under the executive department which are subsequently revoked by the agency in and circulars. The pooling of savings pursuant to the DAP from the allotments made available to the
question or nullified by the Court. different agencies and departments was consistently applied throughout the entire Executive. With the
Executive, through the DBM, being in charge of the third phase of the budget cycle – the budget execution
A case in point is the concurrent appointment of Magdangal B. Elma (Elma) as Chairman of the phase, the President could legitimately adopt a policy like the DAP by virtue of his primary responsibility
Presidential Commission on Good Government (PCGG) and as Chief Presidential Legal Counsel (CPLC) as the Chief Executive of directing the national economy towards growth and development. This is simply
which was declared unconstitutional by this Court in Public Interest Center, Inc. v. Elma. In said case, this because savings could and should be determined only during the budget execution phase.
Court ruled that the concurrent appointment of Elma to these offices is in violation of Section 7, par. 2,
Article IX-B of the 1987 Constitution, since these are incompatible offices. Notably, the appointment of As already mentioned, the implementation of the DAP resulted into the use of savings pooled by the
Elma as Chairman of the PCGG and as CPLC is, without a question, an executive act. Prior to the Executive to finance the PAPs that were not covered in the GAA, or that did not have proper appropriation
declaration of unconstitutionality of the said executive act, certain acts or transactions were made in good covers, as well as to augment items pertaining to other departments of the Government in clear violation
faith and in reliance of the appointment of Elma which cannot just be set aside or invalidated by its of the Constitution. To declare the implementation of the DAP unconstitutional without recognizing that its
subsequent invalidation. prior implementation constituted an operative fact that produced consequences in the real as well as
juristic worlds of the Government and the Nation is to be impractical and unfair. Unless the doctrine is held
In Tan v. Barrios, this Court, in applying the operative fact doctrine, held that despite the invalidity of the to apply, the Executive as the disburser and the offices under it and elsewhere as the recipients could be
jurisdiction of the military courts over civilians, certain operative facts must be acknowledged to have required to undo everything that they had implemented in good faith under the DAP. That scenario would
existed so as not to trample upon the rights of the accused therein. Relevant thereto, in Olaguer v. Military be enormously burdensome for the Government. Equity alleviates such burden.
Commission No. 34, it was ruled that ‘military tribunals pertain to the Executive Department of the
Government and are simply instrumentalities of the executive power, provided by the legislature for the The other side of the coin is that it has been adequately shown as to be beyond debate that the
President as Commander-in-Chief to aid him in properly commanding the army and navy and enforcing implementation of the DAP yielded undeniably positive results that enhanced the economic welfare of the
discipline therein, and utilized under his orders or those of his authorized military representatives.’ country. To count the positive results may be impossible, but the visible ones, like public infrastructure,

83
could easily include roads, bridges, homes for the homeless, hospitals, classrooms and the like. Not to AUGUSTO L. SY JUCO JR., Ph.D., Petitioner,
apply the doctrine of operative fact to the DAP could literally cause the physical undoing of such worthy vs.
results by destruction, and would result in most undesirable wastefulness. FLORENCIO B. ABAD, IN HIS CAPACITY AS THE SECRETARY OF DEPARTMENT OF BUDGET
AND MANAGEMENT; AND HON. FRANKLIN MAGTUNAO DRILON, IN HIS CAP A CITY AS THE
Nonetheless, as Justice Brion has pointed out during the deliberations, the doctrine of operative fact does SENATE PRESIDENT OF THE PHILIPPINES, Respondents.
not always apply, and is not always the consequence of every declaration of constitutional invalidity. It can
be invoked only in situations where the nullification of the effects of what used to be a valid law would x-----------------------x
result in inequity and injustice;212 but where no such result would ensue, the general rule that an
unconstitutional law is totally ineffective should apply. G.R. No. 209136

In that context, as Justice Brion has clarified, the doctrine of operative fact can apply only to the PAPs that MANUELITO R. LUNA, Petitioner,
can no longer be undone, and whose beneficiaries relied in good faith on the validity of the DAP, but vs.
cannot apply to the authors, proponents and implementors of the DAP, unless there are concrete findings SECRETARY FLORENCIO ABAD, IN HIS OFFICIAL CAPACITY AS HEAD OF THE DEPARTMENT OF
of good faith in their favor by the proper tribunals determining their criminal, civil, administrative and other BUDGET AND MANAGEMENT; AND EXECUTIVE SECRETARY PAQUITO OCHOA, IN HIS OFFICIAL
liabilities. CAPACITY AS ALTER EGO OF THE PRESIDENT, Respondents.

WHEREFORE, the Court PARTIALLY GRANTS the petitions for certiorari and prohibition; and x-----------------------x
DECLARES the following acts and practices under the Disbursement Acceleration Program, National
Budget Circular No. 541 and related executive issuances UNCONSTITUTIONAL for being in violation of
Section 25(5), Article VI of the 1987 Constitution and the doctrine of separation of powers, namely: G.R. No. 209155

(a) The withdrawal of unobligated allotments from the implementing agencies, and the declaration of the ATTY. JOSE MALV AR VILLEGAS, JR., Petitioner,
withdrawn unobligated allotments and unreleased appropriations as savings prior to the end of the fiscal vs.
year and without complying with the statutory definition of savings contained in the General Appropriations THE HONORABLE EXECUTIVE SECRETARY PAQUITO N. OCHOA, JR.; AND THE SECRETARY OF
Acts; BUDGET AND MANAGEMENT FLORENCIO B. ABAD, Respondents.

(b) The cross-border transfers of the savings of the Executive to augment the appropriations of other x-----------------------x
offices outside the Executive; and
G.R. No. 209164
(c) The funding of projects, activities and programs that were not covered by any appropriation in the
General Appropriations Act. PHILIPPINE CONSTITUTION ASSOCIATION (PHILCONSA), REPRESENTED BY DEAN FROILAN M.
BACUNGAN, BENJAMIN E. DIOKNO AND LEONOR M. BRIONES, Petitioners,
The Court further DECLARES VOID the use of unprogrammed funds despite the absence of a vs.
certification by the National Treasurer that the revenue collections exceeded the revenue targets for non- DEPARTMENT OF BUDGET AND MANAGEMENT AND/OR HON. FLORENCIO B.
compliance with the conditions provided in the relevant General Appropriations Acts. ABAD, Respondents.

SO ORDERED x-----------------------x

G.R. No. 209287               July 1, 2014 G.R. No. 209260

MARIA CAROLINA P. ARAULLO, CHAIRPERSON, BAGONG ALYANSANG MAKABAYAN; JUDY M. INTEGRATED BAR OF THE PHILIPPINES (IBP), Petitioner,
TAGUIWALO, PROFESSOR, UNIVERSITY OF THE PHILIPPINES DILIMAN, CO-CHAIRPERSON, vs.
PAGBABAGO; HENRI KAHN, CONCERNED CITIZENS MOVEMENT; REP. LUZ ILAGAN, GABRIELA SECRETARY FLORENCIO B. ABAD OF THE DEPARTMENT OF BUDGET AND MANAGEMENT
WOMEN'S PARTY REPRESENTATIVE; REP. CARLOS ISAGANI ZARATE, BAY AN MUNA PARTY- (DBM), Respondent.
LIST REPRESENTATIVE; RENATO M. REYES, JR., SECRETARY GENERAL OF BAYAN; MANUEL
K. DAYRIT, CHAIRMAN, ANG KAPATIRAN PARTY; VENCER MARI E. CRISOSTOMO, x-----------------------x
CHAIRPERSON, ANAKBAYAN; VICTOR VILLANUEVA, CONVENOR, YOUTH ACT NOW, Petitioners,
vs. G.R. No. 209442
BENIGNO SIMEON C. AQUINO III, PRESIDENT OF THE REPUBLIC OF THE PHILIPPINES; PAQUITO
N. OCHOA, JR., EXECUTIVE SECRETARY; AND FLORENCIO B. ABAD, SECRETARY OF THE
GRECO ANTONIOUS BEDA B. BELGICA; BISHOP REUBEN MABANTE AND REV. JOSE L.
DEPARTMENT OF BUDGET AND MANAGEMENT, Respondents.
GONZALEZ, Petitioners,
vs.
x-----------------------x PRESIDENT BENIGNO SIMEON C. AQUINO III, THE SENATE OF THE PHILIPPINES, REPRESENTED
BY SENATE PRESIDENT FRANKLIN M. DRILON; THE HOUSE OF REPRESENTATIVES,
G.R. No. 209135 REPRESENTED BY SPEAKER FELICIANO BELMONTE, JR.; THE EXECUTIVE OFFICE,
REPRESENTED BY EXECUTIVE SECRETARY PAQUITO N. OCHOA, JR.; THE DEPARTMENT OF
BUDGET AND MANAGEMENT, REPRESENTED BY SECRETARY FLORENCIO ABAD; THE

84
DEPARTMENT OF FINANCE, REPRESENTED BY SECRETARY CESAR V. PURISIMA; AND THE On September 25, 2013, Sen. Jinggoy Ejercito Estrada delivered a privilege speech in the Senate of the
BUREAU OF TREASURY, REPRESENTED BY ROSALIA V. DE LEON, Respondents. Philippines to reveal that some Senators, including himself, had been allotted an additional ₱50 Million
each as "incentive" for voting in favor of the impeachment of Chief Justice Renato C. Corona.
x-----------------------x
Responding to Sen. Estrada’s revelation, Secretary Florencio Abad of the DBM issued a public statement
G.R. No. 209517 entitled Abad: Releases to Senators Part of Spending Acceleration Program,1 explaining that the funds
released to the Senators had been part of the DAP, a program designed by the DBM to ramp up spending
to accelerate economic expansion. He clarified that the funds had been released to the Senators based
CONFEDERATION FOR UNITY, RECOGNITION AND ADV AN CEMENT OF GOVERNMENT on their letters of request for funding; and that it was not the first time that releases from the DAP had
EMPLOYEES (COURAGE), REPRESENTED BY ITS 1ST VICE PRESIDENT, SANTIAGO been made because the DAP had already been instituted in 2011 to ramp up spending after sluggish
DASMARINAS, JR.; ROSALINDA NARTATES, FOR HERSELF AND AS NATIONAL PRESIDENT OF disbursements had caused the growth of the gross domestic product (GDP) to slow down. He explained
THE CONSOLIDATED UNION OF EMPLOYEES NATIONAL HOUSING AUTHORITY (CUENHA); that the funds under the DAP were usually taken from (1) unreleased appropriations under Personnel
MANUEL BACLAGON, FOR HIMSELF AND AS PRESIDENT OF THE SOCIAL WELFARE Services;2 (2) unprogrammed funds; (3) carry-over appropriations unreleased from the previous year; and
EMPLOYEES ASSOCIATION OF THE PHILIPPINES, DEPARTMENT OF SOCIAL WELFARE AND (4) budgets for slow-moving items or projects that had been realigned to support faster-disbursing
DEVELOPMENT CENTRAL OFFICE (SWEAP-DSWD CO); ANTONIA PASCUAL, FOR HERSELF AND projects.
AS NATIONAL PRESIDENT OF THE DEPARTMENT OF AGRARIAN REFORM EMPLOYEES
ASSOCIATION (DAREA); ALBERT MAGALANG, FOR HIMSELF AND AS PRESIDENT OF THE
ENVIRONMENT AND MANAGEMENT BUREAU EMPLOYEES UNION (EMBEU); AND MARCIAL The DBM soon came out to claim in its website3 that the DAP releases had been sourced from savings
ARABA, FOR HIMSELF AND AS PRESIDENT OF THE KAPISANAN PARA SA KAGALINGAN NG generated by the Government, and from unprogrammed funds; and that the savings had been derived
MGA KAW ANI NG MMDA (KKKMMDA), Petitioners, from (1) the pooling of unreleased appropriations, like unreleased Personnel Services4 appropriations that
vs. would lapse at the end of the year, unreleased appropriations of slow-moving projects and discontinued
BENIGNO SIMEON C. AQUINO Ill, PRESIDENT OF THE REPUBLIC OF THE PHILIPPINES; PAQUITO projects per zero based budgeting findings;5 and (2) the withdrawal of unobligated allotments also for
OCHOA, JR., EXECUTIVE SECRETARY; AND HON. FLORENCIO B. ABAD, SECRETARY OF THE slow-moving programs and projects that had been earlier released to the agencies of the National
DEPARTMENT OF BUDGET AND MANAGEMENT, Respondents. Government.

x-----------------------x The DBM listed the following as the legal bases for the DAP’s use of savings,6 namely: (1) Section 25(5),
Article VI of the 1987 Constitution, which granted to the President the authority to augment an item for his
office in the general appropriations law; (2) Section 49 (Authority to Use Savings for Certain Purposes)
G.R. No. 209569 and Section 38 (Suspension of Expenditure Appropriations), Chapter 5, Book VI of Executive Order (EO)
No. 292 (Administrative Code of 1987); and (3) the General Appropriations Acts (GAAs) of 2011, 2012
VOLUNTEERS AGAINST CRIME AND CORRUPTION (VACC), REPRESENTED BY DANTE L. and 2013, particularly their provisions on the (a) use of savings; (b) meanings of savings and
JIMENEZ, Petitioner, augmentation; and (c) priority in the use of savings.
vs.
PAQUITO N. OCHOA, EXECUTIVE SECRETARY, AND FLORENCIO B. ABAD, SECRETARY OF THE As for the use of unprogrammed funds under the DAP, the DBM cited as legal bases the special
DEPARTMENT OF BUDGET AND MANAGEMENT, Respondents. provisions on unprogrammed fund contained in the GAAs of 2011, 2012 and 2013.

DECISION The revelation of Sen. Estrada and the reactions of Sec. Abad and the DBM brought the DAP to the
consciousness of the Nation for the first time, and made this present controversy inevitable. That the
BERSAMIN, J.: issues against the DAP came at a time when the Nation was still seething in anger over Congressional
pork barrel – "an appropriation of government spending meant for localized projects and secured solely or
For resolution are the consolidated petitions assailing the constitutionality of the Disbursement primarily to bring money to a representative’s district"7 – excited the Nation as heatedly as the pork barrel
Acceleration Program(DAP), National Budget Circular (NBC) No. 541, and related issuances of the controversy.
Department of Budget and Management (DBM) implementing the DAP.
Nine petitions assailing the constitutionality of the DAP and the issuances relating to the DAP were filed
At the core of the controversy is Section 29(1) of Article VI of the 1987 Constitution, a provision of the within days of each other, as follows: G.R. No. 209135 (Syjuco), on October 7, 2013; G.R. No. 209136
fundamental law that firmly ordains that "[n]o money shall be paid out of the Treasury except in pursuance (Luna), on October 7, 2013; G.R. No. 209155 (Villegas),8 on October 16, 2013; G.R. No. 209164
of an appropriation made by law." The tenor and context of the challenges posed by the petitioners (PHILCONSA), on October 8, 2013; G.R. No. 209260 (IBP), on October 16, 2013; G.R. No. 209287
against the DAP indicate that the DAP contravened this provision by allowing the Executive to allocate (Araullo), on October 17, 2013; G.R. No. 209442 (Belgica), on October 29, 2013; G.R. No. 209517
public money pooled from programmed and unprogrammed funds of its various agencies in the guise of (COURAGE), on November6, 2013; and G.R. No. 209569 (VACC), on November 8, 2013.
the President exercising his constitutional authority under Section 25(5) of the 1987 Constitution to
transfer funds out of savings to augment the appropriations of offices within the Executive Branch of the In G.R. No. 209287 (Araullo), the petitioners brought to the Court’s attention NBC No. 541 (Adoption of
Government. But the challenges are further complicated by the interjection of allegations of transfer of Operational Efficiency Measure – Withdrawal of Agencies’ Unobligated Allotments as of June 30, 2012),
funds to agencies or offices outside of the Executive. alleging that NBC No. 541, which was issued to implement the DAP, directed the withdrawal of
unobligated allotments as of June 30, 2012 of government agencies and offices with low levels of
Antecedents obligations, both for continuing and current allotments.

What has precipitated the controversy? In due time, the respondents filed their Consolidated Comment through the Office of the Solicitor General
(OSG).

The Court directed the holding of oral arguments on the significant issues raised and joined.
85
Issues (2) Circulars and orders, which the respondents identified as related to the DAP, namely:

Under the Advisory issued on November 14, 2013, the presentations of the parties during the oral a. NBC No. 528 dated January 3, 2011 (Guidelines on the Release of Funds for FY 2011);
arguments were limited to the following, to wit:
b. NBC No. 535 dated December 29, 2011 (Guidelines on the Release of Funds for FY 2012);
Procedural Issue:
c. NBC No. 541 dated July 18, 2012 (Adoption of Operational Efficiency Measure – Withdrawal of
A. Whether or not certiorari, prohibition, and mandamus are proper remedies to assail the constitutionality Agencies’ Unobligated Allotments as of June 30, 2012);
and validity of the Disbursement Acceleration Program (DAP), National Budget Circular (NBC) No. 541,
and all other executive issuances allegedly implementing the DAP. Subsumed in this issue are whether d. NBC No. 545 dated January 2, 2013 (Guidelines on the Release of Funds for FY 2013);
there is a controversy ripe for judicial determination, and the standing of petitioners.
e. DBM Circular Letter No. 2004-2 dated January 26, 2004 (Budgetary Treatment of
Substantive Issues: Commitments/Obligations of the National Government);

B. Whether or not the DAP violates Sec. 29, Art. VI of the 1987 Constitution, which provides: "No money f. COA-DBM Joint Circular No. 2013-1 dated March 15, 2013 (Revised Guidelines on the Submission of
shall be paid out of the Treasury except in pursuance of an appropriation made by law." Quarterly Accountability Reports on Appropriations, Allotments, Obligations and Disbursements);

C. Whether or not the DAP, NBC No. 541, and all other executive issuances allegedly implementing the g. NBC No. 440 dated January 30, 1995 (Adoption of a Simplified Fund Release System in the
DAP violate Sec. 25(5), Art. VI of the 1987 Constitution insofar as: Government).

(a)They treat the unreleased appropriations and unobligated allotments withdrawn from government (3) A breakdown of the sources of savings, including savings from discontinued projects and unpaid
agencies as "savings" as the term is used in Sec. 25(5), in relation to the provisions of the GAAs of 2011, appropriations for compensation from 2011 to 2013
2012 and 2013;
On January 28, 2014, the OSG, to comply with the Resolution issued on January 21, 2014 directing the
(b)They authorize the disbursement of funds for projects or programs not provided in the GAAs for the respondents to submit the documents not yet submitted in compliance with the directives of the Court or
Executive Department; and its Members, submitted several evidence packets to aid the Court in understanding the factual bases of
the DAP, to wit:
(c)They "augment" discretionary lump sum appropriations in the GAAs.
(1) First Evidence Packet11 – containing seven memoranda issued by the DBM through Sec. Abad,
D. Whether or not the DAP violates: (1) the Equal Protection Clause, (2) the system of checks and inclusive of annexes, listing in detail the 116 DAP identified projects approved and duly signed by the
balances, and (3) the principle of public accountability enshrined in the 1987 Constitution considering that President, as follows:
it authorizes the release of funds upon the request of legislators.
a. Memorandum for the President dated October 12, 2011 (FY 2011 Proposed Disbursement Acceleration
E. Whether or not factual and legal justification exists to issue a temporary restraining order to restrain the Program (Projects and Sources of Funds);
implementation of the DAP, NBC No. 541, and all other executive issuances allegedly implementing the
DAP. b. Memorandum for the President dated December 12, 2011 (Omnibus Authority to Consolidate
Savings/Unutilized Balances and its Realignment);
In its Consolidated Comment, the OSG raised the matter of unprogrammed funds in order to support its
argument regarding the President’s power to spend. During the oral arguments, the propriety of releasing c. Memorandum for the President dated June 25, 2012 (Omnibus Authority to Consolidate
unprogrammed funds to support projects under the DAP was considerably discussed. The petitioners in Savings/Unutilized Balances and their Realignment);
G.R. No. 209287 (Araullo) and G.R. No. 209442 (Belgica) dwelled on unprogrammed funds in their
respective memoranda. Hence, an additional issue for the oral arguments is stated as follows:
d. Memorandum for the President dated September 4, 2012 (Release of funds for other priority projects
and expenditures of the Government);
F. Whether or not the release of unprogrammed funds under the DAP was in accord with the GAAs.
e. Memorandum for the President dated December 19, 2012 (Proposed Priority Projects and
During the oral arguments held on November 19, 2013, the Court directed Sec. Abad to submit a list of Expenditures of the Government);
savings brought under the DAP that had been sourced from (a) completed programs; (b) discontinued or
abandoned programs; (c) unpaid appropriations for compensation; (d) a certified copy of the President’s
directive dated June 27, 2012 referred to in NBC No. 541; and (e) all circulars or orders issued in relation f. Memorandum for the President dated May 20, 2013 (Omnibus Authority to Consolidate
to the DAP.9 Savings/Unutilized Balances and their Realignment to Fund the Quarterly Disbursement Acceleration
Program); and
In compliance, the OSG submitted several documents, as follows:
g. Memorandum for the President dated September 25, 2013 (Funding for the Task Force Pablo
Rehabilitation Plan).
(1) A certified copy of the Memorandum for the President dated June 25, 2012 (Omnibus Authority to
Consolidate Savings/Unutilized Balances and their Realignment);10
(2) Second Evidence Packet12 – consisting of 15 applications of the DAP, with their corresponding
Special Allotment Release Orders (SAROs) and appropriation covers;

86
(3) Third Evidence Packet13 – containing a list and descriptions of 12 projects under the DAP; of NBC No. 541 were not in the exercise of the taxing or spending power of Congress;20 and that even if
the petitioners had suffered injury, there were plain, speedy and adequate remedies in the ordinary course
(4) Fourth Evidence Packet14 – identifying the DAP-related portions of the Annual Financial Report (AFR) of law available to them, like assailing the regularity of the DAP and related issuances before the
of the Commission on Audit for 2011 and 2012; Commission on Audit (COA) or in the trial courts.21
The respondents aver that the special civil actions of certiorari and prohibition are not proper actions for
directly assailing the constitutionality and validity of the DAP, NBC No. 541, and the other executive
(5) Fifth Evidence Packet15 – containing a letter of Department of Transportation and issuances implementing the DAP.22
Communications(DOTC) Sec. Joseph Abaya addressed to Sec. Abad recommending the withdrawal of
funds from his agency, inclusive of annexes; and
In their memorandum, the respondents further contend that there is no authorized proceeding under the
Constitution and the Rules of Court for questioning the validity of any law unless there is an actual case or
(6) Sixth Evidence Packet16 – a print-out of the Solicitor General’s visual presentation for the January 28, controversy the resolution of which requires the determination of the constitutional question; that the
2014 oral arguments. jurisdiction of the Court is largely appellate; that for a court of law to pass upon the constitutionality of a
law or any act of the Government when there is no case or controversy is for that court to set itself up as a
On February 5, 2014,17 the OSG forwarded the Seventh Evidence Packet,18 which listed the sources of reviewer of the acts of Congress and of the President in violation of the principle of separation of powers;
funds brought under the DAP, the uses of such funds per project or activity pursuant to DAP, and the legal and that, in the absence of a pending case or controversy involving the DAP and NBC No. 541, any
bases thereof. decision herein could amount to a mere advisory opinion that no court can validly render.23

On February 14, 2014, the OSG submitted another set of documents in further compliance with the The respondents argue that it is the application of the DAP to actual situations that the petitioners can
Resolution dated January 28, 2014, viz: question either in the trial courts or in the COA; that if the petitioners are dissatisfied with the ruling either
of the trial courts or of the COA, they can appeal the decision of the trial courts by petition for review on
(1) Certified copies of the certifications issued by the Bureau of Treasury to the effect that the revenue certiorari, or assail the decision or final order of the COA by special civil action for certiorari under Rule 64
collections exceeded the original revenue targets for the years 2011, 2012 and 2013, including collections of the Rules of Court.24
arising from sources not considered in the original revenue targets, which certifications were required for
the release of the unprogrammed funds as provided in Special Provision No. 1 of Article XLV, Article XVI, The respondents’ arguments and submissions on the procedural issue are bereft of merit.
and Article XLV of the 2011, 2012 and 2013 GAAs; and (2) A report on releases of savings of the
Executive Department for the use of the Constitutional Commissions and other branches of the Section 1, Article VIII of the 1987 Constitution expressly provides:
Government, as well as the fund releases to the Senate and the Commission on Elections (COMELEC).
Section 1. The judicial power shall be vested in one Supreme Court and in such lower courts as may be
RULING established by law.

I. Procedural Issue: Judicial power includes the duty of the courts of justice to settle actual controversies involving rights which
are legally demandable and enforceable, and to determine whether or not there has been a grave abuse
a) The petitions under Rule 65 are proper remedies of discretion amounting to lack or excess of jurisdiction on the part of any branch or instrumentality of the
Government.
All the petitions are filed under Rule 65 of the Rules of Court, and include applications for the issuance of
writs of preliminary prohibitory injunction or temporary restraining orders. More specifically, the nature of Thus, the Constitution vests judicial power in the Court and in such lower courts as may be established by
the petitions is individually set forth hereunder, to wit: law. In creating a lower court, Congress concomitantly determines the jurisdiction of that court, and that
court, upon its creation, becomes by operation of the Constitution one of the repositories of judicial
power.25 However, only the Court is a constitutionally created court, the rest being created by Congress
G.R. No. 209135 (Syjuco) Certiorari, Prohibition and Mandamus in its exercise of the legislative power.
G.R. No. 209136 (Luna) Certiorariand Prohibition
The Constitution states that judicial power includes the duty of the courts of justice not only "to settle
G.R. No. 209155 (Villegas) Certiorariand Prohibition actual controversies involving rights which are legally demandable and enforceable" but also "to
determine whether or not there has been a grave abuse of discretion amounting to lack or excess of
G.R. No. 209164 (PHILCONSA)
Certiorariand Prohibition jurisdiction on the part of any branch or instrumentality of the Government." It has thereby expanded the
concept of judicial power, which up to then was confined to its traditional ambit of settling actual
G.R. No. 209260 (IBP) Prohibition controversies involving rights that were legally demandable and enforceable.
G.R. No. 209287 (Araullo) Certiorariand Prohibition
The background and rationale of the expansion of judicial power under the 1987 Constitution were laid out
G.R. No. 209442 (Belgica) Certiorari during the deliberations of the 1986 Constitutional Commission by Commissioner Roberto R. Concepcion
(a former Chief Justice of the Philippines) in his sponsorship of the proposed provisions on the Judiciary,
G.R. No. 209517 (COURAGE)Certiorari and Prohibition where he said:–
G.R. No. 209569 (VACC) Certiorari and Prohibition
The Supreme Court, like all other courts, has one main function: to settle actual controversies involving
The respondents submit that there is no actual controversy that is ripe for adjudication in the absence of conflicts of rights which are demandable and enforceable. There are rights which are guaranteed by law
adverse claims between the parties;19 that the petitioners lacked legal standing to sue because no but cannot be enforced by a judicial party. In a decided case, a husband complained that his wife was
allegations were made to the effect that they had suffered any injury as a result of the adoption of the DAP unwilling to perform her duties as a wife. The Court said: "We can tell your wife what her duties as such
and issuance of NBC No. 541; that their being taxpayers did not immediately confer upon the petitioners are and that she is bound to comply with them, but we cannot force her physically to discharge her main
the legal standing to sue considering that the adoption and implementation of the DAP and the issuance
87
marital duty to her husband. There are some rights guaranteed by law, but they are so personal that to x x x In times of social disquietude or political excitement, the great landmarks of the Constitution are apt
enforce them by actual compulsion would be highly derogatory to human dignity." This is why the first part to be forgotten or marred, if not entirely obliterated. In cases of conflict, the judicial department is the only
of the second paragraph of Section 1 provides that: Judicial power includes the duty of courts to settle constitutional organ which can be called upon to determine the proper allocation of powers between the
actual controversies involving rights which are legally demandable or enforceable… several department and among the integral or constituent units thereof.

The courts, therefore, cannot entertain, much less decide, hypothetical questions. In a presidential system xxxx
of government, the Supreme Court has, also, another important function. The powers of government are
generally considered divided into three branches: the Legislative, the Executive and the Judiciary. Each The Constitution is a definition of the powers of government. Who is to determine the nature, scope and
one is supreme within its own sphere and independent of the others. Because of that supremacy power to extent of such powers? The Constitution itself has provided for the instrumentality of the judiciary as the
determine whether a given law is valid or not is vested in courts of justice. rational way. And when the judiciary mediates to allocate constitutional boundaries, it does not assert any
superiority over the other department; it does not in reality nullify or invalidate an act of the legislature, but
Briefly stated, courts of justice determine the limits of power of the agencies and offices of the government only asserts the solemn and sacred obligation assigned to it by the Constitution to determine conflicting
as well as those of its officers. In other words, the judiciary is the final arbiter on the question whether or claims of authority under the Constitution and to establish for the parties in an actual controversy the
not a branch of government or any of its officials has acted without jurisdiction or in excess of jurisdiction, rights which that instrument secures and guarantees to them. This is in truth all that is involved in what is
or so capriciously as to constitute an abuse of discretion amounting to excess of jurisdiction or lack of termed "judicial supremacy" which properly is the power of judicial review under the Constitution. x x x29
jurisdiction. This is not only a judicial power but a duty to pass judgmenton matters of this nature.
What are the remedies by which the grave abuse of discretion amounting to lack or excess of jurisdiction
This is the background of paragraph 2 of Section 1, which means that the courts cannot hereafter evade on the part of any branch or instrumentality of the Government may be determined under the
the duty to settle matters of this nature, by claiming that such matters constitute a political question. (Bold Constitution?
emphasis supplied)26
The present Rules of Court uses two special civil actions for determining and correcting grave abuse of
Upon interpellation by Commissioner Nolledo, Commissioner Concepcion clarified the scope of judicial discretion amounting to lack or excess of jurisdiction. These are the special civil actions for certiorari and
power in the following manner:– prohibition, and both are governed by Rule 65. A similar remedy of certiorari exists under Rule 64, but the
remedy is expressly applicable only to the judgments and final orders or resolutions of the Commission on
MR. NOLLEDO. x x x Elections and the Commission on Audit.

The second paragraph of Section 1 states: "Judicial power includes the duty of courts of justice to settle The ordinary nature and function of the writ of certiorari in our present system are aptly explained in Delos
actual controversies…" The term "actual controversies" according to the Commissioner should refer to Santos v. Metropolitan Bank and Trust Company:30
questions which are political in nature and, therefore, the courts should not refuse to decide those political
questions. But do I understand it right that this is restrictive or only an example? I know there are cases In the common law, from which the remedy of certiorari evolved, the writ of certiorari was issued out of
which are not actual yet the court can assume jurisdiction. An example is the petition for declaratory relief. Chancery, or the King’s Bench, commanding agents or officers of the inferior courts to return the record of
a cause pending before them, so as to give the party more sure and speedy justice, for the writ would
May I ask the Commissioner’s opinion about that? enable the superior court to determine from an inspection of the record whether the inferior court’s
judgment was rendered without authority. The errors were of such a nature that, if allowed to stand, they
would result in a substantial injury to the petitioner to whom no other remedy was available. If the inferior
MR. CONCEPCION. The Supreme Court has no jurisdiction to grant declaratory judgments. court acted without authority, the record was then revised and corrected in matters of law. The writ of
certiorari was limited to cases in which the inferior court was said to be exceeding its jurisdiction or was
MR. NOLLEDO. The Gentleman used the term "judicial power" but judicial power is not vested in the not proceeding according to essential requirements of law and would lie only to review judicial or quasi-
Supreme Court alone but also in other lower courts as may be created by law. judicial acts.

MR. CONCEPCION. Yes. The concept of the remedy of certiorari in our judicial system remains much the same as it has been in the
common law. In this jurisdiction, however, the exercise of the power to issue the writ of certiorari is largely
MR. NOLLEDO. And so, is this only an example? regulated by laying down the instances or situations in the Rules of Court in which a superior court may
issue the writ of certiorari to an inferior court or officer. Section 1, Rule 65 of the Rules of Court
compellingly provides the requirements for that purpose, viz:
MR. CONCEPCION. No, I know this is not. The Gentleman seems to identify political questions with
jurisdictional questions. But there is a difference.
xxxx
MR. NOLLEDO. Because of the expression "judicial power"?
The sole office of the writ of certiorari is the correction of errors of jurisdiction, which includes the
commission of grave abuse of discretion amounting to lack of jurisdiction. In this regard, mere abuse of
MR. CONCEPCION. No. Judicial power, as I said, refers to ordinary cases but where there is a question
discretion is not enough to warrant the issuance of the writ. The abuse of discretion must be grave, which
as to whether the government had authority or had abused its authority to the extent of lacking jurisdiction
means either that the judicial or quasi-judicial power was exercised in an arbitrary or despotic manner by
or excess of jurisdiction, that is not a political question. Therefore, the court has the duty to decide.27
reason of passion or personal hostility, or that the respondent judge, tribunal or board evaded a positive
duty, or virtually refused to perform the duty enjoined or to act in contemplation of law, such as when such
Our previous Constitutions equally recognized the extent of the power of judicial review and the great judge, tribunal or board exercising judicial or quasi-judicial powers acted in a capricious or whimsical
responsibility of the Judiciary in maintaining the allocation of powers among the three great branches of manner as to be equivalent to lack of jurisdiction.31
Government. Speaking for the Court in Angara v. Electoral Commission,28 Justice Jose P. Laurel intoned:

88
Although similar to prohibition in that it will lie for want or excess of jurisdiction, certiorari is to be The first requisite demands that there be an actual case calling for the exercise of judicial power by the
distinguished from prohibition by the fact that it is a corrective remedy used for the re-examination of Court.37 An actual case or controversy, in the words of Belgica v. Executive Secretary Ochoa:38
some action of an inferior tribunal, and is directed to the cause or proceeding in the lower court and not to
the court itself, while prohibition is a preventative remedy issuing to restrain future action, and is directed x x x is one which involves a conflict of legal rights, an assertion of opposite legal claims, susceptible of
to the court itself.32 The Court expounded on the nature and function of the writ of prohibition in Holy judicial resolution as distinguished from a hypothetical or abstract difference or dispute. In other words,
Spirit Homeowners Association, Inc. v. Defensor:33 "[t]here must be a contrariety of legal rights that can be interpreted and enforced on the basis of existing
law and jurisprudence." Related to the requirement of an actual case or controversy is the requirement of
A petition for prohibition is also not the proper remedy to assail an IRR issued in the exercise of a quasi- "ripeness," meaning that the questions raised for constitutional scrutiny are already ripe for adjudication.
legislative function. Prohibition is an extraordinary writ directed against any tribunal, corporation, board, "A question is ripe for adjudication when the act being challenged has had a direct adverse effect on the
officer or person, whether exercising judicial, quasi-judicial or ministerial functions, ordering said entity or individual challenging it. It is a prerequisite that something had then been accomplished or performed by
person to desist from further proceedings when said proceedings are without or in excess of said entity’s either branch before a court may come into the picture, and the petitioner must allege the existence of an
or person’s jurisdiction, or are accompanied with grave abuse of discretion, and there is no appeal or any immediate or threatened injury to itself as a result of the challenged action." "Withal, courts will decline to
other plain, speedy and adequate remedy in the ordinary course of law. Prohibition lies against judicial or pass upon constitutional issues through advisory opinions, bereft as they are of authority to resolve
ministerial functions, but not against legislative or quasi-legislative functions. Generally, the purpose of a hypothetical or moot questions."
writ of prohibition is to keep a lower court within the limits of its jurisdiction in order to maintain the
administration of justice in orderly channels. Prohibition is the proper remedy to afford relief against An actual and justiciable controversy exists in these consolidated cases. The incompatibility of the
usurpation of jurisdiction or power by an inferior court, or when, in the exercise of jurisdiction in handling perspectives of the parties on the constitutionality of the DAP and its relevant issuances satisfy the
matters clearly within its cognizance the inferior court transgresses the bounds prescribed to it by the law, requirement for a conflict between legal rights. The issues being raised herein meet the requisite ripeness
or where there is no adequate remedy available in the ordinary course of law by which such relief can be considering that the challenged executive acts were already being implemented by the DBM, and there
obtained. Where the principal relief sought is to invalidate an IRR, petitioners’ remedy is an ordinary are averments by the petitioners that such implementation was repugnant to the letter and spirit of the
action for its nullification, an action which properly falls under the jurisdiction of the Regional Trial Court. In Constitution. Moreover, the implementation of the DAP entailed the allocation and expenditure of huge
any case, petitioners’ allegation that "respondents are performing or threatening to perform functions sums of public funds. The fact that public funds have been allocated, disbursed or utilized by reason or on
without or in excess of their jurisdiction" may appropriately be enjoined by the trial court through a writ of account of such challenged executive acts gave rise, therefore, to an actual controversy that is ripe for
injunction or a temporary restraining order. adjudication by the Court.

With respect to the Court, however, the remedies of certiorari and prohibition are necessarily broader in It is true that Sec. Abad manifested during the January 28, 2014 oral arguments that the DAP as a
scope and reach, and the writ of certiorari or prohibition may be issued to correct errors of jurisdiction program had been meanwhile discontinued because it had fully served its purpose, saying: "In conclusion,
committed not only by a tribunal, corporation, board or officer exercising judicial, quasi-judicial or Your Honors, may I inform the Court that because the DAP has already fully served its purpose, the
ministerial functions but also to set right, undo and restrain any act of grave abuse of discretion amounting Administration’s economic managers have recommended its termination to the President. x x x."39
to lack or excess of jurisdiction by any branch or instrumentality of the Government, even if the latter does
not exercise judicial, quasi-judicial or ministerial functions. This application is expressly authorized by the
text of the second paragraph of Section 1, supra. The Solicitor General then quickly confirmed the termination of the DAP as a program, and urged that its
termination had already mooted the challenges to the DAP’s constitutionality, viz:
Thus, petitions for certiorari and prohibition are appropriate remedies to raise constitutional issues and to
review and/or prohibit or nullify the acts of legislative and executive officials.34 DAP as a program, no longer exists, thereby mooting these present cases brought to challenge its
constitutionality. Any constitutional challenge should no longer be at the level of the program, which is
now extinct, but at the level of its prior applications or the specific disbursements under the now defunct
Necessarily, in discharging its duty under Section 1, supra, to set right and undo any act of grave abuse of policy. We challenge the petitioners to pick and choose which among the 116 DAP projects they wish to
discretion amounting to lack or excess of jurisdiction by any branch or instrumentality of the Government, nullify, the full details we will have provided by February 5. We urge this Court to be cautious in limiting
the Court is not at all precluded from making the inquiry provided the challenge was properly brought by the constitutional authority of the President and the Legislature to respond to the dynamic needs of the
interested or affected parties. The Court has been thereby entrusted expressly or by necessary country and the evolving demands of governance, lest we end up straight jacketing our elected
implication with both the duty and the obligation of determining, in appropriate cases, the validity of any representatives in ways not consistent with our constitutional structure and democratic principles.40
assailed legislative or executive action. This entrustment is consistent with the republican system of
checks and balances.35
A moot and academic case is one that ceases to present a justiciable controversy by virtue of
supervening events, so that a declaration thereon would be of no practical use or value.41
Following our recent dispositions concerning the congressional pork barrel, the Court has become more
alert to discharge its constitutional duty. We will not now refrain from exercising our expanded judicial
power in order to review and determine, with authority, the limitations on the Chief Executive’s spending The Court cannot agree that the termination of the DAP as a program was a supervening event that
power. effectively mooted these consolidated cases. Verily, the Court had in the past exercised its power of
judicial review despite the cases being rendered moot and academic by supervening events, like: (1)
when there was a grave violation of the Constitution; (2) when the case involved a situation of exceptional
b) Requisites for the exercise of the character and was of paramount public interest; (3) when the constitutional issue raised required the
power of judicial review were formulation of controlling principles to guide the Bench, the Bar and the public; and (4) when the case was
complied with capable of repetition yet evading review.42

The requisites for the exercise of the power of judicial review are the following, namely: (1) there must Assuming that the petitioners’ several submissions against the DAP were ultimately sustained by the
bean actual case or justiciable controversy before the Court; (2) the question before the Court must be Court here, these cases would definitely come under all the exceptions. Hence, the Court should not
ripe for adjudication; (3) the person challenging the act must be a proper party; and (4) the issue of abstain from exercising its power of judicial review.
constitutionality must be raised at the earliest opportunity and must be the very litis mota of the case.36
Did the petitioners have the legal standing to sue?

89
Legal standing, as a requisite for the exercise of judicial review, refers to "a right of appearance in a court remedied." With respect to taxpayer’s suits, Terr v. Jordan held that "the right of a citizen and a taxpayer
of justice on a given question."43 The concept of legal standing, or locus standi, was particularly to maintain an action in courts to restrain the unlawful use of public funds to his injury cannot be
discussed in De Castro v. Judicial and Bar Council,44 where the Court said: denied."45

In public or constitutional litigations, the Court is often burdened with the determination of the locus standi The Court has cogently observed in Agan, Jr. v. Philippine International Air Terminals Co., Inc.46 that
of the petitioners due to the ever-present need to regulate the invocation of the intervention of the Court to "[s]tanding is a peculiar concept in constitutional law because in some cases, suits are not brought by
correct any official action or policy in order to avoid obstructing the efficient functioning of public officials parties who have been personally injured by the operation of a law or any other government act but by
and offices involved in public service. It is required, therefore, that the petitioner must have a personal concerned citizens, taxpayers or voters who actually sue in the public interest."
stake in the outcome of the controversy, for, as indicated in Agan, Jr. v. Philippine International Air
Terminals Co., Inc.: Except for PHILCONSA, a petitioner in G.R. No. 209164, the petitioners have invoked their capacities as
taxpayers who, by averring that the issuance and implementation of the DAP and its relevant issuances
The question on legal standing is whether such parties have "alleged such a personal stake in the involved the illegal disbursements of public funds, have an interest in preventing the further dissipation of
outcome of the controversy as to assure that concrete adverseness which sharpens the presentation of public funds. The petitioners in G.R. No. 209287 (Araullo) and G.R. No. 209442 (Belgica) also assert their
issues upon which the court so largely depends for illumination of difficult constitutional questions." right as citizens to sue for the enforcement and observance of the constitutional limitations on the political
Accordingly, it has been held that the interest of a person assailing the constitutionality of a statute must branches of the Government.47
be direct and personal. He must be able to show, not only that the law or any government act is invalid,
but also that he sustained or is in imminent danger of sustaining some direct injury as a result of its On its part, PHILCONSA simply reminds that the Court has long recognized its legal standing to bring
enforcement, and not merely that he suffers thereby in some indefinite way. It must appear that the cases upon constitutional issues.48 Luna, the petitioner in G.R. No. 209136, cites his additional capacity
person complaining has been or is about to be denied some right or privilege to which he is lawfully as a lawyer. The IBP, the petitioner in G.R. No. 209260, stands by "its avowed duty to work for the rule of
entitled or that he is about to be subjected to some burdens or penalties by reason of the statute or act law and of paramount importance of the question in this action, not to mention its civic duty as the official
complained of. association of all lawyers in this country."49

It is true that as early as in 1937, in People v. Vera, the Court adopted the direct injury test for determining Under their respective circumstances, each of the petitioners has established sufficient interest in the
whether a petitioner in a public action had locus standi. There, the Court held that the person who would outcome of the controversy as to confer locus standi on each of them.
assail the validity of a statute must have "a personal and substantial interest in the case such that he has
sustained, or will sustain direct injury as a result." Vera was followed in Custodio v. President of the
Senate, Manila Race Horse Trainers’ Association v. De la Fuente, Anti-Chinese League of the Philippines In addition, considering that the issues center on the extent of the power of the Chief Executive to
v. Felix, and Pascual v. Secretary of Public Works. disburse and allocate public funds, whether appropriated by Congress or not, these cases pose issues
that are of transcendental importance to the entire Nation, the petitioners included. As such, the
determination of such important issues call for the Court’s exercise of its broad and wise discretion "to
Yet, the Court has also held that the requirement of locus standi, being a mere procedural technicality, waive the requirement and so remove the impediment to its addressing and resolving the serious
can be waived by the Court in the exercise of its discretion. For instance, in 1949, in Araneta v. Dinglasan, constitutional questions raised."50
the Court liberalized the approach when the cases had "transcendental importance." Some notable
controversies whose petitioners did not pass the direct injury test were allowed to be treated in the same
way as in Araneta v. Dinglasan. II.
Substantive Issues
In the 1975 decision in Aquino v. Commission on Elections, this Court decided to resolve the issues
raised by the petition due to their "far reaching implications," even if the petitioner had no personality to 1.
file the suit. The liberal approach of Aquino v. Commission on Elections has been adopted in several Overview of the Budget System
notable cases, permitting ordinary citizens, legislators, and civic organizations to bring their suits involving
the constitutionality or validity of laws, regulations, and rulings. An understanding of the Budget System of the Philippines will aid the Court in properly appreciating and
justly resolving the substantive issues.
However, the assertion of a public right as a predicate for challenging a supposedly illegal or
unconstitutional executive or legislative action rests on the theory that the petitioner represents the public a) Origin of the Budget System
in general. Although such petitioner may not be as adversely affected by the action complained against as
are others, it is enough that he sufficiently demonstrates in his petition that he is entitled to protection or The term "budget" originated from the Middle English word bouget that had derived from the Latin word
relief from the Court in the vindication of a public right. bulga (which means bag or purse).51

Quite often, as here, the petitioner in a public action sues as a citizen or taxpayer to gain locus standi. In the Philippine setting, Commonwealth Act (CA) No. 246 (Budget Act) defined "budget" as the financial
That is not surprising, for even if the issue may appear to concern only the public in general, such program of the National Government for a designated fiscal year, consisting of the statements of
capacities nonetheless equip the petitioner with adequate interest to sue. In David v. Macapagal-Arroyo, estimated receipts and expenditures for the fiscal year for which it was intended to be effective based on
the Court aptly explains why: the results of operations during the preceding fiscal years. The term was given a different meaning under
Republic Act No. 992 (Revised Budget Act) by describing the budget as the delineation of the services
Case law in most jurisdiction snow allows both "citizen" and "taxpayer" standing in public actions. The and products, or benefits that would accrue to the public together with the estimated unit cost of each type
distinction was first laid down in Beauchamp v. Silk, where it was held that the plaintiff in a taxpayer’s suit of service, product or benefit.52 For a forthright definition, budget should simply be identified as the
is in a different category from the plaintiff in a citizen’s suit. In the former, the plaintiff is affected by the financial plan of the Government,53 or "the master plan of government."54
expenditure of public funds, while in the latter, he is but the mere instrument of the public concern. As held
by the New York Supreme Court in People ex rel Case v. Collins: "In matter of mere public right, The concept of budgeting has not been the product of recent economies. In reality, financing public goals
however…the people are the real parties…It is at least the right, if not the duty, of every citizen to interfere and activities was an idea that existed from the creation of the State.55 To protect the people, the territory
and see that a public offence be properly pursued and punished, and that a public grievance be and sovereignty of the State, its government must perform vital functions that required public

90
expenditures. At the beginning, enormous public expenditures were spent for war activities, preservation (DBCC) as well as policy guidelines and procedures to aid government agencies in the preparation and
of peace and order, security, administration of justice, religion, and supply of limited goods and submission of their budget proposals. The Budget Call is of two kinds, namely: (1) a National Budget Call,
services.56 In order to finance those expenditures, the State raised revenues through taxes and which is addressed to all agencies, including state universities and colleges; and (2) a Corporate Budget
impositions.57 Thus, budgeting became necessary to allocate public revenues for specific government Call, which is addressed to all government-owned and -controlled corporations (GOCCs) and government
functions.58 The State’s budgeting mechanism eventually developed through the years with the growing financial institutions (GFIs).
functions of its government and changes in its market economy.
Following the issuance of the Budget Call, the various departments and agencies submit their respective
The Philippine Budget System has been greatly influenced by western public financial institutions. This is Agency Budget Proposals to the DBM. To boost citizen participation, the current administration has
because of the country’s past as a colony successively of Spain and the United States for a long period of tasked the various departments and agencies to partner with civil society organizations and other citizen-
time. Many aspects of the country’s public fiscal administration, including its Budget System, have been stakeholders in the preparation of the Agency Budget Proposals, which proposals are then presented
naturally patterned after the practices and experiences of the western public financial institutions. At any before a technical panel of the DBM in scheduled budget hearings wherein the various departments and
rate, the Philippine Budget System is presently guided by two principal objectives that are vital to the agencies are given the opportunity to defend their budget proposals. DBM bureaus thereafter review the
development of a progressive democratic government, namely: (1) to carry on all government activities Agency Budget Proposals and come up with recommendations for the Executive Review Board,
under a comprehensive fiscal plan developed, authorized and executed in accordance with the comprised by the DBM Secretary and the DBM’s senior officials. The discussions of the Executive Review
Constitution, prevailing statutes and the principles of sound public management; and (2) to provide for the Board cover the prioritization of programs and their corresponding support vis-à-vis the priority agenda of
periodic review and disclosure of the budgetary status of the Government in such detail so that persons the National Government, and their implementation.
entrusted by law with the responsibility as well as the enlightened citizenry can determine the adequacy of
the budget actions taken, authorized or proposed, as well as the true financial position of the The DBM next consolidates the recommended agency budgets into the National Expenditure Program
Government.59 (NEP)and a Budget of Expenditures and Sources of Financing (BESF). The NEP provides the details of
spending for each department and agency by program, activity or project (PAP), and is submitted in the
b) Evolution of the Philippine Budget System form of a proposed GAA. The Details of Selected Programs and Projects is the more detailed
disaggregation of key PAPs in the NEP, especially those in line with the National Government’s
The budget process in the Philippines evolved from the early years of the American Regime up to the development plan. The Staffing Summary provides the staffing complement of each department and
passage of the Jones Law in 1916. A Budget Office was created within the Department of Finance by the agency, including the number of positions and amounts allocated.
Jones Law to discharge the budgeting function, and was given the responsibility to assist in the
preparation of an executive budget for submission to the Philippine Legislature.60 The NEP and BESF are thereafter presented by the DBM and the DBCC to the President and the Cabinet
for further refinements or reprioritization. Once the NEP and the BESF are approved by the President and
As early as under the 1935 Constitution, a budget policy and a budget procedure were established, and the Cabinet, the DBM prepares the budget documents for submission to Congress. The budget
subsequently strengthened through the enactment of laws and executive acts.61 EO No. 25, issued by documents consist of: (1) the President’s Budget Message, through which the President explains the
President Manuel L. Quezon on April 25, 1936, created the Budget Commission to serve as the agency policy framework and budget priorities; (2) the BESF, mandated by Section 22, Article VII of the
that carried out the President’s responsibility of preparing the budget.62 CA No. 246, the first budget law, Constitution,68 which contains the macroeconomic assumptions, public sector context, breakdown of the
went into effect on January 1, 1938 and established the Philippine budget process. The law also provided expenditures and funding sources for the fiscal year and the two previous years; and (3) the NEP.
a line-item budget as the framework of the Government’s budgeting system,63 with emphasis on the
observance of a "balanced budget" to tie up proposed expenditures with existing revenues. Public or government expenditures are generally classified into two categories, specifically: (1) capital
expenditures or outlays; and (2) current operating expenditures. Capital expenditures are the expenses
CA No. 246 governed the budget process until the passage on June 4, 1954 of Republic Act (RA) No. whose usefulness lasts for more than one year, and which add to the assets of the Government, including
992,whereby Congress introduced performance-budgeting to give importance to functions, projects and investments in the capital of government-owned or controlled corporations and their
activities in terms of expected results.64 RA No. 992 also enhanced the role of the Budget Commission as subsidiaries.69 Current operating expenditures are the purchases of goods and services in current
the fiscal arm of the Government.65 consumption the benefit of which does not extend beyond the fiscal year.70 The two components of
current expenditures are those for personal services (PS), and those for maintenance and other operating
expenses(MOOE).
The 1973 Constitution and various presidential decrees directed a series of budgetary reforms that
culminated in the enactment of PD No. 1177 that President Marcos issued on July30, 1977, and of PD
No. 1405, issued on June 11, 1978. The latter decree converted the Budget Commission into the Ministry Public expenditures are also broadly grouped according to their functions into: (1) economic development
of Budget, and gave its head the rank of a Cabinet member. expenditures (i.e., expenditures on agriculture and natural resources, transportation and communications,
commerce and industry, and other economic development efforts);71 (2) social services or social
development expenditures (i.e., government outlay on education, public health and medicare, labor and
The Ministry of Budget was later renamed the Office of Budget and Management (OBM) under EO No. welfare and others);72 (3) general government or general public services expenditures (i.e., expenditures
711. The OBM became the DBM pursuant to EO No. 292 effective on November 24, 1989. for the general government, legislative services, the administration of justice, and for pensions and
gratuities);73 (4) national defense expenditures (i.e., sub-divided into national security expenditures and
c) The Philippine Budget Cycle66 expenditures for the maintenance of peace and order);74 and (5) public debt.75

Four phases comprise the Philippine budget process, specifically: (1) Budget Preparation; (2) Budget Public expenditures may further be classified according to the nature of funds, i.e., general fund, special
Legislation; (3) Budget Execution; and (4) Accountability. Each phase is distinctly separate from the fund or bond fund.76
others but they overlap in the implementation of the budget during the budget year.
On the other hand, public revenues complement public expenditures and cover all income or receipts of
c.1.Budget Preparation67 the government treasury used to support government expenditures.77

The budget preparation phase is commenced through the issuance of a Budget Call by the DBM. The Classical economist Adam Smith categorized public revenues based on two principal sources, stating:
Budget Call contains budget parameters earlier set by the Development Budget Coordination Committee "The revenue which must defray…the necessary expenses of government may be drawn either, first from

91
some fund which peculiarly belongs to the sovereign or commonwealth, and which is independent of the
revenue of the people, or, secondly, from the revenue of the people."78 Adam Smith’s classification relied
on the two aspects of the nature of the State: first, the State as a juristic person with an artificial
personality, and, second, the State as a sovereign or entity possessing supreme power. Under the first Miscellaneous Operating and
Service Income
aspect, the State could hold property and engage in trade, thereby deriving what is called its quasi private
income or revenues, and which "peculiarly belonged to the sovereign." Under the second aspect, the
State could collect by imposing charges on the revenues of its subjects in the form of taxes.79
Fines and Penalties-Government
Services and Business Operations
In the Philippines, public revenues are generally derived from the following sources, to wit: (1) tax
revenues(i.e., compulsory contributions to finance government activities); 80 (2) capital revenues(i.e.,
proceeds from sales of fixed capital assets or scrap thereof and public domain, and gains on such salesIncome from Grants and
like sale of public lands, buildings and other structures, equipment, and other properties recorded as fixed
assets); 81 (3) grants(i.e., voluntary contributions and aids given to the Government for its operation on
specific purposes in the form of money and/or materials, and do not require any monetary commitment on
the part of the recipient);82 (4) extraordinary income(i.e., repayment of loans and advances made by
government corporations and local governments and the receipts and shares in income of the Banko
Sentral ng Pilipinas, and other receipts);83 and (5) public borrowings(i.e., proceeds of repayable
obligations generally with interest from domestic and foreign creditors of the Government in general, c.2. Budget Legislation86
including the National Government and its political subdivisions).84
The Budget Legislation Phase covers the period commencing from the time Congress receives the
More specifically, public revenues are classified as follows:85 President’s Budget, which is inclusive of the NEPand the BESF, up to the President’s approval of the
GAA. This phase is also known as the Budget Authorization Phase, and involves the significant
General Income Specific Income participation of the Legislative through its deliberations.

Initially, the President’s Budget is assigned to the House of Representatives’ Appropriations Committee
sidy Income from National on First Reading. The Appropriations Committee and its various Sub-Committees schedule and conduct
budget hearings to examine the PAPs of the departments and agencies. Thereafter, the House of
Property Taxes Representatives drafts the General Appropriations Bill (GAB).87
sidy from Central Office
The GABis sponsored, presented and defended by the House of Representatives’ Appropriations
Taxes on Goods and Services Committee and Sub-Committees in plenary session. As with other laws, the GAB is approved on Third
sidy from Regional Reading before the House of Representatives’ version is transmitted to the Senate.88
e/Staff Bureaus
Taxes on International Trade and
After transmission, the Senate conducts its own committee hearings on the GAB. To expedite
me from Government proceedings, the Senate may conduct its committee hearings simultaneously with the House of
Representatives’ deliberations. The Senate’s Finance Committee and its Sub-Committees may submit the
Other Taxes 6.Fines and Penalties-Tax Revenue
proposed amendments to the GAB to the plenary of the Senate only after the House of Representatives
has formally transmitted its version to the Senate. The Senate version of the GAB is likewise approved on
me from Government
Other Specific Income Third Reading.89
ness Operations

The House of Representatives and the Senate then constitute a panel each to sit in the Bicameral
s Revenue Conference Committee for the purpose of discussing and harmonizing the conflicting provisions of their
versions of the GAB. The "harmonized" version of the GAB is next presented to the President for
approval.90 The President reviews the GAB, and prepares the Veto Message where budget items are
subjected to direct veto,91 or are identified for conditional implementation.

rance Income If, by the end of any fiscal year, the Congress shall have failed to pass the GAB for the ensuing fiscal
year, the GAA for the preceding fiscal year shall be deemed re-enacted and shall remain in force and
effect until the GAB is passed by the Congress.92
dend Income

c.3. Budget Execution93


est Income
With the GAA now in full force and effect, the next step is the implementation of the budget. The Budget
of Confiscated Goods and Execution Phase is primarily the function of the DBM, which is tasked to perform the following procedures,
namely: (1) to issue the programs and guidelines for the release of funds; (2) to prepare an Allotment and
Cash Release Program; (3) to release allotments; and (4) to issue disbursement authorities.

ign Exchange (FOREX)

92
The implementation of the GAA is directed by the guidelines issued by the DBM. Prior to this, the various When he assumed office in the middle of 2010, President Aquino made efficiency and transparency in
departments and agencies are required to submit Budget Execution Documents(BED) to outline their government spending a significant focus of his Administration. Yet, although such focus resulted in an
plans and performance targets by laying down the physical and financial plan, the monthly cash program, improved fiscal deficit of 0.5% in the gross domestic product (GDP) from January to July of 2011, it also
the estimate of monthly income, and the list of obligations that are not yet due and demandable. unfortunately decelerated government project implementation and payment schedules.103 The World
Bank observed that the Philippines’ economic growth could be reduced, and potential growth could be
Thereafter, the DBM prepares an Allotment Release Program (ARP)and a Cash Release Program weakened should the Government continue with its underspending and fail to address the large
(CRP).The ARP sets a limit for allotments issued in general and to a specific agency. The CRP fixes the deficiencies in infrastructure.104 The economic situation prevailing in the middle of 2011 thus paved the
monthly, quarterly and annual disbursement levels. way for the development and implementation of the DAP as a stimulus package intended to fast-track
public spending and to push economic growth by investing on high-impact budgetary PAPs to be funded
from the "savings" generated during the year as well as from unprogrammed funds.105 In that respect,
Allotments, which authorize an agency to enter into obligations, are issued by the DBM. Allotments are the DAP was the product of "plain executive policy-making" to stimulate the economy by way of
lesser in scope than appropriations, in that the latter embrace the general legislative authority to spend. accelerated spending.106 The Administration would thereby accelerate government spending by: (1)
Allotments may be released in two forms – through a comprehensive Agency Budget Matrix (ABM),94 or, streamlining the implementation process through the clustering of infrastructure projects of the
individually, by SARO.95 Department of Public Works and Highways (DPWH) and the Department of Education (DepEd),and (2)
front loading PPP-related projects107 due for implementation in the following year.108
Armed with either the ABM or the SARO, agencies become authorized to incur obligations96 on behalf of
the Government in order to implement their PAPs. Obligations may be incurred in various ways, like hiring Did the stimulus package work?
of personnel, entering into contracts for the supply of goods and services, and using utilities.
The March 2012 report of the World Bank,109 released after the initial implementation of the DAP,
In order to settle the obligations incurred by the agencies, the DBM issues a disbursement authority so revealed that the DAP was partially successful. The disbursements under the DAP contributed 1.3
that cash may be allocated in payment of the obligations. A cash or disbursement authority that is percentage points to GDP growth by the fourth quarter of 2011.110 The continued implementation of the
periodically issued is referred to as a Notice of Cash Allocation (NCA),97 which issuance is based upon DAP strengthened growth by 11.8% year on year while infrastructure spending rebounded from a 29%
an agency’s submission of its Monthly Cash Program and other required documents. The NCA specifies contraction to a 34% growth as of September 2013.111
the maximum amount of cash that can be withdrawn from a government servicing bank for the period
indicated. Apart from the NCA, the DBM may issue a Non-Cash Availment Authority(NCAA) to authorize
non-cash disbursements, or a Cash Disbursement Ceiling(CDC) for departments with overseas The DAP thus proved to be a demonstration that expenditure was a policy instrument that the
operations to allow the use of income collected by their foreign posts for their operating requirements. Government could use to direct the economies towards growth and development.112 The Government,
by spending on public infrastructure, would signify its commitment of ensuring profitability for prospective
investors.113 The PAPs funded under the DAP were chosen for this reason based on their: (1) multiplier
Actual disbursement or spending of government funds terminates the Budget Execution Phase and is impact on the economy and infrastructure development; (2) beneficial effect on the poor; and (3)
usually accomplished through the Modified Disbursement Scheme under which disbursements translation into disbursements.114
chargeable against the National Treasury are coursed through the government servicing banks.
b. History of the implementation of
c.4. Accountability98 the DAP, and sources of funds
under the DAP
Accountability is a significant phase of the budget cycle because it ensures that the government funds
have been effectively and efficiently utilized to achieve the State’s socio-economic goals. It also allows the How the Administration’s economic managers conceptualized and developed the DAP, and finally
DBM to assess the performance of agencies during the fiscal year for the purpose of implementing presented it to the President remains unknown because the relevant documents appear to be scarce.
reforms and establishing new policies.
The earliest available document relating to the genesis of the DAP was the memorandum of October
An agency’s accountability may be examined and evaluated through (1) performance targets and 12,2011 from Sec. Abad seeking the approval of the President to implement the proposed DAP. The
outcomes; (2) budget accountability reports; (3) review of agency performance; and (4) audit conducted memorandum, which contained a list of the funding sources for ₱72.11 billion and of the proposed priority
by the Commission on Audit(COA). projects to be funded,115 reads:

2. MEMORANDUM FOR THE PRESIDENT

Nature of the DAP as a fiscal plan xxxx

a. DAP was a program designed to SUBJECT: FY 2011 PROPOSED DISBURSEMENT ACCELERATION PROGRAM (PROJECTS AND
promote economic growth SOURCES OF FUNDS)

Policy is always a part of every budget and fiscal decision of any Administration.99 The national budget DATE: OCTOBER 12, 2011
the Executive prepares and presents to Congress represents the Administration’s "blueprint for public
policy" and reflects the Government’s goals and strategies.100 As such, the national budget becomes a
tangible representation of the programs of the Government in monetary terms, specifying therein the Mr. President, this is to formally confirm your approval of the Disbursement Acceleration Program totaling
PAPs and services for which specific amounts of public funds are proposed and allocated.101 Embodied ₱72.11 billion. We are already working with all the agencies concerned for the immediate execution of the
in every national budget is government spending.102 projects therein.

A. Fund Sources for the Acceleration Program

93
(Descriptions of projects attached as Annex A)
Amount
Action
Fund Sources (In million Description
Requested
Php) GOCCs and GFIs

FY 2011 30,000 Unreleased Personnel Declare as Agency/Project Allotment


Unreleased Services (PS) savings and (SARO and NCA Release) (in Million Php)
Personal appropriations which approve/
Services (PS) will lapse at the end of authorize its use
Appropriations FY 2011 but may be for the 2011 1. LRTA: Rehabilitation of LRT 1 and 2 1,868
pooled as savings and Disbursement
realigned for priority Acceleration
programs that require Program 2. NHA: 11,050
immediate funding
a. Resettlement of North Triangle residents to
Camarin A7 450
FY 2011 482 Unreleased   b. Housing for BFP/BJMP
Unreleased appropriations (slow c. On-site development for families living 500
Appropriations moving projects and along dangerous 10,000
programs for d. Relocation sites for informal settlers
discontinuance) along Iloilo River and its tributaries 100

FY 2010 12,336 Supported by the GFI Approve and 3. PHIL. HEART CENTER: Upgrading of 357
Unprogrammed Dividends authorize its use ageing physical plant and medical equipment
Fund for the 2011
Disbursement
Acceleration 4. CREDIT INFO CORP: Establishment of 75
Program centralized credit information system

FY 2010 21,544 Unreleased With prior 5. PIDS: purchase of land to relocate the PIDS 100
Carryover appropriations (slow approval from office and building construction
Appropriation moving projects and the President in
programs for November 2010
discontinuance) and to declare as 6. HGC: Equity infusion for credit insurance 400
savings from Zero-based Budgeting savings and with and mortgage guaranty operations of HGC
Initiative authority to use
for priority
projects 7. PHIC: Obligations incurred (premium 1,496
subsidy for indigent families) in January-June
2010, booked for payment in Jul[y] – Dec
FY 2011 Budget 7,748 FY 2011 Agency For information 2010. The delay in payment is due to the
items for Budget items that can delay in the certification of the LGU
realignment be realigned within the counterpart. Without it, the NG is obliged to
agency to fund new fast pay the full amount.
disbursing projects
DPWH-3.981 Billion
DA – 2.497 Billion 8. Philpost: Purchase of foreclosed property. 644
DOT – 1.000 Billion Payment of Mandatory Obligations, (GSIS,
DepEd – 270 Million PhilHealth, ECC), Franking Privilege

TOTAL 72.110   9. BSP: First equity infusion out of Php 40B 10,000
capitalization under the BSP Law

B. Projects in the Disbursement Acceleration Program

94
10. PCMC: Capital and Equipment Renovation 280 Irrigation Project 411 411

11. LCOP: 105 17. DAR: 1,293 1,293


a. Pediatric Pulmonary Program a. Agrarian Reform
b. Bio-regenerative Technology Program Communities Project 2 1,293 132
(Stem-Cell Research – subject to legal 35 b. Landowners Compensation 5,432
review and presentation)

70 18. DBM: Conduct of National


Survey of    
Farmers/Fisherfolks/Ips 625 625
12. TIDCORP: NG Equity infusion 570
19. DOJ: Operating requirements
of 50 investigation agents and    
TOTAL 26,945 15 state attorneys 11 11

20. DOT: Preservation of the Cine


Corregidor Complex 25 25
NGAs/LGUs

21. OPAPP: Activities for Peace


Agency/Project Allotment Process (PAMANA- Project    
(SARO) Cash details: budget breakdown,    
(In Million Requirement implementation plan, and    
Php) (NCA) conditions on fund release    
attached as Annex B) 1,819 1,819

13. DOF-BIR: NPSTAR


centralization of data     22. DOST 425 425
processing and others (To be     a. Establishment of National
synchronized with GFMIS     Meterological and Climate    
activities) 758 758 Center 275 275
b. Enhancement of Doppler
Radar Network for National    
14. COA: IT infrastructure Weather Watch, Accurate    
program and hiring of     Forecasting and Flood Early    
additional litigational experts 144 144 Warning 190 190

15. DND-PAF: On Base Housing 23. DOF-BOC: To settle the


Facilities and Communication     principal obligations with    
Equipment 30 30 PDIC consistent with the    
agreement with the CISS and    
SGS 2,800 2,800
16. DA: 2,959 2,223
a. Irrigation, FMRs and
Integrated Community Based Multi-Species     24. OEO-FDCP: Establishment of
Hatchery and Aquasilvi     the National Film Archive and    
Farming 1,629 1,629 local cinematheques, and other    
b. Mindanao Rural local activities 20 20
Development Project 919 183

c. NIA Agno River Integrated

95
25. DPWH: Various infrastructure
projects 5,500 5,500

GOCCs 26,895 26,895


26. DepEd/ERDT/DOST: Thin
Client Cloud Computing    
Project 270 270 NGAs/LGUs 45,165 44,000

27. DOH: Hiring of nurses and For His Excellency’s Consideration


midwives 294 294
(Sgd.) FLORENCIO B. ABAD

28. TESDA: Training Program in [/] APPROVED


partnership with BPO industry    
and other sectors 1,100 1,100
[ ] DISAPPROVED

(Sgd.) H.E. BENIGNO S. AQUINO, III


29. DILG: Performance Challenge
Fund (People Empowered    
Community Driven     OCT 12, 2011
Development with DSWD and    
NAPC) 250 50 The memorandum of October 12, 2011 was followed by another memorandum for the President dated
December 12, 2011116 requesting omnibus authority to consolidate the savings and unutilized balances
for fiscal year 2011. Pertinent portions of the memorandum of December 12, 2011 read:
30. ARMM: Comprehensive Peace
and Development Intervention 8,592 8,592 MEMORANDUM FOR THE PRESIDENT

xxxx
31. DOTC-MRT: Purchase of
additional MRT cars 4,500 -
SUBJECT: Omnibus Authority to Consolidate Savings/Unutilized Balances and its Realignment

DATE: December 12, 2011


32. LGU Support Fund 6,500 6,500

This is to respectfully request for the grant of Omnibus Authority to consolidate savings/unutilized
balances in FY 2011 corresponding to completed or discontinued projects which may be pooled to fund
33. Various Other Local Projects 6,500 6,500
additional projects or expenditures.

In addition, Mr. President, this measure will allow us to undertake projects even if their implementation
34. Development Assistance to the
carries over to 2012 without necessarily impacting on our budget deficit cap next year.
Province of Quezon 750 750

BACKGROUND
TOTAL 45,165 44,000
1.0 The DBM, during the course of performance reviews conducted on the agencies’ operations,
particularly on the implementation of their projects/activities, including expenses incurred in undertaking
C. Summary the same, have identified savings out of the 2011 General Appropriations Act. Said savings correspond to
completed or discontinued projects under certain departments/agencies which may be pooled, for the
following:
Fund Sources
Identified for Allotments Cash 1.1 to provide for new activities which have not been anticipated during preparation of the budget;
Approval for Release Requirements for
(In Million Release in FY
Php) 2011 1.2 to augment additional requirements of on-going priority projects; and

1.3 to provide for deficiencies under the Special Purpose Funds, e.g., PDAF, Calamity Fund, Contingent
Total 72,110 72,110 70,895 Fund

96
1.4 to cover for the modifications of the original allotment class allocation as a result of on-going priority DEC 21, 2011
projects and implementation of new activities
Substantially identical requests for authority to pool savings and to fund proposed projects were contained
2.0 x x x x in various other memoranda from Sec. Abad dated June 25, 2012,117 September 4, 2012,118 December
19, 2012,119 May 20, 2013,120 and September 25, 2013.121 The President apparently approved all the
2.1 x x x requests, withholding approval only of the proposed projects contained in the June 25, 2012
memorandum, as borne out by his marginal note therein to the effect that the proposed projects should
still be "subject to further discussions."122
2.2 x x x
In order to implement the June25, 2012 memorandum, Sec. Abad issued NBC No. 541 (Adoption of
ON THE UTILIZATION OF POOLED SAVINGS Operational Efficiency Measure – Withdrawal of Agencies’ Unobligated Allotments as of June 30,
2012),123 reproduced herein as follows:
3.0 It may be recalled that the President approved our request for omnibus authority to pool
savings/unutilized balances in FY 2010 last November 25, 2010. NATIONAL BUDGET CIRCULAR No. 541

4.0 It is understood that in the utilization of the pooled savings, the DBM shall secure the corresponding July 18, 2012
approval/confirmation of the President. Furthermore, it is assured that the proposed realignments shall be
within the authorized Expenditure level.
TO: All Heads of Departments/Agencies/State Universities and Colleges and other Offices of the National
Government, Budget and Planning Officers; Heads of Accounting Units and All Others Concerned
5.0 Relative thereto, we have identified some expenditure items that may be sourced from the said pooled
appropriations in FY 2010 that will expire on December 31, 2011 and appropriations in FY 2011 that may
be declared as savings to fund additional expenditures. SUBJECT : Adoption of Operational Efficiency Measure – Withdrawal of Agencies’ Unobligated
Allotments as of June 30, 2012
5.1 The 2010 Continuing Appropriations (pooled savings) is proposed to be spent for the projects that we
have identified to be immediate actual disbursements considering that this same fund source will expire 1.0 Rationale
on December 31, 2011.
The DBM, as mandated by Executive Order (EO) No. 292 (Administrative Code of 1987), periodically
5.2 With respect to the proposed expenditure items to be funded from the FY 2011 Unreleased reviews and evaluates the departments/agencies’ efficiency and effectiveness in utilizing budgeted funds
Appropriations, most of these are the same projects for which the DBM is directed by the Office of the for the delivery of services and production of goods, consistent with the government priorities.
President, thru the Executive Secretary, to source funds.
In the event that a measure is necessary to further improve the operational efficiency of the government,
6.0 Among others, the following are such proposed additional projects that have been chosen given their the President is authorized to suspend or stop further use of funds allotted for any agency or expenditure
multiplier impact on economy and infrastructure development, their beneficial effect on the poor, and their authorized in the General Appropriations Act. Withdrawal and pooling of unutilized allotment releases can
translation into disbursements. Please note that we have classified the list of proposed projects as follows: be effected by DBM based on authority of the President, as mandated under Sections 38 and 39, Chapter
5, Book VI of EO 292.
7.0 x x x
For the first five months of 2012, the National Government has not met its spending targets. In order to
accelerate spending and sustain the fiscal targets during the year, expenditure measures have to be
FOR THE PRESIDENT’S APPROVAL implemented to optimize the utilization of available resources.

8.0 Foregoing considered, may we respectfully request for the President’s approval for the following: Departments/agencies have registered low spending levels, in terms of obligations and disbursements per
initial review of their 2012 performance. To enhance agencies’ performance, the DBM conducts
8.1 Grant of omnibus authority to consolidate FY 2011 savings/unutilized balances and its realignment; continuous consultation meetings and/or send call-up letters, requesting them to identify slow-moving
and programs/projects and the factors/issues affecting their performance (both pertaining to internal systems
and those which are outside the agencies’ spheres of control). Also, they are asked to formulate
8.2 The proposed additional projects identified for funding. strategies and improvement plans for the rest of 2012.

For His Excellency’s consideration and approval. Notwithstanding these initiatives, some departments/agencies have continued to post low obligation levels
as of end of first semester, thus resulting to substantial unobligated allotments.
(Sgd.)
In line with this, the President, per directive dated June 27, 2012 authorized the withdrawal of unobligated
allotments of agencies with low levels of obligations as of June 30, 2012, both for continuing and current
[/] APPROVED
allotments. This measure will allow the maximum utilization of available allotments to fund and undertake
other priority expenditures of the national government.
[ ] DISAPPROVED
2.0 Purpose
(Sgd.) H.E. BENIGNO S. AQUINO, III

97
2.1 To provide the conditions and parameters on the withdrawal of unobligated allotments of agencies as 4.2.3 Foreign-Assisted Projects (loan proceeds and peso counterpart);
of June 30, 2012 to fund priority and/or fast-moving programs/projects of the national government;
4.2.4 Special Purpose Funds such as: E-Government Fund, International Commitments Fund, PAMANA,
2.2 To prescribe the reports and documents to be used as bases on the withdrawal of said unobligated Priority Development Assistance Fund, Calamity Fund, Budgetary Support to GOCCs and Allocation to
allotments; and LGUs, among others;

2.3 To provide guidelines in the utilization or reallocation of the withdrawn allotments. 4.2.5 Quick Response Funds; and

3.0 Coverage 4.2.6 Automatic Appropriations i.e., Retirement Life Insurance Premium and Special Accounts in the
General Fund.
3.1 These guidelines shall cover the withdrawal of unobligated allotments as of June 30, 2012 of all
national government agencies (NGAs) charged against FY 2011 Continuing Appropriation (R.A. 5.0 Guidelines
No.10147) and FY 2012 Current Appropriation (R.A. No. 10155), pertaining to:
5.1 National government agencies shall continue to undertake procurement activities notwithstanding the
3.1.1 Capital Outlays (CO); implementation of the policy of withdrawal of unobligated allotments until the end of the third quarter, FY
2012. Even without the allotments, the agency shall proceed in undertaking the procurement processes
3.1.2 Maintenance and Other Operating Expenses (MOOE) related to the implementation of programs (i.e., procurement planning up to the conduct of bidding but short of awarding of contract) pursuant to
and projects, as well as capitalized MOOE; and GPPB Circular Nos. 02-2008 and 01-2009 and DBM Circular Letter No. 2010-9.

3.1.3 Personal Services corresponding to unutilized pension benefits declared as savings by the agencies 5.2 For the purpose of determining the amount of unobligated allotments that shall be withdrawn, all
concerned based on their updated/validated list of pensioners. departments/agencies/operating units (OUs) shall submit to DBM not later than July 30, 2012, the
following budget accountability reports as of June 30, 2012;
3.2 The withdrawal of unobligated allotments may cover the identified programs, projects and activities of
the departments/agencies reflected in the DBM list shown as Annex A or specific programs and projects • Statement of Allotments, Obligations and Balances (SAOB);
as may be identified by the agencies.
• Financial Report of Operations (FRO); and
4.0 Exemption
• Physical Report of Operations.
These guidelines shall not apply to the following:
5.3 In the absence of the June 30, 2012 reports cited under item 5.2 of this Circular, the agency’s latest
4.1 NGAs report available shall be used by DBM as basis for withdrawal of allotment. The DBM shall
compute/approximate the agency’s obligation level as of June 30 to derive its unobligated allotments as of
same period. Example: If the March 31 SAOB or FRO reflects actual obligations of P 800M then the June
4.1.1 Constitutional Offices/Fiscal Autonomy Group, granted fiscal autonomy under the Philippine 30 obligation level shall approximate to ₱1,600 M (i.e., ₱800 M x 2 quarters).
Constitution; and
5.4 All released allotments in FY 2011 charged against R.A. No. 10147 which remained unobligated as of
4.1.2 State Universities and Colleges, adopting the Normative Funding allocation scheme i.e., distribution June 30, 2012 shall be immediately considered for withdrawal. This policy is based on the following
of a predetermined budget ceiling. considerations:

4.2 Fund Sources 5.4.1 The departments/agencies’ approved priority programs and projects are assumed to be
implementation-ready and doable during the given fiscal year; and
4.2.1 Personal Services other than pension benefits;
5.4.2 The practice of having substantial carryover appropriations may imply that the agency has a slower-
4.2.2 MOOE items earmarked for specific purposes or subject to realignment conditions per General than-programmed implementation capacity or agency tends to implement projects within a two-year
Provisions of the GAA: timeframe.

• Confidential and Intelligence Fund; 5.5. Consistent with the President’s directive, the DBM shall, based on evaluation of the reports cited
above and results of consultations with the departments/agencies, withdraw the unobligated allotments as
• Savings from Traveling, Communication, Transportation and Delivery, Repair and Maintenance, of June 30, 2012 through issuance of negative Special Allotment Release Orders (SAROs).
Supplies and Materials and Utility which shall be used for the grant of Collective Negotiation Agreement
incentive benefit; 5.6 DBM shall prepare and submit to the President, a report on the magnitude of withdrawn allotments.
The report shall highlight the agencies which failed to submit the June 30 reports required under this
• Savings from mandatory expenditures which can be realigned only in the last quarter after taking into Circular.
consideration the agency’s full year requirements, i.e., Petroleum, Oil and Lubricants, Water, Illumination,
Power Services, Telephone, other Communication Services and Rent. 5.7 The withdrawn allotments may be:

98
5.7.1 Reissued for the original programs and projects of the agencies/OUs concerned, from which the Taken together, all the issuances showed how the DAP was to be implemented and funded, that is — (1)
allotments were withdrawn; by declaring "savings" coming from the various departments and agencies derived from pooling
unobligated allotments and withdrawing unreleased appropriations; (2) releasing unprogrammed funds;
5.7.2 Realigned to cover additional funding for other existing programs and projects of the agency/OU; or and (3) applying the "savings" and unprogrammed funds to augment existing PAPs or to support other
priority PAPs.
5.7.3 Used to augment existing programs and projects of any agency and to fund priority programs and
projects not considered in the 2012 budget but expected to be started or implemented during the current c. DAP was not an appropriation
year. measure; hence, no appropriation
law was required to adopt or to
implement it
5.8 For items 5.7.1 and 5.7.2 above, agencies/OUs concerned may submit to DBM a Special Budget
Request (SBR), supported with the following:
Petitioners Syjuco, Luna, Villegas and PHILCONSA state that Congress did not enact a law to establish
the DAP, or to authorize the disbursement and release of public funds to implement the DAP. Villegas,
5.8.1 Physical and Financial Plan (PFP); PHILCONSA, IBP, Araullo, and COURAGE observe that the appropriations funded under the DAP were
not included in the 2011, 2012 and 2013 GAAs. To petitioners IBP, Araullo, and COURAGE, the DAP,
5.8.2 Monthly Cash Program (MCP); and being actually an appropriation that set aside public funds for public use, should require an enabling law
for its validity. VACC maintains that the DAP, because it involved huge allocations that were separate and
5.8.3 Proof that the project/activity has started the procurement processes i.e., Proof of Posting and/or distinct from the GAAs, circumvented and duplicated the GAAs without congressional authorization and
Advertisement of the Invitation to Bid. control.

5.9 The deadline for submission of request/s pertaining to these categories shall be until the end of the The petitioners contend in unison that based on how it was developed and implemented the DAP violated
third quarter i.e., September 30, 2012. After said cut-off date, the withdrawn allotments shall be pooled the mandate of Section 29(1), Article VI of the 1987 Constitution that "[n]o money shall be paid out of the
and form part of the overall savings of the national government. Treasury except in pursuance of an appropriation made by law."

5.10 Utilization of the consolidated withdrawn allotments for other priority programs and projects as cited The OSG posits, however, that no law was necessary for the adoption and implementation of the DAP
under item 5.7.3 of this Circular, shall be subject to approval of the President. Based on the approval of because of its being neither a fund nor an appropriation, but a program or an administrative system of
the President, DBM shall issue the SARO to cover the approved priority expenditures subject to prioritizing spending; and that the adoption of the DAP was by virtue of the authority of the President as
submission by the agency/OU concerned of the SBR and supported with PFP and MCP. the Chief Executive to ensure that laws were faithfully executed.

5.11 It is understood that all releases to be made out of the withdrawn allotments (both 2011 and 2012 We agree with the OSG’s position.
unobligated allotments) shall be within the approved Expenditure Program level of the national
government for the current year. The SAROs to be issued shall properly disclose the appropriation source The DAP was a government policy or strategy designed to stimulate the economy through accelerated
of the release to determine the extent of allotment validity, as follows: spending. In the context of the DAP’s adoption and implementation being a function pertaining to the
Executive as the main actor during the Budget Execution Stage under its constitutional mandate to
• For charges under R.A. 10147 – allotments shall be valid up to December 31, 2012; and faithfully execute the laws, including the GAAs, Congress did not need to legislate to adopt or to
implement the DAP. Congress could appropriate but would have nothing more to do during the Budget
Execution Stage. Indeed, appropriation was the act by which Congress "designates a particular fund, or
• For charges under R.A. 10155 – allotments shall be valid up to December 31, 2013.
sets apart a specified portion of the public revenue or of the money in the public treasury, to be applied to
some general object of governmental expenditure, or to some individual purchase or expense."124 As
5.12 Timely compliance with the submission of existing BARs and other reportorial requirements is pointed out in Gonzales v. Raquiza:125 ‘"In a strict sense, appropriation has been defined ‘as nothing
reiterated for monitoring purposes. more than the legislative authorization prescribed by the Constitution that money may be paid out of the
Treasury,’ while appropriation made by law refers to ‘the act of the legislature setting apart or assigning to
6.0 Effectivity a particular use a certain sum to be used in the payment of debt or dues from the State to its
creditors.’"126
This circular shall take effect immediately.
On the other hand, the President, in keeping with his duty to faithfully execute the laws, had sufficient
discretion during the execution of the budget to adapt the budget to changes in the country’s economic
(Sgd.) FLORENCIO B. ABAD
situation.127 He could adopt a plan like the DAP for the purpose. He could pool the savings and identify
Secretary
the PAPs to be funded under the DAP. The pooling of savings pursuant to the DAP, and the identification
of the PAPs to be funded under the DAP did not involve appropriation in the strict sense because the
As can be seen, NBC No. 541 specified that the unobligated allotments of all agencies and departments money had been already set apart from the public treasury by Congress through the GAAs. In such
as of June 30, 2012 that were charged against the continuing appropriations for fiscal year 2011 and the actions, the Executive did not usurp the power vested in Congress under Section 29(1), Article VI of the
2012 GAA (R.A. No. 10155) were subject to withdrawal through the issuance of negative SAROs, but Constitution.
such allotments could be either: (1) reissued for the original PAPs of the concerned agencies from which
they were withdrawn; or (2) realigned to cover additional funding for other existing PAPs of the concerned
3.
agencies; or (3) used to augment existing PAPs of any agency and to fund priority PAPs not considered in
Unreleased appropriations and withdrawn
the 2012 budget but expected to be started or implemented in 2012. Financing the other priority PAPs
unobligated allotments under the DAP
was made subject to the approval of the President. Note here that NBC No. 541 used terminologies like
were not savings, and the use of such
"realignment" and "augmentation" in the application of the withdrawn unobligated allotments.

99
appropriations contravened Section 25(5), The Judiciary, the Constitutional Commissions, and the Ombudsman must have the independence and
Article VI of the 1987 Constitution. flexibility needed in the discharge of their constitutional duties. The imposition of restrictions and
constraints on the manner the independent constitutional offices allocate and utilize the funds
Notwithstanding our appreciation of the DAP as a plan or strategy validly adopted by the Executive to appropriated for their operations is anathema to fiscal autonomy and violative not only of the express
ramp up spending to accelerate economic growth, the challenges posed by the petitioners constrain us to mandate of the Constitution but especially as regards the Supreme Court, of the independence and
dissect the mechanics of the actual execution of the DAP. The management and utilization of the public separation of powers upon which the entire fabric of our constitutional system is based.
wealth inevitably demands a most careful scrutiny of whether the Executive’s implementation of the DAP
was consistent with the Constitution, the relevant GAAs and other existing laws. In the case of the President, the power to transfer funds from one item to another within the Executive has
not been the mere offshoot of established usage, but has emanated from law itself. It has existed since
a. Although executive discretion the time of the American Governors-General.134 Act No. 1902 (An Act authorizing the Governor-General
and flexibility are necessary in to direct any unexpended balances of appropriations be returned to the general fund of the Insular
the execution of the budget, any Treasury and to transfer from the general fund moneys which have been returned thereto), passed on
transfer of appropriated funds May 18, 1909 by the First Philippine Legislature,135 was the first enabling law that granted statutory
should conform to Section 25(5), authority to the President to transfer funds. The authority was without any limitation, for the Act explicitly
Article VI of the Constitution empowered the Governor-General to transfer any unexpended balance of appropriations for any bureau
or office to another, and to spend such balance as if it had originally been appropriated for that bureau or
office.
We begin this dissection by reiterating that Congress cannot anticipate all issues and needs that may
come into play once the budget reaches its execution stage. Executive discretion is necessary at that
stage to achieve a sound fiscal administration and assure effective budget implementation. The heads of From 1916 until 1920, the appropriations laws set a cap on the amounts of funds that could be
offices, particularly the President, require flexibility in their operations under performance budgeting to transferred, thereby limiting the power to transfer funds. Only 10% of the amounts appropriated for
enable them to make whatever adjustments are needed to meet established work goals under changing contingent or miscellaneous expenses could be transferred to a bureau or office, and the transferred
conditions.128 In particular, the power to transfer funds can give the President the flexibility to meet funds were to be used to cover deficiencies in the appropriations also for miscellaneous expenses of said
unforeseen events that may otherwise impede the efficient implementation of the PAPs set by Congress bureau or office.
in the GAA.
In 1921, the ceiling on the amounts of funds to be transferred from items under miscellaneous expenses
Congress has traditionally allowed much flexibility to the President in allocating funds pursuant to the to any other item of a certain bureau or office was removed.
GAAs,129 particularly when the funds are grouped to form lump sum accounts.130 It is assumed that the
agencies of the Government enjoy more flexibility when the GAAs provide broader appropriation During the Commonwealth period, the power of the President to transfer funds continued to be governed
items.131 This flexibility comes in the form of policies that the Executive may adopt during the budget by the GAAs despite the enactment of the Constitution in 1935. It is notable that the 1935 Constitution did
execution phase. The DAP – as a strategy to improve the country’s economic position – was one policy not include a provision on the power to transfer funds. At any rate, a shift in the extent of the President’s
that the President decided to carry out in order to fulfill his mandate under the GAAs. power to transfer funds was again experienced during this era, with the President being given more
flexibility in implementing the budget. The GAAs provided that the power to transfer all or portions of the
Denying to the Executive flexibility in the expenditure process would be counterproductive. In Presidential appropriations in the Executive Department could be made in the "interest of the public, as the President
Spending Power,132 Prof. Louis Fisher, an American constitutional scholar whose specialties have may determine."136
included budget policy, has justified extending discretionary authority to the Executive thusly:
In its time, the 1971 Constitutional Convention wanted to curtail the President’s seemingly unbounded
[T]he impulse to deny discretionary authority altogether should be resisted. There are many number of discretion in transferring funds.137 Its Committee on the Budget and Appropriation proposed to prohibit
reasons why obligations and outlays by administrators may have to differ from appropriations by the transfer of funds among the separate branches of the Government and the independent constitutional
legislators. Appropriations are made many months, and sometimes years, in advance of expenditures. bodies, but to allow instead their respective heads to augment items of appropriations from savings in
Congress acts with imperfect knowledge in trying to legislate in fields that are highly technical and their respective budgets under certain limitations.138 The clear intention of the Convention was to further
constantly undergoing change. New circumstances will develop to make obsolete and mistaken the restrict, not to liberalize, the power to transfer appropriations.139 Thus, the Committee on the Budget and
decisions reached by Congress at the appropriation stage. It is not practicable for Congress to adjust to Appropriation initially considered setting stringent limitations on the power to augment, and suggested that
each new development by passing separate supplemental appropriation bills. Were Congress to control the augmentation of an item of appropriation could be made "by not more than ten percent if the original
expenditures by confining administrators to narrow statutory details, it would perhaps protect its power of item of appropriation to be augmented does not exceed one million pesos, or by not more than five
the purse but it would not protect the purse itself. The realities and complexities of public policy require percent if the original item of appropriation to be augmented exceeds one million pesos."140 But two
executive discretion for the sound management of public funds. members of the Committee objected to the ₱1,000,000.00 threshold, saying that the amount was arbitrary
and might not be reasonable in the future. The Committee agreed to eliminate the ₱1,000,000.00
threshold, and settled on the ten percent limitation.141
xxxx
In the end, the ten percent limitation was discarded during the plenary of the Convention, which adopted
x x x The expenditure process, by its very nature, requires substantial discretion for administrators. They the following final version under Section 16, Article VIII of the 1973 Constitution, to wit:
need to exercise judgment and take responsibility for their actions, but those actions ought to be directed
toward executing congressional, not administrative policy. Let there be discretion, but channel it and use it
to satisfy the programs and priorities established by Congress. (5) No law shall be passed authorizing any transfer of appropriations; however, the President, the Prime
Minister, the Speaker, the Chief Justice of the Supreme Court, and the heads of Constitutional
Commissions may by law be authorized to augment any item in the general appropriations law for their
In contrast, by allowing to the heads of offices some power to transfer funds within their respective offices, respective offices from savings in other items of their respective appropriations.
the Constitution itself ensures the fiscal autonomy of their offices, and at the same time maintains the
separation of powers among the three main branches of the Government. The Court has recognized this,
and emphasized so in Bengzon v. Drilon,133 viz: The 1973 Constitution explicitly and categorically prohibited the transfer of funds from one item to another,
unless Congress enacted a law authorizing the President, the Prime Minister, the Speaker, the Chief

100
Justice of the Supreme Court, and the heads of the Constitutional omissions to transfer funds for the offices from the savings in other items of their respective appropriations. The plain language of the
purpose of augmenting any item from savings in another item in the GAA of their respective offices. The constitutional restriction leaves no room for the petitioner’s posture, which we should now dispose of as
leeway was limited to augmentation only, and was further constricted by the condition that the funds to be untenable.
transferred should come from savings from another item in the appropriation of the office.142
It bears emphasizing that the exception in favor of the high officials named in Section 25(5), Article VI of
On July 30, 1977, President Marcos issued PD No. 1177, providing in its Section 44 that: the Constitution limiting the authority to transfer savings only to augment another item in the GAA is
strictly but reasonably construed as exclusive. As the Court has expounded in Lokin, Jr. v. Commission on
Section 44. Authority to Approve Fund Transfers. The President shall have the authority to transfer any Elections:
fund appropriated for the different departments, bureaus, offices and agencies of the Executive
Department which are included in the General Appropriations Act, to any program, project, or activity of When the statute itself enumerates the exceptions to the application of the general rule, the exceptions
any department, bureau or office included in the General Appropriations Act or approved after its are strictly but reasonably construed. The exceptions extend only as far as their language fairly warrants,
enactment. and all doubts should be resolved in favor of the general provision rather than the exceptions. Where the
general rule is established by a statute with exceptions, none but the enacting authority can curtail the
The President shall, likewise, have the authority to augment any appropriation of the Executive former. Not even the courts may add to the latter by implication, and it is a rule that an express exception
Department in the General Appropriations Act, from savings in the appropriations of another department, excludes all others, although it is always proper in determining the applicability of the rule to inquire
bureau, office or agency within the Executive Branch, pursuant to the provisions of Article VIII, Section 16 whether, in a particular case, it accords with reason and justice.
(5) of the Constitution.
The appropriate and natural office of the exception is to exempt something from the scope of the general
In Demetria v. Alba, however, the Court struck down the first paragraph of Section 44 for contravening words of a statute, which is otherwise within the scope and meaning of such general words.
Section 16(5)of the 1973 Constitution, ruling: Consequently, the existence of an exception in a statute clarifies the intent that the statute shall apply to
all cases not excepted. Exceptions are subject to the rule of strict construction; hence, any doubt will be
resolved in favor of the general provision and against the exception. Indeed, the liberal construction of a
Paragraph 1 of Section 44 of P.D. No. 1177 unduly over-extends the privilege granted under said Section statute will seem to require in many circumstances that the exception, by which the operation of the
16. It empowers the President to indiscriminately transfer funds from one department, bureau, office or statute is limited or abridged, should receive a restricted construction.
agency of the Executive Department to any program, project or activity of any department, bureau or
office included in the General Appropriations Act or approved after its enactment, without regard as to
whether or not the funds to be transferred are actually savings in the item from which the same are to be Accordingly, we should interpret Section 25(5), supra, in the context of a limitation on the President’s
taken, or whether or not the transfer is for the purpose of augmenting the item to which said transfer is to discretion over the appropriations during the Budget Execution Phase.
be made. It does not only completely disregard the standards set in the fundamental law, thereby
amounting to an undue delegation of legislative powers, but likewise goes beyond the tenor thereof. b. Requisites for the valid transfer of
Indeed, such constitutional infirmities render the provision in question null and void.143 appropriated funds under Section
25(5), Article VI of the 1987
It is significant that Demetria was promulgated 25 days after the ratification by the people of the 1987 Constitution
Constitution, whose Section 25(5) of Article VI is identical to Section 16(5), Article VIII of the 1973
Constitution, to wit: The transfer of appropriated funds, to be valid under Section 25(5), supra, must be made upon a
concurrence of the following requisites, namely:
Section 25. x x x
(1) There is a law authorizing the President, the President of the Senate, the Speaker of the House of
xxxx Representatives, the Chief Justice of the Supreme Court, and the heads of the Constitutional
Commissions to transfer funds within their respective offices;
5) No law shall be passed authorizing any transfer of appropriations; however, the President, the
President of the Senate, the Speaker of the House of Representatives, the Chief Justice of the Supreme (2) The funds to be transferred are savings generated from the appropriations for their respective offices;
Court, and the heads of Constitutional Commissions may, by law, be authorized to augment any item in and (3) The purpose of the transfer is to augment an item in the general appropriations law for their
the general appropriations law for their respective offices from savings in other items of their respective respective offices.
appropriations.
b.1. First Requisite–GAAs of 2011 and
xxxx 2012 lacked valid provisions to
authorize transfers of funds under
the DAP; hence, transfers under the
The foregoing history makes it evident that the Constitutional Commission included Section 25(5), supra, DAP were unconstitutional
to keep a tight rein on the exercise of the power to transfer funds appropriated by Congress by the
President and the other high officials of the Government named therein. The Court stated in Nazareth v.
Villar:144 Section 25(5), supra, not being a self-executing provision of the Constitution, must have an implementing
law for it to be operative. That law, generally, is the GAA of a given fiscal year. To comply with the first
requisite, the GAAs should expressly authorize the transfer of funds.
In the funding of current activities, projects, and programs, the general rule should still be that the
budgetary amount contained in the appropriations bill is the extent Congress will determine as sufficient
for the budgetary allocation for the proponent agency. The only exception is found in Section 25 (5), Did the GAAs expressly authorize the transfer of funds?
Article VI of the Constitution, by which the President, the President of the Senate, the Speaker of the
House of Representatives, the Chief Justice of the Supreme Court, and the heads of Constitutional In the 2011 GAA, the provision that gave the President and the other high officials the authority to transfer
Commissions are authorized to transfer appropriations to augmentany item in the GAA for their respective funds was Section 59, as follows:

101
Section 59. Use of Savings. The President of the Philippines, the Senate President, the Speaker of the The OSG represents that "savings" were "appropriations balances," being the difference between the
House of Representatives, the Chief Justice of the Supreme Court, the Heads of Constitutional appropriation authorized by Congress and the actual amount allotted for the appropriation; that the
Commissions enjoying fiscal autonomy, and the Ombudsman are hereby authorized to augment any item definition of "savings" in the GAAs set only the parameters for determining when savings occurred; that it
in this Act from savings in other items of their respective appropriations. was still the President (as well as the other officers vested by the Constitution with the authority to
augment) who ultimately determined when savings actually existed because savings could be determined
In the 2012 GAA, the empowering provision was Section 53, to wit: only during the stage of budget execution; that the President must be given a wide discretion to
accomplish his tasks; and that the withdrawn unobligated allotments were savings inasmuch as they were
clearly "portions or balances of any programmed appropriation…free from any obligation or
Section 53. Use of Savings. The President of the Philippines, the Senate President, the Speaker of the encumbrances which are (i) still available after the completion or final discontinuance or abandonment of
House of Representatives, the Chief Justice of the Supreme Court, the Heads of Constitutional the work, activity or purpose for which the appropriation is authorized…"
Commissions enjoying fiscal autonomy, and the Ombudsman are hereby authorized to augment any item
in this Act from savings in other items of their respective appropriations.
We partially find for the petitioners.
In fact, the foregoing provisions of the 2011 and 2012 GAAs were cited by the DBM as justification for the
use of savings under the DAP.145 In ascertaining the meaning of savings, certain principles should be borne in mind. The first principle is
that Congress wields the power of the purse. Congress decides how the budget will be spent; what PAPs
to fund; and the amounts of money to be spent for each PAP. The second principle is that the Executive,
A reading shows, however, that the aforequoted provisions of the GAAs of 2011 and 2012 were textually as the department of the Government tasked to enforce the laws, is expected to faithfully execute the
unfaithful to the Constitution for not carrying the phrase "for their respective offices" contained in Section GAA and to spend the budget in accordance with the provisions of the GAA.149 The Executive is
25(5), supra. The impact of the phrase "for their respective offices" was to authorize only transfers of expected to faithfully implement the PAPs for which Congress allocated funds, and to limit the
funds within their offices (i.e., in the case of the President, the transfer was to an item of appropriation expenditures within the allocations, unless exigencies result to deficiencies for which augmentation is
within the Executive). The provisions carried a different phrase ("to augment any item in this Act"), and the authorized, subject to the conditions provided by law. The third principle is that in making the President’s
effect was that the 2011 and 2012 GAAs thereby literally allowed the transfer of funds from savings to power to augment operative under the GAA, Congress recognizes the need for flexibility in budget
augment any item in the GAAs even if the item belonged to an office outside the Executive. To that extent execution. In so doing, Congress diminishes its own power of the purse, for it delegates a fraction of its
did the 2011 and 2012 GAAs contravene the Constitution. At the very least, the aforequoted provisions power to the Executive. But Congress does not thereby allow the Executive to override its authority over
cannot be used to claim authority to transfer appropriations from the Executive to another branch, or to a the purse as to let the Executive exceed its delegated authority. And the fourth principle is that savings
constitutional commission. should be actual. "Actual" denotes something that is real or substantial, or something that exists presently
in fact, as opposed to something that is merely theoretical, possible, potential or hypothetical.150
Apparently realizing the problem, Congress inserted the omitted phrase in the counterpart provision in the
2013 GAA, to wit: The foregoing principles caution us to construe savings strictly against expanding the scope of the power
to augment. It is then indubitable that the power to augment was to be used only when the purpose for
Section 52. Use of Savings. The President of the Philippines, the Senate President, the Speaker of the which the funds had been allocated were already satisfied, or the need for such funds had ceased to
House of Representatives, the Chief Justice of the Supreme Court, the Heads of Constitutional exist, for only then could savings be properly realized. This interpretation prevents the Executive from
Commissions enjoying fiscal autonomy, and the Ombudsman are hereby authorized to use savings in unduly transgressing Congress’ power of the purse.
their respective appropriations to augment actual deficiencies incurred for the current year in any item of
their respective appropriations. The definition of "savings" in the GAAs, particularly for 2011, 2012 and 2013, reflected this interpretation
and made it operational, viz:
Even had a valid law authorizing the transfer of funds pursuant to Section 25(5), supra, existed, there still
remained two other requisites to be met, namely: that the source of funds to be transferred were savings Savings refer to portions or balances of any programmed appropriation in this Act free from any obligation
from appropriations within the respective offices; and that the transfer must be for the purpose of or encumbrance which are: (i) still available after the completion or final discontinuance or abandonment
augmenting an item of appropriation within the respective offices. of the work, activity or purpose for which the appropriation is authorized; (ii) from appropriations balances
arising from unpaid compensation and related costs pertaining to vacant positions and leaves of absence
b.2. Second Requisite – There were without pay; and (iii) from appropriations balances realized from the implementation of measures resulting
no savings from which funds in improved systems and efficiencies and thus enabled agencies to meet and deliver the required or
could be sourced for the DAP planned targets, programs and services approved in this Act at a lesser cost.
Were the funds used in the DAP actually savings?
The three instances listed in the GAAs’ aforequoted definition were a sure indication that savings could be
The petitioners claim that the funds used in the DAP — the unreleased appropriations and withdrawn generated only upon the purpose of the appropriation being fulfilled, or upon the need for the
unobligated allotments — were not actual savings within the context of Section 25(5), supra, and the appropriation being no longer existent.
relevant provisions of the GAAs. Belgica argues that "savings" should be understood to refer to the
excess money after the items that needed to be funded have been funded, or those that needed to be The phrase "free from any obligation or encumbrance" in the definition of savings in the GAAs conveyed
paid have been paid pursuant to the budget.146 The petitioners posit that there could be savings only the notion that the appropriation was at that stage when the appropriation was already obligated and the
when the PAPs for which the funds had been appropriated were actually implemented and completed, or appropriation was already released. This interpretation was reinforced by the enumeration of the three
finally discontinued or abandoned. They insist that savings could not be realized with certainty in the instances for savings to arise, which showed that the appropriation referred to had reached the agency
middle of the fiscal year; and that the funds for "slow-moving" PAPs could not be considered as savings level. It could not be otherwise, considering that only when the appropriation had reached the agency
because such PAPs had not actually been abandoned or discontinued yet.147 They stress that NBC No. level could it be determined whether (a) the PAP for which the appropriation had been authorized was
541, by allowing the withdrawn funds to be reissued to the "original program or project from which it was completed, finally discontinued, or abandoned; or (b) there were vacant positions and leaves of absence
withdrawn," conceded that the PAPs from which the supposed savings were taken had not been without pay; or (c) the required or planned targets, programs and services were realized at a lesser cost
completed, abandoned or discontinued.148 because of the implementation of measures resulting in improved systems and efficiencies.

102
The DBM declares that part of the savings brought under the DAP came from "pooling of unreleased 7.0 Despite said reminders and the availability of funds at the department’s disposal, the level of financial
appropriations such as unreleased Personnel Services appropriations which will lapse at the end of the performance of some departments registered below program, with the targeted obligations/disbursements
year, unreleased appropriations of slow moving projects and discontinued projects per Zero-Based for the first semester still not being met.
Budgeting findings."
8.0 In order to maximize the use of the available allotment, all unobligated balances as of June 30, 2012,
The declaration of the DBM by itself does not state the clear legal basis for the treatment of unreleased or both for continuing and current allotments shall be withdrawn and pooled to fund fast moving
unalloted appropriations as savings. programs/projects.

The fact alone that the appropriations are unreleased or unalloted is a mere description of the status of 9.0 It may be emphasized that the allotments to be withdrawn will be based on the list of slow moving
the items as unalloted or unreleased. They have not yet ripened into categories of items from which projects to be identified by the agencies and their catch up plans to be evaluated by the DBM.
savings can be generated. Appropriations have been considered "released" if there has already been an
allotment or authorization to incur obligations and disbursement authority. This means that the DBM has It is apparent from the foregoing text that the withdrawal of unobligated allotments would be based on
issued either an ABM (for those not needing clearance), or a SARO (for those needing clearance), and whether the allotments pertained to slow-moving projects, or not. However, NBC No. 541 did not set in
consequently an NCA, NCAA or CDC, as the case may be. Appropriations remain unreleased, for clear terms the criteria for the withdrawal of unobligated allotments, viz:
instance, because of noncompliance with documentary requirements (like the Special Budget Request),
or simply because of the unavailability of funds. But the appropriations do not actually reach the agencies
to which they were allocated under the GAAs, and have remained with the DBM technically speaking. 3.1. These guidelines shall cover the withdrawal of unobligated allotments as of June 30, 2012 ofall
Ergo, unreleased appropriations refer to appropriations with allotments but without disbursement national government agencies (NGAs) charged against FY 2011 Continuing Appropriation (R.A. No.
authority. 10147) and FY 2012 Current Appropriation (R.A. No. 10155), pertaining to:

For us to consider unreleased appropriations as savings, unless these met the statutory definition of 3.1.1 Capital Outlays (CO);
savings, would seriously undercut the congressional power of the purse, because such appropriations had
not even reached and been used by the agency concerned vis-à-vis the PAPs for which Congress had 3.1.2 Maintenance and Other Operating Expenses (MOOE) related to the implementation of programs
allocated them. However, if an agency has unfilled positions in its plantilla and did not receive an and projects, as well as capitalized MOOE; and
allotment and NCA for such vacancies, appropriations for such positions, although unreleased, may
already constitute savings for that agency under the second instance. 3.1.3 Personal Services corresponding to unutilized pension benefits declared as savings by the agencies
concerned based on their undated/validated list of pensioners.
Unobligated allotments, on the other hand, were encompassed by the first part of the definition of
"savings" in the GAA, that is, as "portions or balances of any programmed appropriation in this Act free A perusal of its various provisions reveals that NBC No. 541 targeted the "withdrawal of unobligated
from any obligation or encumbrance." But the first part of the definition was further qualified by the three allotments of agencies with low levels of obligations"151 "to fund priority and/or fast-moving
enumerated instances of when savings would be realized. As such, unobligated allotments could not be programs/projects."152 But the fact that the withdrawn allotments could be "[r]eissued for the original
indiscriminately declared as savings without first determining whether any of the three instances existed. programs and projects of the agencies/OUs concerned, from which the allotments were
This signified that the DBM’s withdrawal of unobligated allotments had disregarded the definition of withdrawn"153 supported the conclusion that the PAPs had not yet been finally discontinued or
savings under the GAAs. abandoned. Thus, the purpose for which the withdrawn funds had been appropriated was not yet fulfilled,
or did not yet cease to exist, rendering the declaration of the funds as savings impossible.
Justice Carpio has validly observed in his Separate Concurring Opinion that MOOE appropriations are
deemed divided into twelve monthly allocations within the fiscal year; hence, savings could be generated Worse, NBC No. 541 immediately considered for withdrawal all released allotments in 2011 charged
monthly from the excess or unused MOOE appropriations other than the Mandatory Expenditures and against the 2011 GAA that had remained unobligated based on the following considerations, to wit:
Expenditures for Business-type Activities because of the physical impossibility to obligate and spend such
funds as MOOE for a period that already lapsed. Following this observation, MOOE for future months are
5.4.1 The departments/agencies’ approved priority programs and projects are assumed to be
not savings and cannot be transferred.
implementation-ready and doable during the given fiscal year; and

The DBM’s Memorandum for the President dated June 25, 2012 (which became the basis of NBC No.
5.4.2 The practice of having substantial carryover appropriations may imply that the agency has a slower-
541) stated:
than-programmed implementation capacity or agency tends to implement projects within a two-year
timeframe.
ON THE AUTHORITY TO WITHDRAW UNOBLIGATED ALLOTMENTS
Such withdrawals pursuant to NBC No. 541, the circular that affected the unobligated allotments for
5.0 The DBM, during the course of performance reviews conducted on the agencies’ operations, continuing and current appropriations as of June 30, 2012, disregarded the 2-year period of availability of
particularly on the implementation of their projects/activities, including expenses incurred in undertaking the appropriations for MOOE and capital outlay extended under Section 65, General Provisions of the
the same, have been continuously calling the attention of all National Government agencies (NGAs) with 2011 GAA, viz:
low levels of obligations as of end of the first quarter to speedup the implementation of their programs and
projects in the second quarter.
Section 65. Availability of Appropriations. — Appropriations for MOOE and capital outlays authorized in
this Act shall be available for release and obligation for the purpose specified, and under the same special
6.0 Said reminders were made in a series of consultation meetings with the concerned agencies and with provisions applicable thereto, for a period extending to one fiscal year after the end of the year in which
call-up letters sent. such items were appropriated: PROVIDED, That appropriations for MOOE and capital outlays under R.A.
No. 9970 shall be made available up to the end of FY 2011: PROVIDED, FURTHER, That a report on
these releases and obligations shall be submitted to the Senate Committee on Finance and the House
Committee on Appropriations.

103
and Section 63 General Provisions of the 2012 GAA, viz: • Statement of Allotments, Obligation and Balances (SAOB);

Section 63. Availability of Appropriations. — Appropriations for MOOE and capital outlays authorized in • Financial Report of Operations (FRO); and
this Act shall be available for release and obligation for the purpose specified, and under the same special
provisions applicable thereto, for a period extending to one fiscal year after the end of the year in which • Physical Report of Operations.
such items were appropriated: PROVIDED, That a report on these releases and obligations shall be
submitted to the Senate Committee on Finance and the House Committee on Appropriations, either in
printed form or by way of electronic document.154 5.3 In the absence of the June 30, 2012 reports cited under item 5.2 of this Circular, the agency’s latest
report available shall be used by DBM as basis for withdrawal of allotment. The DBM shall
compute/approximate the agency’s obligation level as of June 30 to derive its unobligated allotments as of
Thus, another alleged area of constitutional infirmity was that the DAP and its relevant issuances same period. Example: If the March 31 SAOB or FRO reflects actual obligations of P 800M then the June
shortened the period of availability of the appropriations for MOOE and capital outlays. 30 obligation level shall approximate to ₱1,600 M (i.e., ₱800 M x 2 quarters).

Congress provided a one-year period of availability of the funds for all allotment classes in the 2013 GAA The petitioners assert that no law had authorized the withdrawal and transfer of unobligated allotments
(R.A. No. 10352), to wit: and the pooling of unreleased appropriations; and that the unbridled withdrawal of unobligated allotments
and the retention of appropriated funds were akin to the impoundment of appropriations that could be
Section 63. Availability of Appropriations.— All appropriations authorized in this Act shall be available for allowed only in case of "unmanageable national government budget deficit" under the GAAs,157 thus
release and obligation for the purposes specified, and under the same special provisions applicable violating the provisions of the GAAs of 2011, 2012 and 2013 prohibiting the retention or deduction of
thereto, until the end of FY 2013: PROVIDED, That a report on these releases and obligations shall be allotments.158
submitted to the Senate Committee on Finance and House Committee on Appropriations, either in printed
form or by way of electronic document. In contrast, the respondents emphasize that NBC No. 541 adopted a spending, not saving, policy as a
last-ditch effort of the Executive to push agencies into actually spending their appropriations; that such
Yet, in his memorandum for the President dated May 20, 2013, Sec. Abad sought omnibus authority to policy did not amount to an impoundment scheme, because impoundment referred to the decision of the
consolidate savings and unutilized balances to fund the DAP on a quarterly basis, viz: Executive to refuse to spend funds for political or ideological reasons; and that the withdrawal of
allotments under NBC No. 541 was made pursuant to Section 38, Chapter 5, Book VI of the
7.0 If the level of financial performance of some department will register below program, even with the Administrative Code, by which the President was granted the authority to suspend or otherwise stop
availability of funds at their disposal, the targeted obligations/disbursements for each quarter will not be further expenditure of funds allotted to any agency whenever in his judgment the public interest so
met. It is important to note that these funds will lapse at the end of the fiscal year if these remain required.
unobligated.
The assertions of the petitioners are upheld. The withdrawal and transfer of unobligated allotments and
8.0 To maximize the use of the available allotment, all unobligated balances at the end of every quarter, the pooling of unreleased appropriations were invalid for being bereft of legal support. Nonetheless, such
both for continuing and current allotments shall be withdrawn and pooled to fund fast moving withdrawal of unobligated allotments and the retention of appropriated funds cannot be considered as
programs/projects. impoundment.

9.0 It may be emphasized that the allotments to be withdrawn will be based on the list of slow moving According to Philippine Constitution Association v. Enriquez:159 "Impoundment refers to a refusal by the
projects to be identified by the agencies and their catch up plans to be evaluated by the DBM. President, for whatever reason, to spend funds made available by Congress. It is the failure to spend or
obligate budget authority of any type." Impoundment under the GAA is understood to mean the retention
or deduction of appropriations. The 2011 GAA authorized impoundment only in case of unmanageable
The validity period of the affected appropriations, already given the brief Lifes pan of one year, was further National Government budget deficit, to wit:
shortened to only a quarter of a year under the DBM’s memorandum dated May 20, 2013.
Section 66. Prohibition Against Impoundment of Appropriations. No appropriations authorized under this
The petitioners accuse the respondents of forcing the generation of savings in order to have a larger fund Act shall be impounded through retention or deduction, unless in accordance with the rules and
available for discretionary spending. They aver that the respondents, by withdrawing unobligated regulations to be issued by the DBM: PROVIDED, That all the funds appropriated for the purposes,
allotments in the middle of the fiscal year, in effect deprived funding for PAPs with existing appropriations programs, projects and activities authorized under this Act, except those covered under the
under the GAAs.155 Unprogrammed Fund, shall be released pursuant to Section 33 (3), Chapter 5, Book VI of E.O. No. 292.

The respondents belie the accusation, insisting that the unobligated allotments were being withdrawn Section 67. Unmanageable National Government Budget Deficit. Retention or deduction of appropriations
upon the instance of the implementing agencies based on their own assessment that they could not authorized in this Act shall be effected only in cases where there is an unmanageable national
obligate those allotments pursuant to the President’s directive for them to spend their appropriations as government budget deficit.
quickly as they could in order to ramp up the economy.156
Unmanageable national government budget deficit as used in this section shall be construed to mean that
We agree with the petitioners. (i) the actual national government budget deficit has exceeded the quarterly budget deficit targets
consistent with the full-year target deficit as indicated in the FY 2011 Budget of
Contrary to the respondents’ insistence, the withdrawals were upon the initiative of the DBM itself. The
text of NBC No. 541 bears this out, to wit: Expenditures and Sources of Financing submitted by the President and approved by Congress pursuant
to Section 22, Article VII of the Constitution, or (ii) there are clear economic indications of an impending
5.2 For the purpose of determining the amount of unobligated allotments that shall be withdrawn, all occurrence of such condition, as determined by the Development Budget Coordinating Committee and
departments/agencies/operating units (OUs) shall submit to DBM not later than July 30, 2012, the approved by the President.
following budget accountability reports as of June 30, 2012;

104
The 2012 and 2013 GAAs contained similar provisions. The third requisite for a valid transfer of funds is that the purpose of the transfer should be "to augment an
item in the general appropriations law for the respective offices." The term "augment" means to enlarge or
The withdrawal of unobligated allotments under the DAP should not be regarded as impoundment increase in size, amount, or degree.160
because it entailed only the transfer of funds, not the retention or deduction of appropriations.
The GAAs for 2011, 2012 and 2013 set as a condition for augmentation that the appropriation for the PAP
Nor could Section 68 of the 2011 GAA (and the similar provisions of the 2012 and 2013 GAAs) be item to be augmented must be deficient, to wit: –
applicable. They uniformly stated:
x x x Augmentation implies the existence in this Act of a program, activity, or project with an appropriation,
Section 68. Prohibition Against Retention/Deduction of Allotment. Fund releases from appropriations which upon implementation, or subsequent evaluation of needed resources, is determined to be deficient.
provided in this Act shall be transmitted intact or in full to the office or agency concerned. No retention or In no case shall a non-existent program, activity, or project, be funded by augmentation from savings or
deduction as reserves or overhead shall be made, except as authorized by law, or upon direction of the by the use of appropriations otherwise authorized in this Act.
President of the Philippines. The COA shall ensure compliance with this provision to the extent that sub-
allotments by agencies to their subordinate offices are in conformity with the release documents issued by In other words, an appropriation for any PAP must first be determined to be deficient before it could be
the DBM. augmented from savings. Note is taken of the fact that the 2013 GAA already made this quite clear, thus:

The provision obviously pertained to the retention or deduction of allotments upon their release from the Section 52. Use of Savings. The President of the Philippines, the Senate President, the Speaker of the
DBM, which was a different matter altogether. The Court should not expand the meaning of the provision House of Representatives, the Chief Justice of the Supreme Court, the Heads of Constitutional
by applying it to the withdrawal of allotments. Commissions enjoying fiscal autonomy, and the Ombudsman are hereby authorized to use savings in
their respective appropriations to augment actual deficiencies incurred for the current year in any item of
The respondents rely on Section 38, Chapter 5, Book VI of the Administrative Code of 1987 to justify the their respective appropriations.
withdrawal of unobligated allotments. But the provision authorized only the suspension or stoppage of
further expenditures, not the withdrawal of unobligated allotments, to wit: As of 2013, a total of ₱144.4 billion worth of PAPs were implemented through the DAP.161

Section 38. Suspension of Expenditure of Appropriations.- Except as otherwise provided in the General Of this amount ₱82.5 billion were released in 2011 and ₱54.8 billion in 2012.162 Sec. Abad has reported
Appropriations Act and whenever in his judgment the public interest so requires, the President, upon that 9% of the total DAP releases were applied to the PAPs identified by the legislators.163
notice to the head of office concerned, is authorized to suspend or otherwise stop further expenditure of
funds allotted for any agency, or any other expenditure authorized in the General Appropriations Act, The petitioners disagree, however, and insist that the DAP supported the following PAPs that had not
except for personal services appropriations used for permanent officials and employees. been covered with appropriations in the respective GAAs, namely:

Moreover, the DBM did not suspend or stop further expenditures in accordance with Section 38, supra, (i) ₱1.5 billion for the Cordillera People’s Liberation Army;
but instead transferred the funds to other PAPs.
(ii) ₱1.8 billion for the Moro National Liberation Front;
It is relevant to remind at this juncture that the balances of appropriations that remained unexpended at
the end of the fiscal year were to be reverted to the General Fund.1âwphi1 This was the mandate of
Section 28, Chapter IV, Book VI of the Administrative Code, to wit: (iii) ₱700 million for assistance to Quezon Province;164

Section 28. Reversion of Unexpended Balances of Appropriations, Continuing Appropriations.- (iv) ₱50 million to ₱100 (million) each to certain senators;165
Unexpended balances of appropriations authorized in the General Appropriation Act shall revert to the
unappropriated surplus of the General Fund at the end of the fiscal year and shall not thereafter be (v) ₱10 billion for the relocation of families living along dangerous zones under the National Housing
available for expenditure except by subsequent legislative enactment: Provided, that appropriations for Authority;
capital outlays shall remain valid until fully spent or reverted: provided, further, that continuing
appropriations for current operating expenditures may be specifically recommended and approved as (vi) ₱10 billion and ₱20 billion equity infusion under the Bangko Sentral;
such in support of projects whose effective implementation calls for multi-year expenditure commitments:
provided, finally, that the President may authorize the use of savings realized by an agency during given
(vii) ₱5.4 billion landowners’ compensation under the Department of Agrarian Reform;
year to meet non-recurring expenditures in a subsequent year.

(viii) ₱8.6 billion for the ARMM comprehensive peace and development program;
The balances of continuing appropriations shall be reviewed as part of the annual budget preparation
process and the preparation process and the President may approve upon recommendation of the
Secretary, the reversion of funds no longer needed in connection with the activities funded by said (ix) ₱6.5 billion augmentation of LGU internal revenue allotments
continuing appropriations.
(x) ₱5 billion for crucial projects like tourism road construction under the Department of Tourism and the
The Executive could not circumvent this provision by declaring unreleased appropriations and unobligated Department of Public Works and Highways;
allotments as savings prior to the end of the fiscal year.
(xi) ₱1.8 billion for the DAR-DPWH Tulay ng Pangulo;
b.3. Third Requisite – No funds from
savings could be transferred under (xii) ₱1.96 billion for the DOH-DPWH rehabilitation of regional health units; and
the DAP to augment deficient items
not provided in the GAA

105
(xiii) ₱4 billion for the DepEd-PPP school infrastructure projects.166 a. Generation of new
knowledge and
In refutation, the OSG argues that a total of 116 DAP-financed PAPs were implemented, had technologies and
appropriation covers, and could properly be accounted for because the funds were released following and research
pursuant to the standard practices adopted by the DBM.167 In support of its argument, the OSG has capability building in
submitted seven evidence packets containing memoranda, SAROs, and other pertinent documents priority areas identified as
relative to the implementation and fund transfers under the DAP.168 strategic to National
Development 537,910,000 537,910,000
Upon careful review of the documents contained in the seven evidence packets, we conclude that the
"savings" pooled under the DAP were allocated to PAPs that were not covered by any appropriations in
Aside from this transfer under the DAP to the DREAM project exceeding by almost 300% the
the pertinent GAAs.
appropriation by Congress for the program Generation of new knowledge and technologies and research
capability building in priority areas identified as strategic to National Development, the Executive allotted
For example, the SARO issued on December 22, 2011 for the highly vaunted Disaster Risk, Exposure, funds for personnel services and capital outlays. The Executive thereby substituted its will to that of
Assessment and Mitigation (DREAM) project under the Department of Science and Technology (DOST) Congress. Worse, the Executive had not earlier proposed any amount for personnel services and capital
covered the amount of ₱1.6 Billion,169 broken down as follows: outlays in the NEP that became the basis of the 2011 GAA.170

APPROPRIATION PARTICULARS AMOUNT It is worth stressing in this connection that the failure of the GAAs to set aside any amounts for an
CODE AUTHORIZED expense category sufficiently indicated that Congress purposely did not see fit to fund, much less
implement, the PAP concerned. This indication becomes clearer when even the President himself did not
recommend in the NEP to fund the PAP. The consequence was that any PAP requiring expenditure that
did not receive any appropriation under the GAAs could only be a new PAP, any funding for which would
A.03.a.01.a Generation of new knowledge and technologies and go beyond the authority laid down by Congress in enacting the GAAs. That happened in some instances
research capability building in priority areas under the DAP.
identified as strategic to National Development
Personnel Services
Maintenance and Other Operating Expenses P 43,504,024 In relation to the December 22, 2011 SARO issued to the Philippine Council for Industry, Energy and
Capital Outlays 1,164,517,589 Emerging Technology Research and Development (DOST-PCIEETRD)171 for Establishment of the
391,978,387 Advanced Failure Analysis Laboratory, which reads:
P 1,600,000,000
APPROPRIATION PARTICULARS AMOUNT
CODE AUTHORIZ

the pertinent provision of the 2011 GAA (R.A. No. 10147) showed that Congress had appropriated only
₱537,910,000 for MOOE, but nothing for personnel services and capital outlays, to wit: Development, integration and coordination of the National Research System for Industry,
A.02.a Energy and Emerging Technology and Related Fields
Personnel Maintenance Capital TOTAL Capital Outlays P 300,000,000
Services and Other Outlays
Operating the appropriation code and the particulars appearing in the SARO did not correspond to the program
Expenditures specified in the GAA, whose particulars were Research and Management Services(inclusive of the
following activities: (1) Technological and Economic Assessment for Industry, Energy and Utilities; (2)
Dissemination of Science and Technology Information; and (3) Management of PCIERD Information
Operations System for Industry, Energy and Utilities. Even assuming that Development, integration and coordination
of the National Research System for Industry, Energy and Emerging Technology and Related Fields– the
particulars stated in the SARO – could fall under the broad program description of Research and
Funding Assistance to Science 177,406,0001,887,365,000
49,090,000 2,113,861,000 Management Services– as appearing in the SARO, it would nonetheless remain a new activity by reason
and Technology Activities of its not being specifically stated in the GAA. As such, the DBM, sans legislative authorization, could not
validly fund and implement such PAP under the DAP.

Central Office 1,554,238,000 1,554,238,000 In defending the disbursements, however, the OSG contends that the Executive enjoyed sound discretion
in implementing the budget given the generality in the language and the broad policy objectives identified
under the GAAs;172 and that the President enjoyed unlimited authority to spend the initial appropriations
under his authority to declare and utilize savings,173 and in keeping with his duty to faithfully execute the
laws.

Although the OSG rightly contends that the Executive was authorized to spend in line with its mandate to
faithfully execute the laws (which included the GAAs), such authority did not translate to unfettered
discretion that allowed the President to substitute his own will for that of Congress. He was still required to
remain faithful to the provisions of the GAAs, given that his power to spend pursuant to the GAAs was but
a delegation to him from Congress. Verily, the power to spend the public wealth resided in Congress, not
106
in the Executive.174 Moreover, leaving the spending power of the Executive unrestricted would threaten SECRETARY ABAD:
to undo the principle of separation of powers.175
Well, the first instance had to do with a request from the House of Representatives. They started building
Congress acts as the guardian of the public treasury in faithful discharge of its power of the purse their e-library in 2010 and they had a budget for about 207 Million but they lack about 43 Million to
whenever it deliberates and acts on the budget proposal submitted by the Executive.176 Its power of the complete its 250 Million requirements. Prior to that, the COA, in an audit observation informed the
purse is touted as the very foundation of its institutional strength,177 and underpins "all other legislative Speaker that they had to continue with that construction otherwise the whole building, as well as the
decisions and regulating the balance of influence between the legislative and executive branches of equipments therein may suffer from serious deterioration. And at that time, since the budget of the House
government."178 Such enormous power encompasses the capacity to generate money for the of Representatives was not enough to complete 250 Million, they wrote to the President requesting for an
Government, to appropriate public funds, and to spend the money.179 Pertinently, when it exercises its augmentation of that particular item, which was granted, Your Honor. The second instance in the Memos
power of the purse, Congress wields control by specifying the PAPs for which public money should be is a request from the Commission on Audit. At the time they were pushing very strongly the good
spent. governance programs of the government and therefore, part of that is a requirement to conduct audits as
well as review financial reports of many agencies. And in the performance of that function, the
It is the President who proposes the budget but it is Congress that has the final say on matters of Commission on Audit needed information technology equipment as well as hire consultants and litigators
appropriations.180 For this purpose, appropriation involves two governing principles, namely: (1) "a to help them with their audit work and for that they requested funds from the Executive and the President
Principle of the Public Fisc, asserting that all monies received from whatever source by any part of the saw that it was important for the Commission to be provided with those IT equipments and litigators and
government are public funds;" and (2) "a Principle of Appropriations Control, prohibiting expenditure of consultants and the request was granted, Your Honor.
any public money without legislative authorization."181 To conform with the governing principles, the
Executive cannot circumvent the prohibition by Congress of an expenditure for a PAP by resorting to JUSTICE BERSAMIN:
either public or private funds.182 Nor could the Executive transfer appropriated funds resulting in an
increase in the budget for one PAP, for by so doing the appropriation for another PAP is necessarily These cross border examples, cross border augmentations were not supported by appropriations…
decreased. The terms of both appropriations will thereby be violated.
SECRETARY ABAD:
b.4 Third Requisite – Cross-border
augmentations from savings were
prohibited by the Constitution They were, we were augmenting existing items within their… (interrupted)

By providing that the President, the President of the Senate, the Speaker of the House of JUSTICE BERSAMIN:
Representatives, the Chief Justice of the Supreme Court, and the Heads of the Constitutional
Commissions may be authorized to augment any item in the GAA "for their respective offices," Section No, appropriations before you augmented because this is a cross border and the tenor or text of the
25(5), supra, has delineated borders between their offices, such that funds appropriated for one office are Constitution is quite clear as far as I am concerned. It says here, "The power to augment may only be
prohibited from crossing over to another office even in the guise of augmentation of a deficient item or made to increase any item in the General Appropriations Law for their respective offices." Did you not feel
items. Thus, we call such transfers of funds cross-border transfers or cross-border augmentations. constricted by this provision?

To be sure, the phrase "respective offices" used in Section 25(5), supra, refers to the entire Executive, SECRETARY ABAD:
with respect to the President; the Senate, with respect to the Senate President; the House of
Representatives, with respect to the Speaker; the Judiciary, with respect to the Chief Justice; the Well, as the Constitution provides, the prohibition we felt was on the transfer of appropriations, Your
Constitutional Commissions, with respect to their respective Chairpersons. Honor. What we thought we did was to transfer savings which was needed by the Commission to address
deficiency in an existing item in both the Commission as well as in the House of Representatives; that’s
Did any cross-border transfers or augmentations transpire? how we saw…(interrupted)

During the oral arguments on January 28, 2014, Sec. Abad admitted making some cross-border JUSTICE BERSAMIN:
augmentations, to wit:
So your position as Secretary of Budget is that you could do that?
JUSTICE BERSAMIN:
SECRETARY ABAD:
Alright, the whole time that you have been Secretary of Department of Budget and Management, did the
Executive Department ever redirect any part of savings of the National Government under your control In an extreme instances because…(interrupted)
cross border to another department?
JUSTICE BERSAMIN:
SECRETARY ABAD:
No, no, in all instances, extreme or not extreme, you could do that, that’s your feeling.
Well, in the Memos that we submitted to you, such an instance, Your Honor
SECRETARY ABAD:
JUSTICE BERSAMIN:
Well, in that particular situation when the request was made by the Commission and the House of
Can you tell me two instances? I don’t recall having read your material. Representatives, we felt that we needed to respond because we felt…(interrupted).183

107
The records show, indeed, that funds amounting to ₱143,700,000.00 and ₱250,000,000.00 were May I move to another point, maybe just briefly. I am curious that the position now, I think, of government
transferred under the DAP respectively to the COA184 and the House of Representatives.185 Those is that some transfers of savings is now considered to be, if I’m not mistaken, aid not augmentation. Am I
transfers of funds, which constituted cross-border augmentations for being from the Executive to the COA correct in my hearing of your argument?
and the House of Representatives, are graphed as follows:186
HONORABLE MENDOZA:

That’s our submission, if Your Honor, please.


AMOUNT
(In thousand pesos) JUSTICE LEONEN:
DATE
PURPOSE
RELEASED May I know, Justice, where can we situate this in the text of the Constitution? Where do we actually derive
Reserve Releases
the concepts that transfers of appropriation from one branch to the other or what happened in DAP can be
Imposed considered a said? What particular text in the Constitution can we situate this?

HONORABLE MENDOZA:
on
IT Infrastructure Program and hiring of additional litigation experts 11/11/11 143,700
There is no particular provision or statutory provision for that matter, if Your Honor please. It is drawn from
the fact that the Executive is the executive in-charge of the success of the government.
Completion of the construction of the Legislative Library and Archives 07/23/12 207,034 250,000
Building/Congressional e-library (Savings of HOR) JUSTICE LEONEN:
ves
So, the residual powers labelled in Marcos v. Manglapus would be the basis for this theory of the
government?
The respondents further stated in their memorandum that the President "made available" to the
"Commission on Elections the savings of his department upon [its] request for funds…"187 This was
another instance of a cross-border augmentation. HONORABLE MENDOZA:

The respondents justified all the cross-border transfers thusly: Yes, if Your Honor, please.

99. The Constitution does not prevent the President from transferring savings of his department to another JUSTICE LEONEN:
department upon the latter’s request, provided it is the recipient department that uses such funds to
augment its own appropriation. In such a case, the President merely gives the other department access to A while ago, Justice Carpio mentioned that the remedy is might be to go to Congress. That there are
public funds but he cannot dictate how they shall be applied by that department whose fiscal autonomy is opportunities and there have been opportunities of the President to actually go to Congress and ask for
guaranteed by the Constitution.188 supplemental budgets?

In the oral arguments held on February 18, 2014, Justice Vicente V. Mendoza, representing Congress, HONORABLE MENDOZA:
announced a different characterization of the cross-border transfers of funds as in the nature of "aid"
instead of "augmentation," viz: If there is time to do that, I would say yes.

HONORABLE MENDOZA: JUSTICE LEONEN:

The cross-border transfers, if Your Honors please, is not an application of the DAP. What were these So, the theory of aid rather than augmentation applies in extra-ordinary situation?
cross-border transfers? They are transfers of savings as defined in the various General Appropriations
Act. So, that makes it similar to the DAP, the use of savings. There was a cross-border which appears to
HONORABLE MENDOZA:
be in violation of Section 25, paragraph 5 of Article VI, in the sense that the border was crossed. But
never has it been claimed that the purpose was to augment a deficient item in another department of the
government or agency of the government. The cross-border transfers, if Your Honors please, were in the Very extra-ordinary situations.
nature of [aid] rather than augmentations. Here is a government entity separate and independent from the
Executive Department solely in need of public funds. The President is there 24 hours a day, 7 days a JUSTICE LEONEN:
week. He’s in charge of the whole operation although six or seven heads of government offices are given
the power to augment. Only the President stationed there and in effect in-charge and has the
But Counsel, this would be new doctrine, in case?
responsibility for the failure of any part of the government. You have election, for one reason or another,
the money is not enough to hold election. There would be chaos if no money is given as an aid, not to
augment, but as an aid to a department like COA. The President is responsible in a way that the other HONORABLE MENDOZA:
heads, given the power to augment, are not. So, he cannot very well allow this, if Your Honor please.189
Yes, if Your Honor please.190
JUSTICE LEONEN:

108
Regardless of the variant characterizations of the cross-border transfers of funds, the plain text of Section (50%) of the said savings net of revenue shortfall: PROVIDED, FINALLY, That the release of the balance
25(5), supra, disallowing cross border transfers was disobeyed. Cross-border transfers, whether as of the total savings from programmed appropriations for the year shall be subject to fiscal programming
augmentation, or as aid, were prohibited under Section 25(5), supra. and approval of the President.

4. 2012 GAA
Sourcing the DAP from unprogrammed
funds despite the original revenue targets 1. Release of the Fund. The amounts authorized herein shall be released only when the revenue
not having been exceeded was invalid collections exceed the original revenue targets submitted by the President of the Philippines to Congress
pursuant to Section 22, Article VII of the Constitution: PROVIDED, That collections arising from sources
Funding under the DAP were also sourced from unprogrammed funds provided in the GAAs for 2011, not considered in the aforesaid original revenue targets may be used to cover releases from
2012,and 2013. The respondents stress, however, that the unprogrammed funds were not brought under appropriations in this Fund: PROVIDED, FURTHER, That in case of newly approved loans for foreign-
the DAP as savings, but as separate sources of funds; and that, consequently, the release and use of assisted projects, the existence of a perfected loan agreement for the purpose shall be sufficient basis for
unprogrammed funds were not subject to the restrictions under Section 25(5), supra. the issuance of a SARO covering the loan proceeds.

The documents contained in the Evidence Packets by the OSG have confirmed that the unprogrammed As can be noted, the provisos in both provisions to the effect that "collections arising from sources not
funds were treated as separate sources of funds. Even so, the release and use of the unprogrammed considered in the aforesaid original revenue targets may be used to cover releases from appropriations in
funds were still subject to restrictions, for, to start with, the GAAs precisely specified the instances when this Fund" gave the authority to use such additional revenues for appropriations funded from the
the unprogrammed funds could be released and the purposes for which they could be used. unprogrammed funds. They did not at all waive compliance with the basic requirement that revenue
collections must still exceed the original revenue targets.
The petitioners point out that a condition for the release of the unprogrammed funds was that the revenue
collections must exceed revenue targets; and that the release of the unprogrammed funds was illegal In contrast, the texts of the provisos with regard to additional revenues generated from newly-approved
because such condition was not met.191 foreign loans were clear to the effect that the perfected loan agreement would be in itself "sufficient basis"
for the issuance of a SARO to release the funds but only to the extent of the amount of the loan. In such
The respondents disagree, holding that the release and use of the unprogrammed funds under the DAP instance, the revenue collections need not exceed the revenue targets to warrant the release of the loan
were in accordance with the pertinent provisions of the GAAs. In particular, the DBM avers that the proceeds, and the mere perfection of the loan agreement would suffice.
unprogrammed funds could be availed of when any of the following three instances occur, to wit: (1) the
revenue collections exceeded the original revenue targets proposed in the BESFs submitted by the It can be inferred from the foregoing that under these provisions of the GAAs the additional revenues from
President to Congress; (2) new revenues were collected or realized from sources not originally considered sources not considered in the BESFs must be taken into account in determining if the revenue collections
in the BESFs; or(3) newly-approved loans for foreign assisted projects were secured, or when conditions exceeded the revenue targets. The text of the relevant provision of the 2013 GAA, which was substantially
were triggered for other sources of funds, such as perfected loan agreements for foreign-assisted similar to those of the GAAs for 2011 and 2012, already made this explicit, thus:
projects.192 This view of the DBM was adopted by all the respondents in their Consolidated
Comment.193 1. Release of the Fund. The amounts authorized herein shall be released only when the revenue
collections exceed the original revenue targets submitted by the President of the Philippines to Congress
The BESFs for 2011, 2012 and 2013 uniformly defined "unprogrammed appropriations" as appropriations pursuant to Section 22, Article VII of the Constitution, including collections arising from sources not
that provided standby authority to incur additional agency obligations for priority PAPs when revenue considered in the aforesaid original revenue target, as certified by the BTr: PROVIDED, That in case of
collections exceeded targets, and when additional foreign funds are generated.194 Contrary to the DBM’s newly approved loans for foreign-assisted projects, the existence of a perfected loan agreement for the
averment that there were three instances when unprogrammed funds could be released, the BESFs purpose shall be sufficient basis for the issuance of a SARO covering the loan proceeds.
envisioned only two instances. The third mentioned by the DBM – the collection of new revenues from
sources not originally considered in the BESFs – was not included. This meant that the collection of Consequently, that there were additional revenues from sources not considered in the revenue target
additional revenues from new sources did not warrant the release of the unprogrammed funds. Hence, would not be enough. The total revenue collections must still exceed the original revenue targets to justify
even if the revenues not considered in the BESFs were collected or generated, the basic condition that the release of the unprogrammed funds (other than those from newly-approved foreign loans).
the revenue collections should exceed the revenue targets must still be complied with in order to justify
the release of the unprogrammed funds.
The present controversy on the unprogrammed funds was rooted in the correct interpretation of the
phrase "revenue collections should exceed the original revenue targets." The petitioners take the phrase
The view that there were only two instances when the unprogrammed funds could be released was to mean that the total revenue collections must exceed the total revenue target stated in the BESF, but
bolstered by the following texts of the Special Provisions of the 2011 and 2012 GAAs, to wit: the respondents understand the phrase to refer only to the collections for each source of revenue as
enumerated in the BESF, with the condition being deemed complied with once the revenue collections
2011 GAA from a particular source already exceeded the stated target.

1. Release of Fund. The amounts authorized herein shall be released only when the revenue collections The BESF provided for the following sources of revenue, with the corresponding revenue target stated for
exceed the original revenue targets submitted by the President of the Philippines to Congress pursuant to each source of revenue, to wit:
Section 22, Article VII of the Constitution, including savings generated from programmed appropriations
for the year: PROVIDED, That collections arising from sources not considered in the aforesaid original TAX REVENUES
revenue targets may be used to cover releases from appropriations in this Fund: PROVIDED, FURTHER,
That in case of newly approved loans for foreign-assisted projects, the existence of a perfected loan
agreement for the purpose shall be sufficient basis for the issuance of a SARO covering the loan Taxes on Net Income and Profits
proceeds: PROVIDED, FURTHERMORE, That if there are savings generated from the programmed Taxes on Property
appropriations for the first two quarters of the year, the DBM may, subject to the approval of the President, Taxes on Domestic Goods and Services
release the pertinent appropriations under the Unprogrammed Fund corresponding to only fifty percent
109
General Sales, Turnover or VAT This is to certify that the actual dividend collections remitted to the National Government for the period
Selected Excises on Goods January to May 2013 amounted to ₱12.438 billion compared to the full year program of ₱10.0198 billion
for 2013.
Selected Taxes on Services
Taxes on the Use of Goods or Property or Permission to Perform Activities Moreover, the National Government accounted for the sale of the right to build and operate the NAIA
Other Taxes expressway amounting to ₱11.0 billion in June 2013.199
Taxes on International Trade and Transactions
The certifications reflected that by collecting dividends amounting to ₱23.8 billion in 2011, ₱19.419 billion
NON-TAX REVENUES in 2012, and ₱12.438 billion in 2013 the BTr had exceeded only the ₱5.5 billion in target revenues in the
form of dividends from stocks in each of 2011 and 2012, and only the ₱10 billion in target revenues in the
Fees and Charges form of dividends from stocks in 2013.
BTR Income
However, the requirement that revenue collections exceed the original revenue targets was to be
Government Services construed in light of the purpose for which the unprogrammed funds were incorporated in the GAAs as
Interest on NG Deposits standby appropriations to support additional expenditures for certain priority PAPs should the revenue
Interest on Advances to Government Corporations collections exceed the resource targets assumed in the budget or when additional foreign project loan
Income from Investments proceeds were realized. The unprogrammed funds were included in the GAAs to provide ready cover so
as not to delay the implementation of the PAPs should new or additional revenue sources be realized
during the year.200 Given the tenor of the certifications, the unprogrammed funds were thus not yet
Interest on Bond Holdings supported by the corresponding resources.201

Guarantee Fee The revenue targets stated in the BESF were intended to address the funding requirements of the
Gain on Foreign Exchange proposed programmed appropriations. In contrast, the unprogrammed funds, as standby appropriations,
NG Income Collected by BTr were to be released only when there were revenues in excess of what the programmed appropriations
required. As such, the revenue targets should be considered as a whole, not individually; otherwise, we
Dividends on Stocks would be dealing with artificial revenue surpluses. The requirement that revenue collections must exceed
NG Share from Airport Terminal Fee revenue target should be understood to mean that the revenue collections must exceed the total of the
NG Share from PAGCOR Income revenue targets stated in the BESF. Moreover, to release the unprogrammed funds simply because there
NG Share from MIAA Profit was an excess revenue as to one source of revenue would be an unsound fiscal management measure
because it would disregard the budget plan and foster budget deficits, in contravention of the
Privatization Government’s surplus budget policy.202
Foreign Grants
We cannot, therefore, subscribe to the respondents’ view.
Thus, when the Court required the respondents to submit a certification from the Bureau of Treasury (BTr)
to the effect that the revenue collections had exceeded the original revenue targets,195 they complied by 5.
submitting certifications from the BTr and Department of Finance (DOF) pertaining to only one identified Equal protection, checks and balances,
source of revenue – the dividends from the shares of stock held by the Government in government-owned and public accountability challenges
and controlled corporations.
The DAP is further challenged as violative of the Equal Protection Clause, the system of checks and
To justify the release of the unprogrammed funds for 2011, the OSG presented the certification dated balances, and the principle of public accountability.
March 4, 2011 issued by DOF Undersecretary Gil S. Beltran, as follows:
With respect to the challenge against the DAP under the Equal Protection Clause,203 Luna argues that
This is to certify that under the Budget for Expenditures and Sources of Financing for 2011, the the implementation of the DAP was "unfair as it [was] selective" because the funds released under the
programmed income from dividends from shares of stock in government-owned and controlled DAP was not made available to all the legislators, with some of them refusing to avail themselves of the
corporations is 5.5 billion. DAP funds, and others being unaware of the availability of such funds. Thus, the DAP practised "undue
favoritism" in favor of select legislators in contravention of the Equal Protection Clause.
This is to certify further that based on the records of the Bureau of Treasury, the National Government has
recorded dividend income amounting to ₱23.8 billion as of 31 January 2011.196 Similarly, COURAGE contends that the DAP violated the Equal Protection Clause because no reasonable
classification was used in distributing the funds under the DAP; and that the Senators who supposedly
For 2012, the OSG submitted the certification dated April 26, 2012 issued by National Treasurer Roberto availed themselves of said funds were differently treated as to the amounts they respectively received.
B. Tan, viz:
Anent the petitioners’ theory that the DAP violated the system of checks and balances, Luna submits that
This is to certify that the actual dividend collections remitted to the National Government for the period the grant of the funds under the DAP to some legislators forced their silence about the issues and
January to March 2012 amounted to ₱19.419 billion compared to the full year program of ₱5.5 billion for anomalies surrounding the DAP. Meanwhile, Belgica stresses that the DAP, by allowing the legislators to
2012.197 identify PAPs, authorized them to take part in the implementation and execution of the GAAs, a function
that exclusively belonged to the Executive; that such situation constituted undue and unjustified legislative
encroachment in the functions of the Executive; and that the President arrogated unto himself the power
And, finally, for 2013, the OSG presented the certification dated July 3, 2013 issued by National Treasurer
Rosalia V. De Leon, to wit:

110
of appropriation vested in Congress because NBC No. 541 authorized the use of the funds under the DAP Article 7. Laws are repealed only by subsequent ones, and their violation or non-observance shall not be
for PAPs not considered in the 2012 budget. excused by disuse, or custom or practice to the contrary.

Finally, the petitioners insist that the DAP was repugnant to the principle of public accountability enshrined When the courts declared a law to be inconsistent with the Constitution, the former shall be void and the
in the Constitution,204 because the legislators relinquished the power of appropriation to the Executive, latter shall govern.
and exhibited a reluctance to inquire into the legality of the DAP.
Administrative or executive acts, orders and regulations shall be valid only when they are not contrary to
The OSG counters the challenges, stating that the supposed discrimination in the release of funds under the laws or the Constitution.
the DAP could be raised only by the affected Members of Congress themselves, and if the challenge
based on the violation of the Equal Protection Clause was really against the constitutionality of the DAP, A legislative or executive act that is declared void for being unconstitutional cannot give rise to any right or
the arguments of the petitioners should be directed to the entitlement of the legislators to the funds, not to obligation.206 However, the generality of the rule makes us ponder whether rigidly applying the rule may
the proposition that all of the legislators should have been given such entitlement. at times be impracticable or wasteful. Should we not recognize the need to except from the rigid
application of the rule the instances in which the void law or executive act produced an almost irreversible
The challenge based on the contravention of the Equal Protection Clause, which focuses on the release result?
of funds under the DAP to legislators, lacks factual and legal basis. The allegations about Senators and
Congressmen being unaware of the existence and implementation of the DAP, and about some of them The need is answered by the doctrine of operative fact. The doctrine, definitely not a novel one, has been
having refused to accept such funds were unsupported with relevant data. Also, the claim that the exhaustively explained in De Agbayani v. Philippine National Bank:207
Executive discriminated against some legislators on the ground alone of their receiving less than the
others could not of itself warrant a finding of contravention of the Equal Protection Clause. The denial of
equal protection of any law should be an issue to be raised only by parties who supposedly suffer it, and, The decision now on appeal reflects the orthodox view that an unconstitutional act, for that matter an
in these cases, such parties would be the few legislators claimed to have been discriminated against in executive order or a municipal ordinance likewise suffering from that infirmity, cannot be the source of any
the releases of funds under the DAP. The reason for the requirement is that only such affected legislators legal rights or duties. Nor can it justify any official act taken under it. Its repugnancy to the fundamental
could properly and fully bring to the fore when and how the denial of equal protection occurred, and law once judicially declared results in its being to all intents and purposes a mere scrap of paper. As the
explain why there was a denial in their situation. The requirement was not met here. Consequently, the new Civil Code puts it: ‘When the courts declare a law to be inconsistent with the Constitution, the former
Court was not put in the position to determine if there was a denial of equal protection. To have the Court shall be void and the latter shall govern.’ Administrative or executive acts, orders and regulations shall be
do so despite the inadequacy of the showing of factual and legal support would be to compel it to valid only when they are not contrary to the laws of the Constitution. It is understandable why it should be
speculate, and the outcome would not do justice to those for whose supposed benefit the claim of denial so, the Constitution being supreme and paramount. Any legislative or executive act contrary to its terms
of equal protection has been made. cannot survive.

The argument that the release of funds under the DAP effectively stayed the hands of the legislators from Such a view has support in logic and possesses the merit of simplicity. It may not however be sufficiently
conducting congressional inquiries into the legality and propriety of the DAP is speculative. That realistic. It does not admit of doubt that prior to the declaration of nullity such challenged legislative or
deficiency eliminated any need to consider and resolve the argument, for it is fundamental that executive act must have been in force and had to be complied with. This is so as until after the judiciary,
speculation would not support any proper judicial determination of an issue simply because nothing in an appropriate case, declares its invalidity, it is entitled to obedience and respect. Parties may have
concrete can thereby be gained. In order to sustain their constitutional challenges against official acts of acted under it and may have changed their positions. What could be more fitting than that in a subsequent
the Government, the petitioners must discharge the basic burden of proving that the constitutional litigation regard be had to what has been done while such legislative or executive act was in operation
infirmities actually existed.205 Simply put, guesswork and speculation cannot overcome the presumption and presumed to be valid in all respects. It is now accepted as a doctrine that prior to its being nullified, its
of the constitutionality of the assailed executive act. existence as a fact must be reckoned with. This is merely to reflect awareness that precisely because the
judiciary is the governmental organ which has the final say on whether or not a legislative or executive
measure is valid, a period of time may have elapsed before it can exercise the power of judicial review
We do not need to discuss whether or not the DAP and its implementation through the various circulars that may lead to a declaration of nullity. It would be to deprive the law of its quality of fairness and justice
and memoranda of the DBM transgressed the system of checks and balances in place in our then, if there be no recognition of what had transpired prior to such adjudication.
constitutional system. Our earlier expositions on the DAP and its implementing issuances infringing the
doctrine of separation of powers effectively addressed this particular concern.
In the language of an American Supreme Court decision: ‘The actual existence of a statute, prior to such
a determination [of unconstitutionality], is an operative fact and may have consequences which cannot
Anent the principle of public accountability being transgressed because the adoption and implementation justly be ignored. The past cannot always be erased by a new judicial declaration. The effect of the
of the DAP constituted an assumption by the Executive of Congress’ power of appropriation, we have subsequent ruling as to invalidity may have to be considered in various aspects, with respect to particular
already held that the DAP and its implementing issuances were policies and acts that the Executive could relations, individual and corporate, and particular conduct, private and official.’"
properly adopt and do in the execution of the GAAs to the extent that they sought to implement strategies
to ramp up or accelerate the economy of the country.
The doctrine of operative fact recognizes the existence of the law or executive act prior to the
determination of its unconstitutionality as an operative fact that produced consequences that cannot
6. always be erased, ignored or disregarded. In short, it nullifies the void law or executive act but sustains its
Doctrine of operative fact was applicable effects. It provides an exception to the general rule that a void or unconstitutional law produces no
effect.208 But its use must be subjected to great scrutiny and circumspection, and it cannot be invoked to
After declaring the DAP and its implementing issuances constitutionally infirm, we must now deal with the validate an unconstitutional law or executive act, but is resorted to only as a matter of equity and fair
consequences of the declaration. play.209 It applies only to cases where extraordinary circumstances exist, and only when the
extraordinary circumstances have met the stringent conditions that will permit its application.
Article 7 of the Civil Code provides:
We find the doctrine of operative fact applicable to the adoption and implementation of the DAP. Its
application to the DAP proceeds from equity and fair play. The consequences resulting from the DAP and
its related issuances could not be ignored or could no longer be undone.
111
To be clear, the doctrine of operative fact extends to a void or unconstitutional executive act. The term this score that the operative fact doctrine should be applied to acts and consequences that resulted from
executive act is broad enough to include any and all acts of the Executive, including those that are quasi the implementation of the PARC Resolution approving the SDP of HLI. (Bold underscoring supplied for
legislative and quasi-judicial in nature. The Court held so in Hacienda Luisita, Inc. v. Presidential Agrarian emphasis)
Reform Council:210
In Commissioner of Internal Revenue v. San Roque Power Corporation,211 the Court likewise declared
Nonetheless, the minority is of the persistent view that the applicability of the operative fact doctrine that "for the operative fact doctrine to apply, there must be a ‘legislative or executive measure,’ meaning a
should be limited to statutes and rules and regulations issued by the executive department that are law or executive issuance." Thus, the Court opined there that the operative fact doctrine did not apply to a
accorded the same status as that of a statute or those which are quasi-legislative in nature. Thus, the mere administrative practice of the Bureau of Internal Revenue, viz:
minority concludes that the phrase ‘executive act’ used in the case of De Agbayani v. Philippine National
Bank refers only to acts, orders, and rules and regulations that have the force and effect of law. The Under Section 246, taxpayers may rely upon a rule or ruling issued by the Commissioner from the time
minority also made mention of the Concurring Opinion of Justice Enrique Fernando in Municipality of the rule or ruling is issued up to its reversal by the Commissioner or this Court. The reversal is not given
Malabang v. Benito, where it was supposedly made explicit that the operative fact doctrine applies to retroactive effect. This, in essence, is the doctrine of operative fact. There must, however, be a rule or
executive acts, which are ultimately quasi-legislative in nature. ruling issued by the Commissioner that is relied upon by the taxpayer in good faith. A mere administrative
practice, not formalized into a rule or ruling, will not suffice because such a mere administrative practice
We disagree. For one, neither the De Agbayani case nor the Municipality of Malabang case elaborates may not be uniformly and consistently applied. An administrative practice, if not formalized as a rule or
what ‘executive act’ mean. Moreover, while orders, rules and regulations issued by the President or the ruling, will not be known to the general public and can be availed of only by those with informal contacts
executive branch have fixed definitions and meaning in the Administrative Code and jurisprudence, the with the government agency.
phrase ‘executive act’ does not have such specific definition under existing laws. It should be noted that in
the cases cited by the minority, nowhere can it be found that the term ‘executive act’ is confined to the It is clear from the foregoing that the adoption and the implementation of the DAP and its related
foregoing. Contrarily, the term ‘executive act’ is broad enough to encompass decisions of administrative issuances were executive acts.1avvphi1 The DAP itself, as a policy, transcended a merely administrative
bodies and agencies under the executive department which are subsequently revoked by the agency in practice especially after the Executive, through the DBM, implemented it by issuing various memoranda
question or nullified by the Court. and circulars. The pooling of savings pursuant to the DAP from the allotments made available to the
different agencies and departments was consistently applied throughout the entire Executive. With the
A case in point is the concurrent appointment of Magdangal B. Elma (Elma) as Chairman of the Executive, through the DBM, being in charge of the third phase of the budget cycle – the budget execution
Presidential Commission on Good Government (PCGG) and as Chief Presidential Legal Counsel (CPLC) phase, the President could legitimately adopt a policy like the DAP by virtue of his primary responsibility
which was declared unconstitutional by this Court in Public Interest Center, Inc. v. Elma. In said case, this as the Chief Executive of directing the national economy towards growth and development. This is simply
Court ruled that the concurrent appointment of Elma to these offices is in violation of Section 7, par. 2, because savings could and should be determined only during the budget execution phase.
Article IX-B of the 1987 Constitution, since these are incompatible offices. Notably, the appointment of
Elma as Chairman of the PCGG and as CPLC is, without a question, an executive act. Prior to the As already mentioned, the implementation of the DAP resulted into the use of savings pooled by the
declaration of unconstitutionality of the said executive act, certain acts or transactions were made in good Executive to finance the PAPs that were not covered in the GAA, or that did not have proper appropriation
faith and in reliance of the appointment of Elma which cannot just be set aside or invalidated by its covers, as well as to augment items pertaining to other departments of the Government in clear violation
subsequent invalidation. of the Constitution. To declare the implementation of the DAP unconstitutional without recognizing that its
prior implementation constituted an operative fact that produced consequences in the real as well as
In Tan v. Barrios, this Court, in applying the operative fact doctrine, held that despite the invalidity of the juristic worlds of the Government and the Nation is to be impractical and unfair. Unless the doctrine is held
jurisdiction of the military courts over civilians, certain operative facts must be acknowledged to have to apply, the Executive as the disburser and the offices under it and elsewhere as the recipients could be
existed so as not to trample upon the rights of the accused therein. Relevant thereto, in Olaguer v. Military required to undo everything that they had implemented in good faith under the DAP. That scenario would
Commission No. 34, it was ruled that ‘military tribunals pertain to the Executive Department of the be enormously burdensome for the Government. Equity alleviates such burden.
Government and are simply instrumentalities of the executive power, provided by the legislature for the
President as Commander-in-Chief to aid him in properly commanding the army and navy and enforcing The other side of the coin is that it has been adequately shown as to be beyond debate that the
discipline therein, and utilized under his orders or those of his authorized military representatives.’ implementation of the DAP yielded undeniably positive results that enhanced the economic welfare of the
country. To count the positive results may be impossible, but the visible ones, like public infrastructure,
Evidently, the operative fact doctrine is not confined to statutes and rules and regulations issued by the could easily include roads, bridges, homes for the homeless, hospitals, classrooms and the like. Not to
executive department that are accorded the same status as that of a statute or those which are quasi- apply the doctrine of operative fact to the DAP could literally cause the physical undoing of such worthy
legislative in nature. results by destruction, and would result in most undesirable wastefulness.

Even assuming that De Agbayani initially applied the operative fact doctrine only to executive issuances Nonetheless, as Justice Brion has pointed out during the deliberations, the doctrine of operative fact does
like orders and rules and regulations, said principle can nonetheless be applied, by analogy, to decisions not always apply, and is not always the consequence of every declaration of constitutional invalidity. It can
made by the President or the agencies under the executive department. This doctrine, in the interest of be invoked only in situations where the nullification of the effects of what used to be a valid law would
justice and equity, can be applied liberally and in a broad sense to encompass said decisions of the result in inequity and injustice;212 but where no such result would ensue, the general rule that an
executive branch. In keeping with the demands of equity, the Court can apply the operative fact doctrine unconstitutional law is totally ineffective should apply.
to acts and consequences that resulted from the reliance not only on a law or executive act which is
quasi-legislative in nature but also on decisions or orders of the executive branch which were later In that context, as Justice Brion has clarified, the doctrine of operative fact can apply only to the PAPs that
nullified. This Court is not unmindful that such acts and consequences must be recognized in the higher can no longer be undone, and whose beneficiaries relied in good faith on the validity of the DAP, but
interest of justice, equity and fairness. cannot apply to the authors, proponents and implementors of the DAP, unless there are concrete findings
of good faith in their favor by the proper tribunals determining their criminal, civil, administrative and other
Significantly, a decision made by the President or the administrative agencies has to be complied with liabilities.
because it has the force and effect of law, springing from the powers of the President under the
Constitution and existing laws. Prior to the nullification or recall of said decision, it may have produced WHEREFORE, the Court PARTIALLY GRANTS the petitions for certiorari and prohibition; and
acts and consequences in conformity to and in reliance of said decision, which must be respected. It is on DECLARES the following acts and practices under the Disbursement Acceleration Program, National

112
Budget Circular No. 541 and related executive issuances UNCONSTITUTIONAL for being in violation of Education, Culture and Sports; and (b) the veto of the President of the Special Provision of
Section 25(5), Article VI of the 1987 Constitution and the doctrine of separation of powers, namely: Article XLVIII of the GAA of 1994 (Rollo, pp. 88-90, 104-105)

(a) The withdrawal of unobligated allotments from the implementing agencies, and the declaration of the In G.R. No. 113174, sixteen members of the Senate led by Senate President Edgardo J. Angara, Senator
withdrawn unobligated allotments and unreleased appropriations as savings prior to the end of the fiscal Neptali A. Gonzales, the Chairman of the Committee on Finance, and Senator Raul S. Roco, sought the
year and without complying with the statutory definition of savings contained in the General Appropriations issuance of the writs of certiorari, prohibition and mandamus against the Executive Secretary, the
Acts; Secretary of the Department of Budget and Management, and the National Treasurer.

(b) The cross-border transfers of the savings of the Executive to augment the appropriations of other Suing as members of the Senate and taxpayers, petitioners question: (1) the constitutionality of the
offices outside the Executive; and conditions imposed by the President in the items of the GAA of 1994: (a) for the Supreme Court, (b)
Commission on Audit (COA), (c) Ombudsman, (d) Commission on Human Rights (CHR), (e) Citizen
(c) The funding of projects, activities and programs that were not covered by any appropriation in the Armed Forces Geographical Units (CAFGU'S) and (f) State Universities and Colleges (SUC's); and (2) the
General Appropriations Act. constitutionality of the veto of the special provision in the appropriation for debt service.

The Court further DECLARES VOID the use of unprogrammed funds despite the absence of a In G.R. No. 113766, Senators Alberto G. Romulo and Wigberto Tañada (a co-petitioner in G.R. No.
certification by the National Treasurer that the revenue collections exceeded the revenue targets for non- 113174), together with the Freedom from Debt Coalition, a non-stock domestic corporation, sought the
compliance with the conditions provided in the relevant General Appropriations Acts. issuance of the writs of prohibition and mandamus against the Executive Secretary, the Secretary of the
Department of Budget and Management, the National Treasurer, and the COA.
SO ORDERED
Petitioners Tañada and Romulo sued as members of the Philippine Senate and taxpayers, while petitioner
Freedom from Debt Coalition sued as a taxpayer. They challenge the constitutionality of the Presidential
G.R. No. 113105 August 19, 1994 veto of the special provision in the appropriations for debt service and the automatic appropriation of
funds therefor.
PHILIPPINE CONSTITUTION ASSOCIATION, vs.HON. SALVADOR ENRIQUEZ,
In G.R. No. 11388, Senators Tañada and Romulo sought the issuance of the writs of prohibition and
Once again this Court is called upon to rule on the conflicting claims of authority between the Legislative mandamus against the same respondents in G.R. No. 113766. In this petition, petitioners contest the
and the Executive in the clash of the powers of the purse and the sword. Providing the focus for the constitutionality of: (1) the veto on four special provision added to items in the GAA of 1994 for the Armed
contest between the President and the Congress over control of the national budget are the four cases at Forces of the Philippines (AFP) and the Department of Public Works and Highways (DPWH); and (2) the
bench. Judicial intervention is being sought by a group of concerned taxpayers on the claim that Congress conditions imposed by the President in the implementation of certain appropriations for the CAFGU's, the
and the President have impermissibly exceeded their respective authorities, and by several Senators on DPWH, and the National Housing Authority (NHA).
the claim that the President has committed grave abuse of discretion or acted without jurisdiction in the
exercise of his veto power. Petitioners also sought the issuance of temporary restraining orders to enjoin respondents Secretary of
Budget and Management, National Treasurer and COA from enforcing the questioned provisions of the
I GAA of 1994, but the Court declined to grant said provisional reliefs on the time- honored principle of
according the presumption of validity to statutes and the presumption of regularity to official acts.
House Bill No. 10900, the General Appropriation Bill of 1994 (GAB of 1994), was passed and approved by
both houses of Congress on December 17, 1993. As passed, it imposed conditions and limitations on In view of the importance and novelty of most of the issues raised in the four petitions, the Court invited
certain items of appropriations in the proposed budget previously submitted by the President. It also former Chief Justice Enrique M. Fernando and former Associate Justice Irene Cortes to submit their
authorized members of Congress to propose and identify projects in the "pork barrels" allotted to them respective memoranda as Amicus curiae, which they graciously did.
and to realign their respective operating budgets.
II- Locus Standi
Pursuant to the procedure on the passage and enactment of bills as prescribed by the Constitution,
Congress presented the said bill to the President for consideration and approval. When issues of constitutionality are raised, the Court can exercise its power of judicial review only if the
following requisites are compresent: (1) the existence of an actual and appropriate case; (2) a personal
On December 30, 1993, the President signed the bill into law, and declared the same to have become and substantial interest of the party raising the constitutional question; (3) the exercise of judicial review is
Republic Act No. 7663, entitled "AN ACT APPROPRIATING FUNDS FOR THE OPERATION OF THE pleaded at the earliest opportunity; and (4) the constitutional question is the lis mota of the case (Luz
GOVERNMENT OF THE PHILIPPINES FROM JANUARY ONE TO DECEMBER THIRTY ONE, Farms v. Secretary of the Department of Agrarian Reform, 192 SCRA 51 [1990]; Dumlao v. Commission
NINETEEN HUNDRED AND NINETY-FOUR, AND FOR OTHER PURPOSES" (GAA of 1994). On the on Elections, 95 SCRA 392 [1980]; People v. Vera, 65 Phil. 56 [1937]).
same day, the President delivered his Presidential Veto Message, specifying the provisions of the bill he
vetoed and on which he imposed certain conditions. While the Solicitor General did not question the locus standi of petitioners in G.R. No. 113105, he claimed
that the remedy of the Senators in the other petitions is political (i.e., to override the vetoes) in effect
No step was taken in either House of Congress to override the vetoes. saying that they do not have the requisite legal standing to bring the suits.

In G.R. No. 113105, the Philippine Constitution Association, Exequiel B. Garcia and Ramon A. Gonzales The legal standing of the Senate, as an institution, was recognized in Gonzales v. Macaraig, Jr., 191
as taxpayers, prayed for a writ of prohibition to declare as unconstitutional and void: (a) Article XLI on the SCRA 452 (1990). In said case, 23 Senators, comprising the entire membership of the Upper House of
Countrywide Development Fund, the special provision in Article I entitled Realignment of Allocation for Congress, filed a petition to nullify the presidential veto of Section 55 of the GAA of 1989. The filing of the
Operational Expenses, and Article XLVIII on the Appropriation for Debt Service or the amount suit was authorized by Senate Resolution No. 381, adopted on February 2, 1989, and which reads as
appropriated under said Article XLVIII in excess of the P37.9 Billion allocated for the Department of follows:

113
Authorizing and Directing the Committee on Finance to Bring in the Name of the Senate of the Philippines Article XLI of the GAA of 1994 sets up a Countrywide Development Fund of P2,977,000,000.00 to "be
the Proper Suit with the Supreme Court of the Philippines contesting the Constitutionality of the Veto by used for infrastructure, purchase of ambulances and computers and other priority projects and activities
the President of Special and General Provisions, particularly Section 55, of the General Appropriation Bill and credit facilities to qualified beneficiaries." Said Article provides:
of 1989 (H.B. No. 19186) and For Other Purposes.
COUNTRYWIDE DEVELOPMENT FUND
In the United States, the legal standing of a House of Congress to sue has been recognized (United
States v. American Tel. & Tel. Co., 551 F. 2d 384, 391 [1976]; Notes: Congressional Access To The For Fund requirements of countrywide
Federal Courts, 90 Harvard Law Review 1632 [1977]). development projects P 2,977,000,000
———————
While the petition in G.R. No. 113174 was filed by 16 Senators, including the Senate President and the
Chairman of the Committee on Finance, the suit was not authorized by the Senate itself. Likewise, the New Appropriations, by Purpose
petitions in Current Operating Expenditures
G.R. Nos. 113766 and 113888 were filed without an enabling resolution for the purpose.
A. PURPOSE
Therefore, the question of the legal standing of petitioners in the three cases becomes a preliminary issue
before this Court can inquire into the validity of the presidential veto and the conditions for the
implementation of some items in the GAA of 1994. Personal Maintenance Capital Total
Services and Other Outlays
Operating
We rule that a member of the Senate, and of the House of Representatives for that matter, has the legal Expenses
standing to question the validity of a presidential veto or a condition imposed on an item in an
appropriation bill.
1. For Countrywide
Developments Projects P250,000,000 P2,727,000,000 P2,977,000,000
Where the veto is claimed to have been made without or in excess of the authority vested on the
President by the Constitution, the issue of an impermissible intrusion of the Executive into the domain of TOTAL NEW
the Legislature arises (Notes: Congressional Standing To Challenge Executive Action, 122 University of APPROPRIATIONS P250,000,000 P2,727,000,000 P2,977,000,000
Pennsylvania Law Review 1366 [1974]).
Special Provisions
To the extent the power of Congress are impaired, so is the power of each member thereof, since his
office confers a right to participate in the exercise of the powers of that institution (Coleman v. Miller, 307
U.S. 433 [1939]; Holtzman v. Schlesinger, 484 F. 2d 1307 [1973]). 1. Use and Release of Funds. The amount herein appropriated shall be used for infrastructure, purchase
of ambulances and computers and other priority projects and activities, and credit facilities to qualified
beneficiaries as proposed and identified by officials concerned according to the following allocations:
An act of the Executive which injures the institution of Congress causes a derivative but nonetheless Representatives, P12,500,000 each; Senators, P18,000,000 each; Vice-President,
substantial injury, which can be questioned by a member of Congress (Kennedy v. Jones, 412 F. Supp. P20,000,000; PROVIDED, That, the said credit facilities shall be constituted as a revolving fund to be
353 [1976]). In such a case, any member of Congress can have a resort to the courts. administered by a government financial institution (GFI) as a trust fund for lending operations. Prior years
releases to local government units and national government agencies for this purpose shall be turned
Former Chief Justice Enrique M. Fernando, as Amicus Curiae, noted: over to the government financial institution which shall be the sole administrator of credit facilities released
from this fund.
This is, then, the clearest case of the Senate as a whole or individual Senators as such having a
substantial interest in the question at issue. It could likewise be said that there was the requisite injury to The fund shall be automatically released quarterly by way of Advice of Allotments and Notice of Cash
their rights as Senators. It would then be futile to raise any locus standi issue. Any intrusion into the Allocation directly to the assigned implementing agency not later than five (5) days after the beginning of
domain appertaining to the Senate is to be resisted. Similarly, if the situation were reversed, and it is the each quarter upon submission of the list of projects and activities by the officials concerned.
Executive Branch that could allege a transgression, its officials could likewise file the corresponding
action. What cannot be denied is that a Senator has standing to maintain inviolate the prerogatives, 2. Submission of Quarterly Reports. The Department of Budget and Management shall submit within thirty
powers and privileges vested by the Constitution in his office (Memorandum, p. 14). (30) days after the end of each quarter a report to the Senate Committee on Finance and the House
Committee on Appropriations on the releases made from this Fund. The report shall include the listing of
It is true that the Constitution provides a mechanism for overriding a veto (Art. VI, Sec. 27 [1]). Said the projects, locations, implementing agencies and the endorsing officials (GAA of 1994, p. 1245).
remedy, however, is available only when the presidential veto is based on policy or political considerations
but not when the veto is claimed to be ultra vires. In the latter case, it becomes the duty of the Court to Petitioners claim that the power given to the members of Congress to propose and identify the projects
draw the dividing line where the exercise of executive power ends and the bounds of legislative and activities to be funded by the Countrywide Development Fund is an encroachment by the legislature
jurisdiction begin. on executive power, since said power in an appropriation act in implementation of a law. They argue that
the proposal and identification of the projects do not involve the making of laws or the repeal and
III -G.R. No. 113105 amendment thereof, the only function given to the Congress by the Constitution (Rollo, pp. 78- 86).

1. Countrywide Development Fund Under the Constitution, the spending power called by James Madison as "the power of the purse,"
belongs to Congress, subject only to the veto power of the President. The President may propose the
budget, but still the final say on the matter of appropriations is lodged in the Congress.

114
The power of appropriation carries with it the power to specify the project or activity to be funded under Compensation Insurance Premiums 1,579
the appropriation law. It can be as detailed and as broad as Congress wants it to be. Pag-I.B.I.G. Contributions 1,184
Medicare Premiums 888
The Countrywide Development Fund is explicit that it shall be used "for infrastructure, purchase of Bonus and Cash Gift 14,791
ambulances and computers and other priority projects and activities and credit facilities to qualified Terminal Leave Benefits 2,000
beneficiaries . . ." It was Congress itself that determined the purposes for the appropriation. Personnel Economic Relief Allowance 10,266
Additional Compensation of P500 under A.O. 53 11,130
Others 57,173
Executive function under the Countrywide Development Fund involves implementation of the priority ————
projects specified in the law. Total Other Compensation 103,815
————
The authority given to the members of Congress is only to propose and identify projects to be 01 Total Personal Services 264,032
implemented by the President. Under Article XLI of the GAA of 1994, the President must perforce =======
examine whether the proposals submitted by the members of Congress fall within the specific items of
expenditures for which the Fund was set up, and if qualified, he next determines whether they are in line Maintenance and Other Operating Expenses
with other projects planned for the locality. Thereafter, if the proposed projects qualify for funding under
the Funds, it is the President who shall implement them. In short, the proposals and identifications made
by the members of Congress are merely recommendatory. 02 Traveling Expenses 32,841
03 Communication Services 7,666
04 Repair and Maintenance of Government Facilities 1,220
The procedure of proposing and identifying by members of Congress of particular projects or activities 05 Repair and Maintenance of Government Vehicles 318
under Article XLI of the GAA of 1994 is imaginative as it is innovative. 06 Transportation Services 128
07 Supplies and Materials 20,189
The Constitution is a framework of a workable government and its interpretation must take into account 08 Rents 24,584
the complexities, realities and politics attendant to the operation of the political branches of government. 14 Water/Illumination and Power 6,561
Prior to the GAA of 1991, there was an uneven allocation of appropriations for the constituents of the 15 Social Security Benefits and Other Claims 3,270
members of Congress, with the members close to the Congressional leadership or who hold cards for 17 Training and Seminars Expenses 2,225
"horse-trading," getting more than their less favored colleagues. The members of Congress also had to 18 Extraordinary and Miscellaneous Expenses 9,360
reckon with an unsympathetic President, who could exercise his veto power to cancel from the 23 Advertising and Publication
appropriation bill a pet project of a Representative or Senator. 24 Fidelity Bonds and Insurance Premiums 1,325
29 Other Services 89,778
The Countrywide Development Fund attempts to make equal the unequal. It is also a recognition that ————
individual members of Congress, far more than the President and their congressional colleagues are likely Total Maintenance and Other Operating Expenditures 200,415
to be knowledgeable about the needs of their respective constituents and the priority to be given each ————
project. Total Current Operating Expenditures 464,447
=======
2. Realignment of Operating Expenses
(GAA of 1994, pp. 3-4)
Under the GAA of 1994, the appropriation for the Senate is P472,000,000.00 of which P464,447,000.00 is
appropriated for current operating expenditures, while the appropriation for the House of Representatives The 1994 operating expenditures for the House of Representatives are as follows:
is P1,171,924,000.00 of which P1,165,297,000.00 is appropriated for current operating expenditures
(GAA of 1994, pp. 2, 4, 9, 12). Personal Services

The 1994 operating expenditures for the Senate are as follows: Salaries, Permanent 261,557
Salaries/Wages, Contractual/Emergency 143,643
Personal Services ————
Total Salaries and Wages 405,200
=======
Salaries, Permanent 153,347
Salaries/Wage, Contractual/Emergency 6,870
———— Other Compensation
Total Salaries and Wages 160,217
======= Step Increments 4,312
Honoraria and Commutable
Other Compensation Allowances 4,764
Compensation Insurance
Premiums 1,159
 
Pag-I.B.I.G. Contributions 5,231
Medicare Premiums 2,281
Step Increments 1,073
Honoraria and Commutable Allowances 3,731 Bonus and Cash Gift 35,669

115
Terminal Leave Benefits 29 The proviso of said Article of the Constitution grants the President of the Senate and the Speaker of the
Personnel Economic Relief House of Representatives the power to augment items in an appropriation act for their respective offices
Allowance 21,150 from savings in other items of their appropriations, whenever there is a law authorizing such
Additional Compensation of P500 under A.O. 53 augmentation.
Others 106,140
———— The special provision on realignment of the operating expenses of members of Congress is authorized by
Total Other Compensation 202,863 Section 16 of the General Provisions of the GAA of 1994, which provides:
————
01 Total Personal Services 608,063
======= Expenditure Components. Except by act of the Congress of the Philippines, no change or modification
shall be made in the expenditure items authorized in this Act and other appropriation laws unless in cases
of augmentations from savings in appropriations as authorized under Section 25(5) of Article VI of the
Maintenance and Other Operating Expenses Constitution (GAA of 1994, p. 1273).

02 Traveling Expenses 139,611 Petitioners argue that the Senate President and the Speaker of the House of Representatives, but not the
03 Communication Services 22,514 individual members of Congress are the ones authorized to realign the savings as appropriated.
04 Repair and Maintenance of Government Facilities 5,116
05 Repair and Maintenance of Government Vehicles 1,863
06 Transportation Services 178 Under the Special Provisions applicable to the Congress of the Philippines, the members of Congress
07 Supplies and Materials 55,248 only determine the necessity of the realignment of the savings in the allotments for their operating
10 Grants/Subsidies/Contributions 940 expenses. They are in the best position to do so because they are the ones who know whether there are
14 Water/Illumination and Power 14,458 savings available in some items and whether there are deficiencies in other items of their operating
15 Social Security Benefits and Other Claims 325 expenses that need augmentation. However, it is the Senate President and the Speaker of the House of
17 Training and Seminars Expenses 7,236 Representatives, as the case may be, who shall approve the realignment. Before giving their stamp of
18 Extraordinary and Miscellaneous Expenses 14,474 approval, these two officials will have to see to it that:
20 Anti-Insurgency/Contingency Emergency Expenses 9,400
23 Advertising and Publication 242 (1) The funds to be realigned or transferred are actually savings in the items of expenditures from which
24 Fidelity Bonds and Insurance Premiums 1,420 the same are to be taken; and
29 Other Services 284,209
———— (2) The transfer or realignment is for the purposes of augmenting the items of expenditure to which said
Total Maintenance and Other Operating Expenditures 557,234 transfer or realignment is to be made.
————
Total Current Operating Expenditures 1,165,297
3. Highest Priority for Debt Service
=======

While Congress appropriated P86,323,438,000.00 for debt service (Article XLVII of the GAA of 1994), it
(GAA of 1994, pp. 11-12)
appropriated only P37,780,450,000.00 for the Department of Education Culture and Sports. Petitioners
urged that Congress cannot give debt service the highest priority in the GAA of 1994 (Rollo, pp. 93-94)
The Special Provision Applicable to the Congress of the Philippines provides: because under the Constitution it should be education that is entitled to the highest funding. They invoke
Section 5(5), Article XIV thereof, which provides:
4. Realignment of Allocation for Operational Expenses. A member of Congress may realign his allocation
for operational expenses to any other expenses category provide the total of said allocation is not (5) The State shall assign the highest budgetary priority to education and ensure that teaching will attract
exceeded. (GAA of 1994, p. 14). and retain its rightful share of the best available talents through adequate remuneration and other means
of job satisfaction and fulfillment.
The appropriation for operating expenditures for each House is further divided into expenditures for
salaries, personal services, other compensation benefits, maintenance expenses and other operating This issue was raised in Guingona, Jr. v. Carague, 196 SCRA 221 (1991), where this Court held that
expenses. In turn, each member of Congress is allotted for his own operating expenditure a proportionate Section 5(5), Article XIV of the Constitution, is merely directory, thus:
share of the appropriation for the House to which he belongs. If he does not spend for one items of
expense, the provision in question allows him to transfer his allocation in said item to another item of
While it is true that under Section 5(5), Article XIV of the Constitution, Congress is mandated to "assign
expense.
the highest budgetary priority to education" in order to "insure that teaching will attract and retain its
rightful share of the best available talents through adequate remuneration and other means of job
Petitioners assail the special provision allowing a member of Congress to realign his allocation for satisfaction and fulfillment," it does not thereby follow that the hands of Congress are so hamstrung as to
operational expenses to any other expense category (Rollo, pp. 82-92), claiming that this practice is deprive it the power to respond to the imperatives of the national interest and for the attainment of other
prohibited by Section 25(5), Article VI of the Constitution. Said section provides: state policies or objectives.

No law shall be passed authorizing any transfer of appropriations: however, the President, the President As aptly observed by respondents, since 1985, the budget for education has tripled to upgrade and
of the Senate, the Speaker of the House of Representatives, the Chief Justice of the Supreme Court, and improve the facility of the public school system. The compensation of teachers has been doubled. The
the heads of Constitutional Commissions may, by law, be authorized to augment any item in the general amount of P29,740,611,000.00 set aside for the Department of Education, Culture and Sports under the
appropriations law for their respective offices from savings in other items of their respective General Appropriations Act (R.A. No. 6381), is the highest budgetary allocation among all department
appropriations. budgets. This is a clear compliance with the aforesaid constitutional mandate according highest priority to
education.

116
Having faithfully complied therewith, Congress is certainly not without any power, guided only by its good Petitioners claim that the President cannot veto the Special Provision on the appropriation for debt service
judgment, to provide an appropriation, that can reasonably service our enormous debt, the greater portion without vetoing the entire amount of P86,323,438.00 for said purpose (Rollo, G.R. No. 113105, pp. 93-
of which was inherited from the previous administration. It is not only a matter of honor and to protect the 98; Rollo, G.R. No. 113174, pp. 16-18). The Solicitor General counterposed that the Special Provision did
credit standing of the country. More especially, the very survival of our economy is at stake. Thus, if in the not relate to the item of appropriation for debt service and could therefore be the subject of an item veto
process Congress appropriated an amount for debt service bigger than the share allocated to education, (Rollo, G.R. No. 113105, pp. 54-60; Rollo, G.R. No. 113174, pp. 72-82).
the Court finds and so holds that said appropriation cannot be thereby assailed as unconstitutional.
This issue is a mere rehash of the one put to rest in Gonzales v. Macaraig, Jr., 191 SCRA 452 (1990). In
G.R. No. 113105 that case, the issue was stated by the Court, thus:
G.R. No. 113174
The fundamental issue raised is whether or not the veto by the President of Section 55 of the 1989
Veto of Provision on Debt Ceiling Appropriations Bill (Section 55
FY '89), and subsequently of its counterpart Section 16 of the 1990 Appropriations Bill (Section 16 FY
The Congress added a Special Provision to Article XLVIII (Appropriations for Debt Service) of the GAA of '90), is unconstitutional and without effect.
1994 which provides:
The Court re-stated the issue, just so there would not be any misunderstanding about it, thus:
Special Provisions
The focal issue for resolution is whether or not the President exceeded the item-veto power accorded by
1. Use of the Fund. The appropriation authorized herein shall be used for payment of principal and the Constitution. Or differently put, has the President the power to veto "provisions" of an Appropriations
interest of foreign and domestic indebtedness; PROVIDED, That any payment in excess of the amount Bill?
herein appropriated shall be subject to the approval of the President of the Philippines with the
concurrence of the Congress of the Philippines; PROVIDED, FURTHER, That in no case shall this fund The bases of the petition in Gonzales, which are similar to those invoked in the present case, are stated
be used to pay for the liabilities of the Central Bank Board of Liquidators. as follows:

2. Reporting Requirement. The Bangko Sentral ng Pilipinas and the Department of Finance shall submit a In essence, petitioners' cause is anchored on the following grounds: (1) the President's line-veto power as
quarterly report of actual foreign and domestic debt service payments to the House Committee on regards appropriation bills is limited to item/s and does not cover provision/s; therefore, she exceeded her
Appropriations and Senate Finance Committee within one (1) month after each quarter (GAA of 1944, pp. authority when she vetoed Section 55 (FY '89) and Section 16 (FY '90) which are provisions; (2) when the
1266). President objects to a provision of an appropriation bill, she cannot exercise the item-veto power but
should veto the entire bill; (3) the item-veto power does not carry with it the power to strike out conditions
The President vetoed the first Special Provision, without vetoing the P86,323,438,000.00 appropriation for or restrictions for that would be legislation, in violation of the doctrine of separation of powers; and (4) the
debt service in said Article. According to the President's Veto Message: power of augmentation in Article VI, Section 25 [5] of the 1987 Constitution, has to be provided for by law
and, therefore, Congress is also vested with the prerogative to impose restrictions on the exercise of that
power.
IV. APPROPRIATIONS FOR DEBT SERVICE
The restrictive interpretation urged by petitioners that the President may not veto a provision without
I would like to emphasize that I concur fully with the desire of Congress to reduce the debt burden by vetoing the entire bill not only disregards the basic principle that a distinct and severable part of a bill may
decreasing the appropriation for debt service as well as the inclusion of the Special Provision quoted be the subject of a separate veto but also overlooks the Constitutional mandate that any provision in the
below. Nevertheless, I believe that this debt reduction scheme cannot be validly done through the 1994 general appropriations bill shall relate specifically to some particular appropriation therein and that any
GAA. This must be addressed by revising our debt policy by way of innovative and comprehensive debt such provision shall be limited in its operation to the appropriation to which it relates (1987 Constitution,
reduction programs conceptualized within the ambit of the Medium-Term Philippine Development Plan. Article VI, Section 25 [2]). In other words, in the true sense of the term, a provision in an Appropriations
Bill is limited in its operation to some particular appropriation to which it relates, and does not relate to the
Appropriations for payment of public debt, whether foreign or domestic, are automatically appropriated entire bill.
pursuant to the Foreign Borrowing Act and Section 31 of P.D. No. 1177 as reiterated under Section 26,
Chapter 4, Book VI of E.O. No. 292, the Administrative Code of 1987. I wish to emphasize that the The Court went one step further and ruled that even assuming arguendo that "provisions" are beyond the
constitutionality of such automatic provisions on debt servicing has been upheld by the Supreme Court in executive power to veto, and Section 55
the case of "Teofisto T. Guingona, Jr., and Aquilino Q. Pimentel, Jr. v. Hon. Guillermo N. Carague, in his (FY '89) and Section 16 (FY '90) were not "provisions" in the budgetary sense of the term, they are
capacity as Secretary of Budget and Management, et al.," G.R. No. 94571, dated April 22, 1991. "inappropriate provisions" that should be treated as "items" for the purpose of the President's veto power.

I am, therefore vetoing the following special provision for the reason that the GAA is not the appropriate The Court, citing Henry v. Edwards, La., 346 So. 2d 153 (1977), said that Congress cannot include in a
legislative measure to amend the provisions of the Foreign Borrowing Act, P.D. No. 1177 and E.O. No. general appropriations bill matters that should be more properly enacted in separate legislation, and if it
292: does that, the inappropriate provisions inserted by it must be treated as "item", which can be vetoed by
the President in the exercise of his item-veto power.
Use of the Fund. The appropriation authorized herein shall be used for payment of principal and interest
of foreign and domestic indebtedness: PROVIDED, That any payment in excess of the amount herein It is readily apparent that the Special Provision applicable to the appropriation for debt service insofar as it
appropriated shall be subject to the approval of the President of the Philippines with the concurrence of refers to funds in excess of the amount appropriated in the bill, is an "inappropriate" provision referring to
the Congress of the Philippines: PROVIDED, FURTHER, That in no case shall this fund be used to pay funds other than the P86,323,438,000.00 appropriated in the General Appropriations Act of 1991.
for the liabilities of the Central Bank Board of Liquidators (GAA of 1994, p. 1290).

117
Likewise the vetoed provision is clearly an attempt to repeal Section 31 of P.D. No. 1177 (Foreign exercise of powers of the President given by the Constitution for that would be an unconstitutional
Borrowing Act) and E.O. No. 292, and to reverse the debt payment policy. As held by the Court intrusion into executive prerogative.
in Gonzales, the repeal of these laws should be done in a separate law, not in the appropriations law.
The doctrine of "inappropriate provision" was well elucidated in Henry v. Edwards, supra., thus:
The Court will indulge every intendment in favor of the constitutionality of a veto, the same as it will
presume the constitutionality of an act of Congress (Texas Co. v. State, 254 P. 1060; 31 Ariz, 485, 53 Just as the President may not use his item-veto to usurp constitutional powers conferred on the
A.L.R. 258 [1927]). legislature, neither can the legislature deprive the Governor of the constitutional powers conferred on him
as chief executive officer of the state by including in a general appropriation bill matters more properly
The veto power, while exercisable by the President, is actually a part of the legislative process enacted in separate legislation. The Governor's constitutional power to veto bills of general legislation . . .
(Memorandum of Justice Irene Cortes as Amicus Curiae, pp. 3-7). That is why it is found in Article VI on cannot be abridged by the careful placement of such measures in a general appropriation bill, thereby
the Legislative Department rather than in Article VII on the Executive Department in the Constitution. forcing the Governor to choose between approving unacceptable substantive legislation or vetoing "items"
There is, therefore, sound basis to indulge in the presumption of validity of a veto. The burden shifts on of expenditures essential to the operation of government. The legislature cannot by location of a bill give it
those questioning the validity thereof to show that its use is a violation of the Constitution. immunity from executive veto. Nor can it circumvent the Governor's veto power over substantive
legislation by artfully drafting general law measures so that they appear to be true conditions or limitations
Under his general veto power, the President has to veto the entire bill, not merely parts thereof (1987 on an item of appropriation. Otherwise, the legislature would be permitted to impair the constitutional
Constitution, Art. VI, Sec. 27[1]). The exception to the general veto power is the power given to the responsibilities and functions of a co-equal branch of government in contravention of the separation of
President to veto any particular item or items in a general appropriations bill (1987 Constitution, Art. VI, powers doctrine . . . We are no more willing to allow the legislature to use its appropriation power to
Sec. 27[2]). In so doing, the President must veto the entire item. infringe on the Governor's constitutional right to veto matters of substantive legislation than we are to
allow the Governor to encroach on the Constitutional powers of the legislature. In order to avoid this
result, we hold that, when the legislature inserts inappropriate provisions in a general appropriation bill,
A general appropriations bill is a special type of legislation, whose content is limited to specified sums of such provisions must be treated as "items" for purposes of the Governor's item veto power over general
money dedicated to a specific purpose or a separate fiscal unit (Beckman, The Item Veto Power of the appropriation bills.
Executive,
31 Temple Law Quarterly 27 [1957]).
. . . Legislative control cannot be exercised in such a manner as to encumber the general appropriation bill
with veto-proof "logrolling measures", special interest provisions which could not succeed if separately
The item veto was first introduced by the Organic Act of the Philippines passed by the U.S. Congress on enacted, or "riders", substantive pieces of legislation incorporated in a bill to insure passage without veto .
August 29, 1916. The concept was adopted from some State Constitutions. . . (Emphasis supplied).

Cognizant of the legislative practice of inserting provisions, including conditions, restrictions and Petitioners contend that granting arguendo that the veto of the Special Provision on the ceiling for debt
limitations, to items in appropriations bills, the Constitutional Convention added the following sentence to payment is valid, the President cannot automatically appropriate funds for debt payment without
Section 20(2), Article VI of the 1935 Constitution: complying with the conditions for automatic appropriation under the provisions of R.A. No. 4860 as
amended by P.D. No. 81 and the provisions of P.D. No. 1177 as amended by the Administrative Code of
. . . When a provision of an appropriation bill affect one or more items of the same, the President cannot 1987 and P.D. No. 1967 (Rollo, G.R. No. 113766, pp. 9-15).
veto the provision without at the same time vetoing the particular item or items to which it relates . . . .
Petitioners cannot anticipate that the President will not faithfully execute the laws. The writ of prohibition
In short, under the 1935 Constitution, the President was empowered to veto separately not only items in will not issue on the fear that official actions will be done in contravention of the laws.
an appropriations bill but also "provisions".
The President vetoed the entire paragraph one of the Special Provision of the item on debt service,
While the 1987 Constitution did not retain the aforementioned sentence added to Section 11(2) of Article including the provisions that the appropriation authorized in said item "shall be used for payment of the
VI of the 1935 Constitution, it included the following provision: principal and interest of foreign and domestic indebtedness" and that "in no case shall this fund be used to
pay for the liabilities of the Central Bank Board of Liquidators." These provisions are germane to and have
No provision or enactment shall be embraced in the general appropriations bill unless it relates specifically a direct connection with the item on debt service. Inherent in the power of appropriation is the power to
to some particular appropriation therein. Any such provision or enactment shall be limited in its operation specify how the money shall be spent (Henry v. Edwards, LA, 346 So., 2d., 153). The said provisos, being
to the appropriation to which it relates (Art. VI, Sec. 25[2]). appropriate provisions, cannot be vetoed separately. Hence the item veto of said provisions is void.

In Gonzales, we made it clear that the omission of that sentence of Section 16(2) of the 1935 Constitution We reiterate, in order to obviate any misunderstanding, that we are sustaining the veto of the Special
in the 1987 Constitution should not be interpreted to mean the disallowance of the power of the President Provision of the item on debt service only with respect to the proviso therein requiring that "any payment
to veto a "provision". in excess of the amount herein, appropriated shall be subject to the approval of the President of the
Philippines with the concurrence of the Congress of the Philippines . . ."
As the Constitution is explicit that the provision which Congress can include in an appropriations bill must
"relate specifically to some particular appropriation therein" and "be limited in its operation to the G.R. NO. 113174
appropriation to which it relates," it follows that any provision which does not relate to any particular item, G.R. NO. 113766
or which extends in its operation beyond an item of appropriation, is considered "an inappropriate G.R. NO. 11388
provision" which can be vetoed separately from an item. Also to be included in the category of
"inappropriate provisions" are unconstitutional provisions and provisions which are intended to amend 1. Veto of provisions for revolving funds of SUC's.
other laws, because clearly these kind of laws have no place in an appropriations bill. These are matters
of general legislation more appropriately dealt with in separate enactments. Former Justice Irene Cortes,
as Amicus Curiae, commented that Congress cannot by law establish conditions for and regulate the

118
In the appropriation for State Universities and Colleges (SUC's), the President vetoed special provisions Notwithstanding the aforementioned provisions of the Constitution and existing law, I have noted the
which authorize the use of income and the creation, operation and maintenance of revolving funds. The proliferation of special provisions authorizing the use of agency income as well as the creation, operation
Special Provisions vetoed are the following: and maintenance of revolving funds.

(H. 7) West Visayas State University I would like to underscore the facts that such income were already considered as integral part of the
revenue and financing sources of the National Expenditure Program which I previously submitted to
Equal Sharing of Income. Income earned by the University subject to Section 13 of the special provisions Congress. Hence, the grant of new special provisions authorizing the use of agency income and the
applicable to all State Universities and Colleges shall be equally shared by the University and the establishment of revolving funds over and above the agency appropriations authorized in this Act shall
University Hospital (GAA of 1994, p. 395). effectively reduce the financing sources of the 1994 GAA and, at the same time, increase the level of
expenditures of some agencies beyond the well-coordinated, rationalized levels for such agencies. This
corresponding increases the overall deficit of the National Government (Veto Message, p. 3).
(J. 3) Leyte State College
Petitioners claim that the President acted with grave abuse of discretion when he disallowed by his veto
Revolving Fund for the Operation of LSC House and Human Resources Development Center (HRDC). the "use of income" and the creation of "revolving fund" by the Western Visayas State University and
The income of Leyte State College derived from the operation of its LSC House and HRDC shall be Leyte State Colleges when he allowed other government offices, like the National Stud Farm, to use their
constituted into a Revolving Fund to be deposited in an authorized government depository bank for the income for their operating expenses (Rollo, G.R. No. 113174, pp. 15-16).
operational expenses of these projects/services. The net income of the Revolving Fund at the end of the
year shall be remitted to the National Treasury and shall accrue to the General Fund. The implementing
guidelines shall be issued by the Department of Budget and Management (GAA of 1994, p. 415). There was no undue discrimination when the President vetoed said special provisions while allowing
similar provisions in other government agencies. If some government agencies were allowed to use their
income and maintain a revolving fund for that purpose, it is because these agencies have been enjoying
The vetoed Special Provisions applicable to all SUC's are the following: such privilege before by virtue of the special laws authorizing such practices as exceptions to the "one-
fund policy" (e.g., R.A. No. 4618 for the National Stud Farm, P.D. No. 902-A for the Securities and
12. Use of Income from Extension Services. State Universities and Colleges are authorized to use their Exchange Commission; E.O. No. 359 for the Department of Budget and Management's Procurement
income from their extension services. Subject to the approval of the Board of Regents and the approval of Service).
a special budget pursuant to Sec. 35, Chapter 5, Book VI of E.O.
No. 292, such income shall be utilized solely for faculty development, instructional materials and work 2. Veto of provision on 70% (administrative)/30% (contract) ratio for road maintenance.
study program (GAA of 1994, p. 490).
In the appropriation for the Department of Public Works and Highways, the President vetoed the second
13. Income of State Universities and Colleges. The income of State Universities and Colleges derived paragraph of Special Provision No. 2, specifying the 30% maximum ration of works to be contracted for
from tuition fees and other sources as may be imposed by governing boards other than those accruing to the maintenance of national roads and bridges. The said paragraph reads as follows:
revolving funds created under LOI Nos. 872 and 1026 and those authorized to be recorded as trust
receipts pursuant to Section 40, Chapter 5, Book VI of E.O. No. 292 shall be deposited with the National
Treasury and recorded as a Special Account in the General Fund pursuant to P.D. No. 1234 and P.D. No. 2. Release and Use of Road Maintenance Funds. Funds allotted for the maintenance and repair of roads
1437 for the use of the institution, subject to Section 35, Chapter 5, Book VI of E.O. No. which are provided in this Act for the Department of Public Works and Highways shall be released to the
292L PROVIDED, That disbursements from the Special Account shall not exceed the amount actually respective Engineering District, subject to such rules and regulations as may be prescribed by the
earned and deposited: PROVIDED, FURTHER, That a cash advance on such income may be allowed Department of Budget and Management. Maintenance funds for roads and bridges shall be exempt from
State half of income actually realized during the preceding year and this cash advance shall be charged budgetary reserve.
against income actually earned during the budget year: AND PROVIDED, FINALLY, That in no case shall
such funds be used to create positions, nor for payment of salaries, wages or allowances, except as may Of the amount herein appropriated for the maintenance of national roads and bridges, a maximum of
be specifically approved by the Department of Budge and Management for income-producing activities, or thirty percent (30%) shall be contracted out in accordance with guidelines to be issued by the Department
to purchase equipment or books, without the prior approval of the President of the Philippines pursuant to of Public Works and Highways. The balance shall be used for maintenance by force account.
Letter of Implementation No. 29.
Five percent (5%) of the total road maintenance fund appropriated herein to be applied across the board
All collections of the State Universities and Colleges for fees, charges and receipts intended for private to the allocation of each region shall be set aside for the maintenance of roads which may be converted to
recipient units, including private foundations affiliated with these institutions shall be duly acknowledged or taken over as national roads during the current year and the same shall be released to the central
with official receipts and deposited as a trust receipt before said income shall be subject to Section 35, office of the said department for eventual
Chapter 5, Book VI of E.O. No. 292 sub-allotment to the concerned region and district: PROVIDED, That any balance of the said five percent
(GAA of 1994, p. 490). (5%) shall be restored to the regions on a pro-rata basis for the maintenance of existing national roads.

The President gave his reason for the veto thus: No retention or deduction as reserves or overhead expenses shall be made, except as authorized by law
or upon direction of the President
Pursuant to Section 65 of the Government Auditing Code of the Philippines, Section 44, Chapter 5, Book (GAA of 1994, pp. 785-786; Emphasis supplied).
VI of E.O. No. 292, s. 1987 and Section 22, Article VII of the Constitution, all income earned by all
Government offices and agencies shall accrue to the General Fund of the Government in line with the The President gave the following reason for the veto:
One Fund Policy enunciated by Section 29 (1), Article VI and Section 22, Article VII of the Constitution.
Likewise, the creation and establishment of revolving funds shall be authorized by substantive law While I am cognizant of the well-intended desire of Congress to impose certain restrictions contained in
pursuant to Section 66 of the Government Auditing Code of the Philippines and Section 45, Chapter 5, some special provisions, I am equally aware that many programs, projects and activities of agencies
Book VI of E.O. No. 292. would require some degree of flexibility to ensure their successful implementation and therefore risk their

119
completion. Furthermore, not only could these restrictions and limitations derail and impede program period for the smooth implementation of the law in the case of purchases by the Armed Forces of the
implementation but they may also result in a breach of contractual obligations. Philippines, as implied by Section 11 (Education Drive) of the law itself. This belief, however, cannot
justify his veto of the provision on the purchase of medicines by the AFP.
D.1.a. A study conducted by the Infrastructure Agencies show that for practical intent and purposes,
maintenance by contract could be undertaken to an optimum of seventy percent (70%) and the remaining Being directly related to and inseparable from the appropriation item on purchases of medicines by the
thirty percent (30%) by force account. Moreover, the policy of maximizing implementation through contract AFP, the special provision cannot be vetoed by the President without also vetoing the said item (Bolinao
maintenance is a covenant of the Road and Road Transport Program Loan from the Asian Development Electronics Corporation v. Valencia, 11 SCRA 486 [1964]).
Bank (ADB Loan No. 1047-PHI-1990) and Overseas Economic Cooperation Fund (OECF Loan No. PH-
C17-199). The same is a covenant under the World Bank (IBRD) Loan for the Highway Management 4. Veto of provision on prior approval of Congress for purchase of military equipment.
Project (IBRD Loan
No. PH-3430) obtained in 1992.
In the appropriation for the modernization of the AFP, the President vetoed the underlined proviso of
Special Provision No. 2 on the "Use of Fund," which requires the prior approval of Congress for the
In the light of the foregoing and considering the policy of the government to encourage and maximize release of the corresponding modernization funds, as well as the entire Special Provisions
private sector participation in the regular repair and maintenance of infrastructure facilities, I am directly No. 3 on the "Specific Prohibition":
vetoing the underlined second paragraph of Special Provision No. 2 of the Department of Public Works
and Highways (Veto Message, p. 11).
2. Use of the Fund. Of the amount herein appropriated, priority shall be given for the acquisition of AFP
assets necessary for protecting marine, mineral, forest and other resources within Philippine territorial
The second paragraph of Special Provision No. 2 brings to fore the divergence in policy of Congress and borders and its economic zone, detection, prevention or deterrence of air or surface intrusions and to
the President. While Congress expressly laid down the condition that only 30% of the total appropriation support diplomatic moves aimed at preserving national dignity, sovereignty and patrimony: PROVIDED,
for road maintenance should be contracted out, the President, on the basis of a comprehensive study, That the said modernization fund shall not be released until a Table of Organization and Equipment for
believed that contracting out road maintenance projects at an option of 70% would be more efficient, FY 1994-2000 is submitted to and approved by Congress.
economical and practical.
3. Specific Prohibition. The said Modernization Fund shall not be used for payment of six (6) additional S-
The Special Provision in question is not an inappropriate provision which can be the subject of a veto. It is 211 Trainer planes, 18 SF-260 Trainer planes and 150 armored personnel carriers (GAA of 1994, p. 747).
not alien to the appropriation for road maintenance, and on the other hand, it specified how the said item
shall be expended — 70% by administrative and 30% by contract.
As reason for the veto, the President stated that the said condition and prohibition violate the
Constitutional mandate of non-impairment of contractual obligations, and if allowed, "shall effectively alter
The 1987 Constitution allows the addition by Congress of special provisions, conditions to items in an the original intent of the AFP Modernization Fund to cover all military equipment deemed necessary to
expenditure bill, which cannot be vetoed separately from the items to which they relate so long as they are modernize the Armed Forces of the Philippines" (Veto Message, p. 12).
"appropriate" in the budgetary sense (Art. VII, Sec. 25[2]).
Petitioners claim that Special Provision No. 2 on the "Use of Fund" and Special Provision No. 3 are
The Solicitor General was hard put in justifying the veto of this special provision. He merely argued that conditions or limitations related to the item on the AFP modernization plan.
the provision is a complete turnabout from an entrenched practice of the government to maximize contract
maintenance (Rollo, G.R. No. 113888, pp. 85-86). That is not a ground to veto a provision separate from
the item to which it refers. The requirement in Special Provision No. 2 on the "Use of Fund" for the AFP modernization program that
the President must submit all purchases of military equipment to Congress for its approval, is an exercise
of the "congressional or legislative veto." By way of definition, a congressional veto is a means whereby
The veto of the second paragraph of Special Provision No. 2 of the item for the DPWH is therefore the legislature can block or modify administrative action taken under a statute. It is a form of legislative
unconstitutional. control in the implementation of particular executive actions. The form may be either negative, that is
requiring disapproval of the executive action, or affirmative, requiring approval of the executive action.
3. Veto of provision on purchase of medicines by AFP. This device represents a significant attempt by Congress to move from oversight of the executive to
shared administration (Dixon, The Congressional Veto and Separation of Powers: The Executive on a
In the appropriation for the Armed Forces of the Philippines (AFP), the President vetoed the special Leash,
provision on the purchase by the AFP of medicines in compliance with the Generics Drugs Law (R.A. No. 56 North Carolina Law Review, 423 [1978]).
6675). The vetoed provision reads:
A congressional veto is subject to serious questions involving the principle of separation of powers.
12. Purchase of Medicines. The purchase of medicines by all Armed Forces of the Philippines units,
hospitals and clinics shall strictly comply with the formulary embodied in the National Drug Policy of the However the case at bench is not the proper occasion to resolve the issues of the validity of the legislative
Department of Health (GAA of 1994, p. 748). veto as provided in Special Provisions Nos. 2 and 3 because the issues at hand can be disposed of on
other grounds. Any provision blocking an administrative action in implementing a law or requiring
According to the President, while it is desirable to subject the purchase of medicines to a standard legislative approval of executive acts must be incorporated in a separate and substantive bill. Therefore,
formulary, "it is believed more prudent to provide for a transition period for its adoption and smooth being "inappropriate" provisions, Special Provisions Nos. 2 and 3 were properly vetoed.
implementation in the Armed Forces of the Philippines" (Veto Message, p. 12).
As commented by Justice Irene Cortes in her memorandum as Amicus Curiae: "What Congress cannot
The Special Provision which requires that all purchases of medicines by the AFP should strictly comply do directly by law it cannot do indirectly by attaching conditions to the exercise of that power (of the
with the formulary embodied in the National Drug Policy of the Department of Health is an "appropriate" President as Commander-in-Chief) through provisions in the appropriation law."
provision. it is a mere advertence by Congress to the fact that there is an existing law, the Generics Act of
1988, that requires "the extensive use of drugs with generic names through a rational system of Furthermore, Special Provision No. 3, prohibiting the use of the Modernization Funds for payment of the
procurement and distribution." The President believes that it is more prudent to provide for a transition trainer planes and armored personnel carriers, which have been contracted for by the AFP, is violative of

120
the Constitutional prohibition on the passage of laws that impair the obligation of contracts (Art. III, Sec. this special provision shall be subject to prior Presidential approval pursuant to the provisions of P.D. No.
10), more so, contracts entered into by the Government itself. 1597 and
R.A. No. 6758 (Veto Message, p. 13).
The veto of said special provision is therefore valid.
Petitioners claim that the Congress has required the deactivation of the CAFGU's when it appropriated the
5. Veto of provision on use of savings to augment AFP pension funds. money for payment of the separation pay of the members of thereof. The President, however, directed
that the deactivation should be done in accordance to his timetable, taking into consideration the peace
and order situation in the affected localities.
In the appropriation for the AFP Pension and Gratuity Fund, the President vetoed the new provision
authorizing the Chief of Staff to use savings in the AFP to augment pension and gratuity funds. The
vetoed provision reads: Petitioners complain that the directive of the President was tantamount to an administrative embargo of
the congressional will to implement the Constitution's command to dissolve the CAFGU's (Rollo, G.R. No.
113174,
2. Use of Savings. The Chief of Staff, AFP, is authorized, subject to the approval of the Secretary of p. 14; G.R. No. 113888, pp. 9, 14-16). They argue that the President cannot impair or withhold
National Defense, to use savings in the appropriations provided herein to augment the pension fund being expenditures authorized and appropriated by Congress when neither the Appropriations Act nor other
managed by the AFP Retirement and Separation Benefits System as provided under Sections 2(a) and 3 legislation authorize such impounding (Rollo, G.R. No. 113888, pp. 15-16).
of P.D. No. 361 (GAA of 1994,
p. 746).
The Solicitor General contends that it is the President, as Commander-in-Chief of the Armed Forces of the
Philippines, who should determine when the services of the CAFGU's are no longer needed (Rollo, G.R.
According to the President, the grant of retirement and separation benefits should be covered by direct No. 113888,
appropriations specifically approved for the purpose pursuant to Section 29(1) of Article VI of the pp. 92-95.).
Constitution. Moreover, he stated that the authority to use savings is lodged in the officials enumerated in
Section 25(5) of Article VI of the Constitution (Veto Message, pp. 7-8).
This is the first case before this Court where the power of the President to impound is put in issue.
Impoundment refers to a refusal by the President, for whatever reason, to spend funds made available by
Petitioners claim that the Special Provision on AFP Pension and Gratuity Fund is a condition or limitation Congress. It is the failure to spend or obligate budget authority of any type (Notes: Impoundment of
which is so intertwined with the item of appropriation that it could not be separated therefrom. Funds, 86 Harvard Law Review 1505 [1973]).

The Special Provision, which allows the Chief of Staff to use savings to augment the pension fund for the Those who deny to the President the power to impound argue that once Congress has set aside the fund
AFP being managed by the AFP Retirement and Separation Benefits System is violative of Sections 25(5) for a specific purpose in an appropriations act, it becomes mandatory on the part of the President to
and 29(1) of the Article VI of the Constitution. implement the project and to spend the money appropriated therefor. The President has no discretion on
the matter, for the Constitution imposes on him the duty to faithfully execute the laws.
Under Section 25(5), no law shall be passed authorizing any transfer of appropriations, and under Section
29(1), no money shall be paid out of In refusing or deferring the implementation of an appropriation item, the President in effect exercises a
the Treasury except in pursuance of an appropriation made by law. While Section 25(5) allows as an veto power that is not expressly granted by the Constitution. As a matter of fact, the Constitution does not
exception the realignment of savings to augment items in the general appropriations law for the executive say anything about impounding. The source of the Executive authority must be found elsewhere.
branch, such right must and can be exercised only by the President pursuant to a specific law.
Proponents of impoundment have invoked at least three principal sources of the authority of the
6. Condition on the deactivation of the CAFGU's. President. Foremost is the authority to impound given to him either expressly or impliedly by Congress.
Second is the executive power drawn from the President's role as Commander-in-Chief. Third is the
Congress appropriated compensation for the CAFGU's, including the payment of separation benefits but it Faithful Execution Clause which ironically is the same provision invoked by petitioners herein.
added the following Special Provision:
The proponents insist that a faithful execution of the laws requires that the President desist from
1. CAFGU Compensation and Separation Benefit. The appropriation authorized herein shall be used for implementing the law if doing so would prejudice public interest. An example given is when through
the compensation of CAFGU's including the payment of their separation benefit not exceeding one (1) efficient and prudent management of a project, substantial savings are made. In such a case, it is sheer
year subsistence allowance for the 11,000 members who will be deactivated in 1994. The Chief of Staff, folly to expect the President to spend the entire amount budgeted in the law (Notes: Presidential
AFP, shall, subject to the approval of the Secretary of National Defense, promulgate policies and Impoundment: Constitutional Theories and Political Realities, 61 Georgetown Law Journal 1295 [1973];
procedures for the payment of separation benefit (GAA of 1994, p. 740). Notes; Protecting the Fisc: Executive Impoundment and Congressional Power, 82 Yale Law Journal 1686
[1973).
The President declared in his Veto Message that the implementation of this Special Provision to the item
on the CAFGU's shall be subject to prior Presidential approval pursuant to P.D. No. 1597 and R.A.. No. We do not find anything in the language used in the challenged Special Provision that would imply that
6758. He gave the following reasons for imposing the condition: Congress intended to deny to the President the right to defer or reduce the spending, much less to
deactivate 11,000 CAFGU members all at once in 1994. But even if such is the intention, the
I am well cognizant of the laudable intention of Congress in proposing the amendment of Special appropriation law is not the proper vehicle for such purpose. Such intention must be embodied and
Provision No. 1 of the CAFGU. However, it is premature at this point in time of our peace process to manifested in another law considering that it abrades the powers of the Commander-in-Chief and there
earmark and declare through special provision the actual number of CAFGU members to be deactivated are existing laws on the creation of the CAFGU's to be amended. Again we state: a provision in an
in CY 1994. I understand that the number to be deactivated would largely depend on the result or degree appropriations act cannot
of success of the on-going peace initiatives which are not yet precisely determinable today. I have be used to repeal or amend other laws, in this case, P.D. No. 1597 and R.A. No. 6758.
desisted, therefore, to directly veto said provisions because this would mean the loss of the entire special
provision to the prejudice of its beneficient provisions. I therefore declare that the actual implementation of 7. Condition on the appropriation for the Supreme Court, etc.

121
(a) In the appropriations for the Supreme Court, Ombudsman, COA, and CHR, the Congress added the The said condition is consistent with the Constitutional injunction prescribed under Section 8, Article IX-B
following provisions: of the Constitution which states that "no elective or appointive public officer or employee shall receive
additional, double, or indirect compensation unless specifically authorized by law." I am, therefore,
The Judiciary Special Provisions confident that the heads of the said offices shall maintain fidelity to the law and faithfully adhere to the
well-established principle on compensation standardization (Veto Message, p. 10).
1. Augmentation of any Item in the Court's Appropriations. Any savings in the appropriations for the
Supreme Court and the Lower Courts may be utilized by the Chief Justice of the Supreme Court to Petitioners claim that the conditions imposed by the President violated the independence and fiscal
augment any item of the Court's appropriations for (a) printing of decisions and publication of "Philippine autonomy of the Supreme Court, the Ombudsman, the COA and the CHR.
Reports"; (b) Commutable terminal leaves of Justices and other personnel of the Supreme Court and
payment of adjusted pension rates to retired Justices entitled thereto pursuant to Administrative Matter In the first place, the conditions questioned by petitioners were placed in the GAB by Congress itself, not
No. 91-8-225-C.A.; (c) repair, maintenance, improvement and other operating expenses of the courts' by the President. The Veto Message merely highlighted the Constitutional mandate that additional or
libraries, including purchase of books and periodicals; (d) purchase, maintenance and improvement of indirect compensation can only be given pursuant to law.
printing equipment; (e) necessary expenses for the employment of temporary employees, contractual and
casual employees, for judicial administration; (f) maintenance and improvement of the Court's Electronic In the second place, such statements are mere reminders that the disbursements of appropriations must
Data be made in accordance with law. Such statements may, at worse, be treated as superfluities.
Processing System; (g) extraordinary expenses of the Chief Justice, attendance in international
conferences and conduct of training programs; (h) commutable transportation and representation
allowances and fringe benefits for Justices, Clerks of Court, Court Administrator, Chiefs of Offices and (b) In the appropriation for the COA, the President imposed the condition that the implementation of the
other Court personnel in accordance with the rates prescribed by law; and (i) compensation of attorney- budget of the COA be subject to "the guidelines to be issued by the President."
de-officio: PROVIDED, That as mandated by LOI No. 489 any increase in salary and allowances shall be
subject to the usual procedures and policies as provided for under The provisions subject to said condition reads:
P.D. No. 985 and other pertinent laws (GAA of 1994, p. 1128;
3. Revolving Fund. The income of the Commission on Audit derived from sources authorized by the
Commission on Audit Government Auditing Code of the Philippines (P.D. No. 1445) not exceeding Ten Million Pesos
(P10,000,000) shall be constituted into a revolving fund which shall be used for maintenance, operating
5. Use of Savings. The Chairman of the Commission on Audit is hereby authorized, subject to appropriate and other incidental expenses to enhance audit services and audit-related activities. The fund shall be
accounting and auditing rules and regulations, to use savings for the payment of fringe benefits as may deposited in an authorized government depository ban, and withdrawals therefrom shall be made in
be authorized by law for officials and personnel of the Commission (GAA of 1994, p. 1161; Emphasis accordance with the procedure prescribed by law and implementing rules and
supplied). regulations: PROVIDED, That any interests earned on such deposit shall be remitted at the end of each
quarter to the national Treasury and shall accrue to the General Fund: PROVIDED FURTHER, That the
Commission on Audit shall submit to the Department of Budget and Management a quarterly report of
Office of the Ombudsman income and expenditures of said revolving fund (GAA of 1994, pp. 1160-1161).

6. Augmentation of Items in the appropriation of the Office of the Ombudsman. The Ombudsman is The President cited the "imperative need to rationalize" the implementation, applicability and operation of
hereby authorized, subject to appropriate accounting and auditing rules and regulations to augment items use of income and revolving funds. The Veto Message stated:
of appropriation in the Office of the Ombudsman from savings in other items of appropriation actually
released, for: (a) printing and/or publication of decisions, resolutions, training and information materials;
(b) repair, maintenance and improvement of OMB Central and Area/Sectoral facilities; (c) purchase of . . . I have observed that there are old and long existing special provisions authorizing the use of income
books, journals, periodicals and equipment; and the creation of revolving funds. As a rule, such authorizations should be discouraged. However, I take
(d) payment of commutable representation and transportation allowances of officials and employees who it that these authorizations have legal/statutory basis aside from being already a vested right to the
by reason of their positions are entitled thereto and fringe benefits as may be authorized specifically by agencies concerned which should not be jeopardized through the Veto Message. There is, however,
law for officials and personnel of OMB pursuant to Section 8 of Article IX-B of the Constitution; and (e) for imperative need to rationalize their implementation, applicability and operation. Thus, in order to
other official purposes subject to accounting and auditing rules and regulations (GAA of 1994, p. 1174; substantiate the purpose and intention of said provisions, I hereby declare that the operationalization of
Emphasis supplied). the following provisions during budget implementation shall be subject to the guidelines to be issued by
the President pursuant to Section 35, Chapter 5, Book VI of E.O. No. 292 and Sections 65 and 66 of P.D.
No. 1445 in relation to Sections 2 and 3 of the General Provisions of this Act (Veto Message, p. 6;
Commission on Human Rights Emphasis Supplied.)

1. Use of Savings. The Chairman of the Commission on Human Rights (CHR) is hereby authorized, (c) In the appropriation for the DPWH, the President imposed the condition that in the implementation of
subject to appropriate accounting and auditing rules and regulations, to augment any item of appropriation DPWH projects, the administrative and engineering overhead of 5% and 3% "shall be subject to the
in the office of the CHR from savings in other items of appropriations actually released, for: (a) printing necessary administrative guidelines to be formulated by the Executive pursuant to existing laws." The
and/or publication of decisions, resolutions, training materials and educational publications; (b) repair, condition was imposed because the provision "needs further study" according to the President.
maintenance and improvement of Commission's central and regional facilities; (c) purchase of books,
journals, periodicals and equipment, (d) payment of commutable representation and transportation
allowances of officials and employees who by reason of their positions are entitled thereto and fringe The following provision was made subject to said condition:
benefits, as may be authorized by law for officials and personnel of CHR, subject to accounting and
auditing rules and regulations (GAA of 1994, p. 1178; Emphasis supplied). 9. Engineering and Administrative Overhead. Not more than five percent (5%) of the amount for
infrastructure project released by the Department of Budget and Management shall be deducted by
In his Veto Message, the President expressed his approval of the conditions included in the GAA of 1994. DPWH for administrative overhead, detailed engineering and construction supervision, testing and quality
He noted that: control, and the like, thus insuring that at least ninety-five percent (95%) of the released fund is available

122
for direct implementation of the project. PROVIDED, HOWEVER, That for school buildings, health executed (1987 Constitution, Art. VII, Sec. 17; Planas v. Gil 67 Phil. 62 [1939]). Under the Faithful
centers, day-care centers and barangay halls, the deductible amount shall not exceed three percent (3%). Execution Clause, the President has the power to take "necessary and proper steps" to carry into
execution the law (Schwartz, On Constitutional Law, p. 147 [1977]). These steps are the ones to be
Violation of, or non-compliance with, this provision shall subject the government official or employee embodied in the guidelines.
concerned to administrative, civil and/or criminal sanction under Sections 43 and 80, Book VI of E.O.
No. 292 (GAA of 1994, p. 786). IV - Petitioners chose to avail of the special civil actions but those remedies can be used only when
respondents have acted "without or in excess" of jurisdiction, or "with grave abuse of discretion," (Revised
(d) In the appropriation for the National Housing Authority (NHA), the President imposed the condition that Rules of Court,
allocations for specific projects shall be released and disbursed "in accordance with the housing program Rule 65, Section 2). How can we begrudge the President for vetoing the Special Provision on the
of the government, subject to prior Executive approval." appropriation for debt payment when he merely followed our decision in Gonzales? How can we say that
Congress has abused its discretion when it appropriated a bigger sum for debt payment than the amount
appropriated for education, when it merely followed our dictum in Guingona?
The provision subject to the said condition reads:
Article 8 of the Civil Code of Philippines, provides:
3. Allocations for Specified Projects. The following allocations for the specified projects shall be set aside
for corollary works and used exclusively for the repair, rehabilitation and construction of buildings, roads,
pathwalks, drainage, waterworks systems, facilities and amenities in the area: PROVIDED, That any road Judicial decisions applying or interpreting the laws or the constitution shall from a part of the legal system
to be constructed or rehabilitated shall conform with the specifications and standards set by the of the Philippines.
Department of Public Works and Highways for such kind of road: PROVIDED, FURTHER, That savings
that may be available in the future shall be used for road repair, rehabilitation and construction: The Court's interpretation of the law is part of that law as of the date of its enactment since the court's
interpretation merely establishes the contemporary legislative intent that the construed law purports to
(1) Maharlika Village Road — Not less than P5,000,000 carry into effect (People v. Licera, 65 SCRA 270 [1975]). Decisions of the Supreme Court assume the
same authority as statutes (Floresca v. Philex Mining Corporation, 136 SCRA 141 [1985]).
(2) Tenement Housing Project (Taguig) — Not less than P3,000,000
Even if Guingona and Gonzales are considered hard cases that make bad laws and should be reversed,
such reversal cannot nullify prior acts done in reliance thereof.
(3) Bagong Lipunan Condominium Project (Taguig) — Not less than P2,000,000
WHEREFORE, the petitions are DISMISSED, except with respect to
4. Allocation of Funds. Out of the amount appropriated for the implementation of various projects in (1) G.R. Nos. 113105 and 113766 only insofar as they pray for the annulment of the veto of the special
resettlement areas, Seven Million Five Hundred Thousand Pesos (P7,500,000) shall be allocated to the provision on debt service specifying that the fund therein appropriated "shall be used for payment of the
Dasmariñas Bagong Bayan resettlement area, Eighteen Million Pesos (P18,000,000) to the Carmona principal and interest of foreign and domestic indebtedness" prohibiting the use of the said funds "to pay
Relocation Center Area (Gen. Mariano Alvarez) and Three Million Pesos (P3,000,000) to the Bulihan for the liabilities of the Central Bank Board of Liquidators", and (2) G.R. No. 113888 only insofar as it
Sites and Services, all of which will be for the cementing of roads in accordance with DPWH standards. prays for the annulment of the veto of: (a) the second paragraph of Special Provision No. 2 of the item of
appropriation for the Department of Public Works and Highways (GAA of 1994, pp. 785-786); and (b)
5. Allocation for Sapang Palay. An allocation of Eight Million Pesos (P8,000,000) shall be set aside for the Special Provision No. 12 on the purchase of medicines by the Armed Forces of the Philippines (GAA of
asphalting of seven (7) kilometer main road of Sapang Palay, San Jose Del Monte, Bulacan 1994, p. 748), which is GRANTED. SO ORDERED.
(GAA of 1994, p. 1216).
G.R. No. L-33713 July 30, 1975
The President imposed the conditions: (a) that the "operationalization" of the special provision on
revolving funds of the COA "shall be subject to guidelines to be issued by the President pursuant to EUSEBIO B. GARCIA,vs.HON. ERNESTO S. MATA,
Section 35, Chapter 5,
Book VI of E.O. 292 and Sections 65 and 66 of P.D. No. 1445 in relation to Sections 2 and 3 of the
General Provisions of this Act" (Rollo, G.R. This is a petition for certiorari to review the decision of the Court of First Instance of Quezon City, Branch
No. 113174, pp. 5,7-8); (b) that the implementation of Special Provision No. 9 of the DPWH on the IX, in civil case Q-13466, entitled "Eusebio B. Garcia, petitioner, versus Hon. Ernesto Mata (Juan Ponce
mandatory retention of 5% and 3% of the amounts released by said Department "be subject to the Enrile), et al., respondents," declaring paragraph 11 of the "Special Provisions for the Armed Forces of the
necessary administrative guidelines to be formulated by the Executive pursuant to existing law" (Rollo, Philippines" of Republic Act No. 16001 unconstitutional and therefore invalid and inoperative.
G.R. No. 113888; pp. 10, 14-16); and (c) that the appropriations authorized for the NHA can be released
only "in accordance with the housing program of the government subject to prior Executive approval" We affirm the judgment a quo.
(Rollo, G.R. No. 113888, pp. 10-11;
14-16). The facts material to this case are embodied in the following stipulation submitted jointly by both parties to
the lower court:
The conditions objected to by petitioners are mere reminders that the implementation of the items on
which the said conditions were imposed, should be done in accordance with existing laws, regulations or Petitioner was a reserve officer on active duty with the Armed Forces of the Philippines until his reversion
policies. They did not add anything to what was already in place at the time of the approval of the GAA of to inactive status on 15 November 1960, pursuant to the provisions of Republic Act No. 2332. At the time
1994. of reversion, Petitioner held the rank of Captain with a monthly emolument of P478.00, comprising his
base and longevity pay, quarters and subsistence allowances;
There is less basis to complain when the President said that the expenditures shall be subject to
guidelines he will issue. Until the guidelines are issued, it cannot be determined whether they are proper On June 18, 1955, the date when Republic Act No. 1382 took effect, petitioner had a total of 9 years, 4
or inappropriate. The issuance of administrative guidelines on the use of public funds authorized by months and 12 days of accumulated active commissioned service in the Armed Forces of the Philippines;
Congress is simply an exercise by the President of his constitutional duty to see that the laws are faithfully

123
On July 11, 1956, the date when Republic Act 1600 took effect, petitioner had an accumulated active quarterly report to Congress as to the implementation of the provisions of this paragraph. ( pp. 892-893,
commissioned service of 10 years, 5 months and 5 days in the Armed Forces of the Philippines; RA 1600) (emphasis supplied)

Petitioner's reversion to inactive status on 15 November 1960 was pursuant to the provisions of Republic The petitioner consequently argues that his reversion to inactive status on November 15, 1960 was in
Act 2334, and such reversion was neither for cause, at his own request, nor after court-martial violation of the abovequoted provision which prohibits the reversion to inactive status of reserve officers
proceedings; on active duty with at least ten years of accumulated active commissioned service.

From 15 November 1960 up to the present, petitioner has been on inactive status and as such, he has On the other hand, the respondents contend that the said provision has no relevance or pertinence
neither received any emoluments from the Armed Forces of the Philippines, nor was he ever employed in whatsoever to the budget in question or to any appropriation item contained therein, and is therefore
the Government in any capacity; proscribed by Art. VI, Sec. 19, par. 24 of the 1935 Constitution of the Philippines, which reads:

As a consequence of his reversion to inactive status, petitioner filed the necessary petitions with the No provision or enactment shall be embraced in the general appropriation bill unless it relates specifically
offices of the AFP Chief of Staff, the Secretary of National Defense, and the President, respectively, but to some particular appropriation therein; and any such provision or enactment shall be limited in its
received reply only from the Chief of Staff through the AFP Adjutant General. operation to such appropriation.

On September 17, 1969 the petitioner brought an action for "Mandamus and Recovery of a Sum of A perusal of the challenged provision of R.A. 1600 fails to disclose its relevance or relation to any
Money" in the court a quo to compel the respondents Secretary of National Defense and Chief of Staff of appropriation item therein, or to the Appropriation Act as a whole. From the very first clause of paragraph
the Armed Forces of the Philippines2 to reinstate him in the active commissioned service of the Armed 11 itself, which reads,
Forces of the Philippines, to readjust his rank, and to pay all the emoluments and allowances due to him
from the time of his reversion to inactive status. On December 2, 1970 the trial court dismissed the After the approval of this Act, and when there is no emergency, no reserve officer of the Armed Forces of
petition. The court ruled that paragraph 11 of the "Special Provisions for the Armed Forces of the the Philippines may be called to a tour of active duty for more than two years during any period of five
Philippines" in Republic Act 1600 is "invalid, unconstitutional and inoperative." consecutive years:

The petitioner had a total of 9 years, 4 months and 12 days of accumulated active commissioned service the incongruity and irrelevancy are already evident. While R.A. 1600 appropriated money for the operation
in the AFP when Republic Act 1382 took effect on June 18, 1955. Section I of this law provided: of the Government for the fiscal year 1956-1957, the said paragraph 11 refers to the fundamental
government policy matters of the calling to active duty and the reversion to inactive status of reserve
Reserve officers with at least ten years of active accumulated commissioned service who are still on officers in the AFP. The incongruity and irrelevancy continue throughout the entire paragraph.
active duty at the time of the approval of this Act shall not be reverted into inactive status except for cause
after proper court-martial proceedings or upon their own request: Provided, That for purposes of In the language of the respondents-appellees, "it was indeed a non-appropriation item inserted in an
computing the length of service, six months or more of active service shall be considered one year. appropriation measure in violation of the constitutional inhibition against "riders" to the general
(emphasis supplied) appropriation act." It was indeed a new and completely unrelated provision attached to the Appropriation
Act.
The petitioner's accumulated active commissioned service was thus short of the minimum service
requirement prescribed in the aforequoted provision of R.A. 1382. The paragraph in question also violated Art. VI, Sec. 21, par. 15 of the 1935 Constitution of the Philippines
which provided that "No bill which may be enacted into law shall embrace more than one subject which
On July 11, 1956,3 while the petitioner was yet in the active service, Republic Act 1600 was enacted into shall be expressed in the title of the bill." This constitutional requirement nullified and rendered inoperative
law. Paragraph 11 of the SPECIAL PROVISIONS FOR THE ARMED FORCES OF THE PHILIPPINES any provision contained in the body of an act that was not fairly included in the subject expressed in the
(on page 892 of the Act) provided as follows: title or was not germane to or properly connected with that subject.

11. After the approval of this Act, and when there is no emergency, no reserve officer of the Armed Forces In determining whether a provision contained in an act is embraced in the subject and is properly
of the Philippines may be called to a tour of active duty for more than two years during any period of five connected therewith, the subject to be considered is the one expressed in the title of the act, and every
consecutive years: PROVIDED, That hereafter reserve officers of the Armed Forces of the Philippines on fair intendment and reasonable doubt should be indulged in favor of the validity of the legislative
active duty for more than two years on the date of the approval of this Act except those whose military and enactment. But when an act contains provisions which are clearly not embraced in the subject of the act,
educational training, experience and qualifications are deemed essential to the needs of the service, shall as expressed in the title, such provisions are inoperative and without effect.
be reverted to inactive status within one year from the approval of this Act: PROVIDED, FURTHER, That
reserve officers with at least ten years of active accumulated commissioned service who are still on active We are mindful that the title of an act is not required to be an index to the body of the act. Thus,
duty at the time of the approval of this Act shall not be reverted to inactive status except for cause after in Sumulong vs. Comelec, 73 Phil. 288, 291, this Court held that it is "a sufficient compliance with such
proper court-martial proceedings or upon their request; PROVIDED, FURTHER, That any such reserve requirement if the title expresses the general subject and all the provisions of the statute are germane to
officer reverted to inactive status who has at least five of active commissioned service shall be entitled to that general subject." The constitutional provision was intended to preclude the insertion of riders in
a gratuity equivalent to one month's authorized base and longevity pay in the rank held at the time of such legislation, a rider being a provision not germane to the subject-matter of the bill.6
reversion for every year of active commissioned service; PROVIDED, FURTHER, That any reserve officer
who receives a gratuity under the provisions of this Act shall not except during a National emergency or
mobilization, be called to a tour of active duty within five years from the date of reversion: PROVIDED, The subject of R.A. 1600, as expressed in its title, is restricted to "appropriating funds for the operation of
FURTHER, That the Secretary of National Defense is authorized to extend the tour of active duty of the government." Any provision contained in the body of the act that is fairly included in this restricted
reserve officers who are qualified military pilots and doctors; PROVIDED, FURTHER, That any savings in subject or any matter properly connected therewith is valid and operative. But, if a provision in the body of
the appropriations authorized in this Act for the Department of National Defense notwithstanding any the act is not fairly included in this restricted subject, like the provision relating to the policy matters of
provision of this Act to the contrary and any unexpended balance of certification to accounts payable calling to active duty and reversion to inactive duty of reserve officers of the AFP, such provision is
since 1 July 1949 regardless of purpose of the appropriation shall be made available for the purpose of inoperative and of no effect.
this paragraph: AND PROVIDED, FINALLY, That the Secretary of National Defense shall render a
124
To quote the respondents-appellees on this point: immediate enactment. On January 27, 2005, the House of Representatives approved the bill on second
and third reading.
It is obvious that the statutory provision in question refers to security of reserve officers from reversion to
inactive status, whereas the subject or title of the statute from which it derives its existence refers to House Bill No. 37053 on the other hand, substituted House Bill No. 3105 introduced by Rep. Salacnib F.
appropriations. Verily, it runs contrary to or is repugnant to the above-quoted injunctive provision of the Baterina, and House Bill No. 3381 introduced by Rep. Jacinto V. Paras. Its "mother bill" is House Bill No.
Constitution. Where a conflict arises between a statute and the Constitution, the latter prevails. It should 3555. The House Committee on Ways and Means approved the bill on February 2, 2005. The President
be emphasized that a Constitution is superior to a statute and is precisely called the "supreme law of the also certified it as urgent on February 8, 2005. The House of Representatives approved the bill on second
land" because it is the fundamental or organic law which states the general principles and builds the and third reading on February 28, 2005.
substantial foundation and general framework of law and government, and for that reason a statute
contrary to or in violation of the Constitution is null and void (Talabon vs. Iloilo Provincial Warden, 78 Phil. Meanwhile, the Senate Committee on Ways and Means approved Senate Bill No. 19504 on March 7,
599).1äwphï1.ñët If a law, therefore, happens to infringe upon or violate the fundamental law, courts of 2005, "in substitution of Senate Bill Nos. 1337, 1838 and 1873, taking into consideration House Bill Nos.
justice may step in to nullify its effectiveness (Mabanag vs. Lopez Vito, 78 Phil. 1). 3555 and 3705." Senator Ralph G. Recto sponsored Senate Bill No. 1337, while Senate Bill Nos. 1838
and 1873 were both sponsored by Sens. Franklin M. Drilon, Juan M. Flavier and Francis N. Pangilinan.
Upon the foregoing dissertation, we declare Paragraph 11 of the SPECIAL PROVISIONS FOR THE The President certified the bill on March 11, 2005, and was approved by the Senate on second and third
ARMED FORCES OF THE PHILIPPINES as unconstitutional, invalid and inoperative. Being reading on April 13, 2005.
unconstitutional, it confers no right and affords no protection. In legal contemplation it is as though it has
never been passed.7 On the same date, April 13, 2005, the Senate agreed to the request of the House of Representatives for a
committee conference on the disagreeing provisions of the proposed bills.
Verily, not having shown a clear legal right to the position to which he desires to be restored, the petitioner
cannot compel the respondents to reinstate and/or call him to active duty, promote or readjust his rank, Before long, the Conference Committee on the Disagreeing Provisions of House Bill No. 3555, House Bill
much less pay him back emoluments and allowances. No. 3705, and Senate Bill No. 1950, "after having met and discussed in full free and conference,"
recommended the approval of its report, which the Senate did on May 10, 2005, and with the House of
ACCORDINGLY, the instant petition is denied, and the decision of the lower court dismissing the Representatives agreeing thereto the next day, May 11, 2005.
complaint is hereby affirmed. No pronouncement as to costs.
On May 23, 2005, the enrolled copy of the consolidated House and Senate version was transmitted to the
G.R. No. 168056 September 1, 2005 President, who signed the same into law on May 24, 2005. Thus, came R.A. No. 9337.

ABAKADA GURO PARTY LIST vs.THE HONORABLE EXECUTIVE SECRETARY EDUARDO ERMITA; July 1, 2005 is the effectivity date of R.A. No. 9337.5 When said date came, the Court issued a temporary
restraining order, effective immediately and continuing until further orders, enjoining respondents from
The expenses of government, having for their object the interest of all, should be borne by everyone, and enforcing and implementing the law.
the more man enjoys the advantages of society, the more he ought to hold himself honored in contributing
to those expenses. Oral arguments were held on July 14, 2005. Significantly, during the hearing, the Court speaking through
Mr. Justice Artemio V. Panganiban, voiced the rationale for its issuance of the temporary restraining order
-Anne Robert Jacques Turgot (1727-1781) on July 1, 2005, to wit:

French statesman and economist J. PANGANIBAN : . . . But before I go into the details of your presentation, let me just tell you a little
background. You know when the law took effect on July 1, 2005, the Court issued a TRO at about 5
o’clock in the afternoon. But before that, there was a lot of complaints aired on television and on radio.
Mounting budget deficit, revenue generation, inadequate fiscal allocation for education, increased Some people in a gas station were complaining that the gas prices went up by 10%. Some people were
emoluments for health workers, and wider coverage for full value-added tax benefits … these are the complaining that their electric bill will go up by 10%. Other times people riding in domestic air carrier were
reasons why Republic Act No. 9337 (R.A. No. 9337)1 was enacted. Reasons, the wisdom of which, the complaining that the prices that they’ll have to pay would have to go up by 10%. While all that was being
Court even with its extensive constitutional power of review, cannot probe. The petitioners in these cases, aired, per your presentation and per our own understanding of the law, that’s not true. It’s not true that the
however, question not only the wisdom of the law, but also perceived constitutional infirmities in its e-vat law necessarily increased prices by 10% uniformly isn’t it?
passage.
ATTY. BANIQUED : No, Your Honor.
Every law enjoys in its favor the presumption of constitutionality. Their arguments notwithstanding, J. PANGANIBAN : It is not?
petitioners failed to justify their call for the invalidity of the law. Hence, R.A. No. 9337 is not ATTY. BANIQUED : It’s not, because, Your Honor, there is an Executive Order that granted the Petroleum
unconstitutional. companies some subsidy . . . interrupted
J. PANGANIBAN : That’s correct . . .
LEGISLATIVE HISTORY ATTY. BANIQUED : . . . and therefore that was meant to temper the impact . . . interrupted
J. PANGANIBAN : . . . mitigating measures . . .
R.A. No. 9337 is a consolidation of three legislative bills namely, House Bill Nos. 3555 and 3705, and ATTY. BANIQUED : Yes, Your Honor.
Senate Bill No. 1950. J. PANGANIBAN : As a matter of fact a part of the mitigating measures would be the elimination of the
Excise Tax and the import duties. That is why, it is not correct to say that the VAT as to petroleum dealers
increased prices by 10%.
House Bill No. 35552 was introduced on first reading on January 7, 2005. The House Committee on
ATTY. BANIQUED : Yes, Your Honor.
Ways and Means approved the bill, in substitution of House Bill No. 1468, which Representative (Rep.)
J. PANGANIBAN : And therefore, there is no justification for increasing the retail price by 10% to cover the
Eric D. Singson introduced on August 8, 2004. The President certified the bill on January 7, 2005 for
E-Vat tax. If you consider the excise tax and the import duties, the Net Tax would probably be in the
neighborhood of 7%? We are not going into exact figures I am just trying to deliver a point that different
125
industries, different products, different services are hit differently. So it’s not correct to say that all prices Aside from questioning the so-called stand-by authority of the President to increase the VAT rate to 12%,
must go up by 10%. on the ground that it amounts to an undue delegation of legislative power, petitioners also contend that
ATTY. BANIQUED : You’re right, Your Honor. the increase in the VAT rate to 12% contingent on any of the two conditions being satisfied violates the
due process clause embodied in Article III, Section 1 of the Constitution, as it imposes an unfair and
J. PANGANIBAN : Now. For instance, Domestic Airline companies, Mr. Counsel, are at present imposed additional tax burden on the people, in that: (1) the 12% increase is ambiguous because it does not state
a Sales Tax of 3%. When this E-Vat law took effect the Sales Tax was also removed as a mitigating if the rate would be returned to the original 10% if the conditions are no longer satisfied; (2) the rate is
measure. So, therefore, there is no justification to increase the fares by 10% at best 7%, correct? unfair and unreasonable, as the people are unsure of the applicable VAT rate from year to year; and (3)
the increase in the VAT rate, which is supposed to be an incentive to the President to raise the VAT
collection to at least 2 4/5 of the GDP of the previous year, should only be based on fiscal adequacy.
ATTY. BANIQUED : I guess so, Your Honor, yes.
Petitioners further claim that the inclusion of a stand-by authority granted to the President by the
J. PANGANIBAN : There are other products that the people were complaining on that first day, were Bicameral Conference Committee is a violation of the "no-amendment rule" upon last reading of a bill laid
being increased arbitrarily by 10%. And that’s one reason among many others this Court had to issue down in Article VI, Section 26(2) of the Constitution.
TRO because of the confusion in the implementation. That’s why we added as an issue in this case, even
if it’s tangentially taken up by the pleadings of the parties, the confusion in the implementation of the E-
vat. Our people were subjected to the mercy of that confusion of an across the board increase of 10%, G.R. No. 168461
which you yourself now admit and I think even the Government will admit is incorrect. In some cases, it
should be 3% only, in some cases it should be 6% depending on these mitigating measures and the Thereafter, a petition for prohibition was filed on June 29, 2005, by the Association of Pilipinas Shell
location and situation of each product, of each service, of each company, isn’t it? Dealers, Inc., et al., assailing the following provisions of R.A. No. 9337:

ATTY. BANIQUED : Yes, Your Honor. 1) Section 8, amending Section 110 (A)(2) of the NIRC, requiring that the input tax on depreciable goods
shall be amortized over a 60-month period, if the acquisition, excluding the VAT components, exceeds
J. PANGANIBAN : Alright. So that’s one reason why we had to issue a TRO pending the clarification of all One Million Pesos (₱1, 000,000.00);
these and we wish the government will take time to clarify all these by means of a more detailed
implementing rules, in case the law is upheld by this Court. . . .6 2) Section 8, amending Section 110 (B) of the NIRC, imposing a 70% limit on the amount of input tax to
be credited against the output tax; and
The Court also directed the parties to file their respective Memoranda.
3) Section 12, amending Section 114 (c) of the NIRC, authorizing the Government or any of its political
G.R. No. 168056 subdivisions, instrumentalities or agencies, including GOCCs, to deduct a 5% final withholding tax on
gross payments of goods and services, which are subject to 10% VAT under Sections 106 (sale of goods
and properties) and 108 (sale of services and use or lease of properties) of the NIRC.
Before R.A. No. 9337 took effect, petitioners ABAKADA GURO Party List, et al., filed a petition for
prohibition on May 27, 2005. They question the constitutionality of Sections 4, 5 and 6 of R.A. No. 9337,
amending Sections 106, 107 and 108, respectively, of the National Internal Revenue Code (NIRC). Petitioners contend that these provisions are unconstitutional for being arbitrary, oppressive, excessive,
Section 4 imposes a 10% VAT on sale of goods and properties, Section 5 imposes a 10% VAT on and confiscatory.
importation of goods, and Section 6 imposes a 10% VAT on sale of services and use or lease of
properties. These questioned provisions contain a uniform proviso authorizing the President, upon Petitioners’ argument is premised on the constitutional right of non-deprivation of life, liberty or property
recommendation of the Secretary of Finance, to raise the VAT rate to 12%, effective January 1, 2006, without due process of law under Article III, Section 1 of the Constitution. According to petitioners, the
after any of the following conditions have been satisfied, to wit: contested sections impose limitations on the amount of input tax that may be claimed. Petitioners also
argue that the input tax partakes the nature of a property that may not be confiscated, appropriated, or
. . . That the President, upon the recommendation of the Secretary of Finance, shall, effective January 1, limited without due process of law. Petitioners further contend that like any other property or property
2006, raise the rate of value-added tax to twelve percent (12%), after any of the following conditions has right, the input tax credit may be transferred or disposed of, and that by limiting the same, the government
been satisfied: gets to tax a profit or value-added even if there is no profit or value-added.

(i) Value-added tax collection as a percentage of Gross Domestic Product (GDP) of the previous year Petitioners also believe that these provisions violate the constitutional guarantee of equal protection of the
exceeds two and four-fifth percent (2 4/5%); or law under Article III, Section 1 of the Constitution, as the limitation on the creditable input tax if: (1) the
entity has a high ratio of input tax; or (2) invests in capital equipment; or (3) has several transactions with
the government, is not based on real and substantial differences to meet a valid classification.
(ii) National government deficit as a percentage of GDP of the previous year exceeds one and one-half
percent (1 ½%).
Lastly, petitioners contend that the 70% limit is anything but progressive, violative of Article VI, Section
28(1) of the Constitution, and that it is the smaller businesses with higher input tax to output tax ratio that
Petitioners argue that the law is unconstitutional, as it constitutes abandonment by Congress of its will suffer the consequences thereof for it wipes out whatever meager margins the petitioners make.
exclusive authority to fix the rate of taxes under Article VI, Section 28(2) of the 1987 Philippine
Constitution.
G.R. No. 168463
G.R. No. 168207
Several members of the House of Representatives led by Rep. Francis Joseph G. Escudero filed this
petition for certiorari on June 30, 2005. They question the constitutionality of R.A. No. 9337 on the
On June 9, 2005, Sen. Aquilino Q. Pimentel, Jr., et al., filed a petition for certiorari likewise assailing the following grounds:
constitutionality of Sections 4, 5 and 6 of R.A. No. 9337.

126
1) Sections 4, 5, and 6 of R.A. No. 9337 constitute an undue delegation of legislative power, in violation of 1. Whether Sections 4, 5 and 6 of R.A. No. 9337, amending Sections 106, 107 and 108 of the NIRC,
Article VI, Section 28(2) of the Constitution; violate the following provisions of the Constitution:

2) The Bicameral Conference Committee acted without jurisdiction in deleting the no pass on provisions a. Article VI, Section 28(1), and b. Article VI, Section 28(2)
present in Senate Bill No. 1950 and House Bill No. 3705; and
2. Whether Section 8 of R.A. No. 9337, amending Sections 110(A)(2) and 110(B) of the NIRC; and
3) Insertion by the Bicameral Conference Committee of Sections 27, 28, 34, 116, 117, 119, 121, Section 12 of R.A. No. 9337, amending Section 114(C) of the NIRC, violate the following provisions of the
125,7 148, 151, 236, 237 and 288, which were present in Senate Bill No. 1950, violates Article VI, Section Constitution:
24(1) of the Constitution, which provides that all appropriation, revenue or tariff bills shall originate
exclusively in the House of Representatives a. Article VI, Section 28(1), and b. Article III, Section 1

G.R. No. 168730 RULING OF THE COURT

On the eleventh hour, Governor Enrique T. Garcia filed a petition for certiorari and prohibition on July 20, As a prelude, the Court deems it apt to restate the general principles and concepts of value-added tax
2005, alleging unconstitutionality of the law on the ground that the limitation on the creditable input tax in (VAT), as the confusion and inevitably, litigation, breeds from a fallacious notion of its nature.
effect allows VAT-registered establishments to retain a portion of the taxes they collect, thus violating the
principle that tax collection and revenue should be solely allocated for public purposes and expenditures.
Petitioner Garcia further claims that allowing these establishments to pass on the tax to the consumers is The VAT is a tax on spending or consumption. It is levied on the sale, barter, exchange or lease of goods
inequitable, in violation of Article VI, Section 28(1) of the Constitution. or properties and services.8 Being an indirect tax on expenditure, the seller of goods or services may
pass on the amount of tax paid to the buyer,9 with the seller acting merely as a tax collector.10 The
burden of VAT is intended to fall on the immediate buyers and ultimately, the end-consumers.
RESPONDENTS’ COMMENT
In contrast, a direct tax is a tax for which a taxpayer is directly liable on the transaction or business it
The Office of the Solicitor General (OSG) filed a Comment in behalf of respondents. Preliminarily, engages in, without transferring the burden to someone else.11 Examples are individual and corporate
respondents contend that R.A. No. 9337 enjoys the presumption of constitutionality and petitioners failed income taxes, transfer taxes, and residence taxes.12
to cast doubt on its validity.
In the Philippines, the value-added system of sales taxation has long been in existence, albeit in a
Relying on the case of Tolentino vs. Secretary of Finance, 235 SCRA different mode. Prior to 1978, the system was a single-stage tax computed under the "cost deduction
method" and was payable only by the original sellers. The single-stage system was subsequently
630 (1994), respondents argue that the procedural issues raised by petitioners, i.e., legality of the modified, and a mixture of the "cost deduction method" and "tax credit method" was used to determine the
bicameral proceedings, exclusive origination of revenue measures and the power of the Senate value-added tax payable.13 Under the "tax credit method," an entity can credit against or subtract from
concomitant thereto, have already been settled. With regard to the issue of undue delegation of legislative the VAT charged on its sales or outputs the VAT paid on its purchases, inputs and imports.14
power to the President, respondents contend that the law is complete and leaves no discretion to the
President but to increase the rate to 12% once any of the two conditions provided therein arise. It was only in 1987, when President Corazon C. Aquino issued Executive Order No. 273, that the VAT
system was rationalized by imposing a multi-stage tax rate of 0% or 10% on all sales using the "tax credit
Respondents also refute petitioners’ argument that the increase to 12%, as well as the 70% limitation on method."15
the creditable input tax, the 60-month amortization on the purchase or importation of capital goods
exceeding ₱1,000,000.00, and the 5% final withholding tax by government agencies, is arbitrary, E.O. No. 273 was followed by R.A. No. 7716 or the Expanded VAT Law,16 R.A. No. 8241 or the Improved
oppressive, and confiscatory, and that it violates the constitutional principle on progressive taxation, VAT Law,17 R.A. No. 8424 or the Tax Reform Act of 1997,18 and finally, the presently beleaguered R.A.
among others. No. 9337, also referred to by respondents as the VAT Reform Act.

Finally, respondents manifest that R.A. No. 9337 is the anchor of the government’s fiscal reform agenda. The Court will now discuss the issues in logical sequence.
A reform in the value-added system of taxation is the core revenue measure that will tilt the balance
towards a sustainable macroeconomic environment necessary for economic growth.
PROCEDURAL ISSUE- I.
ISSUES
Whether R.A. No. 9337 violates the following provisions of the Constitution:
The Court defined the issues, as follows:
a. Article VI, Section 24, and
PROCEDURAL ISSUE
b. Article VI, Section 26(2)
Whether R.A. No. 9337 violates the following provisions of the Constitution:
A. The Bicameral Conference Committee
a. Article VI, Section 24, and b. Article VI, Section 26(2)
Petitioners Escudero, et al., and Pimentel, et al., allege that the Bicameral Conference Committee
exceeded its authority by:
SUBSTANTIVE ISSUES
1) Inserting the stand-by authority in favor of the President in Sections 4, 5, and 6 of R.A. No. 9337;

127
2) Deleting entirely the no pass-on provisions found in both the House and Senate bills; are unconstitutional, but whether the bicameral conference committee has strictly complied with the
3) Inserting the provision imposing a 70% limit on the amount of input tax to be credited against the output rules of both houses, thereby remaining within the jurisdiction conferred upon it by Congress.
tax; and
4) Including the amendments introduced only by Senate Bill No. 1950 regarding other kinds of taxes in In the recent case of Fariñas vs. The Executive Secretary,20 the Court En Banc, unanimously reiterated
addition to the value-added tax. and emphasized its adherence to the "enrolled bill doctrine," thus, declining therein petitioners’ plea for
Petitioners now beseech the Court to define the powers of the Bicameral Conference Committee. the Court to go behind the enrolled copy of the bill. Assailed in said case was Congress’s creation of two
sets of bicameral conference committees, the lack of records of said committees’ proceedings, the alleged
It should be borne in mind that the power of internal regulation and discipline are intrinsic in any legislative violation of said committees of the rules of both houses, and the disappearance or deletion of one of the
body for, as unerringly elucidated by Justice Story, "[i]f the power did not exist, it would be utterly provisions in the compromise bill submitted by the bicameral conference committee. It was argued that
impracticable to transact the business of the nation, either at all, or at least with decency, such irregularities in the passage of the law nullified R.A. No. 9006, or the Fair Election Act.
deliberation, and order."19 Thus, Article VI, Section 16 (3) of the Constitution provides that "each House
may determine the rules of its proceedings." Pursuant to this inherent constitutional power to promulgate Striking down such argument, the Court held thus:
and implement its own rules of procedure, the respective rules of each house of Congress provided for
the creation of a Bicameral Conference Committee.
Under the "enrolled bill doctrine," the signing of a bill by the Speaker of the House and the Senate
President and the certification of the Secretaries of both Houses of Congress that it was passed are
Thus, Rule XIV, Sections 88 and 89 of the Rules of House of Representatives provides as follows: conclusive of its due enactment. A review of cases reveals the Court’s consistent adherence to the
rule. The Court finds no reason to deviate from the salutary rule in this case where the
Sec. 88. Conference Committee. – In the event that the House does not agree with the Senate on the irregularities alleged by the petitioners mostly involved the internal rules of Congress, e.g.,
amendment to any bill or joint resolution, the differences may be settled by the conference committees of creation of the 2nd or 3rd Bicameral Conference Committee by the House. This Court is not the
both chambers. proper forum for the enforcement of these internal rules of Congress, whether House or Senate.
Parliamentary rules are merely procedural and with their observance the courts have no concern.
In resolving the differences with the Senate, the House panel shall, as much as possible, adhere to and Whatever doubts there may be as to the formal validity of Rep. Act No. 9006 must be resolved in
support the House Bill. If the differences with the Senate are so substantial that they materially impair the its favor. The Court reiterates its ruling in Arroyo vs. De Venecia, viz.:
House Bill, the panel shall report such fact to the House for the latter’s appropriate action.
But the cases, both here and abroad, in varying forms of expression, all deny to the courts the
Sec. 89. Conference Committee Reports. – . . . Each report shall contain a detailed, sufficiently explicit power to inquire into allegations that, in enacting a law, a House of Congress failed to comply with
statement of the changes in or amendments to the subject measure. its own rules, in the absence of showing that there was a violation of a constitutional provision or
the rights of private individuals. In Osmeña v. Pendatun, it was held: "At any rate, courts have declared
that ‘the rules adopted by deliberative bodies are subject to revocation, modification or waiver at the
... pleasure of the body adopting them.’ And it has been said that "Parliamentary rules are merely
procedural, and with their observance, the courts have no concern. They may be waived or
The Chairman of the House panel may be interpellated on the Conference Committee Report prior to the disregarded by the legislative body." Consequently, "mere failure to conform to parliamentary
voting thereon. The House shall vote on the Conference Committee Report in the same manner and usage will not invalidate the action (taken by a deliberative body) when the requisite number of
procedure as it votes on a bill on third and final reading. members have agreed to a particular measure."21 (Emphasis supplied)

Rule XII, Section 35 of the Rules of the Senate states: The foregoing declaration is exactly in point with the present cases, where petitioners allege irregularities
committed by the conference committee in introducing changes or deleting provisions in the House and
Sec. 35. In the event that the Senate does not agree with the House of Representatives on the provision Senate bills. Akin to the Fariñas case,22 the present petitions also raise an issue regarding the actions
of any bill or joint resolution, the differences shall be settled by a conference committee of both Houses taken by the conference committee on matters regarding Congress’ compliance with its own internal rules.
which shall meet within ten (10) days after their composition. The President shall designate the members As stated earlier, one of the most basic and inherent power of the legislature is the power to formulate
of the Senate Panel in the conference committee with the approval of the Senate. rules for its proceedings and the discipline of its members. Congress is the best judge of how it should
conduct its own business expeditiously and in the most orderly manner. It is also the sole
Each Conference Committee Report shall contain a detailed and sufficiently explicit statement of the
changes in, or amendments to the subject measure, and shall be signed by a majority of the members of concern of Congress to instill discipline among the members of its conference committee if it believes that
each House panel, voting separately. said members violated any of its rules of proceedings. Even the expanded jurisdiction of this Court cannot
apply to questions regarding only the internal operation of Congress, thus, the Court is wont to deny a
review of the internal proceedings of a co-equal branch of government.
A comparative presentation of the conflicting House and Senate provisions and a reconciled version
thereof with the explanatory statement of the conference committee shall be attached to the report.
Moreover, as far back as 1994 or more than ten years ago, in the case of Tolentino vs. Secretary of
Finance,23 the Court already made the pronouncement that "[i]f a change is desired in the practice [of
...
the Bicameral Conference Committee] it must be sought in Congress since this question is not
covered by any constitutional provision but is only an internal rule of each house." 24 To date,
The creation of such conference committee was apparently in response to a problem, not addressed by Congress has not seen it fit to make such changes adverted to by the Court. It seems, therefore, that
any constitutional provision, where the two houses of Congress find themselves in disagreement over Congress finds the practices of the bicameral conference committee to be very useful for purposes of
changes or amendments introduced by the other house in a legislative bill. Given that one of the most prompt and efficient legislative action.
basic powers of the legislative branch is to formulate and implement its own rules of proceedings and to
discipline its members, may the Court then delve into the details of how Congress complies with its
Nevertheless, just to put minds at ease that no blatant irregularities tainted the proceedings of the
internal rules or how it conducts its business of passing legislation? Note that in the present petitions, the
bicameral conference committees, the Court deems it necessary to dwell on the issue. The Court
issue is not whether provisions of the rules of both houses creating the bicameral conference committee
observes that there was a necessity for a conference committee because a comparison of the provisions

128
of House Bill Nos. 3555 and 3705 on one hand, and Senate Bill No. 1950 on the other, reveals that there No similar provision   No similar   Provided for amendments to several
were indeed disagreements. As pointed out in the petitions, said disagreements were as follows: provision NIRC provisions regarding corporate
income, percentage, franchise and
House Bill No. 3555 House Bill No.3705 Senate Bill No. 1950 excise taxes
With regard to "Stand-By Authority" in favor of President The disagreements between the provisions in the House bills and the Senate bill were with regard to (1)
what rate of VAT is to be imposed; (2) whether only the VAT imposed on electricity generation,
Provides for 12% VAT on Provides for 12% VAT in Provides for a single rate of
transmission and distribution companies should not be passed on to consumers, as proposed in the
every sale of goods or general on sales of goods or 10% VAT on sale of goods
Senate bill, or both the VAT imposed on electricity generation, transmission and distribution companies
properties (amending Sec. properties and reduced rates or properties (amending
and the VAT imposed on sale of petroleum products should not be passed on to consumers, as proposed
106 of NIRC); 12% VAT for sale of certain locally Sec. 106 of NIRC), 10%
in the House bill; (3) in what manner input tax credits should be limited; (4) and whether the NIRC
on importation of goods manufactured goods and VAT on sale of services
provisions on corporate income taxes, percentage, franchise and excise taxes should be amended.
(amending Sec. 107 of petroleum products and raw including sale of electricity
NIRC); and 12% VAT on materials to be used in the by generation companies,
sale of services and use manufacture thereof transmission and There being differences and/or disagreements on the foregoing provisions of the House and Senate bills,
or lease of properties (amending Sec. 106 of NIRC); distribution companies, and the Bicameral Conference Committee was mandated by the rules of both houses of Congress to act on
(amending Sec. 108 of 12% VAT on importation of use or lease of properties the same by settling said differences and/or disagreements. The Bicameral Conference Committee acted
NIRC) goods and reduced rates for (amending Sec. 108 of on the disagreeing provisions by making the following changes:
certain imported products NIRC)
including petroleum products 1. With regard to the disagreement on the rate of VAT to be imposed, it would appear from the
(amending Sec. 107 of NIRC); Conference Committee Report that the Bicameral Conference Committee tried to bridge the gap in the
and 12% VAT on sale of difference between the 10% VAT rate proposed by the Senate, and the various rates with 12% as the
services and use or lease of highest VAT rate proposed by the House, by striking a compromise whereby the present 10% VAT rate
properties and a reduced rate would be retained until certain conditions arise, i.e., the value-added tax collection as a percentage of
for certain services including gross domestic product (GDP) of the previous year exceeds 2 4/5%, or National Government deficit as a
power generation (amending percentage of GDP of the previous year exceeds 1½%, when the President, upon recommendation of the
Sec. 108 of NIRC) Secretary of Finance shall raise the rate of VAT to 12% effective January 1, 2006.
With regard to the "no pass-on" provision
No similar provision   Provides that the VAT   Provides that the VAT 2. With regard to the disagreement on whether only the VAT imposed on electricity generation,
imposed on power generation imposed on sales of transmission and distribution companies should not be passed on to consumers or whether both the VAT
and on the sale of petroleum electricity by generation imposed on electricity generation, transmission and distribution companies and the VAT imposed on sale
products shall be absorbed by companies and services of of petroleum products may be passed on to consumers, the Bicameral Conference Committee chose to
generation companies or transmission companies and settle such disagreement by altogether deleting from its Report any no pass-on provision.
sellers, respectively, and shall distribution companies, as
not be passed on to well as those of franchise 3. With regard to the disagreement on whether input tax credits should be limited or not, the Bicameral
consumers grantees of electric utilities Conference Committee decided to adopt the position of the House by putting a limitation on the amount of
shall not apply to residential input tax that may be credited against the output tax, although it crafted its own language as to the
end-users. VAT shall be amount of the limitation on input tax credits and the manner of computing the same by providing thus:
absorbed by generation,
transmission, and distribution
companies. (A) Creditable Input Tax. – . . .
With regard to 70% limit on input tax credit
Provides that the input tax   No similar provision   Provides that the input tax ...
credit for capital goods on credit for capital goods on
which a VAT has been paid which a VAT has been paid Provided, The input tax on goods purchased or imported in a calendar month for use in trade or business
shall be equally distributed shall be equally distributed for which deduction for depreciation is allowed under this Code, shall be spread evenly over the month of
over 5 years or the over 5 years or the acquisition and the fifty-nine (59) succeeding months if the aggregate acquisition cost for such goods,
depreciable life of such depreciable life of such excluding the VAT component thereof, exceeds one million Pesos (₱1,000,000.00): PROVIDED,
capital goods; the input tax capital goods; the input tax however, that if the estimated useful life of the capital good is less than five (5) years, as used for
credit for goods and credit for goods and services depreciation purposes, then the input VAT shall be spread over such shorter period: . . .
services other than capital other than capital goods shall
goods shall not exceed 5% not exceed 90% of the output (B) Excess Output or Input Tax. – If at the end of any taxable quarter the output tax exceeds the input tax,
of the total amount of such VAT. the excess shall be paid by the VAT-registered person. If the input tax exceeds the output tax, the excess
goods and services; and for shall be carried over to the succeeding quarter or quarters: PROVIDED that the input tax inclusive of input
persons engaged in retail VAT carried over from the previous quarter that may be credited in every quarter shall not exceed seventy
trading of goods, the percent (70%) of the output VAT: PROVIDED, HOWEVER, THAT any input tax attributable to zero-rated
allowable input tax credit sales by a VAT-registered person may at his option be refunded or credited against other internal revenue
shall not exceed 11% of the taxes, . . .
total amount of goods
purchased.
4. With regard to the amendments to other provisions of the NIRC on corporate income tax, franchise,
With regard to amendments to be made to NIRC provisions regarding income and percentage and excise taxes, the conference committee decided to include such amendments and
excise taxes

129
basically adopted the provisions found in Senate Bill No. 1950, with some changes as to the rate of the Court recognized the long-standing legislative practice of giving said conference committee ample latitude
tax to be imposed. for compromising differences between the Senate and the House. Thus, in the Tolentino case, it was held
that:
Under the provisions of both the Rules of the House of Representatives and Senate Rules, the Bicameral
Conference Committee is mandated to settle the differences between the disagreeing provisions in the . . . it is within the power of a conference committee to include in its report an entirely new provision that is
House bill and the Senate bill. The term "settle" is synonymous to "reconcile" and "harmonize."25 To not found either in the House bill or in the Senate bill. If the committee can propose an amendment
reconcile or harmonize disagreeing provisions, the Bicameral Conference Committee may then (a) adopt consisting of one or two provisions, there is no reason why it cannot propose several provisions,
the specific provisions of either the House bill or Senate bill, (b) decide that neither provisions in the collectively considered as an "amendment in the nature of a substitute," so long as such amendment is
House bill or the provisions in the Senate bill would germane to the subject of the bills before the committee. After all, its report was not final but needed the
approval of both houses of Congress to become valid as an act of the legislative department. The charge
be carried into the final form of the bill, and/or (c) try to arrive at a compromise between the disagreeing that in this case the Conference Committee acted as a third legislative chamber is thus without
provisions. any basis.31 (Emphasis supplied)

In the present case, the changes introduced by the Bicameral Conference Committee on disagreeing B. R.A. No. 9337 Does Not Violate Article VI, Section 26(2) of the Constitution on the "No-Amendment
provisions were meant only to reconcile and harmonize the disagreeing provisions for it did not inject any Rule"
idea or intent that is wholly foreign to the subject embraced by the original provisions.
Article VI, Sec. 26 (2) of the Constitution, states:
The so-called stand-by authority in favor of the President, whereby the rate of 10% VAT wanted by the
Senate is retained until such time that certain conditions arise when the 12% VAT wanted by the House No bill passed by either House shall become a law unless it has passed three readings on separate days,
shall be imposed, appears to be a compromise to try to bridge the difference in the rate of VAT proposed and printed copies thereof in its final form have been distributed to its Members three days before its
by the two houses of Congress. Nevertheless, such compromise is still totally within the subject of what passage, except when the President certifies to the necessity of its immediate enactment to meet a public
rate of VAT should be imposed on taxpayers. calamity or emergency. Upon the last reading of a bill, no amendment thereto shall be allowed, and the
vote thereon shall be taken immediately thereafter, and the yeas and nays entered in the Journal.
The no pass-on provision was deleted altogether. In the transcripts of the proceedings of the Bicameral
Conference Committee held on May 10, 2005, Sen. Ralph Recto, Chairman of the Senate Panel, Petitioners’ argument that the practice where a bicameral conference committee is allowed to add or
explained the reason for deleting the no pass-on provision in this wise: delete provisions in the House bill and the Senate bill after these had passed three readings is in effect a
circumvention of the "no amendment rule" (Sec. 26 (2), Art. VI of the 1987 Constitution), fails to convince
. . . the thinking was just to keep the VAT law or the VAT bill simple. And we were thinking that no sector the Court to deviate from its ruling in the Tolentino case that:
should be a beneficiary of legislative grace, neither should any sector be discriminated on. The VAT is an
indirect tax. It is a pass on-tax. And let’s keep it plain and simple. Let’s not confuse the bill and put a no Nor is there any reason for requiring that the Committee’s Report in these cases must have undergone
pass-on provision. Two-thirds of the world have a VAT system and in this two-thirds of the globe, I have three readings in each of the two houses. If that be the case, there would be no end to negotiation since
yet to see a VAT with a no pass-though provision. So, the thinking of the Senate is basically simple, let’s each house may seek modification of the compromise bill. . . .
keep the VAT simple.26 (Emphasis supplied)
Art. VI. § 26 (2) must, therefore, be construed as referring only to bills introduced for the first time
Rep. Teodoro Locsin further made the manifestation that the no pass-on provision "never really enjoyed in either house of Congress, not to the conference committee report.32 (Emphasis supplied)
the support of either House."27
The Court reiterates here that the "no-amendment rule" refers only to the procedure to be followed
With regard to the amount of input tax to be credited against output tax, the Bicameral Conference by each house of Congress with regard to bills initiated in each of said respective houses, before
Committee came to a compromise on the percentage rate of the limitation or cap on such input tax credit, said bill is transmitted to the other house for its concurrence or amendment. Verily, to construe said
but again, the change introduced by the Bicameral Conference Committee was totally within the intent of provision in a way as to proscribe any further changes to a bill after one house has voted on it would lead
both houses to put a cap on input tax that may be to absurdity as this would mean that the other house of Congress would be deprived of its constitutional
power to amend or introduce changes to said bill. Thus, Art. VI, Sec. 26 (2) of the Constitution cannot be
credited against the output tax. From the inception of the subject revenue bill in the House of taken to mean that the introduction by the Bicameral Conference Committee of amendments and
Representatives, one of the major objectives was to "plug a glaring loophole in the tax policy and modifications to disagreeing provisions in bills that have been acted upon by both houses of Congress is
administration by creating vital restrictions on the claiming of input VAT tax credits . . ." and "[b]y prohibited.
introducing limitations on the claiming of tax credit, we are capping a major leakage that has placed our
collection efforts at an apparent disadvantage."28 C. R.A. No. 9337 Does Not Violate Article VI, Section 24 of the Constitution on Exclusive Origination of
Revenue Bills
As to the amendments to NIRC provisions on taxes other than the value-added tax proposed in Senate
Bill No. 1950, since said provisions were among those referred to it, the conference committee had to act Coming to the issue of the validity of the amendments made regarding the NIRC provisions on corporate
on the same and it basically adopted the version of the Senate. income taxes and percentage, excise taxes. Petitioners refer to the following provisions, to wit:

Thus, all the changes or modifications made by the Bicameral Conference Committee were germane to Section 27 Rates of Income Tax on Domestic Corporation
subjects of the provisions referred 28(A)(1) Tax on Resident Foreign Corporation
28(B)(1) Inter-corporate Dividends
to it for reconciliation. Such being the case, the Court does not see any grave abuse of discretion 34(B)(1) Inter-corporate Dividends
amounting to lack or excess of jurisdiction committed by the Bicameral Conference Committee. In the Tax on Persons Exempt from VAT
earlier cases of Philippine Judges Association vs. Prado29 and Tolentino vs. Secretary of Finance,30 the

130
Percentage Tax on domestic carriers and keepers of Garage Since there is no question that the revenue bill exclusively originated in the House of Representatives, the
Tax on franchises Senate was acting within its
Tax on banks and Non-Bank Financial Intermediaries
Excise Tax on manufactured oils and other fuels constitutional power to introduce amendments to the House bill when it included provisions in Senate Bill
No. 1950 amending corporate income taxes, percentage, excise and franchise taxes. Verily, Article VI,
Excise Tax on mineral products
Section 24 of the Constitution does not contain any prohibition or limitation on the extent of the
Registration requirements amendments that may be introduced by the Senate to the House revenue bill.
Issuance of receipts or sales or commercial invoices
Disposition of Incremental Revenue Furthermore, the amendments introduced by the Senate to the NIRC provisions that had not been
Petitioners claim that the amendments to these provisions of the NIRC did not at all originate from the touched in the House bills are still in furtherance of the intent of the House in initiating the subject revenue
House. They aver that House Bill No. 3555 proposed amendments only regarding Sections 106, 107, 108, bills. The Explanatory Note of House Bill No. 1468, the very first House bill introduced on the floor, which
110 and 114 of the NIRC, while House Bill No. 3705 proposed amendments only to Sections 106, was later substituted by House Bill No. 3555, stated:
107,108, 109, 110 and 111 of the NIRC; thus, the other sections of the NIRC which the Senate amended
but which amendments were not found in the House bills are not intended to be amended by the House of
Representatives. Hence, they argue that since the proposed amendments did not originate from the One of the challenges faced by the present administration is the urgent and daunting task of solving the
House, such amendments are a violation of Article VI, Section 24 of the Constitution. country’s serious financial problems. To do this, government expenditures must be strictly monitored and
controlled and revenues must be significantly increased. This may be easier said than done, but our fiscal
authorities are still optimistic the government will be operating on a balanced budget by the year 2009. In
The argument does not hold water. fact, several measures that will result to significant expenditure savings have been identified by the
administration. It is supported with a credible package of revenue measures that include measures
Article VI, Section 24 of the Constitution reads: Sec. 24. All appropriation, revenue or tariff bills, bills to improve tax administration and control the leakages in revenues from income taxes and the
authorizing increase of the public debt, bills of local application, and private bills shall originate exclusively value-added tax (VAT). (Emphasis supplied)
in the House of Representatives but the Senate may propose or concur with amendments.
Rep. Eric D. Singson, in his sponsorship speech for House Bill No. 3555, declared that:
In the present cases, petitioners admit that it was indeed House Bill Nos. 3555 and 3705 that initiated the
move for amending provisions of the NIRC dealing mainly with the value-added tax. Upon transmittal of In the budget message of our President in the year 2005, she reiterated that we all acknowledged that on
said House bills to the Senate, the Senate came out with Senate Bill No. 1950 proposing amendments not top of our agenda must be the restoration of the health of our fiscal system.
only to NIRC provisions on the value-added tax but also amendments to NIRC provisions on other kinds
of taxes. Is the introduction by the Senate of provisions not dealing directly with the value- added tax,
which is the only kind of tax being amended in the House bills, still within the purview of the constitutional In order to considerably lower the consolidated public sector deficit and eventually achieve a balanced
provision authorizing the Senate to propose or concur with amendments to a revenue bill that originated budget by the year 2009, we need to seize windows of opportunities which might seem poignant in
from the House? the beginning, but in the long run prove effective and beneficial to the overall status of our
economy. One such opportunity is a review of existing tax rates, evaluating the relevance given
our present conditions.34 (Emphasis supplied)
The foregoing question had been squarely answered in the Tolentino case, wherein the Court held, thus:
Notably therefore, the main purpose of the bills emanating from the House of Representatives is to bring
. . . To begin with, it is not the law – but the revenue bill – which is required by the Constitution to in sizeable revenues for the government
"originate exclusively" in the House of Representatives. It is important to emphasize this, because a bill
originating in the House may undergo such extensive changes in the Senate that the result may be a
rewriting of the whole. . . . At this point, what is important to note is that, as a result of the Senate action, a to supplement our country’s serious financial problems, and improve tax administration and control of the
distinct bill may be produced. To insist that a revenue statute – and not only the bill which initiated leakages in revenues from income taxes and value-added taxes. As these house bills were transmitted to
the legislative process culminating in the enactment of the law – must substantially be the same the Senate, the latter, approaching the measures from the point of national perspective, can introduce
as the House bill would be to deny the Senate’s power not only to "concur with amendments" but amendments within the purposes of those bills. It can provide for ways that would soften the impact of the
also to "propose amendments." It would be to violate the coequality of legislative power of the two VAT measure on the consumer, i.e., by distributing the burden across all sectors instead of putting it
houses of Congress and in fact make the House superior to the Senate. entirely on the shoulders of the consumers. The sponsorship speech of Sen. Ralph Recto on why the
provisions on income tax on corporation were included is worth quoting:

All in all, the proposal of the Senate Committee on Ways and Means will raise ₱64.3 billion in additional
revenues annually even while by mitigating prices of power, services and petroleum products.
…Given, then, the power of the Senate to propose amendments, the Senate can propose its own
version even with respect to bills which are required by the Constitution to originate in the House.
However, not all of this will be wrung out of VAT. In fact, only ₱48.7 billion amount is from the VAT on
twelve goods and services. The rest of the tab – ₱10.5 billion- will be picked by corporations.
...
What we therefore prescribe is a burden sharing between corporate Philippines and the consumer. Why
Indeed, what the Constitution simply means is that the initiative for filing revenue, tariff or tax bills, bills should the latter bear all the pain? Why should the fiscal salvation be only on the burden of the
authorizing an increase of the public debt, private bills and bills of local application must come from the consumer?
House of Representatives on the theory that, elected as they are from the districts, the members of the
House can be expected to be more sensitive to the local needs and problems. On the other hand,
the senators, who are elected at large, are expected to approach the same problems from the The corporate world’s equity is in form of the increase in the corporate income tax from 32 to 35 percent,
national perspective. Both views are thereby made to bear on the enactment of such but up to 2008 only. This will raise ₱10.5 billion a year. After that, the rate will slide back, not to its old rate
laws.33 (Emphasis supplied) of 32 percent, but two notches lower, to 30 percent.

131
Clearly, we are telling those with the capacity to pay, corporations, to bear with this emergency provision SEC. 4. Sec. 106 of the same Code, as amended, is hereby further amended to read as follows:
that will be in effect for 1,200 days, while we put our fiscal house in order. This fiscal medicine will have an
expiry date. SEC. 106. Value-Added Tax on Sale of Goods or Properties. –

For their assistance, a reward of tax reduction awaits them. We intend to keep the length of their sacrifice (A) Rate and Base of Tax. – There shall be levied, assessed and collected on every sale, barter or
brief. We would like to assure them that not because there is a light at the end of the tunnel, this exchange of goods or properties, a value-added tax equivalent to ten percent (10%) of the gross selling
government will keep on making the tunnel long. price or gross value in money of the goods or properties sold, bartered or exchanged, such tax to be paid
by the seller or transferor: provided, that the President, upon the recommendation of the Secretary
The responsibility will not rest solely on the weary shoulders of the small man. Big business will be there of Finance, shall, effective January 1, 2006, raise the rate of value-added tax to twelve percent
to share the burden.35 (12%), after any of the following conditions has been satisfied.

As the Court has said, the Senate can propose amendments and in fact, the amendments made on (i) value-added tax collection as a percentage of Gross Domestic Product (GDP) of the previous
provisions in the tax on income of corporations are germane to the purpose of the house bills which is to year exceeds two and four-fifth percent (2 4/5%) or
raise revenues for the government.
(ii) national government deficit as a percentage of GDP of the previous year exceeds one and one-
Likewise, the Court finds the sections referring to other percentage and excise taxes germane to the half percent (1 ½%).
reforms to the VAT system, as these sections would cushion the effects of VAT on consumers.
Considering that certain goods and services which were subject to percentage tax and excise tax would SEC. 5. Section 107 of the same Code, as amended, is hereby further amended to read as follows:
no longer be VAT-exempt, the consumer would be burdened more as they would be paying the VAT in
addition to these taxes. Thus, there is a need to amend these sections to soften the impact of VAT. Again,
in his sponsorship speech, Sen. Recto said: SEC. 107. Value-Added Tax on Importation of Goods. –

However, for power plants that run on oil, we will reduce to zero the present excise tax on bunker fuel, to (A) In General. – There shall be levied, assessed and collected on every importation of goods a value-
lessen the effect of a VAT on this product. added tax equivalent to ten percent (10%) based on the total value used by the Bureau of Customs in
determining tariff and customs duties, plus customs duties, excise taxes, if any, and other charges, such
tax to be paid by the importer prior to the release of such goods from customs custody: Provided, That
For electric utilities like Meralco, we will wipe out the franchise tax in exchange for a VAT. where the customs duties are determined on the basis of the quantity or volume of the goods, the value-
added tax shall be based on the landed cost plus excise taxes, if any: provided, further, that the
And in the case of petroleum, while we will levy the VAT on oil products, so as not to destroy the VAT President, upon the recommendation of the Secretary of Finance, shall, effective January 1, 2006,
chain, we will however bring down the excise tax on socially sensitive products such as diesel, bunker, raise the rate of value-added tax to twelve percent (12%) after any of the following conditions has
fuel and kerosene. been satisfied.

What do all these exercises point to? These are not contortions of giving to the left hand what was taken (i) value-added tax collection as a percentage of Gross Domestic Product (GDP) of the previous
from the right. Rather, these sprang from our concern of softening the impact of VAT, so that the people year exceeds two and four-fifth percent (2 4/5%) or
can cushion the blow of higher prices they will have to pay as a result of VAT.36
(ii) national government deficit as a percentage of GDP of the previous year exceeds one and one-
The other sections amended by the Senate pertained to matters of tax administration which are necessary half percent (1 ½%).
for the implementation of the changes in the VAT system.
SEC. 6. Section 108 of the same Code, as amended, is hereby further amended to read as follows:
To reiterate, the sections introduced by the Senate are germane to the subject matter and purposes of the
house bills, which is to supplement our country’s fiscal deficit, among others. Thus, the Senate acted SEC. 108. Value-added Tax on Sale of Services and Use or Lease of Properties –
within its power to propose those amendments.
(A) Rate and Base of Tax. – There shall be levied, assessed and collected, a value-added tax equivalent
SUBSTANTIVE ISSUES - I. to ten percent (10%) of gross receipts derived from the sale or exchange of services: provided, that the
President, upon the recommendation of the Secretary of Finance, shall, effective January 1, 2006,
Whether Sections 4, 5 and 6 of R.A. No. 9337, amending Sections 106, 107 and 108 of the NIRC, violate raise the rate of value-added tax to twelve percent (12%), after any of the following conditions has
the following provisions of the Constitution: been satisfied.

a. Article VI, Section 28(1), and (i) value-added tax collection as a percentage of Gross Domestic Product (GDP) of the previous
b. Article VI, Section 28(2) year exceeds two and four-fifth percent (2 4/5%) or
A. No Undue Delegation of Legislative Power (ii) national government deficit as a percentage of GDP of the previous year exceeds one and one-
Petitioners ABAKADA GURO Party List, et al., Pimentel, Jr., et al., and Escudero, et al. contend in half percent (1 ½%). (Emphasis supplied)
common that Sections 4, 5 and 6 of R.A. No. 9337, amending Sections 106, 107 and 108, respectively, of Petitioners allege that the grant of the stand-by authority to the President to increase the VAT rate is a
the NIRC giving the President the stand-by authority to raise the VAT rate from 10% to 12% when a virtual abdication by Congress of its exclusive power to tax because such delegation is not within the
certain condition is met, constitutes undue delegation of the legislative power to tax. purview of Section 28 (2), Article VI of the Constitution, which provides:

The assailed provisions read as follows:

132
The Congress may, by law, authorize the President to fix within specified limits, and may impose, tariff (3) Delegation to the people at large;
rates, import and export quotas, tonnage and wharfage dues, and other duties or imposts within the (4) Delegation to local governments; and
framework of the national development program of the government. (5) Delegation to administrative bodies.
In every case of permissible delegation, there must be a showing that the delegation itself is valid. It is
They argue that the VAT is a tax levied on the sale, barter or exchange of goods and properties as well as valid only if the law (a) is complete in itself, setting forth therein the policy to be executed, carried out, or
on the sale or exchange of services, which cannot be included within the purview of tariffs under the implemented by the delegate;41 and (b) fixes a standard — the limits of which are sufficiently determinate
exempted delegation as the latter refers to customs duties, tolls or tribute payable upon merchandise to and determinable — to which the delegate must conform in the performance of his functions.42 A
the government and usually imposed on goods or merchandise imported or exported. sufficient standard is one which defines legislative policy, marks its limits, maps out its boundaries and
specifies the public agency to apply it. It indicates the circumstances under which the legislative command
is to be effected.43 Both tests are intended to prevent a total transference of legislative authority to the
Petitioners ABAKADA GURO Party List, et al., further contend that delegating to the President the delegate, who is not allowed to step into the shoes of the legislature and exercise a power essentially
legislative power to tax is contrary to republicanism. They insist that accountability, responsibility and legislative.44
transparency should dictate the actions of Congress and they should not pass to the President the
decision to impose taxes. They also argue that the law also effectively nullified the President’s power of
control, which includes the authority to set aside and nullify the acts of her subordinates like the Secretary In People vs. Vera,45 the Court, through eminent Justice Jose P. Laurel, expounded on the concept and
of Finance, by mandating the fixing of the tax rate by the President upon the recommendation of the extent of delegation of power in this wise:
Secretary of Finance.
In testing whether a statute constitutes an undue delegation of legislative power or not, it is usual to
Petitioners Pimentel, et al. aver that the President has ample powers to cause, influence or create the inquire whether the statute was complete in all its terms and provisions when it left the hands of the
conditions provided by the law to bring about either or both the conditions precedent. legislature so that nothing was left to the judgment of any other appointee or delegate of the legislature.

On the other hand, petitioners Escudero, et al. find bizarre and revolting the situation that the imposition ...
of the 12% rate would be subject to the whim of the Secretary of Finance, an unelected bureaucrat,
contrary to the principle of no taxation without representation. They submit that the Secretary of Finance is ‘The true distinction’, says Judge Ranney, ‘is between the delegation of power to make the law,
not mandated to give a favorable recommendation and he may not even give his recommendation. which necessarily involves a discretion as to what it shall be, and conferring an authority or
Moreover, they allege that no guiding standards are provided in the law on what basis and as to how he discretion as to its execution, to be exercised under and in pursuance of the law. The first cannot
will make his recommendation. They claim, nonetheless, that any recommendation of the Secretary of be done; to the latter no valid objection can be made.’
Finance can easily be brushed aside by the President since the former is a mere alter ego of the latter,
such that, ultimately, it is the President who decides whether to impose the increased tax rate or not. ...

A brief discourse on the principle of non-delegation of powers is instructive. It is contended, however, that a legislative act may be made to the effect as law after it leaves the hands
of the legislature. It is true that laws may be made effective on certain contingencies, as by proclamation
The principle of separation of powers ordains that each of the three great branches of government has of the executive or the adoption by the people of a particular community. In Wayman vs. Southard, the
exclusive cognizance of and is supreme in matters falling within its own constitutionally allocated Supreme Court of the United States ruled that the legislature may delegate a power not legislative which it
sphere.37 A logical may itself rightfully exercise. The power to ascertain facts is such a power which may be delegated.
There is nothing essentially legislative in ascertaining the existence of facts or conditions as the
corollary to the doctrine of separation of powers is the principle of non-delegation of powers, as expressed basis of the taking into effect of a law. That is a mental process common to all branches of the
in the Latin maxim: potestas delegata non delegari potest which means "what has been delegated, cannot government. Notwithstanding the apparent tendency, however, to relax the rule prohibiting delegation of
be delegated."38 This doctrine is based on the ethical principle that such as delegated power constitutes legislative authority on account of the complexity arising from social and economic forces at work in this
not only a right but a duty to be performed by the delegate through the instrumentality of his own judgment modern industrial age, the orthodox pronouncement of Judge Cooley in his work on Constitutional
and not through the intervening mind of another.39 Limitations finds restatement in Prof. Willoughby's treatise on the Constitution of the United States in the
following language — speaking of declaration of legislative power to administrative agencies: The
principle which permits the legislature to provide that the administrative agent may determine
With respect to the Legislature, Section 1 of Article VI of the Constitution provides that "the Legislative when the circumstances are such as require the application of a law is defended upon the ground
power shall be vested in the Congress of the Philippines which shall consist of a Senate and a House of that at the time this authority is granted, the rule of public policy, which is the essence of the
Representatives." The powers which Congress is prohibited from delegating are those which are strictly, legislative act, is determined by the legislature. In other words, the legislature, as it is its duty to
or inherently and exclusively, legislative. Purely legislative power, which can never be delegated, has do, determines that, under given circumstances, certain executive or administrative action is to be
been described as the authority to make a complete law – complete as to the time when it shall take taken, and that, under other circumstances, different or no action at all is to be taken. What is thus
effect and as to whom it shall be applicable – and to determine the expediency of its left to the administrative official is not the legislative determination of what public policy demands,
enactment.40 Thus, the rule is that in order that a court may be justified in holding a statute but simply the ascertainment of what the facts of the case require to be done according to the
unconstitutional as a delegation of legislative power, it must appear that the power involved is purely terms of the law by which he is governed. The efficiency of an Act as a declaration of legislative
legislative in nature – that is, one appertaining exclusively to the legislative department. It is the nature of will must, of course, come from Congress, but the ascertainment of the contingency upon which
the power, and not the liability of its use or the manner of its exercise, which determines the validity of its the Act shall take effect may be left to such agencies as it may designate. The legislature, then,
delegation. may provide that a law shall take effect upon the happening of future specified contingencies
leaving to some other person or body the power to determine when the specified contingency has
Nonetheless, the general rule barring delegation of legislative powers is subject to the following arisen. (Emphasis supplied).46
recognized limitations or exceptions:
In Edu vs. Ericta,47 the Court reiterated:
(1) Delegation of tariff powers to the President under Section 28 (2) of Article VI of the Constitution;
(2) Delegation of emergency powers to the President under Section 23 (2) of Article VI of the Constitution;

133
What cannot be delegated is the authority under the Constitution to make laws and to alter and repeal Thus, it is the ministerial duty of the President to immediately impose the 12% rate upon the existence of
them; the test is the completeness of the statute in all its terms and provisions when it leaves the hands of any of the conditions specified by Congress. This is a duty which cannot be evaded by the President.
the legislature. To determine whether or not there is an undue delegation of legislative power, the inquiry Inasmuch as the law specifically uses the word shall, the exercise of discretion by the President does not
must be directed to the scope and definiteness of the measure enacted. The legislative does not come into play. It is a clear directive to impose the 12% VAT rate when the specified conditions are
abdicate its functions when it describes what job must be done, who is to do it, and what is the present. The time of taking into effect of the 12% VAT rate is based on the happening of a certain
scope of his authority. For a complex economy, that may be the only way in which the legislative specified contingency, or upon the ascertainment of certain facts or conditions by a person or body other
process can go forward. A distinction has rightfully been made between delegation of power to than the legislature itself.
make the laws which necessarily involves a discretion as to what it shall be, which constitutionally
may not be done, and delegation of authority or discretion as to its execution to be exercised The Court finds no merit to the contention of petitioners ABAKADA GURO Party List, et al. that the law
under and in pursuance of the law, to which no valid objection can be made. The Constitution is effectively nullified the President’s power of control over the Secretary of Finance by mandating the fixing
thus not to be regarded as denying the legislature the necessary resources of flexibility and practicability. of the tax rate by the President upon the recommendation of the Secretary of Finance. The Court cannot
(Emphasis supplied).48 also subscribe to the position of petitioners

Clearly, the legislature may delegate to executive officers or bodies the power to determine certain facts Pimentel, et al. that the word shall should be interpreted to mean may in view of the phrase "upon the
or conditions, or the happening of contingencies, on which the operation of a statute is, by its terms, made recommendation of the Secretary of Finance." Neither does the Court find persuasive the submission of
to depend, but the legislature must prescribe sufficient standards, policies or limitations on their petitioners Escudero, et al. that any recommendation by the Secretary of Finance can easily be brushed
authority.49 While the power to tax cannot be delegated to executive agencies, details as to the aside by the President since the former is a mere alter ego of the latter.
enforcement and administration of an exercise of such power may be left to them, including the power to
determine the existence of facts on which its operation depends.50
When one speaks of the Secretary of Finance as the alter ego of the President, it simply means that as
head of the Department of Finance he is the assistant and agent of the Chief Executive. The multifarious
The rationale for this is that the preliminary ascertainment of facts as basis for the enactment of legislation executive and administrative functions of the Chief Executive are performed by and through the executive
is not of itself a legislative function, but is simply ancillary to legislation. Thus, the duty of correlating departments, and the acts of the secretaries of such departments, such as the Department of Finance,
information and making recommendations is the kind of subsidiary activity which the legislature may performed and promulgated in the regular course of business, are, unless disapproved or reprobated by
perform through its members, or which it may delegate to others to perform. Intelligent legislation on the the Chief Executive, presumptively the acts of the Chief Executive. The Secretary of Finance, as such,
complicated problems of modern society is impossible in the absence of accurate information on the part occupies a political position and holds office in an advisory capacity, and, in the language of Thomas
of the legislators, and any reasonable method of securing such information is proper.51 The Constitution Jefferson, "should be of the President's bosom confidence" and, in the language of Attorney-General
as a continuously operative charter of government does not require that Congress find for itself Cushing, is "subject to the direction of the President."55

every fact upon which it desires to base legislative action or that it make for itself detailed determinations In the present case, in making his recommendation to the President on the existence of either of the two
which it has declared to be prerequisite to application of legislative policy to particular facts and conditions, the Secretary of Finance is not acting as the alter ego of the President or even her
circumstances impossible for Congress itself properly to investigate.52 subordinate. In such instance, he is not subject to the power of control and direction of the President. He
is acting as the agent of the legislative department, to determine and declare the event upon which its
In the present case, the challenged section of R.A. No. 9337 is the common proviso in Sections 4, 5 and 6 expressed will is to take effect.56 The Secretary of Finance becomes the means or tool by which
which reads as follows: legislative policy is determined and implemented, considering that he possesses all the facilities to gather
data and information and has a much broader perspective to properly evaluate them. His function is to
That the President, upon the recommendation of the Secretary of Finance, shall, effective January 1, gather and collate statistical data and other pertinent information and verify if any of the two conditions
2006, raise the rate of value-added tax to twelve percent (12%), after any of the following conditions has laid out by Congress is present. His personality in such instance is in reality but a projection of that of
been satisfied: Congress. Thus, being the agent of Congress and not of the President, the President cannot alter or
modify or nullify, or set aside the findings of the Secretary of Finance and to substitute the judgment of the
former for that of the latter.
(i) Value-added tax collection as a percentage of Gross Domestic Product (GDP) of the previous year
exceeds two and four-fifth percent (2 4/5%); or
Congress simply granted the Secretary of Finance the authority to ascertain the existence of a fact,
namely, whether by December 31, 2005, the value-added tax collection as a percentage of Gross
(ii) National government deficit as a percentage of GDP of the previous year exceeds one and one-half Domestic Product (GDP) of the previous year exceeds two and four-fifth percent (24/5%) or the national
percent (1 ½%). government deficit as a percentage of GDP of the previous year exceeds one and one-half percent (1½
%). If either of these two instances has occurred, the Secretary of Finance, by legislative mandate, must
The case before the Court is not a delegation of legislative power. It is simply a delegation of submit such information to the President. Then the 12% VAT rate must be imposed by the President
ascertainment of facts upon which enforcement and administration of the increase rate under the law is effective January 1, 2006. There is no undue delegation of legislative power but only of the
contingent. The legislature has made the operation of the 12% rate effective January 1, 2006, contingent discretion as to the execution of a law. This is constitutionally permissible.57 Congress does not
upon a specified fact or condition. It leaves the entire operation or non-operation of the 12% rate upon abdicate its functions or unduly delegate power when it describes what job must be done, who must do it,
factual matters outside of the control of the executive. and what is the scope of his authority; in our complex economy that is frequently the only way in which the
legislative process can go forward.58
No discretion would be exercised by the President. Highlighting the absence of discretion is the fact that
the word shall is used in the common proviso. The use of the word shall connotes a mandatory order. Its As to the argument of petitioners ABAKADA GURO Party List, et al. that delegating to the President the
use in a statute denotes an imperative obligation and is inconsistent with the idea of discretion.53 Where legislative power to tax is contrary to the principle of republicanism, the same deserves scant
the law is clear and unambiguous, it must be taken to mean exactly what it says, and courts have no consideration. Congress did not delegate the power to tax but the mere implementation of the law. The
choice but to see to it that the mandate is obeyed.54 intent and will to increase the VAT rate to 12% came from Congress and the task of the President is to
simply execute the legislative policy. That Congress chose to do so in such a manner is not within the
province of the Court to inquire into, its task being to interpret the law.59

134
The insinuation by petitioners Pimentel, et al. that the President has ample powers to cause, influence or That the first condition amounts to an incentive to the President to increase the VAT collection does not
create the conditions to bring about either or both the conditions precedent does not deserve any merit as render it unconstitutional so long as there is a public purpose for which the law was passed, which in this
this argument is highly speculative. The Court does not rule on allegations which are manifestly case, is mainly to raise revenue. In fact, fiscal adequacy dictated the need for a raise in revenue.
conjectural, as these may not exist at all. The Court deals with facts, not fancies; on realities, not
appearances. When the Court acts on appearances instead of realities, justice and law will be short-lived. The principle of fiscal adequacy as a characteristic of a sound tax system was originally stated by Adam
Smith in his Canons of Taxation (1776), as:
B. The 12% Increase VAT Rate Does Not Impose an Unfair and Unnecessary Additional Tax Burden
IV. Every tax ought to be so contrived as both to take out and to keep out of the pockets of the people as
Petitioners Pimentel, et al. argue that the 12% increase in the VAT rate imposes an unfair and additional little as possible over and above what it brings into the public treasury of the state.63
tax burden on the people. Petitioners also argue that the 12% increase, dependent on any of the 2
conditions set forth in the contested provisions, is ambiguous because it does not state if the VAT rate It simply means that sources of revenues must be adequate to meet government expenditures and their
would be returned to the original 10% if the rates are no longer satisfied. Petitioners also argue that such variations.64
rate is unfair and unreasonable, as the people are unsure of the applicable VAT rate from year to year.
The dire need for revenue cannot be ignored. Our country is in a quagmire of financial woe. During the
Under the common provisos of Sections 4, 5 and 6 of R.A. No. 9337, if any of the two conditions set forth Bicameral Conference Committee hearing, then Finance Secretary Purisima bluntly depicted the country’s
therein are satisfied, the President shall increase the VAT rate to 12%. The provisions of the law are clear. gloomy state of economic affairs, thus:
It does not provide for a return to the 10% rate nor does it empower the President to so revert if, after the
rate is increased to 12%, the VAT collection goes below the 24/5 of the GDP of the previous year or that
the national government deficit as a percentage of GDP of the previous year does not exceed 1½%. First, let me explain the position that the Philippines finds itself in right now. We are in a position where 90
percent of our revenue is used for debt service. So, for every peso of revenue that we currently raise, 90
goes to debt service. That’s interest plus amortization of our debt. So clearly, this is not a sustainable
Therefore, no statutory construction or interpretation is needed. Neither can conditions or limitations be situation. That’s the first fact.
introduced where none is provided for. Rewriting the law is a forbidden ground that only Congress may
tread upon.60
The second fact is that our debt to GDP level is way out of line compared to other peer countries that
borrow money from that international financial markets. Our debt to GDP is approximately equal to our
Thus, in the absence of any provision providing for a return to the 10% rate, which in this case the Court GDP. Again, that shows you that this is not a sustainable situation.
finds none, petitioners’ argument is, at best, purely speculative. There is no basis for petitioners’ fear of a
fluctuating VAT rate because the law itself does not provide that the rate should go back to 10% if the
conditions provided in Sections 4, 5 and 6 are no longer present. The rule is that where the provision of The third thing that I’d like to point out is the environment that we are presently operating in is not as
the law is clear and unambiguous, so that there is no occasion for the court's seeking the legislative intent, benign as what it used to be the past five years.
the law must be taken as it is, devoid of judicial addition or subtraction.61
What do I mean by that?
Petitioners also contend that the increase in the VAT rate, which was allegedly an incentive to the
President to raise the VAT collection to at least 2 4/5 of the GDP of the previous year, should be based on In the past five years, we’ve been lucky because we were operating in a period of basically global growth
fiscal adequacy. and low interest rates. The past few months, we have seen an inching up, in fact, a rapid increase in the
interest rates in the leading economies of the world. And, therefore, our ability to borrow at reasonable
Petitioners obviously overlooked that increase in VAT collection is not the only condition. There is another prices is going to be challenged. In fact, ultimately, the question is our ability to access the financial
condition, i.e., the national government deficit as a percentage of GDP of the previous year exceeds one markets.
and one-half percent (1 ½%).
When the President made her speech in July last year, the environment was not as bad as it is now, at
Respondents explained the philosophy behind these alternative conditions: least based on the forecast of most financial institutions. So, we were assuming that raising 80 billion
would put us in a position where we can then convince them to improve our ability to borrow at lower
rates. But conditions have changed on us because the interest rates have gone up. In fact, just within this
1. VAT/GDP Ratio > 2.8% room, we tried to access the market for a billion dollars because for this year alone, the Philippines will
have to borrow 4 billion dollars. Of that amount, we have borrowed 1.5 billion. We issued last January a
The condition set for increasing VAT rate to 12% have economic or fiscal meaning. If VAT/GDP is less 25-year bond at 9.7 percent cost. We were trying to access last week and the market was not as
than 2.8%, it means that government has weak or no capability of implementing the VAT or that VAT is favorable and up to now we have not accessed and we might pull back because the conditions are not
not effective in the function of the tax collection. Therefore, there is no value to increase it to 12% because very good.
such action will also be ineffectual.
So given this situation, we at the Department of Finance believe that we really need to front-end our deficit
2. Nat’l Gov’t Deficit/GDP >1.5% reduction. Because it is deficit that is causing the increase of the debt and we are in what we call a debt
spiral. The more debt you have, the more deficit you have because interest and debt service eats and
The condition set for increasing VAT when deficit/GDP is 1.5% or less means the fiscal condition of eats more of your revenue. We need to get out of this debt spiral. And the only way, I think, we can get
government has reached a relatively sound position or is towards the direction of a balanced budget out of this debt spiral is really have a front-end adjustment in our revenue base.65
position. Therefore, there is no need to increase the VAT rate since the fiscal house is in a relatively
healthy position. Otherwise stated, if the ratio is more than 1.5%, there is indeed a need to increase the The image portrayed is chilling. Congress passed the law hoping for rescue from an inevitable
VAT rate.62 catastrophe. Whether the law is indeed sufficient to answer the state’s economic dilemma is not for the
Court to judge. In the Fariñas case, the Court refused to consider the various arguments raised therein
that dwelt on the wisdom of Section 14 of R.A. No. 9006 (The Fair Election Act), pronouncing that:

135
. . . policy matters are not the concern of the Court. Government policy is within the exclusive dominion of quarters." In addition, Section 112(B) allows a VAT-registered person to apply for the issuance of a tax
the political branches of the government. It is not for this Court to look into the wisdom or propriety of credit certificate or refund for any unused input taxes, to the extent that such input taxes have not been
legislative determination. Indeed, whether an enactment is wise or unwise, whether it is based on sound applied against the output taxes. Such unused input tax may be used in payment of his other internal
economic theory, whether it is the best means to achieve the desired results, whether, in short, the revenue taxes.
legislative discretion within its prescribed limits should be exercised in a particular manner are matters for
the judgment of the legislature, and the serious conflict of opinions does not suffice to bring them within The non-application of the unutilized input tax in a given quarter is not ad infinitum, as petitioners
the range of judicial cognizance.66 exaggeratedly contend. Their analysis of the effect of the 70% limitation is incomplete and one-sided. It
ends at the net effect that there will be unapplied/unutilized inputs VAT for a given quarter. It does not
In the same vein, the Court in this case will not dawdle on the purpose of Congress or the executive proceed further to the fact that such unapplied/unutilized input tax may be credited in the subsequent
policy, given that it is not for the judiciary to "pass upon questions of wisdom, justice or expediency of periods as allowed by the carry-over provision of Section 110(B) or that it may later on be refunded
legislation."67 through a tax credit certificate under Section 112(B).

II. Therefore, petitioners’ argument must be rejected.

Whether Section 8 of R.A. No. 9337, amending Sections 110(A)(2) and 110(B) of the NIRC; and Section On the other hand, it appears that petitioner Garcia failed to comprehend the operation of the 70%
12 of R.A. No. 9337, amending Section 114(C) of the NIRC, violate the following provisions of the limitation on the input tax. According to petitioner, the limitation on the creditable input tax in effect allows
Constitution: VAT-registered establishments to retain a portion of the taxes they collect, which violates the principle that
tax collection and revenue should be for public purposes and expenditures
a. Article VI, Section 28(1), and b. Article III, Section 1
As earlier stated, the input tax is the tax paid by a person, passed on to him by the seller, when he buys
A. Due Process and Equal Protection Clauses goods. Output tax meanwhile is the tax due to the person when he sells goods. In computing the VAT
payable, three possible scenarios may arise:
Petitioners Association of Pilipinas Shell Dealers, Inc., et al. argue that Section 8 of R.A. No. 9337,
amending Sections 110 (A)(2), 110 (B), and Section 12 of R.A. No. 9337, amending Section 114 (C) of First, if at the end of a taxable quarter the output taxes charged by the seller are equal to the input taxes
the NIRC are arbitrary, oppressive, excessive and confiscatory. Their argument is premised on the that he paid and passed on by the suppliers, then no payment is required;
constitutional right against deprivation of life, liberty of property without due process of law, as embodied
in Article III, Section 1 of the Constitution. Second, when the output taxes exceed the input taxes, the person shall be liable for the excess, which
has to be paid to the Bureau of Internal Revenue (BIR);69 and
Petitioners also contend that these provisions violate the constitutional guarantee of equal protection of
the law. Third, if the input taxes exceed the output taxes, the excess shall be carried over to the succeeding
quarter or quarters. Should the input taxes result from zero-rated or effectively zero-rated transactions,
The doctrine is that where the due process and equal protection clauses are invoked, considering that any excess over the output taxes shall instead be refunded to the taxpayer or credited against other
they are not fixed rules but rather broad standards, there is a need for proof of such persuasive character internal revenue taxes, at the taxpayer’s option.70
as would lead to such a conclusion. Absent such a showing, the presumption of validity must prevail.68
Section 8 of R.A. No. 9337 however, imposed a 70% limitation on the input tax. Thus, a person can credit
Section 8 of R.A. No. 9337, amending Section 110(B) of the NIRC imposes a limitation on the amount of his input tax only up to the extent of 70% of the output tax. In layman’s term, the value-added taxes that a
input tax that may be credited against the output tax. It states, in part: "[P]rovided, that the input tax person/taxpayer paid and passed on to him by a seller can only be credited up to 70% of the value-added
inclusive of the input VAT carried over from the previous quarter that may be credited in every quarter taxes that is due to him on a taxable transaction. There is no retention of any tax collection because the
shall not exceed seventy percent (70%) of the output VAT: …" person/taxpayer has already previously paid the input tax to a seller, and the seller will subsequently remit
such input tax to the BIR. The party directly liable for the payment of the tax is the seller.71 What only
needs to be done is for the person/taxpayer to apply or credit these input taxes, as evidenced by receipts,
Input Tax is defined under Section 110(A) of the NIRC, as amended, as the value-added tax against his output taxes.
due from or paid by a VAT-registered person on the importation of goods or local purchase of good and
services, including lease or use of property, in the course of trade or business, from a VAT-registered
person, and Output Tax is the value-added tax due on the sale or lease of taxable goods or properties or Petitioners Association of Pilipinas Shell Dealers, Inc., et al. also argue that the input tax partakes the
services by any person registered or required to register under the law. nature of a property that may not be confiscated, appropriated, or limited without due process of law.

Petitioners claim that the contested sections impose limitations on the amount of input tax that may be The input tax is not a property or a property right within the constitutional purview of the due process
claimed. In effect, a portion of the input tax that has already been paid cannot now be credited against the clause. A VAT-registered person’s entitlement to the creditable input tax is a mere statutory privilege.
output tax.
The distinction between statutory privileges and vested rights must be borne in mind for persons have no
Petitioners’ argument is not absolute. It assumes that the input tax exceeds 70% of the output tax, and vested rights in statutory privileges. The state may change or take away rights, which were created by the
therefore, the input tax in excess of 70% remains uncredited. However, to the extent that the input tax is law of the state, although it may not take away property, which was vested by virtue of such rights.72
less than 70% of the output tax, then 100% of such input tax is still creditable.
Under the previous system of single-stage taxation, taxes paid at every level of distribution are not
More importantly, the excess input tax, if any, is retained in a business’s books of accounts and remains recoverable from the taxes payable, although it becomes part of the cost, which is deductible from the
creditable in the succeeding quarter/s. This is explicitly allowed by Section 110(B), which provides that "if gross revenue. When Pres. Aquino issued E.O. No. 273 imposing a 10% multi-stage tax on all sales, it
the input tax exceeds the output tax, the excess shall be carried over to the succeeding quarter or was then that the crediting of the input tax paid on purchase or importation of goods and services by VAT-
registered persons against the output tax was introduced.73 This was adopted by the Expanded VAT Law

136
(R.A. No. 7716),74 and The Tax Reform Act of 1997 (R.A. No. 8424).75 The right to credit input tax as nonresident owners. Under the present Section 114(C), these different rates, except for the 10% on lease
against the output tax is clearly a privilege created by law, a privilege that also the law can remove, or in or property rights payment to nonresidents, were deleted, and a uniform rate of 5% is applied.
this case, limit.
The Court observes, however, that the law the used the word final. In tax usage, final, as opposed to
Petitioners also contest as arbitrary, oppressive, excessive and confiscatory, Section 8 of R.A. No. 9337, creditable, means full. Thus, it is provided in Section 114(C): "final value-added tax at the rate of five
amending Section 110(A) of the NIRC, which provides: percent (5%)."

SEC. 110. Tax Credits. –(A) Creditable Input Tax. – … In Revenue Regulations No. 02-98, implementing R.A. No. 8424 (The Tax Reform Act of 1997), the
concept of final withholding tax on income was explained, to wit:
Provided, That the input tax on goods purchased or imported in a calendar month for use in trade or
business for which deduction for depreciation is allowed under this Code, shall be spread evenly over the SECTION 2.57. Withholding of Tax at Source
month of acquisition and the fifty-nine (59) succeeding months if the aggregate acquisition cost for such
goods, excluding the VAT component thereof, exceeds One million pesos (₱1,000,000.00): Provided, (A) Final Withholding Tax. – Under the final withholding tax system the amount of income tax withheld by
however, That if the estimated useful life of the capital goods is less than five (5) years, as used for the withholding agent is constituted as full and final payment of the income tax due from the payee on
depreciation purposes, then the input VAT shall be spread over such a shorter period: Provided, finally, the said income. The liability for payment of the tax rests primarily on the payor as a withholding agent.
That in the case of purchase of services, lease or use of properties, the input tax shall be creditable to the Thus, in case of his failure to withhold the tax or in case of underwithholding, the deficiency tax shall be
purchaser, lessee or license upon payment of the compensation, rental, royalty or fee. collected from the payor/withholding agent. …

The foregoing section imposes a 60-month period within which to amortize the creditable input tax on (B) Creditable Withholding Tax. – Under the creditable withholding tax system, taxes withheld on certain
purchase or importation of capital goods with acquisition cost of ₱1 Million pesos, exclusive of the VAT income payments are intended to equal or at least approximate the tax due of the payee on said income.
component. Such spread out only poses a delay in the crediting of the input tax. Petitioners’ argument is … Taxes withheld on income payments covered by the expanded withholding tax (referred to in Sec.
without basis because the taxpayer is not permanently deprived of his privilege to credit the input tax. 2.57.2 of these regulations) and compensation income (referred to in Sec. 2.78 also of these regulations)
are creditable in nature.
It is worth mentioning that Congress admitted that the spread-out of the creditable input tax in this case
amounts to a 4-year interest-free loan to the government.76 In the same breath, Congress also justified its As applied to value-added tax, this means that taxable transactions with the government are subject to a
move by saying that the provision was designed to raise an annual revenue of 22.6 billion.77 The 5% rate, which constitutes as full payment of the tax payable on the transaction. This represents the net
legislature also dispelled the fear that the provision will fend off foreign investments, saying that foreign VAT payable of the seller. The other 5% effectively accounts for the standard input VAT (deemed input
investors have other tax incentives provided by law, and citing the case of China, where despite a 17.5% VAT), in lieu of the actual input VAT directly or attributable to the taxable transaction.79
non-creditable VAT, foreign investments were not deterred.78 Again, for whatever is the purpose of the
60-month amortization, this involves executive economic policy and legislative wisdom in which the Court
cannot intervene. The Court need not explore the rationale behind the provision. It is clear that Congress intended to treat
differently taxable transactions with the government.80 This is supported by the fact that under the old
provision, the 5% tax withheld by the government remains creditable against the tax liability of the seller or
With regard to the 5% creditable withholding tax imposed on payments made by the government for contractor, to wit:
taxable transactions, Section 12 of R.A. No. 9337, which amended Section 114 of the NIRC, reads:
SEC. 114. Return and Payment of Value-added Tax. –
SEC. 114. Return and Payment of Value-added Tax. –
(C) Withholding of Creditable Value-added Tax. – The Government or any of its political subdivisions,
(C) Withholding of Value-added Tax. – The Government or any of its political subdivisions, instrumentalities or agencies, including government-owned or controlled corporations (GOCCs) shall,
instrumentalities or agencies, including government-owned or controlled corporations (GOCCs) shall, before making payment on account of each purchase of goods from sellers and services rendered by
before making payment on account of each purchase of goods and services which are subject to the contractors which are subject to the value-added tax imposed in Sections 106 and 108 of this Code,
value-added tax imposed in Sections 106 and 108 of this Code, deduct and withhold a final value-added deduct and withhold the value-added tax due at the rate of three percent (3%) of the gross payment for
tax at the rate of five percent (5%) of the gross payment thereof: Provided, That the payment for lease or the purchase of goods and six percent (6%) on gross receipts for services rendered by contractors on
use of properties or property rights to nonresident owners shall be subject to ten percent (10%) every sale or installment payment which shall be creditable against the value-added tax liability of the
withholding tax at the time of payment. For purposes of this Section, the payor or person in control of the seller or contractor: Provided, however, That in the case of government public works contractors, the
payment shall be considered as the withholding agent. withholding rate shall be eight and one-half percent (8.5%): Provided, further, That the payment for lease
or use of properties or property rights to nonresident owners shall be subject to ten percent (10%)
The value-added tax withheld under this Section shall be remitted within ten (10) days following the end of withholding tax at the time of payment. For this purpose, the payor or person in control of the payment
the month the withholding was made. shall be considered as the withholding agent.

Section 114(C) merely provides a method of collection, or as stated by respondents, a more simplified The valued-added tax withheld under this Section shall be remitted within ten (10) days following the end
VAT withholding system. The government in this case is constituted as a withholding agent with respect to of the month the withholding was made. (Emphasis supplied)
their payments for goods and services.
As amended, the use of the word final and the deletion of the word creditable exhibits Congress’s
Prior to its amendment, Section 114(C) provided for different rates of value-added taxes to be withheld -- intention to treat transactions with the government differently. Since it has not been shown that the class
3% on gross payments for purchases of goods; 6% on gross payments for services supplied by subject to the 5% final withholding tax has been unreasonably narrowed, there is no reason to invalidate
contractors other than by public works contractors; 8.5% on gross payments for services supplied by the provision. Petitioners, as petroleum dealers, are not the only ones subjected to the 5% final
public work contractors; or 10% on payment for the lease or use of properties or property rights to withholding tax. It applies to all those who deal with the government.

137
Moreover, the actual input tax is not totally lost or uncreditable, as petitioners believe. Revenue Uniformity in taxation means that all taxable articles or kinds of property of the same class shall be taxed
Regulations No. 14-2005 or the Consolidated Value-Added Tax Regulations 2005 issued by the BIR, at the same rate. Different articles may be taxed at different amounts provided that the rate is uniform on
provides that should the actual input tax exceed 5% of gross payments, the excess may form part of the the same class everywhere with all people at all times.86
cost. Equally, should the actual input tax be less than 5%, the difference is treated as income.81
In this case, the tax law is uniform as it provides a standard rate of 0% or 10% (or 12%) on all goods and
Petitioners also argue that by imposing a limitation on the creditable input tax, the government gets to tax services. Sections 4, 5 and 6 of R.A. No. 9337, amending Sections 106, 107 and 108, respectively, of the
a profit or value-added even if there is no profit or value-added. NIRC, provide for a rate of 10% (or 12%) on sale of goods and properties, importation of goods, and sale
of services and use or lease of properties. These same sections also provide for a 0% rate on certain
Petitioners’ stance is purely hypothetical, argumentative, and again, one-sided. The Court will not engage sales and transaction.
in a legal joust where premises are what ifs, arguments, theoretical and facts, uncertain. Any disquisition
by the Court on this point will only be, as Shakespeare describes life in Macbeth,82 "full of sound and Neither does the law make any distinction as to the type of industry or trade that will bear the 70%
fury, signifying nothing." limitation on the creditable input tax, 5-year amortization of input tax paid on purchase of capital goods or
the 5% final withholding tax by the government. It must be stressed that the rule of uniform taxation does
What’s more, petitioners’ contention assumes the proposition that there is no profit or value-added. It not deprive Congress of the power to classify subjects of taxation, and only demands uniformity within the
need not take an astute businessman to know that it is a matter of exception that a business will sell particular class.87
goods or services without profit or value-added. It cannot be overstressed that a business is created
precisely for profit. R.A. No. 9337 is also equitable. The law is equipped with a threshold margin. The VAT rate of 0% or 10%
(or 12%) does not apply to sales of goods or services with gross annual sales or receipts not exceeding
The equal protection clause under the Constitution means that "no person or class of persons shall be ₱1,500,000.00.88 Also, basic marine and agricultural food products in their original state are still not
deprived of the same protection of laws which is enjoyed by other persons or other classes in the same subject to the tax,89 thus ensuring that prices at the grassroots level will remain accessible. As was stated
place and in like circumstances."83 in Kapatiran ng mga Naglilingkod sa Pamahalaan ng Pilipinas, Inc. vs. Tan:90

The power of the State to make reasonable and natural classifications for the purposes of taxation has The disputed sales tax is also equitable. It is imposed only on sales of goods or services by persons
long been established. Whether it relates to the subject of taxation, the kind of property, the rates to be engaged in business with an aggregate gross annual sales exceeding ₱200,000.00. Small corner sari-
levied, or the amounts to be raised, the methods of assessment, valuation and collection, the State’s sari stores are consequently exempt from its application. Likewise exempt from the tax are sales of farm
power is entitled to presumption of validity. As a rule, the judiciary will not interfere with such power and marine products, so that the costs of basic food and other necessities, spared as they are from the
absent a clear showing of unreasonableness, discrimination, or arbitrariness.84 incidence of the VAT, are expected to be relatively lower and within the reach of the general public.

Petitioners point out that the limitation on the creditable input tax if the entity has a high ratio of input tax, It is admitted that R.A. No. 9337 puts a premium on businesses with low profit margins, and unduly favors
or invests in capital equipment, or has several transactions with the government, is not based on real and those with high profit margins. Congress was not oblivious to this. Thus, to equalize the weighty burden
substantial differences to meet a valid classification. the law entails, the law, under Section 116, imposed a 3% percentage tax on VAT-exempt persons under
Section 109(v), i.e., transactions with gross annual sales and/or receipts not exceeding ₱1.5 Million. This
acts as a equalizer because in effect, bigger businesses that qualify for VAT coverage and VAT-exempt
The argument is pedantic, if not outright baseless. The law does not make any classification in the subject taxpayers stand on equal-footing.
of taxation, the kind of property, the rates to be levied or the amounts to be raised, the methods of
assessment, valuation and collection. Petitioners’ alleged distinctions are based on variables that bear
different consequences. While the implementation of the law may yield varying end results depending on Moreover, Congress provided mitigating measures to cushion the impact of the imposition of the tax on
one’s profit margin and value-added, the Court cannot go beyond what the legislature has laid down and those previously exempt. Excise taxes on petroleum products91 and natural gas92 were reduced.
interfere with the affairs of business. Percentage tax on domestic carriers was removed.93 Power producers are now exempt from paying
franchise tax.94
The equal protection clause does not require the universal application of the laws on all persons or things
without distinction. This might in fact sometimes result in unequal protection. What the clause requires is Aside from these, Congress also increased the income tax rates of corporations, in order to distribute the
equality among equals as determined according to a valid classification. By classification is meant the burden of taxation. Domestic, foreign, and non-resident corporations are now subject to a 35% income tax
grouping of persons or things similar to each other in certain particulars and different from all others in rate, from a previous 32%.95 Intercorporate dividends of non-resident foreign corporations are still subject
these same particulars.85 to 15% final withholding tax but the tax credit allowed on the corporation’s domicile was increased to
20%.96 The Philippine Amusement and Gaming Corporation (PAGCOR) is not exempt from income taxes
anymore.97 Even the sale by an artist of his works or services performed for the production of such works
Petitioners brought to the Court’s attention the introduction of Senate Bill No. 2038 by Sens. S.R. Osmeña was not spared.
III and Ma. Ana Consuelo A.S. – Madrigal on June 6, 2005, and House Bill No. 4493 by Rep. Eric D.
Singson. The proposed legislation seeks to amend the 70% limitation by increasing the same to 90%.
This, according to petitioners, supports their stance that the 70% limitation is arbitrary and confiscatory. All these were designed to ease, as well as spread out, the burden of taxation, which would otherwise rest
On this score, suffice it to say that these are still proposed legislations. Until Congress amends the law, largely on the consumers. It cannot therefore be gainsaid that R.A. No. 9337 is equitable.
and absent any unequivocal basis for its unconstitutionality, the 70% limitation stays.
C. Progressivity of Taxation
B. Uniformity and Equitability of Taxation
Lastly, petitioners contend that the limitation on the creditable input tax is anything but regressive. It is the
Article VI, Section 28(1) of the Constitution reads: smaller business with higher input tax-output tax ratio that will suffer the consequences.

The rule of taxation shall be uniform and equitable. The Congress shall evolve a progressive system of Progressive taxation is built on the principle of the taxpayer’s ability to pay. This principle was also lifted
taxation. from Adam Smith’s Canons of Taxation, and it states:

138
I. The subjects of every state ought to contribute towards the support of the government, as nearly as There being no constitutional impediment to the full enforcement and implementation of R.A. No. 9337,
possible, in proportion to their respective abilities; that is, in proportion to the revenue which they the temporary restraining order issued by the Court on July 1, 2005 is LIFTED upon finality of herein
respectively enjoy under the protection of the state. decision.SO ORDERED.

Taxation is progressive when its rate goes up depending on the resources of the person affected.98 G.R. No. 115455 October 30, 1995

The VAT is an antithesis of progressive taxation. By its very nature, it is regressive. The principle of ARTURO M. TOLENTINO,  vs.THE SECRETARY OF FINANCE and THE COMMISSIONER OF
progressive taxation has no relation with the VAT system inasmuch as the VAT paid by the consumer or INTERNAL REVENUE, 
business for every goods bought or services enjoyed is the same regardless of income. In
These are motions seeking reconsideration of our decision dismissing the petitions filed in these cases for
other words, the VAT paid eats the same portion of an income, whether big or small. The disparity lies in the declaration of unconstitutionality of R.A. No. 7716, otherwise known as the Expanded Value-Added
the income earned by a person or profit margin marked by a business, such that the higher the income or Tax Law. The motions, of which there are 10 in all, have been filed by the several petitioners in these
profit margin, the smaller the portion of the income or profit that is eaten by VAT. A converso, the lower cases, with the exception of the Philippine Educational Publishers Association, Inc. and the Association of
the income or profit margin, the bigger the part that the VAT eats away. At the end of the day, it is really Philippine Booksellers, petitioners in G.R. No. 115931.
the lower income group or businesses with low-profit margins that is always hardest hit.
The Solicitor General, representing the respondents, filed a consolidated comment, to which the
Nevertheless, the Constitution does not really prohibit the imposition of indirect taxes, like the VAT. What Philippine Airlines, Inc., petitioner in G.R. No. 115852, and the Philippine Press Institute, Inc., petitioner in
it simply provides is that Congress shall "evolve a progressive system of taxation." The Court stated in G.R. No. 115544, and Juan T. David, petitioner in G.R. No. 115525, each filed a reply. In turn the Solicitor
the Tolentino case, thus: General filed on June 1, 1995 a rejoinder to the PPI's reply.

The Constitution does not really prohibit the imposition of indirect taxes which, like the VAT, are On June 27, 1995 the matter was submitted for resolution.
regressive. What it simply provides is that Congress shall ‘evolve a progressive system of taxation.’ The
constitutional provision has been interpreted to mean simply that ‘direct taxes are . . . to be preferred [and] I. Power of the Senate to propose amendments to revenue bills. Some of the petitioners (Tolentino,
as much as possible, indirect taxes should be minimized.’ (E. FERNANDO, THE CONSTITUTION OF Kilosbayan, Inc., Philippine Airlines (PAL), Roco, and Chamber of Real Estate and Builders Association
THE PHILIPPINES 221 (Second ed. 1977)) Indeed, the mandate to Congress is not to prescribe, but to (CREBA)) reiterate previous claims made by them that R.A. No. 7716 did not "originate exclusively" in the
evolve, a progressive tax system. Otherwise, sales taxes, which perhaps are the oldest form of indirect House of Representatives as required by Art. VI, §24 of the Constitution. Although they admit that H. No.
taxes, would have been prohibited with the proclamation of Art. VIII, §17 (1) of the 1973 Constitution from 11197 was filed in the House of Representatives where it passed three readings and that afterward it was
which the present Art. VI, §28 (1) was taken. Sales taxes are also regressive. sent to the Senate where after first reading it was referred to the Senate Ways and Means Committee,
they complain that the Senate did not pass it on second and third readings. Instead what the Senate did
Resort to indirect taxes should be minimized but not avoided entirely because it is difficult, if not was to pass its own version (S. No. 1630) which it approved on May 24, 1994. Petitioner Tolentino adds
impossible, to avoid them by imposing such taxes according to the taxpayers' ability to pay. In the case of that what the Senate committee should have done was to amend H. No. 11197 by striking out the text of
the VAT, the law minimizes the regressive effects of this imposition by providing for zero rating of certain the bill and substituting it with the text of S. No. 1630. That way, it is said, "the bill remains a House bill
transactions (R.A. No. 7716, §3, amending §102 (b) of the NIRC), while granting exemptions to other and the Senate version just becomes the text (only the text) of the House bill."
transactions. (R.A. No. 7716, §4 amending §103 of the NIRC)99
The contention has no merit.
CONCLUSION
The enactment of S. No. 1630 is not the only instance in which the Senate proposed an amendment to a
It has been said that taxes are the lifeblood of the government. In this case, it is just an enema, a first-aid House revenue bill by enacting its own version of a revenue bill. On at least two occasions during
measure to resuscitate an economy in distress. The Court is neither blind nor is it turning a deaf ear on the Eighth Congress, the Senate passed its own version of revenue bills, which, in consolidation with
the plight of the masses. But it does not have the panacea for the malady that the law seeks to remedy. House bills earlier passed, became the enrolled bills. These were:
As in other cases, the Court cannot strike down a law as unconstitutional simply because of its yokes.
R.A. No. 7369 (AN ACT TO AMEND THE OMNIBUS INVESTMENTS CODE OF 1987 BY EXTENDING
Let us not be overly influenced by the plea that for every wrong there is a remedy, and that the judiciary FROM FIVE (5) YEARS TO TEN YEARS THE PERIOD FOR TAX AND DUTY EXEMPTION AND TAX
should stand ready to afford relief. There are undoubtedly many wrongs the judicature may not correct, for CREDIT ON CAPITAL EQUIPMENT) which was approved by the President on April 10, 1992. This Act is
instance, those involving political questions. . . . actually a consolidation of H. No. 34254, which was approved by the House on January 29, 1992, and S.
No. 1920, which was approved by the Senate on February 3, 1992.
Let us likewise disabuse our minds from the notion that the judiciary is the repository of remedies for all
political or social ills; We should not forget that the Constitution has judiciously allocated the powers of R.A. No. 7549 (AN ACT GRANTING TAX EXEMPTIONS TO WHOEVER SHALL GIVE REWARD TO
government to three distinct and separate compartments; and that judicial interpretation has tended to the ANY FILIPINO ATHLETE WINNING A MEDAL IN OLYMPIC GAMES) which was approved by the
preservation of the independence of the three, and a zealous regard of the prerogatives of each, knowing President on May 22, 1992. This Act is a consolidation of H. No. 22232, which was approved by the
full well that one is not the guardian of the others and that, for official wrong-doing, each may be brought House of Representatives on August 2, 1989, and S. No. 807, which was approved by the Senate on
to account, either by impeachment, trial or by the ballot box.100 October 21, 1991.

The words of the Court in Vera vs. Avelino101 holds true then, as it still holds true now. All things On the other hand, the Ninth Congress passed revenue laws which were also the result of the
considered, there is no raison d'être for the unconstitutionality of R.A. No. 9337. consolidation of House and Senate bills. These are the following, with indications of the dates on which
the laws were approved by the President and dates the separate bills of the two chambers of Congress
WHEREFORE, Republic Act No. 9337 not being unconstitutional, the petitions in G.R. Nos. 168056, were respectively passed:
168207, 168461, 168463, and 168730, are hereby DISMISSED.

139
1. R.A. NO. 7642 AN ACT RATIONALIZING FURTHER THE STRUCTURE AND ADMINISTRATION OF THE
DOCUMENTARY STAMP TAX, AMENDING FOR THE PURPOSE CERTAIN PROVISIONS OF THE
AN ACT INCREASING THE PENALTIES FOR TAX EVASION, AMENDING FOR THIS PURPOSE THE NATIONAL INTERNAL REVENUE CODE, AS AMENDED, ALLOCATING FUNDS FOR SPECIFIC
PERTINENT SECTIONS OF THE NATIONAL INTERNAL REVENUE CODE (December 28, 1992). PROGRAMS, AND FOR OTHER PURPOSES (December 23, 1993)

House Bill No. 2165, October 5, 1992 House Bill No. 7789, May 31, 1993

Senate Bill No. 32, December 7, 1992 Senate Bill No. 1330, November 18, 1993

2. R.A. NO. 7643 7. R.A. NO. 7717

AN ACT TO EMPOWER THE COMMISSIONER OF INTERNAL REVENUE TO REQUIRE THE AN ACT IMPOSING A TAX ON THE SALE, BARTER OR EXCHANGE OF SHARES OF STOCK LISTED
PAYMENT OF THE VALUE-ADDED TAX EVERY MONTH AND TO ALLOW LOCAL GOVERNMENT AND TRADED THROUGH THE LOCAL STOCK EXCHANGE OR THROUGH INITIAL PUBLIC
UNITS TO SHARE IN VAT REVENUE, AMENDING FOR THIS PURPOSE CERTAIN SECTIONS OF OFFERING, AMENDING FOR THE PURPOSE THE NATIONAL INTERNAL REVENUE CODE, AS
THE NATIONAL INTERNAL REVENUE CODE (December 28, 1992) AMENDED, BY INSERTING A NEW SECTION AND REPEALING CERTAIN SUBSECTIONS THEREOF
(May 5, 1994)
House Bill No. 1503, September 3, 1992
House Bill No. 9187, November 3, 1993
Senate Bill No. 968, December 7, 1992
Senate Bill No. 1127, March 23, 1994
3. R.A. NO. 7646
Thus, the enactment of S. No. 1630 is not the only instance in which the Senate, in the exercise of its
power to propose amendments to bills required to originate in the House, passed its own version of a
AN ACT AUTHORIZING THE COMMISSIONER OF INTERNAL REVENUE TO PRESCRIBE THE PLACE House revenue measure. It is noteworthy that, in the particular case of S. No. 1630, petitioners Tolentino
FOR PAYMENT OF INTERNAL REVENUE TAXES BY LARGE TAXPAYERS, AMENDING FOR THIS and Roco, as members of the Senate, voted to approve it on second and third readings.
PURPOSE CERTAIN PROVISIONS OF THE NATIONAL INTERNAL REVENUE CODE, AS AMENDED
(February 24, 1993)
On the other hand, amendment by substitution, in the manner urged by petitioner Tolentino, concerns a
mere matter of form. Petitioner has not shown what substantial difference it would make if, as the Senate
House Bill No. 1470, October 20, 1992 actually did in this case, a separate bill like S. No. 1630 is instead enacted as a substitute measure,
"taking into Consideration . . . H.B. 11197."
Senate Bill No. 35, November 19, 1992
Indeed, so far as pertinent, the Rules of the Senate only provide:
4. R.A. NO. 7649
RULE XXIX- AMENDMENTS
AN ACT REQUIRING THE GOVERNMENT OR ANY OF ITS POLITICAL SUBDIVISIONS,
INSTRUMENTALITIES OR AGENCIES INCLUDING GOVERNMENT-OWNED OR CONTROLLED §68. Not more than one amendment to the original amendment shall be considered.
CORPORATIONS (GOCCS) TO DEDUCT AND WITHHOLD THE VALUE-ADDED TAX DUE AT THE
RATE OF THREE PERCENT (3%) ON GROSS PAYMENT FOR THE PURCHASE OF GOODS AND SIX
PERCENT (6%) ON GROSS RECEIPTS FOR SERVICES RENDERED BY CONTRACTORS (April 6, No amendment by substitution shall be entertained unless the text thereof is submitted in writing.
1993)
Any of said amendments may be withdrawn before a vote is taken thereon.
House Bill No. 5260, January 26, 1993
§69. No amendment which seeks the inclusion of a legislative provision foreign to the subject matter of a
Senate Bill No. 1141, March 30, 1993 bill (rider) shall be entertained.

5. R.A. NO. 7656 §70-A. A bill or resolution shall not be amended by substituting it with another which covers a subject
distinct from that proposed in the original bill or resolution. (emphasis added).
AN ACT REQUIRING GOVERNMENT-OWNED OR CONTROLLED CORPORATIONS TO DECLARE
DIVIDENDS UNDER CERTAIN CONDITIONS TO THE NATIONAL GOVERNMENT, AND FOR OTHER Nor is there merit in petitioners' contention that, with regard to revenue bills, the Philippine Senate
PURPOSES (November 9, 1993) possesses less power than the U.S. Senate because of textual differences between constitutional
provisions giving them the power to propose or concur with amendments.
House Bill No. 11024, November 3, 1993
Art. I, §7, cl. 1 of the U.S. Constitution reads:
Senate Bill No. 1168, November 3, 1993
All Bills for raising Revenue shall originate in the House of Representatives; but the Senate may propose
or concur with amendments as on other Bills.
6. R.A. NO. 7660

140
Art. VI, §24 of our Constitution reads: (L. TAÑADA AND F. CARREON, POLITICAL LAW OF THE PHILIPPINES 247 (1961))

All appropriation, revenue or tariff bills, bills authorizing increase of the public debt, bills of local The above-mentioned bills are supposed to be initiated by the House of Representatives because it is
application, and private bills shall originate exclusively in the House of Representatives, but the Senate more numerous in membership and therefore also more representative of the people. Moreover, its
may propose or concur with amendments. members are presumed to be more familiar with the needs of the country in regard to the enactment of
the legislation involved.
The addition of the word "exclusively" in the Philippine Constitution and the decision to drop the phrase
"as on other Bills" in the American version, according to petitioners, shows the intention of the framers of The Senate is, however, allowed much leeway in the exercise of its power to propose or concur with
our Constitution to restrict the Senate's power to propose amendments to revenue bills. Petitioner amendments to the bills initiated by the House of Representatives. Thus, in one case, a bill introduced in
Tolentino contends that the word "exclusively" was inserted to modify "originate" and "the words 'as in the U.S. House of Representatives was changed by the Senate to make a proposed inheritance tax a
any other bills' (sic) were eliminated so as to show that these bills were not to be like other bills but must corporation tax. It is also accepted practice for the Senate to introduce what is known as an amendment
be treated as a special kind." by substitution, which may entirely replace the bill initiated in the House of Representatives.

The history of this provision does not support this contention. The supposed indicia of constitutional intent (I. CRUZ, PHILIPPINE POLITICAL LAW 144-145 (1993)).
are nothing but the relics of an unsuccessful attempt to limit the power of the Senate. It will be recalled
that the 1935 Constitution originally provided for a unicameral National Assembly. When it was decided in In sum, while Art. VI, §24 provides that all appropriation, revenue or tariff bills, bills authorizing increase of
1939 to change to a bicameral legislature, it became necessary to provide for the procedure for the public debt, bills of local application, and private bills must "originate exclusively in the House of
lawmaking by the Senate and the House of Representatives. The work of proposing amendments to the Representatives," it also adds, "but the Senate may propose or concur with amendments." In the exercise
Constitution was done by the National Assembly, acting as a constituent assembly, some of whose of this power, the Senate may propose an entirely new bill as a substitute measure. As petitioner
members, jealous of preserving the Assembly's lawmaking powers, sought to curtail the powers of the Tolentino states in a high school text, a committee to which a bill is referred may do any of the following:
proposed Senate. Accordingly they proposed the following provision:
(1) to endorse the bill without changes; (2) to make changes in the bill omitting or adding sections or
All bills appropriating public funds, revenue or tariff bills, bills of local application, and private bills shall altering its language; (3) to make and endorse an entirely new bill as a substitute, in which case it will be
originate exclusively in the Assembly, but the Senate may propose or concur with amendments. In case of known as a committee bill; or (4) to make no report at all.
disapproval by the Senate of any such bills, the Assembly may repass the same by a two-thirds vote of all
its members, and thereupon, the bill so repassed shall be deemed enacted and may be submitted to the
President for corresponding action. In the event that the Senate should fail to finally act on any such bills, (A. TOLENTINO, THE GOVERNMENT OF THE PHILIPPINES 258 (1950))
the Assembly may, after thirty days from the opening of the next regular session of the same legislative
term, reapprove the same with a vote of two-thirds of all the members of the Assembly. And upon such To except from this procedure the amendment of bills which are required to originate in the House by
reapproval, the bill shall be deemed enacted and may be submitted to the President for corresponding prescribing that the number of the House bill and its other parts up to the enacting clause must be
action. preserved although the text of the Senate amendment may be incorporated in place of the original body of
the bill is to insist on a mere technicality. At any rate there is no rule prescribing this form. S. No. 1630, as
The special committee on the revision of laws of the Second National Assembly vetoed the proposal. It a substitute measure, is therefore as much an amendment of H. No. 11197 as any which the Senate
deleted everything after the first sentence. As rewritten, the proposal was approved by the National could have made.
Assembly and embodied in Resolution No. 38, as amended by Resolution No. 73. (J. ARUEGO, KNOW
YOUR CONSTITUTION 65-66 (1950)). The proposed amendment was submitted to the people and II. S. No. 1630 a mere amendment of H. No. 11197. Petitioners' basic error is that they assume that S.
ratified by them in the elections held on June 18, 1940. No. 1630 is an independent and distinct bill. Hence their repeated references to its certification that it was
passed by the Senate "in substitution of S.B. No. 1129, taking into consideration P.S. Res. No. 734
This is the history of Art. VI, §18 (2) of the 1935 Constitution, from which Art. VI, §24 of the present and H.B. No. 11197," implying that there is something substantially different between the reference to S.
Constitution was derived. It explains why the word "exclusively" was added to the American text from No. 1129 and the reference to H. No. 11197. From this premise, they conclude that R.A. No. 7716
which the framers of the Philippine Constitution borrowed and why the phrase "as on other Bills" was not originated both in the House and in the Senate and that it is the product of two "half-baked bills because
copied. Considering the defeat of the proposal, the power of the Senate to propose amendments must be neither H. No. 11197 nor S. No. 1630 was passed by both houses of Congress."
understood to be full, plenary and complete "as on other Bills." Thus, because revenue bills are required
to originate exclusively in the House of Representatives, the Senate cannot enact revenue measures of its In point of fact, in several instances the provisions of S. No. 1630, clearly appear to be mere amendments
own without such bills. After a revenue bill is passed and sent over to it by the House, however, the of the corresponding provisions of H. No. 11197. The very tabular comparison of the provisions of H. No.
Senate certainly can pass its own version on the same subject matter. This follows from the coequality of 11197 and S. No. 1630 attached as Supplement A to the basic petition of petitioner Tolentino, while
the two chambers of Congress. showing differences between the two bills, at the same time indicates that the provisions of the Senate bill
were precisely intended to be amendments to the House bill.
That this is also the understanding of book authors of the scope of the Senate's power to concur is clear
from the following commentaries: Without H. No. 11197, the Senate could not have enacted S. No. 1630. Because the Senate bill was a
mere amendment of the House bill, H. No. 11197 in its original form did not have to pass the Senate on
The power of the Senate to propose or concur with amendments is apparently without restriction. It would second and three readings. It was enough that after it was passed on first reading it was referred to the
seem that by virtue of this power, the Senate can practically re-write a bill required to come from the Senate Committee on Ways and Means. Neither was it required that S. No. 1630 be passed by the House
House and leave only a trace of the original bill. For example, a general revenue bill passed by the lower of Representatives before the two bills could be referred to the Conference Committee.
house of the United States Congress contained provisions for the imposition of an inheritance tax . This
was changed by the Senate into a corporation tax. The amending authority of the Senate was declared by There is legislative precedent for what was done in the case of H. No. 11197 and S. No. 1630. When the
the United States Supreme Court to be sufficiently broad to enable it to make the alteration. [Flint v. Stone House bill and Senate bill, which became R.A. No. 1405 (Act prohibiting the disclosure of bank deposits),
Tracy Company, 220 U.S. 107, 55 L. ed. 389]. were referred to a conference committee, the question was raised whether the two bills could be the

141
subject of such conference, considering that the bill from one house had not been passed by the other This provision of the 1973 document, with slight modification, was adopted in Art. VI, §26 (2) of the
and vice versa. As Congressman Duran put the question: present Constitution, thus:

MR. DURAN. Therefore, I raise this question of order as to procedure: If a House bill is passed by the (2) No bill passed by either House shall become a law unless it has passed three readings on separate
House but not passed by the Senate, and a Senate bill of a similar nature is passed in the Senate but days, and printed copies thereof in its final form have been distributed to its Members three days before
never passed in the House, can the two bills be the subject of a conference, and can a law be enacted its passage, except when the President certifies to the necessity of its immediate enactment to meet a
from these two bills? I understand that the Senate bill in this particular instance does not refer to public calamity or emergency. Upon the last reading of a bill, no amendment thereto shall be allowed, and
investments in government securities, whereas the bill in the House, which was introduced by the the vote thereon shall be taken immediately thereafter, and the yeas and nays entered in the Journal.
Speaker, covers two subject matters: not only investigation of deposits in banks but also investigation of
investments in government securities. Now, since the two bills differ in their subject matter, I believe that The exception is based on the prudential consideration that if in all cases three readings on separate days
no law can be enacted. are required and a bill has to be printed in final form before it can be passed, the need for a law may be
rendered academic by the occurrence of the very emergency or public calamity which it is meant to
Ruling on the point of order raised, the chair (Speaker Jose B. Laurel, Jr.) said: address.

THE SPEAKER. The report of the conference committee is in order. It is precisely in cases like this where Petitioners further contend that a "growing budget deficit" is not an emergency, especially in a country like
a conference should be had. If the House bill had been approved by the Senate, there would have been the Philippines where budget deficit is a chronic condition. Even if this were the case, an enormous
no need of a conference; but precisely because the Senate passed another bill on the same subject budget deficit does not make the need for R.A. No. 7716 any less urgent or the situation calling for its
matter, the conference committee had to be created, and we are now considering the report of that enactment any less an emergency.
committee.
Apparently, the members of the Senate (including some of the petitioners in these cases) believed that
(2 CONG. REC. NO. 13, July 27, 1955, pp. 3841-42 (emphasis added)) there was an urgent need for consideration of S. No. 1630, because they responded to the call of the
President by voting on the bill on second and third readings on the same day. While the judicial
III. The President's certification. The fallacy in thinking that H. No. 11197 and S. No. 1630 are distinct and department is not bound by the Senate's acceptance of the President's certification, the respect due
unrelated measures also accounts for the petitioners' (Kilosbayan's and PAL's) contention that because coequal departments of the government in matters committed to them by the Constitution and the
the President separately certified to the need for the immediate enactment of these measures, his absence of a clear showing of grave abuse of discretion caution a stay of the judicial hand.
certification was ineffectual and void. The certification had to be made of the version of the same revenue
bill which at the moment was being considered. Otherwise, to follow petitioners' theory, it would be At any rate, we are satisfied that S. No. 1630 received thorough consideration in the Senate where it was
necessary for the President to certify as many bills as are presented in a house of Congress even though discussed for six days. Only its distribution in advance in its final printed form was actually dispensed with
the bills are merely versions of the bill he has already certified. It is enough that he certifies the bill which, by holding the voting on second and third readings on the same day (March 24, 1994). Otherwise,
at the time he makes the certification, is under consideration. Since on March 22, 1994 the Senate was sufficient time between the submission of the bill on February 8, 1994 on second reading and its approval
considering S. No. 1630, it was that bill which had to be certified. For that matter on June 1, 1993 the on March 24, 1994 elapsed before it was finally voted on by the Senate on third reading.
President had earlier certified H. No. 9210 for immediate enactment because it was the one which at that
time was being considered by the House. This bill was later substituted, together with other bills, by H. No. The purpose for which three readings on separate days is required is said to be two-fold: (1) to inform the
11197. members of Congress of what they must vote on and (2) to give them notice that a measure is
progressing through the enacting process, thus enabling them and others interested in the measure to
As to what Presidential certification can accomplish, we have already explained in the main decision that prepare their positions with reference to it. (1 J. G. SUTHERLAND, STATUTES AND STATUTORY
the phrase "except when the President certifies to the necessity of its immediate enactment, etc." in Art. CONSTRUCTION §10.04, p. 282 (1972)). These purposes were substantially achieved in the case of R.A.
VI, §26 (2) qualifies not only the requirement that "printed copies [of a bill] in its final form [must be] No. 7716.
distributed to the members three days before its passage" but also the requirement that before a bill can
become a law it must have passed "three readings on separate days." There is not only textual support for IV. Power of Conference Committee. It is contended (principally by Kilosbayan, Inc. and the Movement of
such construction but historical basis as well. Attorneys for Brotherhood, Integrity and Nationalism, Inc. (MABINI)) that in violation of the constitutional
policy of full public disclosure and the people's right to know (Art. II, §28 and Art. III, §7) the Conference
Art. VI, §21 (2) of the 1935 Constitution originally provided: Committee met for two days in executive session with only the conferees present.

(2) No bill shall be passed by either House unless it shall have been printed and copies thereof in its final As pointed out in our main decision, even in the United States it was customary to hold such sessions with
form furnished its Members at least three calendar days prior to its passage, except when the President only the conferees and their staffs in attendance and it was only in 1975 when a new rule was adopted
shall have certified to the necessity of its immediate enactment. Upon the last reading of a bill, no requiring open sessions. Unlike its American counterpart, the Philippine Congress has not adopted a rule
amendment thereof shall be allowed and the question upon its passage shall be taken immediately prescribing open hearings for conference committees.
thereafter, and the yeas and nays entered on the Journal.
It is nevertheless claimed that in the United States, before the adoption of the rule in 1975, at least staff
When the 1973 Constitution was adopted, it was provided in Art. VIII, §19 (2): members were present. These were staff members of the Senators and Congressmen, however, who
may be presumed to be their confidential men, not stenographers as in this case who on the last two days
(2) No bill shall become a law unless it has passed three readings on separate days, and printed copies of the conference were excluded. There is no showing that the conferees themselves did not take notes of
thereof in its final form have been distributed to the Members three days before its passage, except when their proceedings so as to give petitioner Kilosbayan basis for claiming that even in secret diplomatic
the Prime Minister certifies to the necessity of its immediate enactment to meet a public calamity or negotiations involving state interests, conferees keep notes of their meetings. Above all, the public's right
emergency. Upon the last reading of a bill, no amendment thereto shall be allowed, and the vote thereon to know was fully served because the Conference Committee in this case submitted a report showing the
shall be taken immediately thereafter, and the yeas and nays entered in the Journal. changes made on the differing versions of the House and the Senate.

142
Petitioners cite the rules of both houses which provide that conference committee reports must contain "a (Id. at 710. (emphasis added))
detailed, sufficiently explicit statement of the changes in or other amendments." These changes are
shown in the bill attached to the Conference Committee Report. The members of both houses could thus It is interesting to note the following description of conference committees in the Philippines in a 1979
ascertain what changes had been made in the original bills without the need of a statement detailing the study:
changes.
Conference committees may be of two types: free or instructed. These committees may be given
The same question now presented was raised when the bill which became R.A. No. 1400 (Land Reform instructions by their parent bodies or they may be left without instructions. Normally the conference
Act of 1955) was reported by the Conference Committee. Congressman Bengzon raised a point of order. committees are without instructions, and this is why they are often critically referred to as "the little
He said: legislatures." Once bills have been sent to them, the conferees have almost unlimited authority to change
the clauses of the bills and in fact sometimes introduce new measures that were not in the original
MR. BENGZON. My point of order is that it is out of order to consider the report of the conference legislation. No minutes are kept, and members' activities on conference committees are difficult to
committee regarding House Bill No. 2557 by reason of the provision of Section 11, Article XII, of the Rules determine. One congressman known for his idealism put it this way: "I killed a bill on export incentives for
of this House which provides specifically that the conference report must be accompanied by a detailed my interest group [copra] in the conference committee but I could not have done so anywhere else." The
statement of the effects of the amendment on the bill of the House. This conference committee report is conference committee submits a report to both houses, and usually it is accepted. If the report is not
not accompanied by that detailed statement, Mr. Speaker. Therefore it is out of order to consider it. accepted, then the committee is discharged and new members are appointed.

Petitioner Tolentino, then the Majority Floor Leader, answered: (R. Jackson, Committees in the Philippine Congress, in COMMITTEES AND LEGISLATURES: A
COMPARATIVE ANALYSIS 163 (J. D. LEES AND M. SHAW, eds.)).
MR. TOLENTINO. Mr. Speaker, I should just like to say a few words in connection with the point of order
raised by the gentleman from Pangasinan. In citing this study, we pass no judgment on the methods of conference committees. We cite it only to say
that conference committees here are no different from their counterparts in the United States whose vast
There is no question about the provision of the Rule cited by the gentleman from Pangasinan, but this powers we noted in Philippine Judges Association v. Prado, supra. At all events, under Art. VI, §16(3)
provision applies to those cases where only portions of the bill have been amended. In this case before each house has the power "to determine the rules of its proceedings," including those of its committees.
us an entire bill is presented; therefore, it can be easily seen from the reading of the bill what the Any meaningful change in the method and procedures of Congress or its committees must therefore be
provisions are. Besides, this procedure has been an established practice. sought in that body itself.

After some interruption, he continued: V. The titles of S. No. 1630 and H. No. 11197. PAL maintains that R.A. No. 7716 violates Art. VI, §26 (1)
of the Constitution which provides that "Every bill passed by Congress shall embrace only one subject
which shall be expressed in the title thereof." PAL contends that the amendment of its franchise by the
MR. TOLENTINO. As I was saying, Mr. Speaker, we have to look into the reason for the provisions of the withdrawal of its exemption from the VAT is not expressed in the title of the law.
Rules, and the reason for the requirement in the provision cited by the gentleman from Pangasinan is
when there are only certain words or phrases inserted in or deleted from the provisions of the bill included
in the conference report, and we cannot understand what those words and phrases mean and their Pursuant to §13 of P.D. No. 1590, PAL pays a franchise tax of 2% on its gross revenue "in lieu of all other
relation to the bill. In that case, it is necessary to make a detailed statement on how those words and taxes, duties, royalties, registration, license and other fees and charges of any kind, nature, or description,
phrases will affect the bill as a whole; but when the entire bill itself is copied verbatim in the conference imposed, levied, established, assessed or collected by any municipal, city, provincial or national authority
report, that is not necessary. So when the reason for the Rule does not exist, the Rule does not exist. or government agency, now or in the future."

(2 CONG. REC. NO. 2, p. 4056. (emphasis added)) PAL was exempted from the payment of the VAT along with other entities by §103 of the National Internal
Revenue Code, which provides as follows:
Congressman Tolentino was sustained by the chair. The record shows that when the ruling was appealed,
it was upheld by viva voce and when a division of the House was called, it was sustained by a vote of 48 §103. Exempt transactions. — The following shall be exempt from the value-added tax:
to 5. (Id.,
p. 4058) xxx xxx xxx
(q) Transactions which are exempt under special laws or international agreements to which the
Nor is there any doubt about the power of a conference committee to insert new provisions as long as Philippines is a signatory.
these are germane to the subject of the conference. As this Court held in Philippine Judges Association R.A. No. 7716 seeks to withdraw certain exemptions, including that granted to PAL, by amending §103,
v. Prado, 227 SCRA 703 (1993), in an opinion written by then Justice Cruz, the jurisdiction of the as follows:
conference committee is not limited to resolving differences between the Senate and the House. It may §103. Exempt transactions. — The following shall be exempt from the value-added tax:
propose an entirely new provision. What is important is that its report is subsequently approved by the xxx xxx xxx
respective houses of Congress. This Court ruled that it would not entertain allegations that, because new (q) Transactions which are exempt under special laws, except those granted under Presidential Decree
provisions had been added by the conference committee, there was thereby a violation of the Nos. 66, 529, 972, 1491, 1590. . . .
constitutional injunction that "upon the last reading of a bill, no amendment thereto shall be allowed." The amendment of §103 is expressed in the title of R.A. No. 7716 which reads:
AN ACT RESTRUCTURING THE VALUE-ADDED TAX (VAT) SYSTEM, WIDENING ITS TAX BASE AND
ENHANCING ITS ADMINISTRATION, AND FOR THESE PURPOSES AMENDING AND REPEALING
Applying these principles, we shall decline to look into the petitioners' charges that an amendment was THE RELEVANT PROVISIONS OF THE NATIONAL INTERNAL REVENUE CODE, AS AMENDED, AND
made upon the last reading of the bill that eventually became R.A. No. 7354 and that copies thereof in its FOR OTHER PURPOSES.
final form were not distributed among the members of each House. Both the enrolled bill and the
legislative journals certify that the measure was duly enacted i.e., in accordance with Article VI, Sec. 26
(2) of the Constitution. We are bound by such official assurances from a coordinate department of the By stating that R.A. No. 7716 seeks to "[RESTRUCTURE] THE VALUE-ADDED TAX (VAT) SYSTEM
government, to which we owe, at the very least, a becoming courtesy. [BY] WIDENING ITS TAX BASE AND ENHANCING ITS ADMINISTRATION, AND FOR THESE

143
PURPOSES AMENDING AND REPEALING THE RELEVANT PROVISIONS OF THE NATIONAL On the other hand, in Minneapolis Star & Tribune Co. v. Minnesota Comm'r of Revenue, 460 U.S. 575, 75
INTERNAL REVENUE CODE, AS AMENDED AND FOR OTHER PURPOSES," Congress thereby clearly L. Ed. 2d 295 (1983), the tax was found to be discriminatory because although it could have been made
expresses its intention to amend any provision of the NIRC which stands in the way of accomplishing the liable for the sales tax or, in lieu thereof, for the use tax on the privilege of using, storing or consuming
purpose of the law. tangible goods, the press was not. Instead, the press was exempted from both taxes. It was, however,
later made to pay a special use tax on the cost of paper and ink which made these items "the only items
PAL asserts that the amendment of its franchise must be reflected in the title of the law by specific subject to the use tax that were component of goods to be sold at retail." The U.S. Supreme Court held
reference to P.D. No. 1590. It is unnecessary to do this in order to comply with the constitutional that the differential treatment of the press "suggests that the goal of regulation is not related to
requirement, since it is already stated in the title that the law seeks to amend the pertinent provisions of suppression of expression, and such goal is presumptively unconstitutional." It would therefore appear
the NIRC, among which is §103(q), in order to widen the base of the VAT. Actually, it is the bill which that even a law that favors the press is constitutionally suspect. (See the dissent of Rehnquist, J. in that
becomes a law that is required to express in its title the subject of legislation. The titles of H. No. 11197 case)
and S. No. 1630 in fact specifically referred to §103 of the NIRC as among the provisions sought to be
amended. We are satisfied that sufficient notice had been given of the pendency of these bills in Nor is it true that only two exemptions previously granted by E.O. No. 273 are withdrawn "absolutely and
Congress before they were enacted into what is now R.A. unqualifiedly" by R.A. No. 7716. Other exemptions from the VAT, such as those previously granted to
No. 7716. PAL, petroleum concessionaires, enterprises registered with the Export Processing Zone Authority, and
many more are likewise totally withdrawn, in addition to exemptions which are partially withdrawn, in an
In Philippine Judges Association v. Prado, supra, a similar argument as that now made by PAL was effort to broaden the base of the tax.
rejected. R.A. No. 7354 is entitled AN ACT CREATING THE PHILIPPINE POSTAL CORPORATION,
DEFINING ITS POWERS, FUNCTIONS AND RESPONSIBILITIES, PROVIDING FOR REGULATION OF The PPI says that the discriminatory treatment of the press is highlighted by the fact that transactions,
THE INDUSTRY AND FOR OTHER PURPOSES CONNECTED THEREWITH. It contained a provision which are profit oriented, continue to enjoy exemption under R.A. No. 7716. An enumeration of some of
repealing all franking privileges. It was contended that the withdrawal of franking privileges was not these transactions will suffice to show that by and large this is not so and that the exemptions are granted
expressed in the title of the law. In holding that there was sufficient description of the subject of the law in for a purpose. As the Solicitor General says, such exemptions are granted, in some cases, to encourage
its title, including the repeal of franking privileges, this Court held: agricultural production and, in other cases, for the personal benefit of the end-user rather than for profit.
The exempt transactions are:
To require every end and means necessary for the accomplishment of the general objectives of the
statute to be expressed in its title would not only be unreasonable but would actually render legislation (a) Goods for consumption or use which are in their original state (agricultural, marine and forest products,
impossible. [Cooley, Constitutional Limitations, 8th Ed., p. 297] As has been correctly explained: cotton seeds in their original state, fertilizers, seeds, seedlings, fingerlings, fish, prawn livestock and
poultry feeds) and goods or services to enhance agriculture (milling of palay, corn, sugar cane and raw
The details of a legislative act need not be specifically stated in its title, but matter germane to the subject sugar, livestock, poultry feeds, fertilizer, ingredients used for the manufacture of feeds).
as expressed in the title, and adopted to the accomplishment of the object in view, may properly be
included in the act. Thus, it is proper to create in the same act the machinery by which the act is to be (b) Goods used for personal consumption or use (household and personal effects of citizens returning to
enforced, to prescribe the penalties for its infraction, and to remove obstacles in the way of its execution. the Philippines) or for professional use, like professional instruments and implements, by persons coming
If such matters are properly connected with the subject as expressed in the title, it is unnecessary that to the Philippines to settle here.
they should also have special mention in the title. (Southern Pac. Co. v. Bartine, 170 Fed. 725)
(c) Goods subject to excise tax such as petroleum products or to be used for manufacture of petroleum
(227 SCRA at 707-708) products subject to excise tax and services subject to percentage tax.

VI. Claims of press freedom and religious liberty. We have held that, as a general proposition, the press is (d) Educational services, medical, dental, hospital and veterinary services, and services rendered under
not exempt from the taxing power of the State and that what the constitutional guarantee of free press employer-employee relationship.
prohibits are laws which single out the press or target a group belonging to the press for special treatment
or which in any way discriminate against the press on the basis of the content of the publication, and R.A. (e) Works of art and similar creations sold by the artist himself.
No. 7716 is none of these.
(f) Transactions exempted under special laws, or international agreements.
Now it is contended by the PPI that by removing the exemption of the press from the VAT while
maintaining those granted to others, the law discriminates against the press. At any rate, it is averred,
"even nondiscriminatory taxation of constitutionally guaranteed freedom is unconstitutional." (g) Export-sales by persons not VAT-registered.

With respect to the first contention, it would suffice to say that since the law granted the press a privilege, (h) Goods or services with gross annual sale or receipt not exceeding P500,000.00.
the law could take back the privilege anytime without offense to the Constitution. The reason is simple: by
granting exemptions, the State does not forever waive the exercise of its sovereign prerogative. (Respondents' Consolidated Comment on the Motions for Reconsideration, pp. 58-60)

Indeed, in withdrawing the exemption, the law merely subjects the press to the same tax burden to which The PPI asserts that it does not really matter that the law does not discriminate against the press because
other businesses have long ago been subject. It is thus different from the tax involved in the cases "even nondiscriminatory taxation on constitutionally guaranteed freedom is unconstitutional." PPI cites in
invoked by the PPI. The license tax in Grosjean v. American Press Co., 297 U.S. 233, 80 L. Ed. 660 support of this assertion the following statement in Murdock v. Pennsylvania, 319 U.S. 105, 87 L. Ed.
(1936) was found to be discriminatory because it was laid on the gross advertising receipts only of 1292 (1943):
newspapers whose weekly circulation was over 20,000, with the result that the tax applied only to 13 out
of 124 publishers in Louisiana. These large papers were critical of Senator Huey Long who controlled the The fact that the ordinance is "nondiscriminatory" is immaterial. The protection afforded by the First
state legislature which enacted the license tax. The censorial motivation for the law was thus evident. Amendment is not so restricted. A license tax certainly does not acquire constitutional validity because it
classifies the privileges protected by the First Amendment along with the wares and merchandise of

144
hucksters and peddlers and treats them all alike. Such equality in treatment does not save the ordinance. government and no obligation of contract can extend to the defeat of that authority. (Norman v. Baltimore
Freedom of press, freedom of speech, freedom of religion are in preferred position. and Ohio R.R., 79 L. Ed. 885 (1935)).

The Court was speaking in that case of a license tax, which, unlike an ordinary tax, is mainly for It is next pointed out that while §4 of R.A. No. 7716 exempts such transactions as the sale of agricultural
regulation. Its imposition on the press is unconstitutional because it lays a prior restraint on the exercise of products, food items, petroleum, and medical and veterinary services, it grants no exemption on the sale
its right. Hence, although its application to others, such those selling goods, is valid, its application to the of real property which is equally essential. The sale of real property for socialized and low-cost housing is
press or to religious groups, such as the Jehovah's Witnesses, in connection with the latter's sale of exempted from the tax, but CREBA claims that real estate transactions of "the less poor," i.e., the middle
religious books and pamphlets, is unconstitutional. As the U.S. Supreme Court put it, "it is one thing to class, who are equally homeless, should likewise be exempted.
impose a tax on income or property of a preacher. It is quite another thing to exact a tax on him for
delivering a sermon." The sale of food items, petroleum, medical and veterinary services, etc., which are essential goods and
services was already exempt under §103, pars. (b) (d) (1) of the NIRC before the enactment of R.A. No.
A similar ruling was made by this Court in American Bible Society v. City of Manila, 101 Phil. 386 (1957) 7716. Petitioner is in error in claiming that R.A. No. 7716 granted exemption to these transactions, while
which invalidated a city ordinance requiring a business license fee on those engaged in the sale of subjecting those of petitioner to the payment of the VAT. Moreover, there is a difference between the
general merchandise. It was held that the tax could not be imposed on the sale of bibles by the American "homeless poor" and the "homeless less poor" in the example given by petitioner, because the second
Bible Society without restraining the free exercise of its right to propagate. group or middle class can afford to rent houses in the meantime that they cannot yet buy their own
homes. The two social classes are thus differently situated in life. "It is inherent in the power to tax that the
The VAT is, however, different. It is not a license tax. It is not a tax on the exercise of a privilege, much State be free to select the subjects of taxation, and it has been repeatedly held that 'inequalities which
less a constitutional right. It is imposed on the sale, barter, lease or exchange of goods or properties or result from a singling out of one particular class for taxation, or exemption infringe no constitutional
the sale or exchange of services and the lease of properties purely for revenue purposes. To subject the limitation.'" (Lutz v. Araneta, 98 Phil. 148, 153 (1955). Accord, City of Baguio v. De Leon, 134 Phil. 912
press to its payment is not to burden the exercise of its right any more than to make the press pay income (1968); Sison, Jr. v. Ancheta, 130 SCRA 654, 663 (1984); Kapatiran ng mga Naglilingkod sa Pamahalaan
tax or subject it to general regulation is not to violate its freedom under the Constitution. ng Pilipinas, Inc. v. Tan, 163 SCRA 371 (1988)).

Additionally, the Philippine Bible Society, Inc. claims that although it sells bibles, the proceeds derived Finally, it is contended, for the reasons already noted, that R.A. No. 7716 also violates Art. VI, §28(1)
from the sales are used to subsidize the cost of printing copies which are given free to those who cannot which provides that "The rule of taxation shall be uniform and equitable. The Congress shall evolve a
afford to pay so that to tax the sales would be to increase the price, while reducing the volume of sale. progressive system of taxation."
Granting that to be the case, the resulting burden on the exercise of religious freedom is so incidental as
to make it difficult to differentiate it from any other economic imposition that might make the right to Equality and uniformity of taxation means that all taxable articles or kinds of property of the same class be
disseminate religious doctrines costly. Otherwise, to follow the petitioner's argument, to increase the tax taxed at the same rate. The taxing power has the authority to make reasonable and natural classifications
on the sale of vestments would be to lay an impermissible burden on the right of the preacher to make a for purposes of taxation. To satisfy this requirement it is enough that the statute or ordinance applies
sermon. equally to all persons, forms and corporations placed in similar situation. (City of Baguio v. De
Leon, supra; Sison, Jr. v. Ancheta, supra)
On the other hand the registration fee of P1,000.00 imposed by §107 of the NIRC, as amended by §7 of
R.A. No. 7716, although fixed in amount, is really just to pay for the expenses of registration and Indeed, the VAT was already provided in E.O. No. 273 long before R.A. No. 7716 was enacted. R.A. No.
enforcement of provisions such as those relating to accounting in §108 of the NIRC. That the PBS 7716 merely expands the base of the tax. The validity of the original VAT Law was questioned
distributes free bibles and therefore is not liable to pay the VAT does not excuse it from the payment of in Kapatiran ng Naglilingkod sa Pamahalaan ng Pilipinas, Inc. v. Tan, 163 SCRA 383 (1988) on grounds
this fee because it also sells some copies. At any rate whether the PBS is liable for the VAT must be similar to those made in these cases, namely, that the law was "oppressive, discriminatory, unjust and
decided in concrete cases, in the event it is assessed this tax by the Commissioner of Internal Revenue. regressive in violation of Art. VI, §28(1) of the Constitution." (At 382) Rejecting the challenge to the law,
this Court held:
VII. Alleged violations of the due process, equal protection and contract clauses and the rule on taxation.
CREBA asserts that R.A. No. 7716 (1) impairs the obligations of contracts, (2) classifies transactions as As the Court sees it, EO 273 satisfies all the requirements of a valid tax. It is uniform. . . .
covered or exempt without reasonable basis and (3) violates the rule that taxes should be uniform and
equitable and that Congress shall "evolve a progressive system of taxation." The sales tax adopted in EO 273 is applied similarly on all goods and services sold to the public, which
are not exempt, at the constant rate of 0% or 10%.
With respect to the first contention, it is claimed that the application of the tax to existing contracts of the
sale of real property by installment or on deferred payment basis would result in substantial increases in The disputed sales tax is also equitable. It is imposed only on sales of goods or services by persons
the monthly amortizations to be paid because of the 10% VAT. The additional amount, it is pointed out, is engaged in business with an aggregate gross annual sales exceeding P200,000.00. Small corner sari-sari
something that the buyer did not anticipate at the time he entered into the contract. stores are consequently exempt from its application. Likewise exempt from the tax are sales of farm and
marine products, so that the costs of basic food and other necessities, spared as they are from the
The short answer to this is the one given by this Court in an early case: "Authorities from numerous incidence of the VAT, are expected to be relatively lower and within the reach of the general public.
sources are cited by the plaintiffs, but none of them show that a lawful tax on a new subject, or an
increased tax on an old one, interferes with a contract or impairs its obligation, within the meaning of the (At 382-383)
Constitution. Even though such taxation may affect particular contracts, as it may increase the debt of one
person and lessen the security of another, or may impose additional burdens upon one class and release
the burdens of another, still the tax must be paid unless prohibited by the Constitution, nor can it be said The CREBA claims that the VAT is regressive. A similar claim is made by the Cooperative Union of the
that it impairs the obligation of any existing contract in its true legal sense." (La Insular v. Machuca Go- Philippines, Inc. (CUP), while petitioner Juan T. David argues that the law contravenes the mandate of
Tauco and Nubla Co-Siong, 39 Phil. 567, 574 (1919)). Indeed not only existing laws but also "the Congress to provide for a progressive system of taxation because the law imposes a flat rate of 10% and
reservation of the essential attributes of sovereignty, is . . . read into contracts as a postulate of the legal thus places the tax burden on all taxpayers without regard to their ability to pay.
order." (Philippine-American Life Ins. Co. v. Auditor General, 22 SCRA 135, 147 (1968)) Contracts must
be understood as having been made in reference to the possible exercise of the rightful authority of the

145
The Constitution does not really prohibit the imposition of indirect taxes which, like the VAT, are petitioner's members have not even been assessed the VAT. Petitioner's case is not made concrete by a
regressive. What it simply provides is that Congress shall "evolve a progressive system of taxation." The series of hypothetical questions asked which are no different from those dealt with in advisory opinions.
constitutional provision has been interpreted to mean simply that "direct taxes are . . . to be preferred
[and] as much as possible, indirect taxes should be minimized." (E. FERNANDO, THE CONSTITUTION The difficulty confronting petitioner is thus apparent. He alleges arbitrariness. A mere allegation, as here,
OF THE PHILIPPINES 221 (Second ed. (1977)). Indeed, the mandate to Congress is not to prescribe, but does not suffice. There must be a factual foundation of such unconstitutional taint. Considering that
to evolve, a progressive tax system. Otherwise, sales taxes, which perhaps are the oldest form of indirect petitioner here would condemn such a provision as void on its face, he has not made out a case. This is
taxes, would have been prohibited with the proclamation of Art. VIII, §17(1) of the 1973 Constitution from merely to adhere to the authoritative doctrine that where the due process and equal protection clauses are
which the present Art. VI, §28(1) was taken. Sales taxes are also regressive. invoked, considering that they are not fixed rules but rather broad standards, there is a need for proof of
such persuasive character as would lead to such a conclusion. Absent such a showing, the presumption
Resort to indirect taxes should be minimized but not avoided entirely because it is difficult, if not of validity must prevail.
impossible, to avoid them by imposing such taxes according to the taxpayers' ability to pay. In the case of
the VAT, the law minimizes the regressive effects of this imposition by providing for zero rating of certain (Sison, Jr. v. Ancheta, 130 SCRA at 661)
transactions (R.A. No. 7716, §3, amending §102 (b) of the NIRC), while granting exemptions to other
transactions. (R.A. No. 7716, §4, amending §103 of the NIRC).
Adjudication of these broad claims must await the development of a concrete case. It may be that
postponement of adjudication would result in a multiplicity of suits. This need not be the case, however.
Thus, the following transactions involving basic and essential goods and services are exempted from the Enforcement of the law may give rise to such a case. A test case, provided it is an actual case and not an
VAT: abstract or hypothetical one, may thus be presented.

(a) Goods for consumption or use which are in their original state (agricultural, marine and forest products, Nor is hardship to taxpayers alone an adequate justification for adjudicating abstract issues. Otherwise,
cotton seeds in their original state, fertilizers, seeds, seedlings, fingerlings, fish, prawn livestock and adjudication would be no different from the giving of advisory opinion that does not really settle legal
poultry feeds) and goods or services to enhance agriculture (milling of palay, corn sugar cane and raw issues.
sugar, livestock, poultry feeds, fertilizer, ingredients used for the manufacture of feeds).
We are told that it is our duty under Art. VIII, §1, ¶2 to decide whenever a claim is made that "there has
(b) Goods used for personal consumption or use (household and personal effects of citizens returning to been a grave abuse of discretion amounting to lack or excess of jurisdiction on the part of any branch or
the Philippines) and or professional use, like professional instruments and implements, by persons instrumentality of the government." This duty can only arise if an actual case or controversy is before us.
coming to the Philippines to settle here. Under Art . VIII, §5 our jurisdiction is defined in terms of "cases" and all that Art. VIII, §1, ¶2 can plausibly
mean is that in the exercise of that jurisdiction we have the judicial power to determine questions of grave
(c) Goods subject to excise tax such as petroleum products or to be used for manufacture of petroleum abuse of discretion by any branch or instrumentality of the government.
products subject to excise tax and services subject to percentage tax.
Put in another way, what is granted in Art. VIII, §1, ¶2 is "judicial power," which is "the power of a court to
(d) Educational services, medical, dental, hospital and veterinary services, and services rendered under hear and decide cases pending between parties who have the right to sue and be sued in the courts of
employer-employee relationship. law and equity" (Lamb v. Phipps, 22 Phil. 456, 559 (1912)), as distinguished from legislative and executive
power. This power cannot be directly appropriated until it is apportioned among several courts either by
(e) Works of art and similar creations sold by the artist himself. the Constitution, as in the case of Art. VIII, §5, or by statute, as in the case of the Judiciary Act of 1948
(R.A. No. 296) and the Judiciary Reorganization Act of 1980 (B.P. Blg. 129). The power thus apportioned
constitutes the court's "jurisdiction," defined as "the power conferred by law upon a court or judge to take
(f) Transactions exempted under special laws, or international agreements. cognizance of a case, to the exclusion of all others." (United States v. Arceo, 6 Phil. 29 (1906)) Without an
actual case coming within its jurisdiction, this Court cannot inquire into any allegation of grave abuse of
(g) Export-sales by persons not VAT-registered. discretion by the other departments of the government.

(h) Goods or services with gross annual sale or receipt not exceeding P500,000.00. VIII. Alleged violation of policy towards cooperatives. On the other hand, the Cooperative Union of the
Philippines (CUP), after briefly surveying the course of legislation, argues that it was to adopt a definite
(Respondents' Consolidated Comment on the Motions for Reconsideration, pp. 58-60) policy of granting tax exemption to cooperatives that the present Constitution embodies provisions on
cooperatives. To subject cooperatives to the VAT would therefore be to infringe a constitutional policy.
Petitioner claims that in 1973, P.D. No. 175 was promulgated exempting cooperatives from the payment
On the other hand, the transactions which are subject to the VAT are those which involve goods and
of income taxes and sales taxes but in 1984, because of the crisis which menaced the national economy,
services which are used or availed of mainly by higher income groups. These include real properties held
this exemption was withdrawn by P.D. No. 1955; that in 1986, P.D. No. 2008 again granted cooperatives
primarily for sale to customers or for lease in the ordinary course of trade or business, the right or privilege
exemption from income and sales taxes until December 31, 1991, but, in the same year, E.O. No. 93
to use patent, copyright, and other similar property or right, the right or privilege to use industrial,
revoked the exemption; and that finally in 1987 the framers of the Constitution "repudiated the previous
commercial or scientific equipment, motion picture films, tapes and discs, radio, television, satellite
actions of the government adverse to the interests of the cooperatives, that is, the repeated revocation of
transmission and cable television time, hotels, restaurants and similar places, securities, lending
the tax exemption to cooperatives and instead upheld the policy of strengthening the cooperatives by way
investments, taxicabs, utility cars for rent, tourist buses, and other common carriers, services of franchise
of the grant of tax exemptions," by providing the following in Art. XII:
grantees of telephone and telegraph.

§1. The goals of the national economy are a more equitable distribution of opportunities, income, and
The problem with CREBA's petition is that it presents broad claims of constitutional violations by tendering
wealth; a sustained increase in the amount of goods and services produced by the nation for the benefit of
issues not at retail but at wholesale and in the abstract. There is no fully developed record which can
the people; and an expanding productivity as the key to raising the quality of life for all, especially the
impart to adjudication the impact of actuality. There is no factual foundation to show in the concrete the
underprivileged.
application of the law to actual contracts and exemplify its effect on property rights. For the fact is that

146
The State shall promote industrialization and full employment based on sound agricultural development These two consolidated special civil actions for prohibition challenge, in G.R. No. 109289, the
and agrarian reform, through industries that make full and efficient use of human and natural resources, constitutionality of Republic Act No. 7496, also commonly known as the Simplified Net Income Taxation
and which are competitive in both domestic and foreign markets. However, the State shall protect Filipino Scheme ("SNIT"), amending certain provisions of the National Internal Revenue Code and, in
enterprises against unfair foreign competition and trade practices. G.R. No. 109446, the validity of Section 6, Revenue Regulations No. 2-93, promulgated by public
respondents pursuant to said law.
In the pursuit of these goals, all sectors of the economy and all regions of the country shall be given
optimum opportunity to develop. Private enterprises, including corporations, cooperatives, and similar Petitioners claim to be taxpayers adversely affected by the continued implementation of the amendatory
collective organizations, shall be encouraged to broaden the base of their ownership. legislation.

§15. The Congress shall create an agency to promote the viability and growth of cooperatives as In G.R. No. 109289, it is asserted that the enactment of Republic Act
instruments for social justice and economic development. No. 7496 violates the following provisions of the Constitution:

Petitioner's contention has no merit. In the first place, it is not true that P.D. No. 1955 singled out Article VI, Section 26(1) — Every bill passed by the Congress shall embrace only one subject which shall
cooperatives by withdrawing their exemption from income and sales taxes under P.D. No. 175, §5. What be expressed in the title thereof.
P.D. No. 1955, §1 did was to withdraw the exemptions and preferential treatments theretofore granted to
private business enterprises in general, in view of the economic crisis which then beset the nation. It is Article VI, Section 28(1) — The rule of taxation shall be uniform and equitable. The Congress shall evolve
true that after P.D. No. 2008, §2 had restored the tax exemptions of cooperatives in 1986, the exemption a progressive system of taxation.
was again repealed by E.O. No. 93, §1, but then again cooperatives were not the only ones whose
exemptions were withdrawn. The withdrawal of tax incentives applied to all, including government and
private entities. In the second place, the Constitution does not really require that cooperatives be granted Article III, Section 1 — No person shall be deprived of . . . property without due process of law, nor shall
tax exemptions in order to promote their growth and viability. Hence, there is no basis for petitioner's any person be denied the equal protection of the laws.
assertion that the government's policy toward cooperatives had been one of vacillation, as far as the grant
of tax privileges was concerned, and that it was to put an end to this indecision that the constitutional In G.R. No. 109446, petitioners, assailing Section 6 of Revenue Regulations No. 2-93, argue that public
provisions cited were adopted. Perhaps as a matter of policy cooperatives should be granted tax respondents have exceeded their rule-making authority in applying SNIT to general professional
exemptions, but that is left to the discretion of Congress. If Congress does not grant exemption and there partnerships.
is no discrimination to cooperatives, no violation of any constitutional policy can be charged.
The Solicitor General espouses the position taken by public respondents.
Indeed, petitioner's theory amounts to saying that under the Constitution cooperatives are exempt from
taxation. Such theory is contrary to the Constitution under which only the following are exempt from The Court has given due course to both petitions. The parties, in compliance with the Court's directive,
taxation: charitable institutions, churches and parsonages, by reason of Art. VI, §28 (3), and non-stock, have filed their respective memoranda.
non-profit educational institutions by reason of Art. XIV, §4 (3).
G.R. No. 109289
CUP's further ground for seeking the invalidation of R.A. No. 7716 is that it denies cooperatives the equal
protection of the law because electric cooperatives are exempted from the VAT. The classification
Petitioner contends that the title of House Bill No. 34314, progenitor of Republic Act No. 7496, is a
between electric and other cooperatives (farmers cooperatives, producers cooperatives, marketing
misnomer or, at least, deficient for being merely entitled, "Simplified Net Income Taxation Scheme for the
cooperatives, etc.) apparently rests on a congressional determination that there is greater need to provide
Self-Employed
cheaper electric power to as many people as possible, especially those living in the rural areas, than there
and Professionals Engaged in the Practice of their Profession" (Petition in G.R. No. 109289).
is to provide them with other necessities in life. We cannot say that such classification is unreasonable.

The full text of the title actually reads:


We have carefully read the various arguments raised against the constitutional validity of R.A. No. 7716.
We have in fact taken the extraordinary step of enjoining its enforcement pending resolution of these
cases. We have now come to the conclusion that the law suffers from none of the infirmities attributed to it An Act Adopting the Simplified Net Income Taxation Scheme For The Self-Employed and Professionals
by petitioners and that its enactment by the other branches of the government does not constitute a grave Engaged In The Practice of Their Profession, Amending Sections 21 and 29 of the National Internal
abuse of discretion. Any question as to its necessity, desirability or expediency must be addressed to Revenue Code, as Amended.
Congress as the body which is electorally responsible, remembering that, as Justice Holmes has said,
"legislators are the ultimate guardians of the liberties and welfare of the people in quite as great a degree The pertinent provisions of Sections 21 and 29, so referred to, of the National Internal Revenue Code, as
as are the courts." (Missouri, Kansas & Texas Ry. Co. v. May, 194 U.S. 267, 270, 48 L. Ed. 971, 973 now amended, provide:
(1904)). It is not right, as petitioner in G.R. No. 115543 does in arguing that we should enforce the public
accountability of legislators, that those who took part in passing the law in question by voting for it in Sec. 21. Tax on citizens or residents. —
Congress should later thrust to the courts the burden of reviewing measures in the flush of enactment.
This Court does not sit as a third branch of the legislature, much less exercise a veto power over
legislation. xxx xxx xxx

WHEREFORE, the motions for reconsideration are denied with finality and the temporary restraining order (f) Simplified Net Income Tax for the Self-Employed and/or Professionals Engaged in the Practice of
previously issued is hereby lifted.SO ORDERED. Profession. — A tax is hereby imposed upon the taxable net income as determined in Section 27 received
during each taxable year from all sources, other than income covered by paragraphs (b), (c), (d) and (e) of
this section by every individual whether
G.R. No. 109289 October 3, 1994 a citizen of the Philippines or an alien residing in the Philippines who is self-employed or practices his
profession herein, determined in accordance with the following schedule:
RUFINO R. TAN, petitioner,vs. RAMON R. DEL ROSARIO,
147
Not over P10,000 3% Uniformity of taxation, like the kindred concept of equal protection, merely requires that all subjects or
objects of taxation, similarly situated, are to be treated alike both in privileges and liabilities (Juan Luna
Over P10,000 P300 + 9% Subdivision vs. Sarmiento, 91 Phil. 371). Uniformity does not forfend classification as long as: (1) the
but not over P30,000 of excess over P10,000 standards that are used therefor are substantial and not arbitrary, (2) the categorization is germane to
achieve the legislative purpose, (3) the law applies, all things being equal, to both present and future
conditions, and (4) the classification applies equally well to all those belonging to the same class (Pepsi
Over P30,000 P2,100 + 15% Cola vs. City of Butuan, 24 SCRA 3; Basco vs. PAGCOR, 197 SCRA 52).
but not over P120,00 of excess over P30,000
What may instead be perceived to be apparent from the amendatory law is the legislative intent to
Over P120,000 P15,600 + 20% increasingly shift the income tax system towards the schedular approach2 in the income taxation of
but not over P350,000 of excess over P120,000 individual taxpayers and to maintain, by and large, the present global treatment3 on taxable corporations.
We certainly do not view this classification to be arbitrary and inappropriate.
Over P350,000 P61,600 + 30%
of excess over P350,000 Petitioner gives a fairly extensive discussion on the merits of the law, illustrating, in the process, what he
believes to be an imbalance between the tax liabilities of those covered by the amendatory law and those
Sec. 29. Deductions from gross income. — In computing taxable income subject to tax under Sections who are not. With the legislature primarily lies the discretion to determine the nature (kind), object
21(a), 24(a), (b) and (c); and 25 (a)(1), there shall be allowed as deductions the items specified in (purpose), extent (rate), coverage (subjects) and situs (place) of taxation. This court cannot freely delve
paragraphs (a) to (i) of this section: Provided, however, That in computing taxable income subject to tax into those matters which, by constitutional fiat, rightly rest on legislative judgment. Of course, where a tax
under Section 21 (f) in the case of individuals engaged in business or practice of profession, only the measure becomes so unconscionable and unjust as to amount to confiscation of property, courts will not
following direct costs shall be allowed as deductions: hesitate to strike it down, for, despite all its plenitude, the power to tax cannot override constitutional
proscriptions. This stage, however, has not been demonstrated to have been reached within any
(a) Raw materials, supplies and direct labor; appreciable distance in this controversy before us.
(b) Salaries of employees directly engaged in activities in the course of or pursuant to the business or
practice of their profession; Having arrived at this conclusion, the plea of petitioner to have the law declared unconstitutional for being
(c) Telecommunications, electricity, fuel, light and water; violative of due process must perforce fail. The due process clause may correctly be invoked only when
(d) Business rentals; there is a clear contravention of inherent or constitutional limitations in the exercise of the tax power. No
(e) Depreciation; such transgression is so evident to us.
(f) Contributions made to the Government and accredited relief organizations for the rehabilitation of
calamity stricken areas declared by the President; and G.R. No. 109446
(g) Interest paid or accrued within a taxable year on loans contracted from accredited financial institutions
which must be proven to have been incurred in connection with the conduct of a taxpayer's profession,
trade or business. The several propositions advanced by petitioners revolve around the question of whether or not public
For individuals whose cost of goods sold and direct costs are difficult to determine, a maximum of forty respondents have exceeded their authority in promulgating Section 6, Revenue Regulations No. 2-93, to
per cent (40%) of their gross receipts shall be allowed as deductions to answer for business or carry out Republic Act No. 7496.
professional expenses as the case may be.
The questioned regulation reads:
On the basis of the above language of the law, it would be difficult to accept petitioner's view that the
amendatory law should be considered as having now adopted a gross income, instead of as having still Sec. 6. General Professional Partnership — The general professional partnership (GPP) and the partners
retained the net income, taxation scheme. The allowance for deductible items, it is true, may have comprising the GPP are covered by R. A. No. 7496. Thus, in determining the net profit of the partnership,
significantly been reduced by the questioned law in comparison with that which has prevailed prior to the only the direct costs mentioned in said law are to be deducted from partnership income. Also, the
amendment; limiting, however, allowable deductions from gross income is neither discordant with, nor expenses paid or incurred by partners in their individual capacities in the practice of their profession which
opposed to, the net income tax concept. The fact of the matter is still that various deductions, which are are not reimbursed or paid by the partnership but are not considered as direct cost, are not deductible
by no means inconsequential, continue to be well provided under the new law. from his gross income.

Article VI, Section 26(1), of the Constitution has been envisioned so as (a) to prevent log-rolling legislation The real objection of petitioners is focused on the administrative interpretation of public respondents that
intended to unite the members of the legislature who favor any one of unrelated subjects in support of the would apply SNIT to partners in general professional partnerships. Petitioners cite the pertinent
whole act, (b) to avoid surprises or even fraud upon the legislature, and (c) to fairly apprise the people, deliberations in Congress during its enactment of Republic Act No. 7496, also quoted by the Honorable
through such publications of its proceedings as are usually made, of the subjects of legislation.1 The Hernando B. Perez, minority floor leader of the House of Representatives, in the latter's privilege speech
above objectives of the fundamental law appear to us to have been sufficiently met. Anything else would by way of commenting on the questioned implementing regulation of public respondents following the
be to require a virtual compendium of the law which could not have been the intendment of the effectivity of the law, thusly:
constitutional mandate.
MR. ALBANO, Now Mr. Speaker, I would like to get the correct impression of this bill. Do we speak here
Petitioner intimates that Republic Act No. 7496 desecrates the constitutional requirement that taxation of individuals who are earning, I mean, who earn through business enterprises and therefore, should file
"shall be uniform and equitable" in that the law would now attempt to tax single proprietorships and an income tax return?
professionals differently from the manner it imposes the tax on corporations and partnerships. The
contention clearly forgets, however, that such a system of income taxation has long been the prevailing MR. PEREZ. That is correct, Mr. Speaker. This does not apply to corporations. It applies only to
rule even prior to Republic Act No. 7496. individuals.

(See Deliberations on H. B. No. 34314, August 6, 1991, 6:15 P.M.; Emphasis ours).

148
Other deliberations support this position, to wit: Corporations, (3) Estates under Judicial Settlement and (4) Irrevocable Trusts (irrevocable both as
to corpus and as to income).
MR. ABAYA . . . Now, Mr. Speaker, did I hear the Gentleman from Batangas say that this bill is intended
to increase collections as far as individuals are concerned and to make collection of taxes equitable? Partnerships are, under the Code, either "taxable partnerships" or "exempt partnerships." Ordinarily,
partnerships, no matter how created or organized, are subject to income tax (and thus alluded to as
MR. PEREZ. That is correct, Mr. Speaker. "taxable partnerships") which, for purposes of the above categorization, are by law assimilated to be
within the context of, and so legally contemplated as, corporations. Except for few variances, such as in
the application of the "constructive receipt rule" in the derivation of income, the income tax approach is
(Id. at 6:40 P.M.; Emphasis ours). alike to both juridical persons. Obviously, SNIT is not intended or envisioned, as so correctly pointed out
in the discussions in Congress during its deliberations on Republic Act 7496, aforequoted, to cover
In fact, in the sponsorship speech of Senator Mamintal Tamano on the Senate version of the SNITS, it is corporations and partnerships which are independently subject to the payment of income tax.
categorically stated, thus:
"Exempt partnerships," upon the other hand, are not similarly identified as corporations nor even
This bill, Mr. President, is not applicable to business corporations or to partnerships; it is only with respect considered as independent taxable entities for income tax purposes. A general professional partnership is
to individuals and professionals. (Emphasis ours) such an example.4 Here, the partners themselves, not the partnership (although it is still obligated to file
an income tax return [mainly for administration and data]), are liable for the payment of income tax in
The Court, first of all, should like to correct the apparent misconception that general professional their individual capacity computed on their respective and distributive shares of profits. In the
partnerships are subject to the payment of income tax or that there is a difference in the tax treatment determination of the tax liability, a partner does so as an individual, and there is no choice on the matter.
between individuals engaged in business or in the practice of their respective professions and partners in In fine, under the Tax Code on income taxation, the general professional partnership is deemed to be no
general professional partnerships. The fact of the matter is that a general professional partnership, unlike more than a mere mechanism or a flow-through entity in the generation of income by, and the ultimate
an ordinary business partnership (which is treated as a corporation for income tax purposes and so distribution of such income to, respectively, each of the individual partners.
subject to the corporate income tax), is not itself an income taxpayer. The income tax is imposed not on
the professional partnership, which is tax exempt, but on the partners themselves in their individual Section 6 of Revenue Regulation No. 2-93 did not alter, but merely confirmed, the above standing rule as
capacity computed on their distributive shares of partnership profits. Section 23 of the Tax Code, which now so modified by Republic Act
has not been amended at all by Republic Act 7496, is explicit: No. 7496 on basically the extent of allowable deductions applicable to all individual income taxpayers on
their non-compensation income. There is no evident intention of the law, either before or after the
Sec. 23. Tax liability of members of general professional partnerships. — (a) Persons exercising a amendatory legislation, to place in an unequal footing or in significant variance the income tax treatment
common profession in general partnership shall be liable for income tax only in their individual capacity, of professionals who practice their respective professions individually and of those who do it through a
and the share in the net profits of the general professional partnership to which any taxable partner would general professional partnership.
be entitled whether distributed or otherwise, shall be returned for taxation and the tax paid in accordance
with the provisions of this Title. WHEREFORE, the petitions are DISMISSED. No special pronouncement on costs.SO ORDERED

(b) In determining his distributive share in the net income of the partnership, each partner — G.R. No. L-114783 December 8, 1994

(1) Shall take into account separately his distributive share of the partnership's income, gain, loss, ROBERT V. TOBIAS vs.HON. CITY MAYOR BENJAMIN S. ABALOS
deduction, or credit to the extent provided by the pertinent provisions of this Code, and
Invoking their rights as taxpayers and as residents of Mandaluyong, herein petitioners assail the
(2) Shall be deemed to have elected the itemized deductions, unless he declares his distributive share of constitutionality of Republic Act No. 7675, otherwise known as "An Act Converting the Municipality of
the gross income undiminished by his share of the deductions. Mandaluyong into a Highly Urbanized City to be known as the City of Mandaluyong."

There is, then and now, no distinction in income tax liability between a person who practices his Prior to the enactment of the assailed statute, the municipalities of Mandaluyong and San Juan belonged
profession alone or individually and one who does it through partnership (whether registered or not) with to only one legislative district. Hon. Ronaldo Zamora, the incumbent congressional representative of this
others in the exercise of a common profession. Indeed, outside of the gross compensation income tax and legislative district, sponsored the bill which eventually became R.A. No. 7675. President Ramos signed
the final tax on passive investment income, under the present income tax system all individuals deriving R.A. No. 7675 into law on February 9, 1994.
income from any source whatsoever are treated in almost invariably the same manner and under a
common set of rules. Pursuant to the Local Government Code of 1991, a plebiscite was held on April 10, 1994. The people of
Mandaluyong were asked whether they approved of the conversion of the Municipality of Mandaluyong
We can well appreciate the concern taken by petitioners if perhaps we were to consider Republic Act No. into a highly urbanized city as provided under R.A. No. 7675. The turnout at the plebiscite was only
7496 as an entirely independent, not merely as an amendatory, piece of legislation. The view can easily 14.41% of the voting population. Nevertheless, 18,621 voted "yes" whereas 7,911 voted "no." By virtue of
become myopic, however, when the law is understood, as it should be, as only forming part of, and these results, R.A. No. 7675 was deemed ratified and in effect.
subject to, the whole income tax concept and precepts long obtaining under the National Internal Revenue
Code. To elaborate a little, the phrase "income taxpayers" is an all embracing term used in the Tax Code, Petitioners now come before this Court, contending that R.A. No. 7675, specifically Article VIII, Section 49
and it practically covers all persons who derive taxable income. The law, in levying the tax, adopts the thereof, is unconstitutional for being violative of three specific provisions of the Constitution.
most comprehensive tax situs of nationality and residence of the taxpayer (that renders citizens,
regardless of residence, and resident aliens subject to income tax liability on their income from all
sources) and of the generally accepted and internationally recognized income taxable base (that can Article VIII, Section 49 of R.A. No. 7675 provides:
subject non-resident aliens and foreign corporations to income tax on their income from Philippine
sources). In the process, the Code classifies taxpayers into four main groups, namely: (1) Individuals, (2) As a highly-urbanized city, the City of Mandaluyong shall have its own legislative district with the first
representative to be elected in the next national elections after the passage of this Act. The remainder of

149
the former legislative district of San Juan/Mandaluyong shall become the new legislative district of San necessarily includes and contemplates the subject treated under Section 49 regarding the creation of a
Juan with its first representative to be elected at the same election. separate congressional district for Mandaluyong.

Petitioner's first objection to the aforequoted provision of R.A. No. 7675 is that it contravenes the "one Moreover, a liberal construction of the "one title-one subject" rule has been invariably adopted by this
subject-one bill" rule, as enunciated in Article VI, Section 26(1) of the Constitution, to wit: court so as not to cripple or impede legislation. Thus, in Sumulong v. Comelec (73 Phil. 288 [1941]), we
ruled that the constitutional requirement as now expressed in Article VI, Section 26(1) "should be given a
Sec. 26(1). Every bill passed by the Congress shall embrace only one subject which shall be expressed in practical rather than a technical construction. It should be sufficient compliance with such requirement if
the title thereof. the title expresses the general subject and all the provisions are germane to that general subject."

Petitioners allege that the inclusion of the assailed Section 49 in the subject law resulted in the latter The liberal construction of the "one title-one subject" rule had been further elucidated in Lidasan v.
embracing two principal subjects, namely: (1) the conversion of Mandaluyong into a highly urbanized city; Comelec (21 SCRA 496 [1967]), to wit:
and (2) the division of the congressional district of San Juan/Mandaluyong into two separate districts.
Of course, the Constitution does not require Congress to employ in the title of an enactment, language of
Petitioners contend that the second aforestated subject is not germane to the subject matter of R.A. No. such precision as to mirror, fully index or catalogue all the contents and the minute details therein. It
7675 since the said law treats of the conversion of Mandaluyong into a highly urbanized city, as suffices if the title should serve the purpose of the constitutional demand that it inform the legislators, the
expressed in the title of the law. Therefore, since Section 49 treats of a subject distinct from that stated in persons interested in the subject of the bill and the public, of the nature, scope and consequences of the
the title of the law, the "one subject-one bill" rule has not been complied with. proposed law and its operation" (emphasis supplied).

Petitioners' second and third objections involve Article VI, Sections 5(1) and (4) of the Constitution, which Proceeding now to the other constitutional issues raised by petitioners to the effect that there is no
provide, to wit: mention in the assailed law of any census to show that Mandaluyong and San Juan had each attained the
minimum requirement of 250,000 inhabitants to justify their separation into two legislative districts, the
same does not suffice to strike down the validity of R.A. No. 7675. The said Act enjoys the presumption of
Sec. 5(1). The House of Representatives shall be composed of not more than two hundred and fifty having passed through the regular congressional processes, including due consideration by the members
members, unless otherwise fixed by law, who shall be elected from legislative districts apportioned among of Congress of the minimum requirements for the establishment of separate legislative districts. At any
the provinces, cities, and the Metropolitan Manila area in accordance with the number of their respective rate, it is not required that all laws emanating from the legislature must contain all relevant data
inhabitants, and on the basis of a uniform and progressive ratio, and those who, as provided by law, shall considered by Congress in the enactment of said laws.
be elected through a party list system of registered national, regional and sectoral parties or
organizations.
As to the contention that the assailed law violates the present limit on the number of representatives as
set forth in the Constitution, a reading of the applicable provision, Article VI, Section 5(1), as aforequoted,
Sec. 5(4). Within three years following the return of every census, the Congress shall make a shows that the present limit of 250 members is not absolute. The Constitution clearly provides that the
reapportionment of legislative districts based on the standard provided in this section. House of Representatives shall be composed of not more than 250 members, "unless otherwise provided
by law." The inescapable import of the latter clause is that the present composition of Congress may be
Petitioners argue that the division of San Juan and Mandaluyong into separate congressional districts increased, if Congress itself so mandates through a legislative enactment. Therefore, the increase in
under Section 49 of the assailed law has resulted in an increase in the composition of the House of congressional representation mandated by R.A. No. 7675 is not unconstitutional.
Representatives beyond that provided in Article VI, Sec. 5(1) of the Constitution. Furthermore, petitioners
contend that said division was not made pursuant to any census showing that the subject municipalities Thus, in the absence of proof that Mandaluyong and San Juan do not qualify to have separate legislative
have attained the minimum population requirements. And finally, petitioners assert that Section 49 has the districts, the assailed Section 49 of R.A.
effect of preempting the right of Congress to reapportion legislative districts pursuant to Sec. 5(4) as No. 7675 must be allowed to stand.
aforecited.
As to the contention that Section 49 of R.A. No. 7675 in effect preempts the right of Congress to
The contentions are devoid of merit. reapportion legislative districts, the said argument borders on the absurd since petitioners overlook the
glaring fact that it was Congress itself which drafted, deliberated upon and enacted the assailed law,
Anent the first issue, we agree with the observation of the Solicitor General that the statutory conversion including Section 49 thereof. Congress cannot possibly preempt itself on a right which pertains to itself.
of Mandaluyong into a highly urbanized city with a population of not less than two hundred fifty thousand
indubitably ordains compliance with the "one city-one representative" proviso in the Constitution: Aside from the constitutional objections to R.A. No. 7675, petitioners present further arguments against
the validity thereof.
. . . Each city with a population of at least two hundred fifty thousand, or each province, shall have at least
one representative" (Article VI, Section 5(3), Constitution). Petitioners contend that the people of San Juan should have been made to participate in the plebiscite on
R.A. No. 7675 as the same involved a change in their legislative district. The contention is bereft of merit
Hence, it is in compliance with the aforestated constitutional mandate that the creation of a separate since the principal subject involved in the plebiscite was the conversion of Mandaluyong into a highly
congressional district for the City of Mandaluyong is decreed under Article VIII, Section 49 of R.A. No. urbanized city. The matter of separate district representation was only ancillary thereto. Thus, the
7675. inhabitants of San Juan were properly excluded from the said plebiscite as they had nothing to do with the
change of status of neighboring Mandaluyong.
Contrary to petitioners' assertion, the creation of a separate congressional district for Mandaluyong is not
a subject separate and distinct from the subject of its conversion into a highly urbanized city but is a Similarly, petitioners' additional argument that the subject law has resulted in "gerrymandering," which is
natural and logical consequence of its conversion into a highly urbanized city. Verily, the title of R.A. No. the practice of creating legislative districts to favor a particular candidate or party, is not worthy of
7675, "An Act Converting the Municipality of Mandaluyong Into a Highly Urbanized City of Mandaluyong" credence. As correctly observed by the Solicitor General, it should be noted that Rep. Ronaldo Zamora,
the author of the assailed law, is the incumbent representative of the former San Juan/Mandaluyong

150
district, having consistently won in both localities. By dividing San Juan/Mandaluyong, Rep. Zamora's 6. WHEREAS, the rampant and unregulated showing of obscene videogram features constitutes a clear
constituency has in fact been diminished, which development could hardly be considered as favorable to and present danger to the moral and spiritual well-being of the youth, and impairs the mandate of the
him.WHEREFORE, the petition is hereby DISMISSED for lack of merit. SO ORDERED. Constitution for the State to support the rearing of the youth for civic efficiency and the development of
moral character and promote their physical, intellectual, and social well-being;
G.R. No. L-75697
7. WHEREAS, civic-minded citizens and groups have called for remedial measures to curb these blatant
VALENTIN TIO vs.VIDEOGRAM REGULATORY BOARD, malpractices which have flaunted our censorship and copyright laws;

This petition was filed on September 1, 1986 by petitioner on his own behalf and purportedly on behalf of 8. WHEREAS, in the face of these grave emergencies corroding the moral values of the people and
other videogram operators adversely affected. It assails the constitutionality of Presidential Decree No. betraying the national economic recovery program, bold emergency measures must be adopted with
1987 entitled "An Act Creating the Videogram Regulatory Board" with broad powers to regulate and dispatch; ... (Numbering of paragraphs supplied).
supervise the videogram industry (hereinafter briefly referred to as the BOARD). The Decree was
promulgated on October 5, 1985 and took effect on April 10, 1986, fifteen (15) days after completion of its Petitioner's attack on the constitutionality of the DECREE rests on the following grounds:
publication in the Official Gazette. 1. Section 10 thereof, which imposes a tax of 30% on the gross receipts payable to the local government
is a RIDER and the same is not germane to the subject matter thereof;
On November 5, 1985, a month after the promulgation of the abovementioned decree, Presidential 2. The tax imposed is harsh, confiscatory, oppressive and/or in unlawful restraint of trade in violation of
Decree No. 1994 amended the National Internal Revenue Code providing, inter alia: the due process clause of the Constitution;
3. There is no factual nor legal basis for the exercise by the President of the vast powers conferred upon
him by Amendment No. 6;
SEC. 134. Video Tapes. — There shall be collected on each processed video-tape cassette, ready for 4. There is undue delegation of power and authority;
playback, regardless of length, an annual tax of five pesos; Provided, That locally manufactured or 5. The Decree is an ex-post facto law; and
imported blank video tapes shall be subject to sales tax. 6. There is over regulation of the video industry as if it were a nuisance, which it is not.
We shall consider the foregoing objections in seriatim.
On October 23, 1986, the Greater Manila Theaters Association, Integrated Movie Producers, Importers
and Distributors Association of the Philippines, and Philippine Motion Pictures Producers Association, 1. The Constitutional requirement that "every bill shall embrace only one subject which shall be expressed
hereinafter collectively referred to as the Intervenors, were permitted by the Court to intervene in the case, in the title thereof" 1 is sufficiently complied with if the title be comprehensive enough to include the
over petitioner's opposition, upon the allegations that intervention was necessary for the complete general purpose which a statute seeks to achieve. It is not necessary that the title express each and every
protection of their rights and that their "survival and very existence is threatened by the unregulated end that the statute wishes to accomplish. The requirement is satisfied if all the parts of the statute are
proliferation of film piracy." The Intervenors were thereafter allowed to file their Comment in Intervention. related, and are germane to the subject matter expressed in the title, or as long as they are not
inconsistent with or foreign to the general subject and title. 2 An act having a single general subject,
The rationale behind the enactment of the DECREE, is set out in its preambular clauses as follows: indicated in the title, may contain any number of provisions, no matter how diverse they may be, so long
as they are not inconsistent with or foreign to the general subject, and may be considered in furtherance
1. WHEREAS, the proliferation and unregulated circulation of videograms including, among others, of such subject by providing for the method and means of carrying out the general object." 3 The rule also
videotapes, discs, cassettes or any technical improvement or variation thereof, have greatly prejudiced is that the constitutional requirement as to the title of a bill should not be so narrowly construed as to
the operations of moviehouses and theaters, and have caused a sharp decline in theatrical attendance by cripple or impede the power of legislation. 4 It should be given practical rather than technical
at least forty percent (40%) and a tremendous drop in the collection of sales, contractor's specific, construction. 5
amusement and other taxes, thereby resulting in substantial losses estimated at P450 Million annually in
government revenues; Tested by the foregoing criteria, petitioner's contention that the tax provision of the DECREE is a rider is
without merit. That section reads, inter alia:
2. WHEREAS, videogram(s) establishments collectively earn around P600 Million per annum from rentals,
sales and disposition of videograms, and such earnings have not been subjected to tax, thereby depriving Section 10. Tax on Sale, Lease or Disposition of Videograms. — Notwithstanding any provision of law to
the Government of approximately P180 Million in taxes each year; the contrary, the province shall collect a tax of thirty percent (30%) of the purchase price or rental rate, as
the case may be, for every sale, lease or disposition of a videogram containing a reproduction of any
3. WHEREAS, the unregulated activities of videogram establishments have also affected the viability of motion picture or audiovisual program. Fifty percent (50%) of the proceeds of the tax collected shall
the movie industry, particularly the more than 1,200 movie houses and theaters throughout the country, accrue to the province, and the other fifty percent (50%) shall acrrue to the municipality where the tax is
and occasioned industry-wide displacement and unemployment due to the shutdown of numerous collected; PROVIDED, That in Metropolitan Manila, the tax shall be shared equally by the City/Municipality
moviehouses and theaters; and the Metropolitan Manila Commission.

4. "WHEREAS, in order to ensure national economic recovery, it is imperative for the Government to x x x           x x x          x x x
create an environment conducive to growth and development of all business industries, including the
movie industry which has an accumulated investment of about P3 Billion; The foregoing provision is allied and germane to, and is reasonably necessary for the accomplishment of,
the general object of the DECREE, which is the regulation of the video industry through the Videogram
5. WHEREAS, proper taxation of the activities of videogram establishments will not only alleviate the dire Regulatory Board as expressed in its title. The tax provision is not inconsistent with, nor foreign to that
financial condition of the movie industry upon which more than 75,000 families and 500,000 workers general subject and title. As a tool for regulation 6 it is simply one of the regulatory and control
depend for their livelihood, but also provide an additional source of revenue for the Government, and at mechanisms scattered throughout the DECREE. The express purpose of the DECREE to include taxation
the same time rationalize the heretofore uncontrolled distribution of videograms; of the video industry in order to regulate and rationalize the heretofore uncontrolled distribution of
videograms is evident from Preambles 2 and 5, supra. Those preambles explain the motives of the
lawmaker in presenting the measure. The title of the DECREE, which is the creation of the Videogram
Regulatory Board, is comprehensive enough to include the purposes expressed in its Preamble and

151
reasonably covers all its provisions. It is unnecessary to express all those objectives in the title or that the concerned being "subject to the direction and control of the BOARD." That the grant of such authority
latter be an index to the body of the DECREE. 7 might be the source of graft and corruption would not stigmatize the DECREE as unconstitutional. Should
the eventuality occur, the aggrieved parties will not be without adequate remedy in law.
2. Petitioner also submits that the thirty percent (30%) tax imposed is harsh and oppressive, confiscatory,
and in restraint of trade. However, it is beyond serious question that a tax does not cease to be valid 5. The DECREE is not violative of the ex post facto principle. An ex post facto law is, among other
merely because it regulates, discourages, or even definitely deters the activities taxed. 8 The power to categories, one which "alters the legal rules of evidence, and authorizes conviction upon less or different
impose taxes is one so unlimited in force and so searching in extent, that the courts scarcely venture to testimony than the law required at the time of the commission of the offense." It is petitioner's position that
declare that it is subject to any restrictions whatever, except such as rest in the discretion of the authority Section 15 of the DECREE in providing that:
which exercises it. 9 In imposing a tax, the legislature acts upon its constituents. This is, in general, a
sufficient security against erroneous and oppressive taxation. 10 All videogram establishments in the Philippines are hereby given a period of forty-five (45) days after the
effectivity of this Decree within which to register with and secure a permit from the BOARD to engage in
The tax imposed by the DECREE is not only a regulatory but also a revenue measure prompted by the the videogram business and to register with the BOARD all their inventories of videograms, including
realization that earnings of videogram establishments of around P600 million per annum have not been videotapes, discs, cassettes or other technical improvements or variations thereof, before they could be
subjected to tax, thereby depriving the Government of an additional source of revenue. It is an end-user sold, leased, or otherwise disposed of. Thereafter any videogram found in the possession of any person
tax, imposed on retailers for every videogram they make available for public viewing. It is similar to the engaged in the videogram business without the required proof of registration by the BOARD, shall be
30% amusement tax imposed or borne by the movie industry which the theater-owners pay to the prima facie evidence of violation of the Decree, whether the possession of such videogram be for private
government, but which is passed on to the entire cost of the admission ticket, thus shifting the tax burden showing and/or public exhibition.
on the buying or the viewing public. It is a tax that is imposed uniformly on all videogram operators.
raises immediately a prima facie evidence of violation of the DECREE when the required proof of
The levy of the 30% tax is for a public purpose. It was imposed primarily to answer the need for regulating registration of any videogram cannot be presented and thus partakes of the nature of an ex post facto law.
the video industry, particularly because of the rampant film piracy, the flagrant violation of intellectual
property rights, and the proliferation of pornographic video tapes. And while it was also an objective of the The argument is untenable. As this Court held in the recent case of Vallarta vs. Court of Appeals, et al. 15
DECREE to protect the movie industry, the tax remains a valid imposition.
... it is now well settled that "there is no constitutional objection to the passage of a law providing that the
The public purpose of a tax may legally exist even if the motive which impelled the legislature to impose presumption of innocence may be overcome by a contrary presumption founded upon the experience of
the tax was to favor one industry over another. 11 human conduct, and enacting what evidence shall be sufficient to overcome such presumption of
innocence" (People vs. Mingoa 92 Phil. 856 [1953] at 858-59, citing 1 COOLEY, A TREATISE ON THE
It is inherent in the power to tax that a state be free to select the subjects of taxation, and it has been CONSTITUTIONAL LIMITATIONS, 639-641). And the "legislature may enact that when certain facts have
repeatedly held that "inequities which result from a singling out of one particular class for taxation or been proved that they shall be prima facie evidence of the existence of the guilt of the accused and shift
exemption infringe no constitutional limitation". 12 Taxation has been made the implement of the state's the burden of proof provided there be a rational connection between the facts proved and the ultimate
police power.13 facts presumed so that the inference of the one from proof of the others is not unreasonable and arbitrary
because of lack of connection between the two in common experience". 16
At bottom, the rate of tax is a matter better addressed to the taxing legislature.
Applied to the challenged provision, there is no question that there is a rational connection between the
3. Petitioner argues that there was no legal nor factual basis for the promulgation of the DECREE by the fact proved, which is non-registration, and the ultimate fact presumed which is violation of the DECREE,
former President under Amendment No. 6 of the 1973 Constitution providing that "whenever in the besides the fact that the prima facie presumption of violation of the DECREE attaches only after a forty-
judgment of the President ... , there exists a grave emergency or a threat or imminence thereof, or five-day period counted from its effectivity and is, therefore, neither retrospective in character.
whenever the interim Batasang Pambansa or the regular National Assembly fails or is unable to act
adequately on any matter for any reason that in his judgment requires immediate action, he may, in order 6. We do not share petitioner's fears that the video industry is being over-regulated and being eased out
to meet the exigency, issue the necessary decrees, orders, or letters of instructions, which shall form part of existence as if it were a nuisance. Being a relatively new industry, the need for its regulation was
of the law of the land." apparent. While the underlying objective of the DECREE is to protect the moribund movie industry, there
is no question that public welfare is at bottom of its enactment, considering "the unfair competition posed
In refutation, the Intervenors and the Solicitor General's Office aver that the 8th "whereas" clause by rampant film piracy; the erosion of the moral fiber of the viewing public brought about by the availability
sufficiently summarizes the justification in that grave emergencies corroding the moral values of the of unclassified and unreviewed video tapes containing pornographic films and films with brutally violent
people and betraying the national economic recovery program necessitated bold emergency measures to sequences; and losses in government revenues due to the drop in theatrical attendance, not to mention
be adopted with dispatch. Whatever the reasons "in the judgment" of the then President, considering that the fact that the activities of video establishments are virtually untaxed since mere payment of Mayor's
the issue of the validity of the exercise of legislative power under the said Amendment still pends permit and municipal license fees are required to engage in business. 17
resolution in several other cases, we reserve resolution of the question raised at the proper time.
The enactment of the Decree since April 10, 1986 has not brought about the "demise" of the video
4. Neither can it be successfully argued that the DECREE contains an undue delegation of legislative industry. On the contrary, video establishments are seen to have proliferated in many places
power. The grant in Section 11 of the DECREE of authority to the BOARD to "solicit the direct assistance notwithstanding the 30% tax imposed.
of other agencies and units of the government and deputize, for a fixed and limited period, the heads or
personnel of such agencies and units to perform enforcement functions for the Board" is not a delegation In the last analysis, what petitioner basically questions is the necessity, wisdom and expediency of the
of the power to legislate but merely a conferment of authority or discretion as to its execution, DECREE. These considerations, however, are primarily and exclusively a matter of legislative concern.
enforcement, and implementation. "The true distinction is between the delegation of power to make the
law, which necessarily involves a discretion as to what it shall be, and conferring authority or discretion as Only congressional power or competence, not the wisdom of the action taken, may be the basis for
to its execution to be exercised under and in pursuance of the law. The first cannot be done; to the latter, declaring a statute invalid. This is as it ought to be. The principle of separation of powers has in the main
no valid objection can be made." 14 Besides, in the very language of the decree, the authority of the wisely allocated the respective authority of each department and confined its jurisdiction to such a sphere.
BOARD to solicit such assistance is for a "fixed and limited period" with the deputized agencies
152
There would then be intrusion not allowable under the Constitution if on a matter left to the discretion of a The text of Republic Act No. 3836 reads:
coordinate branch, the judiciary would substitute its own. If there be adherence to the rule of law, as there
ought to be, the last offender should be courts of justice, to which rightly litigants submit their controversy AN ACT AMENDING SUBSECTION (c), SECTION TWELVE OF COMMONWEALTH ACT NUMBERED
precisely to maintain unimpaired the supremacy of legal norms and prescriptions. The attack on the ONE HUNDRED EIGHTY-SIX, AS AMENDED BY REPUBLIC ACT NUMBERED THIRTY HUNDRED
validity of the challenged provision likewise insofar as there may be objections, even if valid and cogent NINETY-SIX:
on its wisdom cannot be sustained. 18
Be it enacted by the Senate and House of Representatives of the Philippines in Congress assembled:
In fine, petitioner has not overcome the presumption of validity which attaches to a challenged statute. We
find no clear violation of the Constitution which would justify us in pronouncing Presidential Decree No.
1987 as unconstitutional and void. SECTION 1. Subsection (c), Section twelve of Commonwealth Act Numbered One Hundred eighty-six, as
amended by Republic Act Numbered Thirty hundred ninety-six, is further amended to read as follows:
WHEREFORE, the instant Petition is hereby dismissed.
"(c) Retirement is likewise allowed to a member, regardless of age, who has rendered at least twenty
years of service. The benefit shall, in addition to the return of his personal contributions plus interest and
No costs. the payment of the corresponding employer's premiums described in subsection (a) of Section five hereof,
without interest, be only a gratuity equivalent to one month's salary for every year of service, based on the
SO ORDERED. highest rate received, but not to exceed twenty-four months: Provided, That the retiring officer or
employee has been in the service of the said employer or office for at least four years immediately
G.R. No. L-23326      December 18, 1965 preceding his retirement.

PHILIPPINE CONSTITUTION ASSOCIATION, INC vs.PEDRO M. GIMENEZ "Retirement is also allowed to a senator or a member of the House of Representatives and to an elective
officer of either House of the Congress, regardless of age, provided that in the case of a Senator or
Member, he must have served at least twelve years as a Senator and/or as a member of the House of
We are called upon in this case to decide the grave and fundamental problem of the constitutionality of Representatives, and, in the case of an elective officer of either House, he must have served the
Republic Act No. 3836 "insofar as the same allows retirement gratuity and commutation of vacation and government for at least twelve years, not less than four years of which must have been rendered as such
sick leave to Senators and Representatives, and to the elective officials of both houses (of Congress)." elective officer: Provided, That the gratuity payable to a retiring senator, member of the House of
The suit was instituted by the Philippine Constitution Association, Inc. (Philconsa, for short), a non-profit Representatives, or elective officer, of either House, shall be equivalent to one year's salary for every four
civic organization, duly incorporated under Philippine laws, by way of a petition for prohibition with years of service in the government and the same shall be exempt from any tax whatsoever and shall be
preliminary injunction to restrain the Auditor General of the Philippines and the disbursing officers of both neither liable to attachment or execution nor refundable in case of reinstatement or re-election of the
Houses of Congress from "passing in audit the vouchers, and from countersigning the checks or treasury retiree.
warrants for the payment to any former Senator or former Member of the House of Representatives of
retirement and vacation gratuities pursuant to Republic Act No. 3836; and likewise restraining the
respondent disbursing officers of the House and Senate, respectively, and their successors in office from "This gratuity is payable by the employer or office concerned which is hereby authorized to provide the
paying the said retirement and vacation gratuities." necessary appropriation or pay the same from any unexpended items of appropriations or savings in its
appropriations or saving in its appropriations.
It is argued that the above-numbered Republic Act, at least to the end that it provided for the retirement of
the members of Congress in the manner and terms that it did, is unconstitutional and void. The challenge "Elective or appointive officials and employees paid gratuity under this subsection shall be entitled to the
to the constitutionality of the law is centered on the following propositions: commutation of the unused vacation and sick leave, based on the highest rate received, which they may
have to their credit at the time of retirement."
1. The provision for the retirement of the members and certain officers of Congress is not expressed in the
title of the bill, in violation of section 21 (1) of Article VI of the Constitution. SECTION 2. This Act shall take effect upon its approval.

2. The provision on retirement gratuity is an attempt to circumvent the Constitutional ban on increase of Approved, June 22, 1963.
salaries of the members of Congress during their term of office, contrary to the provisions of Article VI,
Section 14 of the Constitution. The Solicitor General's Office, in representation of the respondent, filed its answer on September 8, 1964,
and contends, by way of special and affirmative defenses that:
3. The same provision constitutes "selfish class legislation" because it allows members and officers of
Congress to retire after twelve (12) years of service and gives them a gratuity equivalent to one year 1. The grant of retirement or pension benefits under Republic Act No. 3836 to the officers objected to by
salary for every four years of service, which is not refundable in case of reinstatement or re-election of the the petitioner does not constitute "forbidden compensation" within the meaning of Section 14 of Article VI
retiree, while all other officers and employees of the government can retire only after at least twenty (20) of the Philippine Constitution.
years of service and are given a gratuity which is only equivalent to one month salary for every year of
service, which, in any case, cannot exceed 24 months. 2. The title of the law in question sufficiently complies with the provisions of Section 21, Article VI, of the
Constitution that "no bill which may be enacted into law shall embrace more than one subject which shall
4. The provision on vacation and sick leave, commutable at the highest rate received, insofar as members be expressed in the title of the bill.
of Congress are concerned, is another attempt of the legislators to further increase their compensation in
violation of the Constitution. 3. The law in question does not constitute legislation.

The text of Republic Act No. 3836

153
4. Certain indispensable parties, specifically the elected officers of Congress who are authorized to funds, which may be enjoined at the request of the taxpayers."1 This legislation (Republic Act 3836)
approve vouchers for payments for funds under the law in question, and the claimants to the vouchers to involves the disbursement of public funds.
be presented for payment under said items, were not included in the petition.
We are not, however, unmindful of the ruling laid down by the Supreme Court of the United States in the
5. The petitioner has no standing to institute this suit. case of Massachusetts v. Mellon, 262 U.S. 447, holding that:

6. The payment of commutable vacation and sick leave benefits under the said Act is merely "in the ... the relation of a taxpayer of the United States to the Federal Government is very different. His interest
nature of a basis for computing the gratuity due each retiring member" and, therefore, is not an indirect in the moneys of the Treasury — partly realized from taxation and partly from other sources — is shared
scheme to increase their salary. with millions of others; is comparatively minute and indeterminable; and the effect upon future taxation of
any payment out of the funds, so remote, fluctuating and uncertain, that no basis is afforded for an appeal
A brief historical background of Republic Act No. 3836 to the preventive powers of equity.

Republic Act No. 3836 was originally House Bill No. 6051, which was introduced by Congressmen Marcial The general view in the United States, which is followed here, is stated in the American Jurisprudence,
R. Pimentel of Camarines Norte and Marcelino R. Veloso of the Third District of Leyte, on May 6, 1963. thus —
On the same date, it was referred to the Committee on Civil Service. which on the following May 8,
submitted its REPORT No. 3129, recommending approval of the bill with amendments, among others, In the determination of the degree of interest essential to give the requisite standing to attack the
that the word "TWENTY" in the bill as filed — representing the number of years that a senator or member constitutionality of a statute the general rule is that not only persons individually affected, but
must serve in Congress to entitle him to retirement under the bill — must be reduced to "TWELVE" years, also taxpayers have sufficient interest in preventing the illegal expenditure of moneys raised by taxation
and that the following words were inserted, namely, "AND THE SAME (referring to gratuity) SHALL BE and may therefore question the constitutionality of statutes requiring expenditure of public moneys. (11
EXEMPT FROM ANY TAX WHATSOEVER AND SHALL NOT BE LIABLE FROM ATTACHMENT OR Am. Jur. 761; emphasis supplied.)
EXECUTION NOR REFUNDABLE IN CASE OF REINSTATEMENT OR REELECTION OF THE
RETIREE." On May 8, 1963, the bill with the proposed amendments was approved on second reading. It As far as the first point is concerned, We hold, therefore, that the contention of the Solicitor General is
was passed on third reading on May 13, 1963, and on the same day was sent to the Senate, which, in untenable.
turn, on May 23, 1963, passed it without amendment. The bill was finally approved on June 22, 1963. As
explained in the EXPLANATORY NOTE attached to the bill, among others —
Second legal point —Whether or not Republic Act No. 3836 falls within the prohibition embodied in Art. VI,
section 14 of the Constitution.
The inclusion of members of Congress in subsection (c), Section 12 of C.A. 186, as amended, will enable
them to retire voluntarily, regardless of age, after serving a minimum of twenty years as a Member of
Congress. This gratuity will insure the security of the family of the retiring member of Congress with the The first constitutional question is whether Republic Act 3836 violates Section 14, Article VI, of the
latter engaging in other activities which may detract from his exalted position and usefulness as lawmaker. Constitution, which reads as follows:
It is expected that with this assurance of security for his loved ones, deserving and well-intentioned but
poor men will be attracted to serve their people in Congress. The senators and the Members of the House of Representatives shall, unless otherwise provided by law,
receive an annual compensation of seven thousand two hundred pesos each, including per diems
As finally approved, the law (Subsection [c], paragraph 2, Section 1, R.A. 3836) allows a Senator or a and other emoluments or allowances, and exclusive only of travelling expenses to and from their
Member of the House of Representatives and an elective officer of either House of Congress to retire respective districts in the case of Members of the House of Representative and to and from their places of
regardless of age. To be eligible for retirement, he must have served for at least twelve years as such residence in the case of Senators, when attending sessions of the Congress. No increase in said
Senator and/or as member of the House of Representatives. For an elective officer of either House, he compensation shall take effect until after the expiration of the full term of all the Members of the Senate
must have served the government for at least twelve years, of which not less than four years must have and of the House of Representatives approving such increase. Until otherwise provided by law, the
been rendered as such elective officer. The gratuity payable by the employer or office concerned is President of the Senate and the Speaker of the House of Representatives shall each receive an annual
equivalent to one year's salary for every four years of service in the government. Said gratuity is exempt compensation of sixteen thousand pesos (emphasis supplied)
from taxation, not liable to attachment or execution, and not refundable in case of reinstatement or re-
election of the retiree. Before discussing this point, it is worthy to note that the Constitution embodies some limitations and
prohibitions upon the members of Congress, to wit:
First legal point — personality of the Petitioner to bring suit.
1. They may not hold any other office or employment in the Government without forfeiting their respective
The first point to be considered is whether petitioner Philconsa has a standing to institute this action. This seats;
Court has not hesitated to examine past decisions involving this matter. This Court has repeatedly held
that when the petitioner, like in this case, is composed of substantial taxpayers, and the outcome will 2. They shall not be appointed, during the time for which they are elected, to any civil office which may
affect their vital interests, they are allowed to bring this suit. (Pascual v. Secretary, G.R. No. L-10405, have been created or the emoluments whereof shall have been increased while they were members of
December 29, 1960; and Gonzales v. Hechanova, 60 Off. Gaz. 802 [1963]). Congress; (Section 16, Article VI, Constitution)

The petitioner, Philconsa, is precisely a non-profit, civic organization composed of several leaders from all 3. They cannot be financially interested in any franchise;
walks of life whose main objective is to uphold the principles of the Constitution.
4. They cannot appear in any civil case wherein the Government is an adverse party;
In rejecting the motion to dismiss in the case of Pascual v. Secretary, supra, this Court stated, among
other things, that "there are many decisions nullifying, at the instance of the taxpayers, laws providing the 5. They cannot appear as counsel before any Electoral Tribunal; and
disbursement of public funds, upon the theory that the expenditure of public funds by an officer of the
State for the purpose of administering an unconstitutional act constitutes a misappropriation of such

154
6. They cannot appear as counsel in any criminal case where an officer or employee of the Government is office, the emoluments of which have been increased during the session of the Legislature of which he
accused. (Section 17, Article VI, Constitution) was a member, until after the expiration of his term of office in the Legislature, the word "emoluments"
does not refer to the fixed salary alone, but includes fees and compensation as the incumbent of the office
In addition to the above prohibitions, the Anti-Graft Law (Republic Act 3019) also prohibits members of is by law entitled to receive because he holds such office and performed some service required of the
Congress to have any special interest in any specific business which will directly or indirectly be favored occupant thereof."
by any law or resolution authored by them during their term of office.
From the decisions of these cases, it is evident that retirement benefit is a form or another species of
It is thus clear that the Constitutional Convention wisely surrounded the Constitution with these limitations emolument, because it is a part of compensation for services of one possessing any office.
and prohibitions upon Members of Congress. This is a practical demonstration or application of the
principle of the and balances which is one of the peculiar characteristics of our Constitution. In the light of Republic Act No. 3836 provides for an increase in the emoluments of Senators and Members of the
this background, can We conclude that Congress can validly enact Republic Act 3836, providing House of Representatives, to take effect upon the approval of said Act, which was on June 22, 1963.
retirement benefits to its members, without violating the provisions in the aforementioned Article VI, Retirement benefits were immediately available thereunder, without awaiting the expiration of the full term
Section 14, of the Constitution, regarding increase of the compensation act including other emoluments? of all the Members of the Senate and the House of Representatives approving such increase. Such
provision clearly runs counter to the prohibition in Article VI, Section 14 of the Constitution.
It is worthy to note that the original salary for the members of the National Assembly (unicameral body)
was fixed at P5,000.00 per annum each. This was raised to P7,200 per annum by the enactment of the Third Legal Point — Whether or not the law in question violates the equal protection clause of the
1940 Constitutional amendment, when the unicameral body, the National Assembly, was changed to Constitution.
Congress, composed of two bodies, the Senate and the House of Representatives. Again, in 1964, by the
enactment of Republic Act 4143, the salary for the Members of Congress was raised to P32,000.00 per Another reason in support of the conclusion reached herein is that the features of said Republic Act 3836
annum for each of them; and for the President of the Senate and the Speaker of the House of are patently discriminatory, and therefore violate the equal protection clause of the Constitution. (Art. III,
Representatives, to P40,000.00 per annum each. Sec. 1, part. 1.)

Likewise, it is significant that, as stated above, when the Constitutional Convention first determined the In the first place, while the said law grants retirement benefits to Senators and Members of the House of
compensation for the Members of Congress, the amount fixed by it was only P5,000.00 per annum, but it Representatives who are elective officials, it does not include other elective officials such as the governors
embodies a special proviso which reads as follows: "No increase in said compensation shall take effect of provinces and the members of the provincial boards, and the elective officials of the municipalities and
until after the expiration of the full term of all the members of the National Assembly elected subsequent to chartered cities.
approval of such increase." In other words, under the original constitutional provision regarding the power
of the National Assembly to increase the salaries of its members, no increase would take effect until after
the expiration of the full term of the members of the Assembly elected subsequent to the approval of such The principle of equal protection of law embodied in our Constitution has been fully explained by Us in the
increase. (See Aruego, The Framing of the Constitution, Vol. 1, pp. 296-300; Sinco, Philippine case of People v. Vera, 65 Phil. 56, 126, where We stated that the classification to be reasonable must be
Government and Political Law, 4th ed., p. 187) based upon substantial distinctions which make real differences and must be germane to the purposes of
the law.
This goes to show how zealous were the members of the Constitutional Convention in guarding against
the temptation for members of Congress to increase their salaries. However, the original strict prohibition As well stated by Willoughby on the Constitution of the United States (second edition), p. 1937, the
was modified by the subsequent provision when the Constitutional amendments were approved in 19402 principle of the requirement of equal protection of law applies to all persons similarly situated. Why limit
the application of the benefits of Republic Act 3836 to the elected members of Congress? We feel that the
classification here is not reasonable. (See also Sinco, Philippine Political Law, 11th ed. [1962]; Selected
The Constitutional provision in the aforementioned Section 14, Article VI, includes in the term Essays on Constitutional Law [1938-62], p. 789; The Equal Protection of the Laws, 37 Cal. Law Rev. 341.)
compensation "other emoluments." This is the pivotal point on this fundamental question as to whether
the retirement benefits as provided for in Republic Act 3836 fall within the purview of the term "other
emoluments." Secondly, all members of Congress under Republic Act 3836 are given retirement benefits after serving
twelve years, not necessarily continuous, whereas, most government officers and employees are given
retirement benefits after serving for at least twenty years. In fact, the original bill of Act 3836 provided for
Most of the authorities and decided cases have regarded "emolument" as "the profit arising from office or twenty years of service.
employment; that which is received as compensation for services or which is annexed to the possession
of an office, as salary, fees and perquisites.3
In the third place, all government officers and employees are given only one retirement benefit
irrespective of their length of service in the government, whereas, under Republic Act 3836, because of
In another set of cases, "emolument" has been defined as "the profit arising from office or employment; no age limitation, a Senator or Member of the House of Representatives upon being elected for 24 years
that which is received as compensation for services, or which is annexed to the possession of office, as will be entitled to two retirement benefits or equivalent to six years' salary.
salary, fees and perquisites; advantage, gain, public or private." The gain, profit or advantage which is
contemplated in the definition or significance of the word "emolument" as applied to public officers, clearly
comprehends, We think, a gain, profit, or advantage which is pecuniary in character. (citing Taxpayers' Also, while the payment of retirement benefits (annuity) to an employee who had been retired and
League of Cargon County v. McPherson, 54 P. 2d. 897, 90l.: 49 Wy. 26; 106 A.L.R. 767) reappointed is suspended during his new employment (under Commonwealth Act 186, as amended), this
is not so under Republic Act 3836.
In Schieffelin v. Berry, 216 N.Y.S. (citing Wright v. Craig, 202 App. Div. 684, 195 N.Y.S. 391, affirmed 234
N.Y. 548, 138 N.E. 441), it has been established that pensions and retirement allowances are part of Lastly, it is peculiar that Republic Act 3836 grants retirement benefits to officials who are not members of
compensation of public officials; otherwise their payment would be unconstitutional. the Government Service Insurance System. Most grantees of retirement benefits under the various
retirement laws have to be members or must at least contribute a portion of their monthly salaries to the
System.4
In another case, State v. Schmahl, 145 N.W. 795, 125 Minn. 104, it is stated that "as used in Article 4,
section 9, of the Constitution of Minnesota, providing that no Senator or Representative shall hold any

155
The arguments advanced against the discriminatory features of Republic Act 3836, as far as Members of It is to be observed that under Republic Act 3836, amending the first paragraph of section 12, subsection
Congress are concerned, apply with equal force to the elected officers of each House, such as the (c) of Commonwealth Act 186, as amended by Republic Acts Nos. 660 and. 3096, the retirement benefits
Secretaries and the Sergeants-at-arms. Under Republic Act 3836, the Secretaries and Sergeants-at-arms are granted to members of the Government Service Insurance System, who have rendered at least twenty
of each House are given the benefits of retirement without having served for twenty years as required with years of service regardless of age. This paragraph is related and germane to the subject of
other officers and employees of the Government. Commonwealth Act No. 186.

Fourth Legal Point — Whether or not the title of Republic Act No. 3836 is germane to the subject matter On the other hand, the succeeding paragraph of Republic Act 3836 refers to members of Congress and to
expressed in the act. elective officers thereof who are not members of the Government Service Insurance System. To provide
retirement benefits, therefore, for these officials, would relate to subject matter which is not germane to
Another Constitutional point to determine is whether the title of Republic Act 3836 complies with the Commonwealth Act No. 186. In other words, this portion of the amendment (re retirement benefits for
requirement of paragraph 1, section 21, Article VI of the Constitution, which reads as follows: Members of Congress and elected officers, such as the Secretary and Sergeants-at-arms for each House)
is not related in any manner to the subject of Commonwealth Act 186 establishing the Government
Service Insurance System and which provides for both retirement and insurance benefits to its members.
No bill which may be enacted into law shall embrace more than one subject which shall be expressed in
the title of the bill.
Parenthetically, it may be added that the purpose of the requirement that the subject of an Act should be
expressed in its title is fully explained by Cooley, thus: (1) to prevent surprise or fraud upon the
We are not unmindful of the fact that there has been a general disposition in all courts to construe the Legislature; and (2) to fairly apprise the people, through such publication of legislation that are being
constitutional provision with reference to the subject and title of the Act, liberally. considered, in order that they may have the opportunity of being heard thereon by petition or otherwise, if
they shall so desire (Cooley, Constitutional Limitations, 8th ed., Vol. 1, p. 162; See also Martin, Political
It is the contention of petitioner that the said title of Republic Act 3836 gives no inkling or notice Law Reviewer, Book One [1965], p. 119)
whatsoever to the public regarding the retirement gratuities and commutable vacation and sick leave
privileges to members of Congress. It is claimed that petitioner learned of this law for the first time only With respect to sufficiency of title this Court has ruled in two cases:
when Jose Velasco, disbursing officer of the House, testified on January 30, 1964, before Justice
Labrador, in connection with the hearing of the case, and he revealed that in 1963, Congress enacted the
retirement law for its members. In fact the Appropriation Act for the fiscal year 1964-65, Republic Act No. The Constitutional requirement with respect to titles of statutes as sufficient to reflect their contents is
4164, provides: satisfied if all parts of a law relate to the subject expressed in its title, and it is not necessary that the title
be a complete index of the content. (People v. Carlos, 78 Phil. 535)
13. For payment of retirement gratuities of members of the Senate pursuant to the provisions of Republic
Act No. 3836: PROVIDED, That no portion of this Appropriation shall be transferred to any other item until The Constitutional requirement that the subject of an act shall be expressed in its title should be
all approved claims shall have been paid — P210,000.00. reasonably construed so as not to interfere unduly with the enactment of necessary legislation. It should
be given a practical, rather than technical, construction. It should be a sufficient compliance with such
requirement if the title expresses the general subject and all the provisions of the statute are germane to
In the appropriations for the House of Representatives the following items appear: that general subject. (Sumulong v. The Commission on Elections, 73 Phil. 288, 291)

7. For government share of premiums on life insurance and retirement of Members and employees of the The requirement that the subject of an act shall be expressed in its title is wholly illustrated and explained
House of Representatives, as provided for under Republic Act No. 1616 — P300,000.00 in Central Capiz v. Ramirez, 40 Phil. 883. In this case, the question raised was whether Commonwealth
Act 2784, known as the Public Land Act, was limited in its application to lands of the public domain or
8. For payment of the cash commutation of the accumulated vacation and sick leaves as provided for whether its provisions also extended to agricultural lands held in private ownership. The Court held that
under Republic Act No. 611, and retirement gratuities of Members and employees of the House of the act was limited to lands of the public domain as indicated in its title, and did not include private
Representatives under Republic Act No. 1616 —P1,300,000.00. agricultural lands. The Court further stated that this provision of the Constitution expressing the subject
matter of an Act in its title is not a mere rule of legislative procedure, directory to Congress, but it
In the Appropriations Act of 1965-1966 (Republic Act No. 4642), the following item appears in the is mandatory. It is the duty of the Court to declare void any statute not conforming to this constitutional
appropriations for the Senate: provision. (See Walker v. State, 49 Alabama 329; Cooley, Constitutional Limitations, pp. 162-164;5 See
also Agcaoili v. Suguitan, 48 Phil. 676; Sutherland on Statutory Construction, Sec. 111.)
13. For payment of retirement gratuities of Senate personnel pursuant to the provisions of Republic Act
No. 1616: PROVIDED, That no portion of this appropriation shall be transferred to any other item until all In the light of the history and analysis of Republic Act 3836, We conclude that the title of said Republic Act
approved claims shall have been paid — P100,000.00. 3836 is void as it is not germane to the subject matter and is a violation of the aforementioned paragraph
1, section 21, Article VI of the Constitution.
It is thus clear that in the Appropriations Act for 1965-1966, the item in the Senate for P210,000.00 to
implement Republic Act 3836 was eliminated. In short, Republic Act 3836 violates three constitutional provisions, namely: first, the prohibition regarding
increase in the salaries of Members of Congress; second, the equal protection clause; and third, the
prohibition that the title of a bill shall not embrace more than one subject.
In the appropriations for the House (1965-1966), the following items appear:

IN VIEW OF THE FOREGOING CONSIDERATIONS, Republic Act No. 3836 is hereby declared null and
7. For government share of premiums on life insurance and retirement of Members and employees of the
void, in so far as it refers to the retirement of Members of Congress and the elected officials thereof, as
House Of Representatives as provided for under Republic Act No. 1616 — P1,200,000.00.
being unconstitutional. The restraining order issued in our resolution on December 6, 1965 is hereby
made permanent. No costs
8. For payment of the cash commutation of the accumulated vacation and sick leaves as provided for
under Republic Act No. 611, and retirement gratuities of Members and employees of the House of
Representatives under Republic Act No. 1616 — P1,700,000.00.

156
157

You might also like